РСшСниС Π·Π°Π΄Π°Ρ‡ ΠΏΠΎ Ρ„ΠΈΠ·ΠΈΠΊΠ΅ с Π±Π»ΠΎΠΊΠ°ΠΌΠΈ: ΠŸΠΎΠ΄Π²ΠΈΠΆΠ½Ρ‹ΠΉ ΠΈ Π½Π΅ΠΏΠΎΠ΄Π²ΠΈΠΆΠ½Ρ‹ΠΉ Π±Π»ΠΎΠΊ, с ΠΏΡ€ΠΈΠΌΠ΅Ρ€Π°ΠΌΠΈ Π·Π°Π΄Π°Ρ‡

Π‘ΠΎΠ΄Π΅Ρ€ΠΆΠ°Π½ΠΈΠ΅

ΠŸΠΎΠ΄Π²ΠΈΠΆΠ½Ρ‹ΠΉ ΠΈ Π½Π΅ΠΏΠΎΠ΄Π²ΠΈΠΆΠ½Ρ‹ΠΉ Π±Π»ΠΎΠΊ, с ΠΏΡ€ΠΈΠΌΠ΅Ρ€Π°ΠΌΠΈ Π·Π°Π΄Π°Ρ‡

Π‘Π»ΠΎΠΊΠΈ относят ΠΊ простым ΠΌΠ΅Ρ…Π°Π½ΠΈΠ·ΠΌΠ°ΠΌ. Π’ Π³Ρ€ΡƒΠΏΠΏΡƒ этих устройств, ΠΊΠΎΡ‚ΠΎΡ€Ρ‹Π΅ слуТат для прСобразования силы, ΠΏΠΎΠΌΠΈΠΌΠΎ Π±Π»ΠΎΠΊΠΎΠ² относят Ρ€Ρ‹Ρ‡Π°Π³, Π½Π°ΠΊΠ»ΠΎΠ½Π½ΡƒΡŽ ΠΏΠ»ΠΎΡΠΊΠΎΡΡ‚ΡŒ.

Π˜Π·Π³ΠΎΡ‚Π°Π²Π»ΠΈΠ²Π°ΡŽΡ‚ΡΡ Π±Π»ΠΎΠΊΠΈ Π² Π²ΠΈΠ΄Π΅ дисков (колСс, Π½ΠΈΠ·ΠΊΠΈΡ… Ρ†ΠΈΠ»ΠΈΠ½Π΄Ρ€ΠΎΠ² ΠΈ Ρ‚. ΠΏ.), ΠΈΠΌΠ΅ΡŽΡ‰ΠΈΡ… ΠΆΠ΅Π»ΠΎΠ±, Ρ‡Π΅Ρ€Π΅Π· ΠΊΠΎΡ‚ΠΎΡ€Ρ‹ΠΉ ΠΏΡ€ΠΎΠΏΡƒΡΠΊΠ°ΡŽΡ‚ Π²Π΅Ρ€Π΅Π²ΠΊΡƒ (торс, ΠΊΠ°Π½Π°Ρ‚, Ρ†Π΅ΠΏΡŒ).

НСподвиТный Π±Π»ΠΎΠΊ

НСподвиТным называСтся Π±Π»ΠΎΠΊ, с Π·Π°ΠΊΡ€Π΅ΠΏΠ»Π΅Π½Π½ΠΎΠΉ осью (рис.1). Он Π½Π΅ пСрСмСщаСтся ΠΏΡ€ΠΈ подъСмС Π³Ρ€ΡƒΠ·Π°. НСподвиТный Π±Π»ΠΎΠΊ ΠΌΠΎΠΆΠ½ΠΎ Ρ€Π°ΡΡΠΌΠ°Ρ‚Ρ€ΠΈΠ²Π°Ρ‚ΡŒ ΠΊΠ°ΠΊ Ρ€Ρ‹Ρ‡Π°Π³, ΠΊΠΎΡ‚ΠΎΡ€Ρ‹ΠΉ ΠΈΠΌΠ΅Π΅Ρ‚ Ρ€Π°Π²Π½Ρ‹Π΅ ΠΏΠ»Π΅Ρ‡ΠΈ.

УсловиСм равновСсия Π±Π»ΠΎΠΊΠ° являСтся условиС равновСсия ΠΌΠΎΠΌΠ΅Π½Ρ‚ΠΎΠ² сил, ΠΏΡ€ΠΈΠ»ΠΎΠΆΠ΅Π½Π½Ρ‹Ρ… ΠΊ Π½Π΅ΠΌΡƒ:

Β  Β 

Π‘Π»ΠΎΠΊ Π½Π° рис.1 Π±ΡƒΠ΄Π΅Ρ‚ Π½Π°Ρ…ΠΎΠ΄ΠΈΡ‚ΡŒΡΡ Π² равновСсии, Ссли силы натяТСния Π½ΠΈΡ‚Π΅ΠΉ Ρ€Π°Π²Π½Ρ‹:

Β  Β 

Ρ‚Π°ΠΊ ΠΊΠ°ΠΊ ΠΏΠ»Π΅Ρ‡ΠΈ этих сил ΠΎΠ΄ΠΈΠ½Π°ΠΊΠΎΠ²Ρ‹ (ОА=ΠžΠ’). НСподвиТный Π±Π»ΠΎΠΊ Π½Π΅ Π΄Π°Π΅Ρ‚ Π²Ρ‹ΠΈΠ³Ρ€Ρ‹ΡˆΠ° Π² силС, Π½ΠΎ ΠΎΠ½ позволяСт ΠΈΠ·ΠΌΠ΅Π½ΠΈΡ‚ΡŒ Π½Π°ΠΏΡ€Π°Π²Π»Π΅Π½ΠΈΠ΅ дСйствия силы. Π’ΡΠ½ΡƒΡ‚ΡŒ Π·Π° Π²Π΅Ρ€Π΅Π²ΠΊΡƒ, которая ΠΈΠ΄Π΅Ρ‚ свСрху часто ΡƒΠ΄ΠΎΠ±Π½Π΅Π΅, Ρ‡Π΅ΠΌ Π·Π° Π²Π΅Ρ€Π΅Π²ΠΊΡƒ, которая ΠΈΠ΄Π΅Ρ‚ снизу.

Если масса Π³Ρ€ΡƒΠ·Π°, привязанного ΠΊ ΠΎΠ΄Π½ΠΎΠΌΡƒ ΠΈΠ· ΠΊΠΎΠ½Ρ†ΠΎΠ² Π²Π΅Ρ€Π΅Π²ΠΊΠΈ, ΠΏΠ΅Ρ€Π΅ΠΊΠΈΠ½ΡƒΡ‚ΠΎΠΉ Ρ‡Π΅Ρ€Π΅Π· Π½Π΅ΠΏΠΎΠ΄Π²ΠΈΠΆΠ½Ρ‹ΠΉ Π±Π»ΠΎΠΊ Ρ€Π°Π²Π½Π° m, Ρ‚ΠΎ для Ρ‚ΠΎΠ³ΠΎ, Ρ‡Ρ‚ΠΎΠ±Ρ‹ Π΅Π³ΠΎ ΠΏΠΎΠ΄Π½ΠΈΠΌΠ°Ρ‚ΡŒ, ΠΊ Π΄Ρ€ΡƒΠ³ΠΎΠΌΡƒ ΠΊΠΎΠ½Ρ†Ρƒ Π²Π΅Ρ€Π΅Π²ΠΊΠΈ слСдуСт ΠΏΡ€ΠΈΠΊΠ»Π°Π΄Ρ‹Π²Π°Ρ‚ΡŒ силу F, Ρ€Π°Π²Π½ΡƒΡŽ:

Β  Β 

ΠΏΡ€ΠΈ условии, Ρ‡Ρ‚ΠΎ силу трСния Π² Π±Π»ΠΎΠΊΠ΅ ΠΌΡ‹ Π½Π΅ ΡƒΡ‡ΠΈΡ‚Ρ‹Π²Π°Π΅ΠΌ. Если Π½Π΅ΠΎΠ±Ρ…ΠΎΠ΄ΠΈΠΌΠΎ ΡƒΡ‡Π΅ΡΡ‚ΡŒ Ρ‚Ρ€Π΅Π½ΠΈΠ΅ Π² Π±Π»ΠΎΠΊΠ΅, Ρ‚ΠΎ вводят коэффициСнт сопротивлСния (k), Ρ‚ΠΎΠ³Π΄Π°:

Β  Β 

Π—Π°ΠΌΠ΅Π½ΠΎΠΉ Π±Π»ΠΎΠΊΠ° ΠΌΠΎΠΆΠ΅Ρ‚ ΡΠ»ΡƒΠΆΠΈΡ‚ΡŒ гладкая нСподвиТная ΠΎΠΏΠΎΡ€Π°. Π§Π΅Ρ€Π΅Π· Ρ‚Π°ΠΊΡƒΡŽ ΠΎΠΏΠΎΡ€Ρƒ ΠΏΠ΅Ρ€Π΅ΠΊΠΈΠ΄Ρ‹Π²Π°ΡŽΡ‚ Π²Π΅Ρ€Π΅Π²ΠΊΡƒ (ΠΊΠ°Π½Π°Ρ‚), которая ΡΠΊΠΎΠ»ΡŒΠ·ΠΈΡ‚ ΠΏΠΎ ΠΎΠΏΠΎΡ€Π΅, Π½ΠΎ ΠΏΡ€ΠΈ этом растСт сила трСния.

НСподвиТный Π±Π»ΠΎΠΊ Π²Ρ‹ΠΈΠ³Ρ€Ρ‹ΡˆΠ° Π² Ρ€Π°Π±ΠΎΡ‚Π΅ Π½Π΅ Π΄Π°Π΅Ρ‚. ΠŸΡƒΡ‚ΠΈ, ΠΊΠΎΡ‚ΠΎΡ€Ρ‹Π΅ проходят Ρ‚ΠΎΡ‡ΠΊΠΈ прилоТСния сил, ΠΎΠ΄ΠΈΠ½Π°ΠΊΠΎΠ²Ρ‹, Ρ€Π°Π²Π½Ρ‹ силы, ΡΠ»Π΅Π΄ΠΎΠ²Π°Ρ‚Π΅Π»ΡŒΠ½ΠΎ, Ρ€Π°Π²Π½Ρ‹ Ρ€Π°Π±ΠΎΡ‚Ρ‹.

ΠšΠΎΠΌΠ±ΠΈΠ½Π°Ρ†ΠΈΡ Π½Π΅ΠΏΠΎΠ΄Π²ΠΈΠΆΠ½Ρ‹Ρ… Π±Π»ΠΎΠΊΠΎΠ²

Для Ρ‚ΠΎΠ³ΠΎ Ρ‡Ρ‚ΠΎΠ±Ρ‹ ΠΏΠΎΠ»ΡƒΡ‡ΠΈΡ‚ΡŒ Π²Ρ‹ΠΈΠ³Ρ€Ρ‹Ρˆ Π² силС, примСняя Π½Π΅ΠΏΠΎΠ΄Π²ΠΈΠΆΠ½Ρ‹Π΅ Π±Π»ΠΎΠΊΠΈ ΠΏΡ€ΠΈΠΌΠ΅Π½ΡΡŽΡ‚ ΠΊΠΎΠΌΠ±ΠΈΠ½Π°Ρ†ΠΈΡŽ Π±Π»ΠΎΠΊΠΎΠ², Π½Π°ΠΏΡ€ΠΈΠΌΠ΅Ρ€, Π΄Π²ΠΎΠΉΠ½ΠΎΠΉ Π±Π»ΠΎΠΊ. ΠŸΡ€ΠΈ Π±Π»ΠΎΠΊΠΈ Π΄ΠΎΠ»ΠΆΠ½Ρ‹ ΠΈΠΌΠ΅Ρ‚ΡŒ Ρ€Π°Π·Π½Ρ‹Π΅ Π΄ΠΈΠ°ΠΌΠ΅Ρ‚Ρ€Ρ‹. Π˜Ρ… ΡΠΎΠ΅Π΄ΠΈΠ½ΡΡŽΡ‚ Π½Π΅ΠΏΠΎΠ΄Π²ΠΈΠΆΠ½ΠΎ ΠΌΠ΅ΠΆΠ΄Ρƒ собой ΠΈ Π½Π°ΡΠ°ΠΆΠΈΠ²Π°ΡŽΡ‚ Π½Π° Π΅Π΄ΠΈΠ½ΡƒΡŽ ось. К ΠΊΠ°ΠΆΠ΄ΠΎΠΌΡƒ Π±Π»ΠΎΠΊΡƒ прикрСпляСтся Π²Π΅Ρ€Π΅Π²ΠΊΠ°, Ρ‡Ρ‚ΠΎ ΠΎΠ½Π° ΠΌΠΎΠΆΠ΅Ρ‚ Π½Π°ΠΌΠ°Ρ‚Ρ‹Π²Π°Ρ‚ΡŒΡΡ Π½Π° Π±Π»ΠΎΠΊ ΠΈΠ»ΠΈ ΡΠΌΠ°Ρ‚Ρ‹Π²Π°Ρ‚ΡŒΡΡ с Π½Π΅Π³ΠΎ Π±Π΅Π· скольТСния.

ΠŸΠ»Π΅Ρ‡ΠΈ сил Π² Ρ‚Π°ΠΊΠΎΠΌ случаС Π±ΡƒΠ΄ΡƒΡ‚ Π½Π΅Ρ€Π°Π²Π½Ρ‹ΠΌΠΈ. Π”Π²ΠΎΠΉΠ½ΠΎΠΉ Π±Π»ΠΎΠΊ дСйствуСт ΠΊΠ°ΠΊ Ρ€Ρ‹Ρ‡Π°Π³ с ΠΏΠ»Π΅Ρ‡Π°ΠΌΠΈ Ρ€Π°Π·Π½ΠΎΠΉ Π΄Π»ΠΈΠ½Ρ‹. На рис.2 ΠΈΠ·ΠΎΠ±Ρ€Π°ΠΆΠ΅Π½Π° схСма Π΄Π²ΠΎΠΉΠ½ΠΎΠ³ΠΎ Π±Π»ΠΎΠΊΠ°.

УсловиС равновСсия для Ρ€Ρ‹Ρ‡Π°Π³Π° Π½Π° рис.2 станСт Ρ„ΠΎΡ€ΠΌΡƒΠ»Π°:

Β  Β 

Π”Π²ΠΎΠΉΠ½ΠΎΠΉ Π±Π»ΠΎΠΊ ΠΌΠΎΠΆΠ΅Ρ‚ ΠΏΡ€Π΅ΠΎΠ±Ρ€Π°Π·ΠΎΠ²Ρ‹Π²Π°Ρ‚ΡŒ силу. ΠŸΡ€ΠΈΠΊΠ»Π°Π΄Ρ‹Π²Π°Ρ ΠΌΠ΅Π½ΡŒΡˆΡƒΡŽ силу ΠΊ Π²Π΅Ρ€Π΅Π²ΠΊΠ΅, Π½Π°ΠΌΠΎΡ‚Π°Π½Π½ΠΎΠΉ Π½Π° Π±Π»ΠΎΠΊ большого радиуса, ΠΏΠΎΠ»ΡƒΡ‡Π°ΡŽΡ‚ силу, которая дСйствуСт со стороны Π²Π΅Ρ€Π΅Π²ΠΊΠΈ, Π½Π°Π²ΠΈΡ‚ΠΎΠΉ Π½Π° Π±Π»ΠΎΠΊ мСньшСго радиуса.

ΠŸΠΎΠ΄Π²ΠΈΠΆΠ½Ρ‹ΠΉ Π±Π»ΠΎΠΊ

ΠŸΠΎΠ΄Π²ΠΈΠΆΠ½Ρ‹ΠΌ Π±Π»ΠΎΠΊΠΎΠΌ Π½Π°Π·Ρ‹Π²Π°ΡŽΡ‚ Π±Π»ΠΎΠΊ, ось ΠΊΠΎΡ‚ΠΎΡ€ΠΎΠ³ΠΎ пСрСмСщаСтся совмСстно с Π³Ρ€ΡƒΠ·ΠΎΠΌ. На рис. 2 ΠΏΠΎΠ΄Π²ΠΈΠΆΠ½Ρ‹ΠΉ Π±Π»ΠΎΠΊ ΠΌΠΎΠΆΠ½ΠΎ Ρ€Π°ΡΡΠΌΠ°Ρ‚Ρ€ΠΈΠ²Π°Ρ‚ΡŒ ΠΊΠ°ΠΊ Ρ€Ρ‹Ρ‡Π°Π³ с ΠΏΠ»Π΅Ρ‡Π°ΠΌΠΈ Ρ€Π°Π·Π½ΠΎΠΉ Π²Π΅Π»ΠΈΡ‡ΠΈΠ½Ρ‹. Π’ этом случаС Ρ‚ΠΎΡ‡ΠΊΠ° О являСтся Ρ‚ΠΎΡ‡ΠΊΠΎΠΉ ΠΎΠΏΠΎΡ€Ρ‹ Ρ€Ρ‹Ρ‡Π°Π³Π°. OA – ΠΏΠ»Π΅Ρ‡ΠΎ силы ; OB – ΠΏΠ»Π΅Ρ‡ΠΎ силы . Рассмотрим рис. 3. ΠŸΠ»Π΅Ρ‡ΠΎ силы Π² Π΄Π²Π° Ρ€Π°Π·Π° большС, Ρ‡Π΅ΠΌ ΠΏΠ»Π΅Ρ‡ΠΎ силы , ΡΠ»Π΅Π΄ΠΎΠ²Π°Ρ‚Π΅Π»ΡŒΠ½ΠΎ, для равновСсия Π½Π΅ΠΎΠ±Ρ…ΠΎΠ΄ΠΈΠΌΠΎ, Ρ‡Ρ‚ΠΎΠ±Ρ‹ Π²Π΅Π»ΠΈΡ‡ΠΈΠ½Π° силы F Π±Ρ‹Π»Π° Π² Π΄Π²Π° Ρ€Π°Π·Π° мСньшС, Ρ‡Π΅ΠΌ ΠΌΠΎΠ΄ΡƒΠ»ΡŒ силы P:

МоТно ΡΠ΄Π΅Π»Π°Ρ‚ΡŒ Π²Ρ‹Π²ΠΎΠ΄ ΠΎ Ρ‚ΠΎΠΌ, Ρ‡Ρ‚ΠΎ ΠΏΡ€ΠΈ ΠΏΠΎΠΌΠΎΡ‰ΠΈ ΠΏΠΎΠ΄Π²ΠΈΠΆΠ½ΠΎΠ³ΠΎ Π±Π»ΠΎΠΊΠ° ΠΌΡ‹ ΠΏΠΎΠ»ΡƒΡ‡Π°Π΅ΠΌ Π²Ρ‹ΠΈΠ³Ρ€Ρ‹Ρˆ Π² силС Π² Π΄Π²Π° Ρ€Π°Π·Π°. УсловиС равновСсия ΠΏΠΎΠ΄Π²ΠΈΠΆΠ½ΠΎΠ³ΠΎ Π±Π»ΠΎΠΊΠ° Π±Π΅Π· ΡƒΡ‡Π΅Ρ‚Π° силы трСния запишСм ΠΊΠ°ΠΊ:

Β  Β 

Если ΠΏΠΎΠΏΡ‹Ρ‚Π°Ρ‚ΡŒΡΡ ΡƒΡ‡Π΅ΡΡ‚ΡŒ силу трСния Π² Π±Π»ΠΎΠΊΠ΅, Ρ‚ΠΎ вводят коэффициСнт сопротивлСния Π±Π»ΠΎΠΊΠ° (k) ΠΈ ΠΏΠΎΠ»ΡƒΡ‡Π°ΡŽΡ‚:

Β  Β 

Иногда ΠΏΡ€ΠΈΠΌΠ΅Π½ΡΡŽΡ‚ сочСтаниС ΠΏΠΎΠ΄Π²ΠΈΠΆΠ½ΠΎΠ³ΠΎ ΠΈ Π½Π΅ΠΏΠΎΠ΄Π²ΠΈΠΆΠ½ΠΎΠ³ΠΎ Π±Π»ΠΎΠΊΠ°. Π’ Ρ‚Π°ΠΊΠΎΠΌ сочСтании Π½Π΅ΠΏΠΎΠ΄Π²ΠΈΠΆΠ½Ρ‹ΠΉ Π±Π»ΠΎΠΊ ΠΈΡΠΏΠΎΠ»ΡŒΠ·ΡƒΡŽΡ‚ для удобства. Он Π½Π΅ Π΄Π°Π΅Ρ‚ Π²Ρ‹ΠΈΠ³Ρ€Ρ‹ΡˆΠ° Π² силС, Π½ΠΎ позволяСт ΠΈΠ·ΠΌΠ΅Π½ΡΡ‚ΡŒ Π½Π°ΠΏΡ€Π°Π²Π»Π΅Π½ΠΈΠ΅ дСйствия силы. ΠŸΠΎΠ΄Π²ΠΈΠΆΠ½Ρ‹ΠΉ Π±Π»ΠΎΠΊ ΠΏΡ€ΠΈΠΌΠ΅Π½ΡΡŽΡ‚ для измСнСния Π²Π΅Π»ΠΈΡ‡ΠΈΠ½Ρ‹ ΠΏΡ€ΠΈΠ»Π°Π³Π°Π΅ΠΌΠΎΠ³ΠΎ усилия. Если ΠΊΠΎΠ½Ρ†Ρ‹ Π²Π΅Ρ€Π΅Π²ΠΊΠΈ, ΠΎΡ…Π²Π°Ρ‚Ρ‹Π²Π°ΡŽΡ‰Π΅ΠΉ Π±Π»ΠΎΠΊ, ΡΠΎΡΡ‚Π°Π²Π»ΡΡŽΡ‚ с Π³ΠΎΡ€ΠΈΠ·ΠΎΠ½Ρ‚ΠΎΠΌ ΠΎΠ΄ΠΈΠ½Π°ΠΊΠΎΠ²Ρ‹Π΅ ΡƒΠ³Π»Ρ‹, Ρ‚ΠΎ ΠΎΡ‚Π½ΠΎΡˆΠ΅Π½ΠΈΠ΅ силы, ΠΎΠΊΠ°Π·Ρ‹Π²Π°ΡŽΡ‰Π΅ΠΉ воздСйствиС Π½Π° Π³Ρ€ΡƒΠ· ΠΊ вСсу Ρ‚Π΅Π»Π°, Ρ€Π°Π²Π½Π° ΠΎΡ‚Π½ΠΎΡˆΠ΅Π½ΠΈΡŽ радиуса Π±Π»ΠΎΠΊΠ° ΠΊ Ρ…ΠΎΡ€Π΄Π΅ Π΄ΡƒΠ³ΠΈ, ΠΊΠΎΡ‚ΠΎΡ€ΡƒΡŽ ΠΎΡ…Π²Π°Ρ‚Ρ‹Π²Π°Π΅Ρ‚ Π²Π΅Ρ€Π΅Π²ΠΊΠ°. Π’ случаС ΠΏΠ°Ρ€Π°Π»Π»Π΅Π»ΡŒΠ½ΠΎΡΡ‚ΠΈ Π²Π΅Ρ€Π΅Π²ΠΎΠΊ, сила нСобходимая для подъСма Π³Ρ€ΡƒΠ·Π° потрСбуСтся Π² Π΄Π²Π° Ρ€Π°Π·Π° мСньшС, Ρ‡Π΅ΠΌ вСс ΠΏΠΎΠ΄Π½ΠΈΠΌΠ°Π΅ΠΌΠΎΠ³ΠΎ Π³Ρ€ΡƒΠ·Π°.

Π—ΠΎΠ»ΠΎΡ‚ΠΎΠ΅ ΠΏΡ€Π°Π²ΠΈΠ»ΠΎ ΠΌΠ΅Ρ…Π°Π½ΠΈΠΊΠΈ

ΠŸΡ€ΠΎΡΡ‚Ρ‹Π΅ ΠΌΠ΅Ρ…Π°Π½ΠΈΠ·ΠΌΡ‹ Π²Ρ‹ΠΈΠ³Ρ€Ρ‹ΡˆΠ° Π² Ρ€Π°Π±ΠΎΡ‚Π΅ Π½Π΅ Π΄Π°ΡŽΡ‚. Π’ΠΎ сколько ΠΌΡ‹ ΠΏΠΎΠ»ΡƒΡ‡Π°Π΅ΠΌ Π²Ρ‹ΠΈΠ³Ρ€Ρ‹Ρˆ Π² силС, Π²ΠΎ ΡΡ‚ΠΎΠ»ΡŒΠΊΠΎ ΠΆΠ΅ Ρ€Π°Π· ΠΏΡ€ΠΎΠΈΠ³Ρ€Ρ‹Π²Π°Π΅ΠΌ Π² расстоянии. Π’Π°ΠΊ ΠΊΠ°ΠΊ Ρ€Π°Π±ΠΎΡ‚Π° Ρ€Π°Π²Π½Π° скалярному ΠΏΡ€ΠΎΠΈΠ·Π²Π΅Π΄Π΅Π½ΠΈΡŽ сила Π½Π° ΠΏΠ΅Ρ€Π΅ΠΌΠ΅Ρ‰Π΅Π½ΠΈΠ΅, ΡΠ»Π΅Π΄ΠΎΠ²Π°Ρ‚Π΅Π»ΡŒΠ½ΠΎ, ΠΎΠ½Π° Π½Π΅ измСнится ΠΏΡ€ΠΈ использовании ΠΏΠΎΠ΄Π²ΠΈΠΆΠ½ΠΎΠ³ΠΎ (ΠΊΠ°ΠΊ ΠΈ Π½Π΅ΠΏΠΎΠ΄Π²ΠΈΠΆΠ½ΠΎΠ³ΠΎ) Π±Π»ΠΎΠΊΠΎΠ².

Π’ Π²ΠΈΠ΄Π΅ Ρ„ΠΎΡ€ΠΌΡƒΠ»Ρ‹ Β«Π·ΠΎΠ»ΠΎΡ‚ΠΎΠ΅ ΠΏΡ€Π°Π²ΠΈΠ»ΠΎβ„– ΠΌΠΎΠΆΠ½ΠΎ Π·Π°ΠΏΠΈΡΠ°Ρ‚ΡŒ Ρ‚Π°ΠΊ:

Β  Β 

Π³Π΄Π΅ – ΠΏΡƒΡ‚ΡŒ, ΠΊΠΎΡ‚ΠΎΡ€Ρ‹ΠΉ ΠΏΡ€ΠΎΡ…ΠΎΠ΄ΠΈΡ‚ Ρ‚ΠΎΡ‡ΠΊΠ° прилоТСния силы – ΠΏΡƒΡ‚ΡŒ ΠΏΡ€ΠΎΡ…ΠΎΠ΄ΠΈΠΌΡ‹ΠΉ Ρ‚ΠΎΡ‡ΠΊΠΎΠΉ прилоТСния силы .

Π—ΠΎΠ»ΠΎΡ‚ΠΎΠ΅ ΠΏΡ€Π°Π²ΠΈΠ»ΠΎ являСтся самой простой Ρ„ΠΎΡ€ΠΌΡƒΠ»ΠΈΡ€ΠΎΠ²ΠΊΠΎΠΉ Π·Π°ΠΊΠΎΠ½Π° сохранСния энСргии. Π­Ρ‚ΠΎ ΠΏΡ€Π°Π²ΠΈΠ»ΠΎ распространяСтся Π½Π° случаи, Ρ€Π°Π²Π½ΠΎΠΌΠ΅Ρ€Π½ΠΎΠ³ΠΎ ΠΈΠ»ΠΈ ΠΏΠΎΡ‡Ρ‚ΠΈ Ρ€Π°Π²Π½ΠΎΠΌΠ΅Ρ€Π½ΠΎΠ³ΠΎ двиТСния ΠΌΠ΅Ρ…Π°Π½ΠΈΠ·ΠΌΠΎΠ². Расстояния ΠΏΠΎΡΡ‚ΡƒΠΏΠ°Ρ‚Π΅Π»ΡŒΠ½ΠΎΠ³ΠΎ двиТСния ΠΊΠΎΠ½Ρ†ΠΎΠ² Π²Π΅Ρ€Π΅Π²ΠΎΠΊ связаны с радиусами Π±Π»ΠΎΠΊΠΎΠ² ( ΠΈ ) ΠΊΠ°ΠΊ:

Β  Β 

ΠŸΠΎΠ»ΡƒΡ‡ΠΈΠΌ, Ρ‡Ρ‚ΠΎ для выполнСния Β«Π·ΠΎΠ»ΠΎΡ‚ΠΎΠ³ΠΎ ΠΏΡ€Π°Π²ΠΈΠ»Π°Β» для Π΄Π²ΠΎΠΉΠ½ΠΎΠ³ΠΎ Π±Π»ΠΎΠΊΠ° Π½Π΅ΠΎΠ±Ρ…ΠΎΠ΄ΠΈΠΌΠΎ, Ρ‡Ρ‚ΠΎΠ±Ρ‹:

Β  Β 

Если силы ΠΈ ΡƒΡ€Π°Π²Π½ΠΎΠ²Π΅ΡˆΠ΅Π½Ρ‹, Ρ‚ΠΎ Π±Π»ΠΎΠΊ покоится ΠΈΠ»ΠΈ двиТСтся Ρ€Π°Π²Π½ΠΎΠΌΠ΅Ρ€Π½ΠΎ.

ΠŸΡ€ΠΈΠΌΠ΅Ρ€Ρ‹ Ρ€Π΅ΡˆΠ΅Π½ΠΈΡ Π·Π°Π΄Π°Ρ‡

силы ΠΈ Π±Π»ΠΎΠΊΠΈ, 8 класс.

ΠŸΡ€ΠΎΠ΄ΠΎΠ»ΠΆΠ°Π΅ΠΌ ΠΏΠΎΠ΄Π³ΠΎΡ‚ΠΎΠ²ΠΊΡƒ ΠΊ ΠΎΠ»ΠΈΠΌΠΏΠΈΠ°Π΄Π°ΠΌ. БСгодня закрСпляСм Ρ‚Π΅ΠΌΡƒ «статика». ΠŸΠΎΠ³ΠΎΠ²ΠΎΡ€ΠΈΠΌ ΠΏΡ€ΠΎ Π±Π»ΠΎΠΊΠΈ, посчитаСм силы, установим равновСсиС.

Π—Π°Π΄Π°Ρ‡Π° 1. Π§Π΅Ρ€Π½Ρ‹ΠΉ ящик, привязанный Ρ‡Π΅Ρ€Π΅Π· систСму Π±Π»ΠΎΠΊΠΎΠ² ΠΈ Π½ΠΈΡ‚Π΅ΠΉ ΠΊ стСнкС, покоится Π½Π° Π³ΠΎΡ€ΠΈΠ·ΠΎΠ½Ρ‚Π°Π»ΡŒΠ½ΠΎΠΉ повСрхности. Π§Ρ‚ΠΎΠ±Ρ‹ ΠΏΡ€Π΅ΠΎΠ΄ΠΎΠ»Π΅Ρ‚ΡŒ Ρ‚Ρ€Π΅Π½ΠΈΠ΅ ΠΈ ΡΠ΄Π²ΠΈΠ½ΡƒΡ‚ΡŒ Π΅Π³ΠΎ с мСста, нСпосрСдствСнно ΠΊ Π½Π΅ΠΌΡƒ Π½Π΅ΠΎΠ±Ρ…ΠΎΠ΄ΠΈΠΌΠΎ ΠΏΡ€ΠΈΠ»ΠΎΠΆΠΈΡ‚ΡŒ Π³ΠΎΡ€ΠΈΠ·ΠΎΠ½Ρ‚Π°Π»ΡŒΠ½ΡƒΡŽ силу Ρ‡ΡƒΡ‚ΡŒ большС   Н. ΠšΠ°ΠΊΡƒΡŽ ΠΌΠΈΠ½ΠΈΠΌΠ°Π»ΡŒΠ½ΡƒΡŽ силу Π½Π°Π΄ΠΎ ΠΏΡ€ΠΈΠΊΠ»Π°Π΄Ρ‹Π²Π°Ρ‚ΡŒ ΠΊ Ρ‡Π΅Ρ€Π½ΠΎΠΌΡƒ ящику, Ρ‡Ρ‚ΠΎΠ±Ρ‹ ΠΎΠ½ оставался Π½Π΅ΠΏΠΎΠ΄Π²ΠΈΠΆΠ½Ρ‹ΠΌ, Ссли ΠΊ Π²Π΅Ρ€Π΅Π²ΠΊΠ΅ ΠΏΡ€ΠΈΠ»ΠΎΠΆΠ΅Π½Π° сила   Н? ΠžΡ‚Π²Π΅Ρ‚ Π΄Π°Ρ‚ΡŒ Π² ΠΡŒΡŽΡ‚ΠΎΠ½Π°Ρ…, ΠΎΠΊΡ€ΡƒΠ³Π»ΠΈΠ² Π΄ΠΎ Ρ†Π΅Π»Ρ‹Ρ…. Π‘Ρ‡ΠΈΡ‚Π°Ρ‚ΡŒ, Ρ‡Ρ‚ΠΎΒ Β Β ΠΌ/c.

Рисунок 1

Расставим силы:

Рисунок 2

Π’Π΅ΠΏΠ΅Ρ€ΡŒ Π²ΠΈΠ΄Π½ΠΎ, Ρ‡Ρ‚ΠΎ Π½Π° Π±Π»ΠΎΠΊ Π΄Π΅ΠΉΡΡ‚Π²ΡƒΡŽΡ‚ Ρ‚Ρ€ΠΈ силы , поэтому общая сила Ρ€Π°Π²Π½Π° 45 Н. 16 ΠΈΠ· Π½ΠΈΡ… Β«ΡΡŠΠ΅ΡΡ‚Β» сила трСния, поэтому, Ρ‡Ρ‚ΠΎΠ±Ρ‹ ΡƒΠ΄Π΅Ρ€ΠΆΠ°Ρ‚ΡŒ Ρ‚Π°ΠΊΠΎΠΉ ящик, Π½Π΅ Ρ…Π²Π°Ρ‚Π°Π΅Ρ‚ Н.

ΠžΡ‚Π²Π΅Ρ‚: 29 Н.

Π—Π°Π΄Π°Ρ‡Π° 2. ВсС Π±Π»ΠΎΠΊΠΈ Π² систСмС, прСдставлСнной Π½Π° Ρ€ΠΈΡΡƒΠ½ΠΊΠ΅Β βˆ’Β Π½Π΅Π²Π΅ΡΠΎΠΌΡ‹Π΅. Масса Π»Π΅Π²ΠΎΠ³ΠΎ Ρ‚Π΅Π»Π°Β Β Β ΠΊΠ³. ΠŸΡ€ΠΈ ΠΊΠ°ΠΊΠΎΠΉ массС  правого Ρ‚Π΅Π»Π° систСма останСтся Π² равновСсии?Β ΠžΡ‚Π²Π΅Ρ‚ Π΄Π°Ρ‚ΡŒ Π² ΠΊΠΈΠ»ΠΎΠ³Ρ€Π°ΠΌΠΌΠ°Ρ…, ΠΎΠΊΡ€ΡƒΠ³Π»ΠΈΠ² Π΄ΠΎ Ρ†Π΅Π»Ρ‹Ρ….Β Π‘Ρ‡ΠΈΡ‚Π°Ρ‚ΡŒ, Ρ‡Ρ‚ΠΎΒ Β  ΠΌ/c.

Рисунок 3

Расставим силы:

Рисунок 4

Π’Π΅ΠΏΠ΅Ρ€ΡŒ запишСм условия равновСсия:

Β  Β 

Β  Β 

ΠžΡ‚ΠΊΡƒΠ΄Π°

Β  Β 

И

Β  Β 

ΠžΡ‚Π²Π΅Ρ‚: 2 ΠΊΠ³.

Π—Π°Π΄Π°Ρ‡Π° 3. БпасатСли с ΠΏΠΎΠΌΠΎΡ‰ΡŒΡŽ Π²Π΅Ρ€Π΅Π²ΠΎΠΊ, ΠΏΠ΅Ρ€Π΅ΠΊΠΈΠ½ΡƒΡ‚Ρ‹Ρ… Ρ‡Π΅Ρ€Π΅Π· систСму Π±Π»ΠΎΠΊΠΎΠ², ΠΏΠ΅Ρ€Π΅ΠΌΠ΅Ρ‰Π°ΡŽΡ‚ Ρ€Π°Π²Π½ΠΎΠΌΠ΅Ρ€Π½ΠΎ ΠΈ прямолинСйно ΠΌΠ°ΡΡΠΈΠ²Π½ΡƒΡŽ ΠΏΠ»ΠΈΡ‚Ρƒ Ρ‚Π°ΠΊ, ΠΊΠ°ΠΊ ΠΏΠΎΠΊΠ°Π·Π°Π½ΠΎ Π½Π° рисункС. Π‘ ΠΊΠ°ΠΊΠΎΠΉ Ρ€Π΅Π·ΡƒΠ»ΡŒΡ‚ΠΈΡ€ΡƒΡŽΡ‰Π΅ΠΉ силой Π²Π΅Ρ€Ρ‘Π²ΠΊΠΈ Π΄Π΅ΠΉΡΡ‚Π²ΡƒΡŽΡ‚ Π½Π° ΠΏΠ»ΠΈΡ‚Ρƒ? БпасатСли тянут свой ΠΊΠΎΠ½Π΅Ρ† Π²Π΅Ρ€Π΅Π²ΠΊΠΈ с силой   Н. Массами Π²Π΅Ρ€Π΅Π²ΠΎΠΊ ΠΈ Π±Π»ΠΎΠΊΠΎΠ² ΠΏΡ€Π΅Π½Π΅Π±Ρ€Π΅Ρ‡ΡŒ. ΠžΡ‚Π²Π΅Ρ‚ Π΄Π°Ρ‚ΡŒ в Н, ΠΎΠΊΡ€ΡƒΠ³Π»ΠΈΠ² Π΄ΠΎ Ρ†Π΅Π»Ρ‹Ρ….Β Π‘Ρ‡ΠΈΡ‚Π°Ρ‚ΡŒ, Ρ‡Ρ‚ΠΎΒ Β  ΠΌ/c.

Π—Π°ΠΌΠ΅Ρ‡Π°Π½ΠΈΠ΅: трСбуСтся Π½Π°ΠΉΡ‚ΠΈ Ρ‚ΠΎΠ»ΡŒΠΊΠΎ силу, с ΠΊΠΎΡ‚ΠΎΡ€ΠΎΠΉ Π½ΠΈΡ‚ΠΈ Π΄Π΅ΠΉΡΡ‚Π²ΡƒΡŽΡ‚Β Π½Π΅ΠΏΠΎΡΡ€Π΅Π΄ΡΡ‚Π²Π΅Π½Π½ΠΎΒ Π½Π° ΠΏΠ»ΠΈΡ‚Ρƒ. Π‘ΠΈΠ»Ρƒ, Π΄Π΅ΠΉΡΡ‚Π²ΡƒΡŽΡ‰ΡƒΡŽ Π½Π° ΠΏΠ»ΠΈΡ‚Ρƒ со стороны Π²Π΅Ρ€Ρ…Π½Π΅Π³ΠΎ крСплСния Π² ΠΎΡ‚Π²Π΅Ρ‚ Π²ΠΊΠ»ΡŽΡ‡Π°Ρ‚ΡŒ Π½Π΅ Π½Π°Π΄ΠΎ.

Рисунок 5

Расставим силы:

Рисунок 6

Π’Π΅ΠΏΠ΅Ρ€ΡŒ Π²ΠΈΠ΄Π½ΠΎ, Ρ‡Ρ‚ΠΎ Β«Π·Π° Π½ΠΈΡ‚ΠΊΠΈΒ» ΠΏΠ»ΠΈΡ‚Ρƒ тянут Н, Π° полная сила (с ΡƒΡ‡Π΅Ρ‚ΠΎΠΌ Π²Π΅Ρ€Ρ…Π½Π΅Π³ΠΎ крСплСния – Н.

ΠžΡ‚Π²Π΅Ρ‚: 600 Н.

Π—Π°Π΄Π°Ρ‡Π° 4. ΠšΠ°ΠΊΡƒΡŽ Π³ΠΎΡ€ΠΈΠ·ΠΎΠ½Ρ‚Π°Π»ΡŒΠ½ΡƒΡŽ силу Π½Π°Π΄ΠΎ ΠΏΡ€ΠΈΠΊΠ»Π°Π΄Ρ‹Π²Π°Ρ‚ΡŒ ΠΊ ΡˆΠΊΠ°Ρ„Ρƒ, Ρ‡Ρ‚ΠΎΠ±Ρ‹ ΡƒΠ΄Π΅Ρ€ΠΆΠ°Ρ‚ΡŒ Π΅Π³ΠΎ Π½Π° мСстС? ΠœΠ°ΡΡΡ‹ Π³Ρ€ΡƒΠ·ΠΎΠ² Ρ€Π°Π²Π½Ρ‹Β Β Β ΠΊΠ³,Β Β Β ΠΊΠ³. ΠžΡ‚Π²Π΅Ρ‚ Π΄Π°Ρ‚ΡŒ Π² ΠΡŒΡŽΡ‚ΠΎΠ½Π°Ρ…, ΠΎΠΊΡ€ΡƒΠ³Π»ΠΈΠ² Π΄ΠΎ Ρ†Π΅Π»Ρ‹Ρ…. Π‘Ρ‡ΠΈΡ‚Π°Ρ‚ΡŒ, Ρ‡Ρ‚ΠΎΒ Β  ΠΌ/c.

Рисунок 7

На ΡˆΠΊΠ°Ρ„ Π΄Π΅ΠΉΡΡ‚Π²ΡƒΡŽΡ‚ Π΄Π²Π΅ силы тяТСсти: ΠΏΠ΅Ρ€Π²ΠΎΠ³ΠΎ Π³Ρ€ΡƒΠ·Π° (20 Н) ΠΈ Π²Ρ‚ΠΎΡ€ΠΎΠ³ΠΎ (50 Н). Π˜Ρ‚ΠΎΠ³ΠΎ 70 Н.

ΠžΡ‚Π²Π΅Ρ‚: 70 Н.

Π—Π°Π΄Π°Ρ‡Π° 5. Π”Π²Π° ящика покоятся Π½Π° Π³ΠΎΡ€ΠΈΠ·ΠΎΠ½Ρ‚Π°Π»ΡŒΠ½ΠΎΠΉ повСрхности. Π§Ρ‚ΠΎΠ±Ρ‹ ΠΏΡ€Π΅ΠΎΠ΄ΠΎΠ»Π΅Ρ‚ΡŒ Ρ‚Ρ€Π΅Π½ΠΈΠ΅ ΠΈ ΡΠ΄Π²ΠΈΠ½ΡƒΡ‚ΡŒ с мСста Π»Π΅Π²Ρ‹ΠΉ ящик, ΠΊ Π½Π΅ΠΌΡƒ Π½Π΅ΠΎΠ±Ρ…ΠΎΠ΄ΠΈΠΌΠΎ ΠΏΡ€ΠΈΠ»ΠΎΠΆΠΈΡ‚ΡŒ Π³ΠΎΡ€ΠΈΠ·ΠΎΠ½Ρ‚Π°Π»ΡŒΠ½ΡƒΡŽ силу Ρ‡ΡƒΡ‚ΡŒ большС  26 Н. Π§Ρ‚ΠΎΠ±Ρ‹ ΡΠ΄Π²ΠΈΠ½ΡƒΡ‚ΡŒ ΠΏΡ€Π°Π²Ρ‹ΠΉΒ βˆ’Β Ρ‡ΡƒΡ‚ΡŒ большС  14 Н. Π―Ρ‰ΠΈΠΊΠΈ соСдинили Π½ΠΈΡ‚ΡŒΡŽ, ΠΏΠ΅Ρ€Π΅Π±Ρ€ΠΎΡˆΠ΅Π½Π½ΠΎΠΉ Ρ‡Π΅Ρ€Π΅Π· Π±Π»ΠΎΠΊΠΈ, ΠΏΡ€ΠΈΠΊΡ€Π΅ΠΏΠ»Ρ‘Π½Π½Ρ‹Π΅ ΠΊ ящикам Ρ‚Π°ΠΊ, ΠΊΠ°ΠΊ ΠΏΠΎΠΊΠ°Π·Π°Π½ΠΎ Π½Π° рисункС. ΠšΠ°ΠΊΡƒΡŽ ΠΌΠΈΠ½ΠΈΠΌΠ°Π»ΡŒΠ½ΡƒΡŽ силу Π½Π°Π΄ΠΎ ΠΏΡ€ΠΈΠ»ΠΎΠΆΠΈΡ‚ΡŒ ΠΊ ΠΊΠΎΠ½Ρ†Ρƒ Π½ΠΈΡ‚ΠΈ, Ρ‡Ρ‚ΠΎΠ±Ρ‹ расстояниС ΠΌΠ΅ΠΆΠ΄Ρƒ ящиками Π½Π°Ρ‡Π°Π»ΠΎ ΡƒΠΌΠ΅Π½ΡŒΡˆΠ°Ρ‚ΡŒΡΡ? ΠžΡ‚Π²Π΅Ρ‚ Π΄Π°Ρ‚ΡŒ Π² ΠΡŒΡŽΡ‚ΠΎΠ½Π°Ρ…, ΠΎΠΊΡ€ΡƒΠ³Π»ΠΈΠ² Π΄ΠΎ Ρ†Π΅Π»Ρ‹Ρ….

Рисунок 8

РасстояниС Π±ΡƒΠ΄Π΅Ρ‚ ΡƒΠΌΠ΅Π½ΡŒΡˆΠ°Ρ‚ΡŒΡΡ ΠΏΡ€ΠΈ сдвигС любого ΠΈΠ· ящиков, поэтому Π½ΡƒΠΆΠ½ΠΎ Π²Ρ‹ΡΡΠ½ΠΈΡ‚ΡŒ, ΠΊΠ°ΠΊΠΎΠΉ Π»Π΅Π³Ρ‡Π΅ ΡΠ΄Π²ΠΈΠ½ΡƒΡ‚ΡŒ. Для этого расставляСм силы:

Рисунок 9

Π’Π΅ΠΏΠ΅Ρ€ΡŒ Π²ΠΈΠ΄Π½ΠΎ, Ρ‡Ρ‚ΠΎ Π½Π° Π»Π΅Π²Ρ‹ΠΉ ящик дСйствуСт сила , Π° Π½Π° ΠΏΡ€Π°Π²Ρ‹ΠΉ – . Если

Β  Β 

Во Н, а Ссли

Β  Β 

Π’ΠΎ Н. ΠŸΠΎΡΡ‚ΠΎΠΌΡƒ ΠΎΡ‚Π²Π΅Ρ‚ – 7 Н. Π­Ρ‚ΠΎΠ³ΠΎ Π±ΡƒΠ΄Π΅Ρ‚ достаточно, Ρ‡Ρ‚ΠΎΠ±Ρ‹ ΡΠ΄Π²ΠΈΠ½ΡƒΡ‚ΡŒ ΠΏΡ€Π°Π²Ρ‹ΠΉ ящик ΠΈ Ρ‚Π΅ΠΌ самым ΡΠΎΠΊΡ€Π°Ρ‚ΠΈΡ‚ΡŒ расстояниС.

ΠŸΠΎΠ΄Π²ΠΈΠΆΠ½Ρ‹Π΅ ΠΈ Π½Π΅ΠΏΠΎΠ΄Π²ΠΈΠΆΠ½Ρ‹Π΅ Π±Π»ΠΎΠΊΠΈ. Π€ΠΈΠ·ΠΈΠΊΠ°, 7 класс: ΡƒΡ€ΠΎΠΊΠΈ, тСсты, задания.

1. Π‘Π»ΠΎΠΊ, ΠΎΠΏΡ€Π΅Π΄Π΅Π»Π΅Π½ΠΈΠ΅ экономии силы

Π‘Π»ΠΎΠΆΠ½ΠΎΡΡ‚ΡŒ: Π»Ρ‘Π³ΠΊΠΎΠ΅

1
2. Π‘Π»ΠΎΠΊ, показания Π΄ΠΈΠ½Π°ΠΌΠΎΠΌΠ΅Ρ‚Ρ€ΠΎΠ² Π² систСмС Π±Π»ΠΎΠΊΠΎΠ²

Π‘Π»ΠΎΠΆΠ½ΠΎΡΡ‚ΡŒ: Π»Ρ‘Π³ΠΊΠΎΠ΅

3
3. Показания Π΄ΠΈΠ½Π°ΠΌΠΎΠΌΠ΅Ρ‚Ρ€ΠΎΠ² Π² Ρ€Π°Π·Π½Ρ‹Ρ… полоТСниях

Π‘Π»ΠΎΠΆΠ½ΠΎΡΡ‚ΡŒ: Π»Ρ‘Π³ΠΊΠΎΠ΅

2
4. БистСмы Π½Π΅ΠΏΠΎΠ΄Π²ΠΈΠΆΠ½Ρ‹Ρ… Π±Π»ΠΎΠΊΠΎΠ²

Π‘Π»ΠΎΠΆΠ½ΠΎΡΡ‚ΡŒ: Π»Ρ‘Π³ΠΊΠΎΠ΅

1
5. Показания Π΄ΠΈΠ½Π°ΠΌΠΎΠΌΠ΅Ρ‚Ρ€ΠΎΠ² Π² систСмах Π½Π΅ΠΏΠΎΠ΄Π²ΠΈΠΆΠ½Ρ‹Ρ… Π±Π»ΠΎΠΊΠΎΠ² с ΠΎΠ΄Π½ΠΈΠΌ ΠΈΠ»ΠΈ двумя Π³Ρ€ΡƒΠ·ΠΈΠΊΠ°ΠΌΠΈ

Π‘Π»ΠΎΠΆΠ½ΠΎΡΡ‚ΡŒ: Π»Ρ‘Π³ΠΊΠΎΠ΅

1
6. БистСмы ΠΈΠ· Π΄Π²ΡƒΡ… Π±Π»ΠΎΠΊΠΎΠ²

Π‘Π»ΠΎΠΆΠ½ΠΎΡΡ‚ΡŒ: Π»Ρ‘Π³ΠΊΠΎΠ΅

2
7. Π Ρ‹Ρ‡Π°Π³ ΠΈ ΠΏΠΎΠ΄Π²ΠΈΠΆΠ½Ρ‹ΠΉ Π±Π»ΠΎΠΊ

Π‘Π»ΠΎΠΆΠ½ΠΎΡΡ‚ΡŒ: срСднСС

4
8. НСподвиТный ΠΈ ΠΏΠΎΠ΄Π²ΠΈΠΆΠ½Ρ‹ΠΉ Π±Π»ΠΎΠΊΠΈ

Π‘Π»ΠΎΠΆΠ½ΠΎΡΡ‚ΡŒ: срСднСС

2
9. ВСс ΠΏΠΎΠ΄Π²ΠΈΠΆΠ½ΠΎΠ³ΠΎ Π±Π»ΠΎΠΊΠ° ΠΈ Π³Ρ€ΡƒΠ·Π°

Π‘Π»ΠΎΠΆΠ½ΠΎΡΡ‚ΡŒ: срСднСС

2
10. БистСма Π½Π΅ΠΏΠΎΠ΄Π²ΠΈΠΆΠ½ΠΎΠ³ΠΎ ΠΈ ΠΏΠΎΠ΄Π²ΠΈΠΆΠ½ΠΎΠ³ΠΎ Π±Π»ΠΎΠΊΠ°

Π‘Π»ΠΎΠΆΠ½ΠΎΡΡ‚ΡŒ: срСднСС

2
11. Масса Π³Ρ€ΡƒΠ·Π° (ΠΏΠΎΠ΄Π²ΠΈΠΆΠ½Ρ‹ΠΉ Π±Π»ΠΎΠΊ)

Π‘Π»ΠΎΠΆΠ½ΠΎΡΡ‚ΡŒ: срСднСС

4
12. Π‘ΠΈΠ»Π°, с ΠΊΠΎΡ‚ΠΎΡ€ΠΎΠΉ Π΄Π°Π²ΠΈΡ‚ Π½Π° зСмлю Ρ€Π°Π±ΠΎΡ‡ΠΈΠΉ (Π½Π΅ΠΏΠΎΠ΄Π²ΠΈΠΆΠ½Ρ‹ΠΉ Π±Π»ΠΎΠΊ)

Π‘Π»ΠΎΠΆΠ½ΠΎΡΡ‚ΡŒ: срСднСС

3
13. БистСма ΠΈΠ· Π΄Π²ΡƒΡ… ΠΏΠΎΠ΄Π²ΠΈΠΆΠ½Ρ‹Ρ… ΠΈ Π΄Π²ΡƒΡ… Π½Π΅ΠΏΠΎΠ΄Π²ΠΈΠΆΠ½Ρ‹Ρ… Π±Π»ΠΎΠΊΠΎΠ²

Π‘Π»ΠΎΠΆΠ½ΠΎΡΡ‚ΡŒ: слоТноС

2
14. Π”Π°Π²Π»Π΅Π½ΠΈΠ΅ Π½Π° ΠΎΠΏΠΎΡ€Ρƒ (ΠΏΠΎΠ΄Π²ΠΈΠΆΠ½Ρ‹ΠΉ Π±Π»ΠΎΠΊ)

Π‘Π»ΠΎΠΆΠ½ΠΎΡΡ‚ΡŒ: слоТноС

5
15. ΠŸΡ€ΠΎΡΡ‚Ρ‹Π΅ ΠΌΠ΅Ρ…Π°Π½ΠΈΠ·ΠΌΡ‹

Π‘Π»ΠΎΠΆΠ½ΠΎΡΡ‚ΡŒ: слоТноС

5

Π‘Π»ΠΎΠΊΠΈ.

УсловиС равновСсия Π±Π»ΠΎΠΊΠΎΠ²

Β«ΠšΡ‚ΠΎ ΠΎΠ²Π»Π°Π΄Π΅Π» творСниями АрхимСда,

Π±ΡƒΠ΄Π΅Ρ‚ мСньшС ΡƒΠ΄ΠΈΠ²Π»ΡΡ‚ΡŒΡΡ открытиям

самых Π²Π΅Π»ΠΈΠΊΠΈΡ… людСй нашСго Π²Ρ€Π΅ΠΌΠ΅Π½ΠΈΒ»

Π“.Π’. Π›Π΅ΠΉΠ±Π½ΠΈΡ†

Данная Ρ‚Π΅ΠΌΠ° посвящСна Ρ€Π΅ΡˆΠ΅Π½ΠΈΡŽ Π·Π°Π΄Π°Ρ‡ Π½Π° Ρ‚Π΅ΠΌΡƒ Β«Π‘Π»ΠΎΠΊΠΈ. УсловиС равновСсия Π±Π»ΠΎΠΊΠΎΠ²Β».

Π—Π°Π΄Π°Ρ‡Π° 1. ΠšΠ°ΠΊΡƒΡŽ ΠΌΠΈΠ½ΠΈΠΌΠ°Π»ΡŒΠ½ΡƒΡŽ силу Π½ΡƒΠΆΠ½ΠΎ ΠΏΡ€ΠΈΠ»ΠΎΠΆΠΈΡ‚ΡŒ ΠΊ ΠΊΠΎΠ½Ρ†Ρƒ Π²Π΅Ρ€Π΅Π²ΠΊΠΈ для подъСма мСшка Ρ†Π΅ΠΌΠ΅Π½Ρ‚Π° массой 50 ΠΊΠ³ с ΠΏΠΎΠΌΠΎΡ‰ΡŒΡŽ Π΄Π°Π½Π½ΠΎΠΉ систСмы Π±Π»ΠΎΠΊΠΎΠ²? На ΠΊΠ°ΠΊΡƒΡŽ высоту Π±ΡƒΠ΄Π΅Ρ‚ поднят мСшок ΠΏΡ€ΠΈ ΡΠΎΠ²Π΅Ρ€ΡˆΠ΅Π½ΠΈΠΈ этой силой Ρ€Π°Π±ΠΎΡ‚Ρ‹ Π² 2500 Π”ΠΆ? Π‘Ρ‡ΠΈΡ‚Π°Ρ‚ΡŒ Π±Π»ΠΎΠΊΠΈ ΠΈΠ΄Π΅Π°Π»ΡŒΠ½Ρ‹ΠΌΠΈ.

Π”ΠΠΠž:

Π Π•Π¨Π•ΠΠ˜Π•:

ΠŸΠΎΠ΄Π²ΠΈΠΆΠ½Ρ‹ΠΉ Π±Π»ΠΎΠΊ, позволяСт ΠΏΠΎΠ»ΡƒΡ‡ΠΈΡ‚ΡŒ Π²Ρ‹ΠΈΠ³Ρ€Ρ‹Ρˆ Π² силС Π² Π΄Π²Π° Ρ€Π°Π·Π°. Π—Π½Π°Ρ‡ΠΈΡ‚, минимальная сила, ΠΊΠΎΡ‚ΠΎΡ€ΡƒΡŽ Π½ΡƒΠΆΠ½ΠΎ ΠΏΡ€ΠΈΠ»ΠΎΠΆΠΈΡ‚ΡŒ ΠΊ ΠΊΠΎΠ½Ρ†Ρƒ Π²Π΅Ρ€Π΅Π²ΠΊΠΈ для подъСма мСшка Ρ†Π΅ΠΌΠ΅Π½Ρ‚Π°, Ρ€Π°Π²Π½Π° ΠΏΠΎΠ»ΠΎΠ²ΠΈΠ½Π΅ вСса этого мСшка

ВСс мСшка Ρ†Π΅ΠΌΠ΅Π½Ρ‚Π°:

Вогда минимальная сила

Π Π°Π±ΠΎΡ‚Π° силы опрСдСляСтся ΠΏΠΎ Ρ„ΠΎΡ€ΠΌΡƒΠ»Π΅

Богласно Β«Π—ΠΎΠ»ΠΎΡ‚ΠΎΠΌΡƒ ΠΏΡ€Π°Π²ΠΈΠ»Ρƒ ΠΌΠ΅Ρ…Π°Π½ΠΈΠΊΠΈΒ», Ссли ΠΌΡ‹ Π²Ρ‹ΠΈΠ³Ρ€Ρ‹Π²Π°Π΅ΠΌ Π² силС, Ρ‚ΠΎ Π²ΠΎ ΡΡ‚ΠΎΠ»ΡŒΠΊΠΎ ΠΆΠ΅ Ρ€Π°Π· ΠΏΡ€ΠΎΠΈΠ³Ρ€Ρ‹Π²Π°Π΅ΠΌ Π² ΠΏΡƒΡ‚ΠΈ. Π’Π°ΠΊ ΠΊΠ°ΠΊ ΠΏΠΎΠ΄Π²ΠΈΠΆΠ½Ρ‹ΠΉ Π±Π»ΠΎΠΊ Π΄Π°Π΅Ρ‚ Π½Π°ΠΌ Π²Ρ‹ΠΈΠ³Ρ€Ρ‹Ρˆ Π² силС Π² Π΄Π²Π° Ρ€Π°Π·Π°, Ρ‚ΠΎ Π² ΠΏΡƒΡ‚ΠΈ ΠΌΡ‹ ΠΏΡ€ΠΎΠΈΠ³Ρ€Π°Π΅ΠΌ Ρ‚Π°ΠΊΠΆΠ΅ Π² Π΄Π²Π° Ρ€Π°Π·Π°, Ρ‚ΠΎ Π΅ΡΡ‚ΡŒ высота подъСма мСшка Ρ†Π΅ΠΌΠ΅Π½Ρ‚Π° Π±ΡƒΠ΄Π΅Ρ‚ Π² Π΄Π²Π° Ρ€Π°Π·Π° мСньшС высоты подъСма Ρ‚ΠΎΡ‡ΠΊΠΈ прилоТСния силы

Высоту подъСма Ρ‚ΠΎΡ‡ΠΊΠΈ прилоТСния силы ΠΎΠΏΡ€Π΅Π΄Π΅Π»ΠΈΠΌ ΠΊΠ°ΠΊ ΠΎΡ‚Π½ΠΎΡˆΠ΅Π½ΠΈΠ΅ Ρ€Π°Π±ΠΎΡ‚Ρ‹, ΡΠΎΠ²Π΅Ρ€ΡˆΠ΅Π½Π½ΠΎΠΉ силой, ΠΊ ΠΌΠΎΠ΄ΡƒΠ»ΡŽ этой силы

Π’ΠΎΠ³Π΄Π° искомая высота

ΠžΡ‚Π²Π΅Ρ‚: 250 Н; 5 ΠΌ.

Π—Π°Π΄Π°Ρ‡Π° 2. Π’ систСмС, ΠΈΠ·ΠΎΠ±Ρ€Π°ΠΆΠ΅Π½Π½ΠΎΠΉ Π½Π° рисункС, масса самого ΠΏΡ€Π°Π²ΠΎΠ³ΠΎ Π³Ρ€ΡƒΠ·Π° Ρ€Π°Π²Π½Π° 1,5 ΠΊΠ³, Π° массы всСх Π±Π»ΠΎΠΊΠΎΠ² ΠΎΠ΄ΠΈΠ½Π°ΠΊΠΎΠ²Ρ‹ ΠΈ Ρ€Π°Π²Π½Ρ‹ 0,4 ΠΊΠ³. БистСма ΡƒΡ€Π°Π²Π½ΠΎΠ²Π΅ΡˆΠ΅Π½Π° ΠΈ Π½Π΅ΠΏΠΎΠ΄Π²ΠΈΠΆΠ½Π°. ΠžΠΏΡ€Π΅Π΄Π΅Π»ΠΈΡ‚Π΅ массы ΠΎΡΡ‚Π°Π»ΡŒΠ½Ρ‹Ρ… Π³Ρ€ΡƒΠ·ΠΎΠ², Ссли массой троса ΠΈ Ρ‚Ρ€Π΅Π½ΠΈΠ΅ΠΌ Π² Π±Π»ΠΎΠΊΠ°Ρ… ΠΌΠΎΠΆΠ½ΠΎ ΠΏΡ€Π΅Π½Π΅Π±Ρ€Π΅Ρ‡ΡŒ.

Π”ΠΠΠž:

Π Π•Π¨Π•ΠΠ˜Π•:

Если трос являСтся нСвСсомым ΠΈ нСрастяТимым, Π° Ρ‚Π°ΠΊ ΠΆΠ΅ ΠΏΡ€ΠΈ отсутствии трСния Π² Π±Π»ΠΎΠΊΠ°Ρ…, силы натяТСния троса Π΄ΠΎΠ»ΠΆΠ½Ρ‹ Π±Ρ‹Ρ‚ΡŒ Ρ€Π°Π²Π½Ρ‹ ΠΌΠ΅ΠΆΠ΄Ρƒ собой

ΠŸΠΎΡΠΊΠΎΠ»ΡŒΠΊΡƒ систСма ΡƒΡ€Π°Π²Π½ΠΎΠ²Π΅ΡˆΠ΅Π½Π°, Ρ‚ΠΎ:

Π‘ Π΄Ρ€ΡƒΠ³ΠΎΠΉ стороны, трос  ΠΏΠ΅Ρ€Π΅Π΄Π°Π΅Ρ‚ ΠΏΡ€ΠΎΠΈΠ·Π²ΠΎΠ΄ΠΈΠΌΠΎΠ΅ Π½Π° Π½Π΅Π³ΠΎ воздСйствиС Ρ€Π°Π²Π½ΠΎΠΌΠ΅Ρ€Π½ΠΎ ΠΏΠΎ всСй своСй Π΄Π»ΠΈΠ½Π΅. Π’Π°ΠΊ ΠΊΠ°ΠΊ Π½Π° Π±Π»ΠΎΠΊΠΈ с ΠΎΠ±Π΅ΠΈΡ… сторон Π΄Π΅ΠΉΡΡ‚Π²ΡƒΡŽΡ‚ силы натяТСния T, Π° Π½Π° ось ΠΊΠ°ΠΆΠ΄ΠΎΠ³ΠΎ Π±Π»ΠΎΠΊΠ° дСйствуСт вСс подвСшСнного Π½Π° Π½Π΅Π³ΠΎ Π³Ρ€ΡƒΠ·Π° ΠΈ вСс самого Π±Π»ΠΎΠΊΠ°, Ρ‚ΠΎ ΠΏΠΎΠ»ΡƒΡ‡Π°Π΅ΠΌ, Ρ‡Ρ‚ΠΎ Π² равновСсии сила тяТСсти, Π΄Π΅ΠΉΡΡ‚Π²ΡƒΡŽΡ‰Π°Ρ Π½Π° ΠΊΠ°ΠΆΠ΄Ρ‹ΠΉ ΠΈΠ· Π±Π»ΠΎΠΊΠΎΠ² с Π³Ρ€ΡƒΠ·ΠΎΠΌ, ΡƒΡ€Π°Π²Π½ΠΎΠ²Π΅ΡˆΠΈΠ²Π°Π΅Ρ‚ΡΡ ΡƒΠ΄Π²ΠΎΠ΅Π½Π½ΠΎΠΉ силой натяТСния троса

ΠŸΠΎΡΠΊΠΎΠ»ΡŒΠΊΡƒ систСма ΡƒΡ€Π°Π²Π½ΠΎΠ²Π΅ΡˆΠ΅Π½Π°, Ρ‚ΠΎ:

ΠžΡ‚Π²Π΅Ρ‚: 2,6 ΠΊΠ³.

Π—Π°Π΄Π°Ρ‡Π° 3. ΠŸΠ»ΠΈΡ‚Π° массой 120 ΠΊΠ³ Π±Ρ‹Π»Π° Ρ€Π°Π²Π½ΠΎΠΌΠ΅Ρ€Π½ΠΎ поднята с ΠΏΠΎΠΌΠΎΡ‰ΡŒΡŽ ΠΏΠΎΠ΄Π²ΠΈΠΆΠ½ΠΎΠ³ΠΎ Π±Π»ΠΎΠΊΠ° Π½Π° высоту 16 ΠΌ Π·Π° 40 с. Бчитая ΠšΠŸΠ” ΠΌΠ΅Ρ…Π°Π½ΠΈΠ·ΠΌΠ° 80%, Π° массу Π±Π»ΠΎΠΊΠ° β€” 10 ΠΊΠ³, ΠΎΠΏΡ€Π΅Π΄Π΅Π»ΠΈΡ‚Π΅ ΠΏΠΎΠ»Π½ΡƒΡŽ Ρ€Π°Π±ΠΎΡ‚Ρƒ ΠΈ Ρ€Π°Π·Π²ΠΈΠ²Π°Π΅ΠΌΡƒΡŽ ΠΌΠΎΡ‰Π½ΠΎΡΡ‚ΡŒ. Π‘Ρ‡ΠΈΡ‚Π°Ρ‚ΡŒ, Ρ‡Ρ‚ΠΎ Π² Π±Π»ΠΎΠΊΠ΅ ΠΎΡ‚ΡΡƒΡ‚ΡΡ‚Π²ΡƒΡŽΡ‚ силы трСния.

Π”ΠΠΠž:

Π Π•Π¨Π•ΠΠ˜Π•:

ΠšΠΎΡΡ„Ρ„ΠΈΡ†ΠΈΠ΅Π½Ρ‚Π° ΠΏΠΎΠ»Π΅Π·Π½ΠΎΠ³ΠΎ дСйствия ΠΌΠ΅Ρ…Π°Π½ΠΈΠ·ΠΌΠ° опрСдСляСтся ΠΎΡ‚Π½ΠΎΡˆΠ΅Π½ΠΈΠ΅ΠΌ ΠΏΠΎΠ»Π΅Π·Π½ΠΎΠΉ Ρ€Π°Π±ΠΎΡ‚Ρ‹ ΠΌΠ΅Ρ…Π°Π½ΠΈΠ·ΠΌΠ° ΠΊΠΎ всСй Π·Π°Ρ‚Ρ€Π°Ρ‡Π΅Π½Π½ΠΎΠΉ ΠΈΠΌ Ρ€Π°Π±ΠΎΡ‚Π΅

ПолСзная Ρ€Π°Π±ΠΎΡ‚Π°:

Π‘ΠΈΠ»Π° тяТСсти ΠΏΠ»ΠΈΡ‚Ρ‹ ΠΈ Π±Π»ΠΎΠΊΠ°:

Π’ΠΎΠ³Π΄Π°

ЗатрачСнная Ρ€Π°Π±ΠΎΡ‚Π°:

ЗатрачСнная ΠΌΠΎΡ‰Π½ΠΎΡΡ‚ΡŒ:

ΠžΡ‚Π²Π΅Ρ‚: 26 ΠΊΠ”ΠΆ; 650 Π’Ρ‚.

Π—Π°Π΄Π°Ρ‡Π° 4. На рисункС ΠΈΠ·ΠΎΠ±Ρ€Π°ΠΆΠ΅Π½Π° систСма Π³Ρ€ΡƒΠ·ΠΎΠ². ΠœΠ°ΡΡΡ‹ Ρ‚Π΅Π» соотвСтствСнно Ρ€Π°Π²Π½Ρ‹ 0,2 ΠΊΠ³ ΠΈ 0,4 ΠΊΠ³. ΠžΠΏΡ€Π΅Π΄Π΅Π»ΠΈΡ‚Π΅ силу натяТСния Π½ΠΈΡ‚ΠΈ. Π‘Ρ‡ΠΈΡ‚Π°Ρ‚ΡŒ Π½ΠΈΡ‚ΡŒ ΠΈ Π±Π»ΠΎΠΊΠΈ ΠΈΠ΄Π΅Π°Π»ΡŒΠ½Ρ‹ΠΌΠΈ?

Π”ΠΠΠž:

Π Π•Π¨Π•ΠΠ˜Π•:

Π’Π°ΠΊ ΠΊΠ°ΠΊ Π½ΠΈΡ‚ΡŒ ΠΈ Π±Π»ΠΎΠΊΠΈ ΠΈΠ΄Π΅Π°Π»ΡŒΠ½Ρ‹, Ρ‚ΠΎ:

На основании II Π·Π°ΠΊΠΎΠ½Π° ΠΡŒΡŽΡ‚ΠΎΠ½Π°:

Π’Π°ΠΊ ΠΊΠ°ΠΊ Π±Π»ΠΎΠΊ 2 ΠΏΠΎΠ΄Π²ΠΈΠΆΠ½Ρ‹ΠΉ:

Π’ΠΎΠ³Π΄Π° ΠΏΠΎΠ»ΡƒΡ‡Π°Π΅ΠΌ

Π’Ρ‹Ρ€Π°Π·ΠΈΠΌ ΠΈΠ· Π²Ρ‚ΠΎΡ€ΠΎΠ³ΠΎ уравнСния ускорСниС a2, подставим ΠΏΠΎΠ»ΡƒΡ‡Π΅Π½Π½ΠΎΠ΅ Π²Ρ‹Ρ€Π°ΠΆΠ΅Π½ΠΈΠ΅ Π² ΠΏΠ΅Ρ€Π²ΠΎΠ΅ ΡƒΡ€Π°Π²Π½Π΅Π½ΠΈΠ΅

ΠŸΡ€Π΅ΠΎΠ±Ρ€Π°Π·ΡƒΠ΅ΠΌ ΠΏΠΎΠ»ΡƒΡ‡Π΅Π½Π½ΠΎΠ΅ ΡƒΡ€Π°Π²Π½Π΅Π½ΠΈΠ΅ ΠΈ Π²Ρ‹Ρ€Π°Π·ΠΈΠΌ ΠΈΠ· Π½Π΅Π³ΠΎ ΠΈΡΠΊΠΎΠΌΡƒΡŽ силу натяТСния Π½ΠΈΡ‚ΠΈ

Β 

ΠžΡ‚Π²Π΅Ρ‚: 2 Н.

100 ballov.kz ΠΎΠ±Ρ€Π°Π·ΠΎΠ²Π°Ρ‚Π΅Π»ΡŒΠ½Ρ‹ΠΉ ΠΏΠΎΡ€Ρ‚Π°Π» для ΠΏΠΎΠ΄Π³ΠΎΡ‚ΠΎΠ²ΠΊΠΈ ΠΊ ЕНВ ΠΈ КВА

Π’ 2021 Π³ΠΎΠ΄Ρƒ казахстанскиС школьники Π±ΡƒΠ΄ΡƒΡ‚ ΡΠ΄Π°Π²Π°Ρ‚ΡŒ ΠΏΠΎ-Π½ΠΎΠ²ΠΎΠΌΡƒ Π•Π΄ΠΈΠ½ΠΎΠ΅ Π½Π°Ρ†ΠΈΠΎΠ½Π°Π»ΡŒΠ½ΠΎΠ΅ тСстированиС. Помимо Ρ‚ΠΎΠ³ΠΎ, Ρ‡Ρ‚ΠΎ Π³Π»Π°Π²Π½Ρ‹ΠΉ ΡˆΠΊΠΎΠ»ΡŒΠ½Ρ‹ΠΉ экзамСн Π±ΡƒΠ΄Π΅Ρ‚ ΠΏΡ€ΠΎΡ…ΠΎΠ΄ΠΈΡ‚ΡŒ элСктронно, выпускникам прСдоставят Π²ΠΎΠ·ΠΌΠΎΠΆΠ½ΠΎΡΡ‚ΡŒ ΠΈΡΠΏΡ‹Ρ‚Π°Ρ‚ΡŒ свою ΡƒΠ΄Π°Ρ‡Ρƒ Π΄Π²Π°ΠΆΠ΄Ρ‹. ΠšΠΎΡ€Ρ€Π΅ΡΠΏΠΎΠ½Π΄Π΅Π½Ρ‚Β zakon.kz побСсСдовал с Π²ΠΈΡ†Π΅-министром образования ΠΈ Π½Π°ΡƒΠΊΠΈ ΠœΠΈΡ€Π°ΡΠΎΠΌ Π”Π°ΡƒΠ»Π΅Π½ΠΎΠ²Ρ‹ΠΌ ΠΈ ΡƒΠ·Π½Π°Π», ΠΊ Ρ‡Π΅ΠΌΡƒ Π³ΠΎΡ‚ΠΎΠ²ΠΈΡ‚ΡŒΡΡ Π±ΡƒΠ΄ΡƒΡ‰ΠΈΠΌ Π°Π±ΠΈΡ‚ΡƒΡ€ΠΈΠ΅Π½Ρ‚Π°ΠΌ.

β€” О ΠΏΠ΅Ρ€Π΅Π²ΠΎΠ΄Π΅ ЕНВ Π½Π° элСктронный Ρ„ΠΎΡ€ΠΌΠ°Ρ‚ Π³ΠΎΠ²ΠΎΡ€ΠΈΠ»ΠΎΡΡŒ Π½Π΅ Ρ€Π°Π·. И Π²ΠΎΡ‚, с 2021 Π³ΠΎΠ΄Π° тСстированиС Π½Π°Ρ‡Π½ΡƒΡ‚ ΠΏΡ€ΠΎΠ²ΠΎΠ΄ΠΈΡ‚ΡŒ ΠΏΠΎ-Π½ΠΎΠ²ΠΎΠΌΡƒ. ΠœΠΈΡ€Π°Ρ ΠœΡƒΡ…Ρ‚Π°Ρ€ΠΎΠ²ΠΈΡ‡, расскаТитС, ΠΊΠ°ΠΊ это Π±ΡƒΠ΄Π΅Ρ‚?

β€”Β ΠŸΠΎ ΡΠΎΠ΄Π΅Ρ€ΠΆΠ°Π½ΠΈΡŽ всС остаСтся ΠΏΠΎ-ΠΏΡ€Π΅ΠΆΠ½Π΅ΠΌΡƒ, Π½ΠΎ мСняСтся Ρ„ΠΎΡ€ΠΌΠ°Ρ‚. Если Ρ€Π°Π½ΡŒΡˆΠ΅ школьник садился Π·Π° ΠΏΠ°Ρ€Ρ‚Ρƒ ΠΈ Π΅ΠΌΡƒ Π²Ρ‹Π΄Π°Π²Π°Π»ΠΈ Π±ΡƒΠΌΠ°ΠΆΠ½Ρ‹ΠΉ Π²Π°Ρ€ΠΈΠ°Π½Ρ‚ ΠΊΠ½ΠΈΠΆΠΊΠΈ ΠΈ лист ΠΎΡ‚Π²Π΅Ρ‚Π°, Ρ‚ΠΎ Ρ‚Π΅ΠΏΠ΅Ρ€ΡŒ тСст Π±ΡƒΠ΄ΡƒΡ‚ ΡΠ΄Π°Π²Π°Ρ‚ΡŒ Π·Π° ΠΊΠΎΠΌΠΏΡŒΡŽΡ‚Π΅Ρ€ΠΎΠΌ Π² элСктронном Ρ„ΠΎΡ€ΠΌΠ°Ρ‚Π΅. Π£ ΠΊΠ°ΠΆΠ΄ΠΎΠ³ΠΎ выпускника Π±ΡƒΠ΄Π΅Ρ‚ своС мСсто, ΠΎΠ³ΠΎΡ€ΠΎΠΆΠ΅Π½Π½ΠΎΠ΅ оргстСклом.

Π—Π°Ρ€Π΅Π³ΠΈΡΡ‚Ρ€ΠΈΡ€ΠΎΠ²Π°Ρ‚ΡŒΡΡ ΠΌΠΎΠΆΠ½ΠΎ Π±ΡƒΠ΄Π΅Ρ‚ элСктронно Π½Π° сайтС ΠΠ°Ρ†ΠΈΠΎΠ½Π°Π»ΡŒΠ½ΠΎΠ³ΠΎ Ρ†Π΅Π½Ρ‚Ρ€Π° тСстирования. Но, удобство Π² Ρ‚ΠΎΠΌ, Ρ‡Ρ‚ΠΎ школьник сам смоТСт Π²Ρ‹Π±Ρ€Π°Ρ‚ΡŒ Π΄Π°Ρ‚Ρƒ, врСмя ΠΈ мСсто сдачи тСстирования.

ΠšΡ€ΠΎΠΌΠ΅ Ρ‚ΠΎΠ³ΠΎ, Π² этом Π³ΠΎΠ΄Ρƒ ЕНВ для ΠΏΡ€Π΅Ρ‚Π΅Π½Π΄ΡƒΡŽΡ‰ΠΈΡ… Π½Π° Π³Ρ€Π°Π½Ρ‚ Π±ΡƒΠ΄Π΅Ρ‚ Π΄Π»ΠΈΡ‚ΡŒΡΡ Ρ‚Ρ€ΠΈ мСсяца, ΠΈ Π² Ρ‚Π΅Ρ‡Π΅Π½ΠΈΠ΅ 100 Π΄Π½Π΅ΠΉ ΡΠ΄Π°Ρ‚ΡŒ Π΅Π³ΠΎ ΠΌΠΎΠΆΠ½ΠΎ Π±ΡƒΠ΄Π΅Ρ‚ Π΄Π²Π° Ρ€Π°Π·Π°.

β€” РасскаТитС ΠΏΠΎΠΏΠΎΠ΄Ρ€ΠΎΠ±Π½Π΅Π΅?

β€” Π’ ΠΌΠ°Ρ€Ρ‚Π΅ ΠΏΡ€ΠΎΠΉΠ΄Π΅Ρ‚ тСстированиС для ΠΆΠ΅Π»Π°ΡŽΡ‰ΠΈΡ… ΠΏΠΎΡΡ‚ΡƒΠΏΠΈΡ‚ΡŒ Π½Π° ΠΏΠ»Π°Ρ‚Π½ΠΎΠΉ основС, Π° для ΠΏΡ€Π΅Ρ‚Π΅Π½Π΄ΡƒΡŽΡ‰ΠΈΡ… Π½Π° Π³Ρ€Π°Π½Ρ‚ ΠΌΡ‹ Π²Π²Π΅Π»ΠΈ Π½ΠΎΠ²Ρ‹Π΅ ΠΏΡ€Π°Π²ΠΈΠ»Π°. Школьник, Ρ‡Ρ‚ΠΎΠ±Ρ‹ ΠΏΠΎΡΡ‚ΡƒΠΏΠΈΡ‚ΡŒ Π½Π° Π³Ρ€Π°Π½Ρ‚, ΠΏΠΎ ТСланию ΠΌΠΎΠΆΠ΅Ρ‚ ΡΠ΄Π°Ρ‚ΡŒ ЕНВ Π΄Π²Π° Ρ€Π°Π·Π° Π² Π°ΠΏΡ€Π΅Π»Π΅, ΠΌΠ°Π΅ ΠΈΠ»ΠΈ Π² июнС, Π° Π½Π°ΠΈΠ»ΡƒΡ‡ΡˆΠΈΠΉ Ρ€Π΅Π·ΡƒΠ»ΡŒΡ‚Π°Ρ‚ ΠΎΡ‚ΠΏΡ€Π°Π²ΠΈΡ‚ΡŒ Π½Π° конкурс. Но Π΅ΡΡ‚ΡŒ ΠΎΠ³Ρ€Π°Π½ΠΈΡ‡Π΅Π½ΠΈΠ΅ β€” Π΄Π²Π° Ρ€Π°Π·Π° Π² ΠΎΠ΄ΠΈΠ½ дСнь ΡΠ΄Π°Π²Π°Ρ‚ΡŒ тСст нСльзя. К ΠΏΡ€ΠΈΠΌΠ΅Ρ€Ρƒ, Ссли Ρ‚Ρ‹ сдал ЕНВ Π² Π°ΠΏΡ€Π΅Π»Π΅, Ρ‚ΠΎ ΠΏΠΎΡ‚ΠΎΠΌ ΠΏΠΎΠ²Ρ‚ΠΎΡ€Π½ΠΎ ΠΌΠΎΠΆΠ½ΠΎ ΠΏΠ΅Ρ€Π΅ΡΠ΄Π°Ρ‚ΡŒ Π΅Π³ΠΎ Ρ‡Π΅Ρ€Π΅Π· нСсколько Π΄Π½Π΅ΠΉ ΠΈΠ»ΠΈ Π² ΠΌΠ°Π΅, июнС. ΠœΡ‹ Ρ€Π΅ΠΊΠΎΠΌΠ΅Π½Π΄ΡƒΠ΅ΠΌ всС-Ρ‚Π°ΠΊΠΈ Π±Ρ€Π°Ρ‚ΡŒ нСбольшой ΠΏΠ΅Ρ€Π΅Ρ€Ρ‹Π², Ρ‡Ρ‚ΠΎΠ±Ρ‹ Π΅Ρ‰Π΅ Π»ΡƒΡ‡ΡˆΠ΅ ΠΏΠΎΠ΄Π³ΠΎΡ‚ΠΎΠ²ΠΈΡ‚ΡŒΡΡ. Но Π² любом случаС это Π²Ρ‹Π±ΠΎΡ€ школьника.

β€” БистСма оцСнивания останСтся ΠΏΡ€Π΅ΠΆΠ½Π΅ΠΉ?

β€” ΠšΠΎΠ»ΠΈΡ‡Π΅ΡΡ‚Π²ΠΎ ΠΏΡ€Π΅Π΄ΠΌΠ΅Ρ‚ΠΎΠ² остаСтся ΠΏΡ€Π΅ΠΆΠ½ΠΈΠΌ β€” Ρ‚Ρ€ΠΈ ΠΎΠ±ΡΠ·Π°Ρ‚Π΅Π»ΡŒΠ½Ρ‹Ρ… ΠΏΡ€Π΅Π΄ΠΌΠ΅Ρ‚Π° ΠΈ Π΄Π²Π° Π½Π° Π²Ρ‹Π±ΠΎΡ€. Если Π² Π±ΡƒΠΌΠ°ΠΆΠ½ΠΎΠΌ Ρ„ΠΎΡ€ΠΌΠ°Ρ‚Π΅ Π·Π°ΠΊΡ€Π°ΡˆΠ΅Π½Π½Ρ‹ΠΉ Π²Π°Ρ€ΠΈΠ°Π½Ρ‚ ΠΎΡ‚Π²Π΅Ρ‚Π° ΡƒΠΆΠ΅ нСльзя Π±Ρ‹Π»ΠΎ ΠΈΡΠΏΡ€Π°Π²ΠΈΡ‚ΡŒ, Ρ‚ΠΎ Π² элСктронном Ρ„ΠΎΡ€ΠΌΠ°Ρ‚Π΅ школьник смоТСт Π²Π΅Ρ€Π½ΡƒΡ‚ΡŒΡΡ ΠΊ вопросу ΠΈ ΠΏΠΎΠΌΠ΅Π½ΡΡ‚ΡŒ ΠΎΡ‚Π²Π΅Ρ‚, Π½ΠΎ Π΄ΠΎ Ρ‚ΠΎΠ³ΠΎ, ΠΊΠ°ΠΊ Π·Π°Π²Π΅Ρ€ΡˆΠΈΠ» тСст.

Π‘Π°ΠΌΠΎΠ΅ Π³Π»Π°Π²Π½ΠΎΠ΅ β€” Ρ€Π΅Π·ΡƒΠ»ΡŒΡ‚Π°Ρ‚Ρ‹ тСста ΠΌΠΎΠΆΠ½ΠΎ Π±ΡƒΠ΄Π΅Ρ‚ ΠΏΠΎΠ»ΡƒΡ‡ΠΈΡ‚ΡŒ сразу ΠΆΠ΅ послС наТатия ΠΊΠ½ΠΎΠΏΠΊΠΈ Β«Π·Π°Π²Π΅Ρ€ΡˆΠΈΡ‚ΡŒ тСстированиС». РаньшС ΡƒΡ…ΠΎΠ΄ΠΈΠ»ΠΎ ΠΎΡ‡Π΅Π½ΡŒ ΠΌΠ½ΠΎΠ³ΠΎ Π²Ρ€Π΅ΠΌΠ΅Π½ΠΈ Π½Π° ΠΏΡ€ΠΎΠ²Π΅Ρ€ΠΊΡƒ ΠΎΡ‚Π²Π΅Ρ‚ΠΎΠ², Π΄Π΅Ρ‚ΠΈ ΠΈ Ρ€ΠΎΠ΄ΠΈΡ‚Π΅Π»ΠΈ ΠΏΠ΅Ρ€Π΅ΠΆΠΈΠ²Π°Π»ΠΈ, ΠΆΠ΄Π°Π»ΠΈ Π²Π΅Ρ‡Π΅Ρ€Π°, Ρ‡Ρ‚ΠΎΠ±Ρ‹ ΡƒΠ·Π½Π°Ρ‚ΡŒ Ρ€Π΅Π·ΡƒΠ»ΡŒΡ‚Π°Ρ‚. БСйчас ΠΌΡ‹ всС Π°Π²Ρ‚ΠΎΠΌΠ°Ρ‚ΠΈΠ·ΠΈΡ€ΠΎΠ²Π°Π»ΠΈ ΠΈ Π½Π°Π±Ρ€Π°Π½Π½ΠΎΠ΅ количСство Π±Π°Π»Π»ΠΎΠ² Π±ΡƒΠ΄Π΅Ρ‚ Π²Ρ‹Π²Π΅Π΄Π΅Π½ΠΎ Π½Π° экран сразу ΠΆΠ΅ послС Π·Π°Π²Π΅Ρ€ΡˆΠ΅Π½ΠΈΡ тСстирования.
МаксимальноС количСство Π±Π°Π»Π»ΠΎΠ² остаСтся ΠΏΡ€Π΅ΠΆΠ½ΠΈΠΌ β€” 140.

β€” А апСлляция?

β€” Если ΡΠ΄Π°ΡŽΡ‰ΠΈΠΉ Π½Π΅ Π±ΡƒΠ΄Π΅Ρ‚ согласСн с ΠΊΠ°ΠΊΠΈΠΌΠΈ-Ρ‚ΠΎ вопросами, посчитаСт ΠΈΡ… Π½Π΅ΠΊΠΎΡ€Ρ€Π΅ΠΊΡ‚Π½Ρ‹ΠΌΠΈ, Ρ‚ΠΎ ΠΎΠ½ сразу ΠΆΠ΅ Π½Π° мСстС смоТСт ΠΏΠΎΠ΄Π°Ρ‚ΡŒ заявку Π½Π° Π°ΠΏΠ΅Π»Π»ΡΡ†ΠΈΡŽ. НС Π½ΡƒΠΆΠ½ΠΎ Π±ΡƒΠ΄Π΅Ρ‚ ΠΆΠ΄Π°Ρ‚ΡŒ ΡΠ»Π΅Π΄ΡƒΡŽΡ‰Π΅Π³ΠΎ дня, ΠΈΠ΄Ρ‚ΠΈ Π² Ρ†Π΅Π½Ρ‚Ρ€ тСстирования, Π²ΡƒΠ· ΠΈΠ»ΠΈ ΡˆΠΊΠΎΠ»Ρƒ, всС это Π±ΡƒΠ΄Π΅Ρ‚ элСктронно.

β€” Π‘ ΡƒΡ‡Π΅Ρ‚ΠΎΠΌ Ρ‚ΠΎΠ³ΠΎ, Ρ‡Ρ‚ΠΎ школьникам Π½Π΅ придСтся Π²Ρ€ΡƒΡ‡Π½ΡƒΡŽ Π·Π°ΠΊΡ€Π°ΡˆΠΈΠ²Π°Ρ‚ΡŒ листы ΠΎΡ‚Π²Π΅Ρ‚ΠΎΠ², Π±ΡƒΠ΄Π΅Ρ‚ Π»ΠΈ ΠΈΠ·ΠΌΠ΅Π½Π΅Π½ΠΎ врСмя сдачи тСстирования?

β€” ΠœΡ‹ Ρ€Π΅ΡˆΠΈΠ»ΠΈ ΠΎΡΡ‚Π°Π²ΠΈΡ‚ΡŒ ΠΏΡ€Π΅ΠΆΠ½Π΅Π΅ врСмя β€” 240 ΠΌΠΈΠ½ΡƒΡ‚. Но Ρ‚Π΅ΠΏΠ΅Ρ€ΡŒ, ΠΊΠ°ΠΊ Π²Ρ‹ ΠΎΡ‚ΠΌΠ΅Ρ‚ΠΈΠ»ΠΈ, школьникам Π½Π΅ Π½ΡƒΠΆΠ½ΠΎ Π±ΡƒΠ΄Π΅Ρ‚ Ρ‚Ρ€Π°Ρ‚ΠΈΡ‚ΡŒ час Π½Π° Ρ‚ΠΎ, Ρ‡Ρ‚ΠΎΠ±Ρ‹ ΠΏΡ€Π°Π²ΠΈΠ»ΡŒΠ½ΠΎ Π·Π°ΠΊΡ€Π°ΡΠΈΡ‚ΡŒ лист ΠΎΡ‚Π²Π΅Ρ‚ΠΎΠ², ΠΎΠ½ΠΈ спокойно смогут ΠΈΡΠΏΠΎΠ»ΡŒΠ·ΠΎΠ²Π°Ρ‚ΡŒ это врСмя Π½Π° Ρ€Π΅ΡˆΠ΅Π½ΠΈΠ΅ Π·Π°Π΄Π°Ρ‡.

β€” НС сСкрСт, Ρ‡Ρ‚ΠΎ Π² Π½Π΅ΠΊΠΎΡ‚ΠΎΡ€Ρ‹Ρ… сСлах ΠΈ ΠΎΡ‚Π΄Π°Π»Π΅Π½Π½Ρ‹Ρ… насСлСнных ΠΏΡƒΠ½ΠΊΡ‚Π°Ρ… Π½Π΅ Ρ…Π²Π°Ρ‚Π°Π΅Ρ‚ ΠΊΠΎΠΌΠΏΡŒΡŽΡ‚Π΅Ρ€ΠΎΠ². Как сСльскиС школьники Π±ΡƒΠ΄ΡƒΡ‚ ΡΠ΄Π°Π²Π°Ρ‚ΡŒ ЕНВ ΠΏΠΎ Π½ΠΎΠ²ΠΎΠΌΡƒ Ρ„ΠΎΡ€ΠΌΠ°Ρ‚Ρƒ?

β€” Π—Π°Π΄Π°Ρ‡Π° Π² Ρ‚ΠΎΠΌ, Ρ‡Ρ‚ΠΎΠ±Ρ‹ ΠΏΡ€Π°Π²ΠΈΠ»ΡŒΠ½ΠΎ Π²Ρ‹Π±Ρ€Π°Ρ‚ΡŒ врСмя ΠΈ Π΄Π°Ρ‚Ρƒ тСстирования. Π¦Π΅Π½Ρ‚Ρ€Ρ‹ тСстирования Π΅ΡΡ‚ΡŒ Π²ΠΎ всСх Ρ€Π΅Π³ΠΈΠΎΠ½Π°Ρ…, Π² Нур-Π‘ΡƒΠ»Ρ‚Π°Π½Π΅, Алматы ΠΈ Π¨Ρ‹ΠΌΠΊΠ΅Π½Ρ‚Π΅ ΠΈΡ… нСсколько. Школьники, ΠΏΡ€ΠΎΠΆΠΈΠ²Π°ΡŽΡ‰ΠΈΠ΅ Π² ΠΎΡ‚Π΄Π°Π»Π΅Π½Π½Ρ‹Ρ… насСлСнных ΠΏΡƒΠ½ΠΊΡ‚Π°Ρ…, ΠΊΠ°ΠΊ ΠΈ Ρ€Π°Π½ΡŒΡˆΠ΅ смогут ΠΏΡ€ΠΈΠ΅Ρ…Π°Ρ‚ΡŒ Π² Π³ΠΎΡ€ΠΎΠ΄, Π³Π΄Π΅ Π΅ΡΡ‚ΡŒ эти Ρ†Π΅Π½Ρ‚Ρ€Ρ‹, ΠΈ ΡΠ΄Π°Ρ‚ΡŒ тСстированиС.

β€” На сколько ΠΏΡ€ΠΎΡ†Π΅Π½Ρ‚ΠΎΠ² Π±ΡƒΠ΄Π΅Ρ‚ ΠΎΠ±Π½ΠΎΠ²Π»Π΅Π½Π° Π±Π°Π·Π° вопросов?

β€” Π‘Π°Π·Π° вопросов Π΅ΠΆΠ΅Π³ΠΎΠ΄Π½ΠΎ обновляСтся ΠΊΠ°ΠΊ ΠΌΠΈΠ½ΠΈΠΌΡƒΠΌ Π½Π° 30%. Π’ этом Π³ΠΎΠ΄Ρƒ ΠΌΡ‹ Π΄ΠΎΠ±Π°Π²ΠΈΠ»ΠΈ контСкстныС задания, Ρ‚ΠΎ Ρ‡Ρ‚ΠΎ школьники всСгда просили. ΠœΡ‹ ΡƒΠ΄Π΅Π»ΠΈΠ»ΠΈ большоС Π²Π½ΠΈΠΌΠ°Π½ΠΈΠ΅ истории ΠšΠ°Π·Π°Ρ…ΡΡ‚Π°Π½Π° ΠΈ всСмирной истории β€” ΠΈΡΠΊΠ»ΡŽΡ‡ΠΈΠ»ΠΈ практичСски всС Π΄Π°Ρ‚Ρ‹. Для нас Π³Π»Π°Π²Π½ΠΎΠ΅ Π½Π΅ Π·Π°Π·ΡƒΠ±Ρ€ΠΈΠ²Π°Π½ΠΈΠ΅ Π΄Π°Ρ‚, Π° ΠΏΠΎΠ½ΠΈΠΌΠ°Π½ΠΈΠ΅ значСния историчСских событий. Но ΠΏΠΎ ΠΊΠ°ΠΆΠ΄ΠΎΠΌΡƒ ΠΏΡ€Π΅Π΄ΠΌΠ΅Ρ‚Ρƒ Π±ΡƒΠ΄ΡƒΡ‚ контСкстныС вопросы.

β€” По Π²Π°ΡˆΠ΅ΠΌΡƒ мнСнию систСма справится с Π²ΠΎΠ·ΠΌΠΎΠΆΠ½Ρ‹ΠΌΠΈ хакСрскими Π°Ρ‚Π°ΠΊΠ°ΠΌΠΈ, Π²Π·Π»ΠΎΠΌΠ°ΠΌΠΈ?

β€” Π˜Π½Ρ„ΠΎΡ€ΠΌΠ°Ρ†ΠΈΠΎΠ½Π½Π°Ρ Π±Π΅Π·ΠΎΠΏΠ°ΡΠ½ΠΎΡΡ‚ΡŒ β€” это пСрвостСпСнный ΠΈ ΠΏΡ€ΠΈΠΎΡ€ΠΈΡ‚Π΅Ρ‚Π½Ρ‹ΠΉ вопрос. Π¦Π΅Π½Ρ‚Ρ€Π°Π»ΡŒΠ½Ρ‹ΠΉ Π°ΠΏΠΏΠ°Ρ€Π°Ρ‚ всСй систСмы находится Π² Нур-Π‘ΡƒΠ»Ρ‚Π°Π½Π΅. Бвязь с Ρ€Π΅Π³ΠΈΠΎΠ½Π°Π»ΡŒΠ½Ρ‹ΠΌΠΈ Ρ†Π΅Π½Ρ‚Ρ€Π°ΠΌΠΈ сдачи ЕНВ проводится ΠΏΠΎ Π·Π°ΠΊΡ€Ρ‹Ρ‚ΠΎΠΌΡƒ VPN-ΠΊΠ°Π½Π°Π»Ρƒ. ΠšΠΎΠ΄Ρ‹ ΠΏΡ€Π°Π²ΠΈΠ»ΡŒΠ½Ρ‹Ρ… ΠΎΡ‚Π²Π΅Ρ‚ΠΎΠ² Ρ‚ΠΎΠ»ΡŒΠΊΠΎ Π² ΠΠ°Ρ†ΠΈΠΎΠ½Π°Π»ΡŒΠ½ΠΎΠΌ Ρ†Π΅Π½Ρ‚Ρ€Π΅ тСстирования.

ΠšΡ€ΠΎΠΌΠ΅ Ρ‚ΠΎΠ³ΠΎ, Π΄ΠΎΠΏΠΎΠ»Π½ΠΈΡ‚Π΅Π»ΡŒΠ½ΠΎ Ρ‡Π΅Ρ€Π΅Π· Π“Π’Π‘ ΠšΠΠ‘ (ГосударствСнная тСхничСская слуТба) всС тСсты проходят ΠΏΡ€ΠΎΠ²Π΅Ρ€ΠΊΡƒ Π½Π° ΠΏΡ€Π΅Π΄ΠΌΠ΅Ρ‚ Π²ΠΎΠ·ΠΌΠΎΠΆΠ½ΠΎΠ³ΠΎ Π²ΠΌΠ΅ΡˆΠ°Ρ‚Π΅Π»ΡŒΡΡ‚Π²Π°. Π—Π΄Π΅ΡΡŒ всС Π½Π΅ просто, это ΡΠΏΠ΅Ρ†ΠΈΠ°Π»ΡŒΠ½Ρ‹Π΅ Π·Π°Ρ‰ΠΈΡ‰Π΅Π½Π½Ρ‹Π΅ ΠΊΠ°Π½Π°Π»Ρ‹ связи.

β€” А Ρ‡Ρ‚ΠΎ с санитарными трСбованиями? НуТно Π»ΠΈ Π±ΡƒΠ΄Π΅Ρ‚ школьникам ΡΠ΄Π°Π²Π°Ρ‚ΡŒ ПЦР-тСст ΠΏΠ΅Ρ€Π΅Π΄ ЕНВ?

β€” ПЦР-тСст ΡΠ΄Π°Π²Π°Ρ‚ΡŒ Π½Π΅ Π½ΡƒΠΆΠ½ΠΎ Π±ΡƒΠ΄Π΅Ρ‚. Π’Ρ€Π΅Π±ΠΎΠ²Π°Π½ΠΈΠ΅ ΠΏΠΎ маскам Π±ΡƒΠ΄Π΅Ρ‚. ΠŸΡ€ΠΈ нСобходимости Π¦Π΅Π½Ρ‚Ρ€ Π½Π°Ρ†ΠΈΠΎΠ½Π°Π»ΡŒΠ½ΠΎΠ³ΠΎ тСстирования Π±ΡƒΠ΄Π΅Ρ‚ Π²Ρ‹Π΄Π°Π²Π°Ρ‚ΡŒ маски школьникам Π²ΠΎ врСмя сдачи ЕНВ. И, ΠΊΠΎΠ½Π΅Ρ‡Π½ΠΎ ΠΆΠ΅, Π±ΡƒΠ΄Π΅ΠΌ ΠΈΠ·ΠΌΠ΅Ρ€ΡΡ‚ΡŒ Ρ‚Π΅ΠΌΠΏΠ΅Ρ€Π°Ρ‚ΡƒΡ€Ρƒ. Π‘ΠΎΡ†ΠΈΠ°Π»ΡŒΠ½Π°Ρ дистанция Π±ΡƒΠ΄Π΅Ρ‚ ΡΠΎΠ±Π»ΡŽΠ΄Π°Ρ‚ΡŒΡΡ Π² ΠΊΠ°ΠΆΠ΄ΠΎΠΉ Π°ΡƒΠ΄ΠΈΡ‚ΠΎΡ€ΠΈΠΈ.

β€” Бколько Ρ‡Π΅Π»ΠΎΠ²Π΅ΠΊ Π±ΡƒΠ΄Π΅Ρ‚ ΡΠΈΠ΄Π΅Ρ‚ΡŒ Π² ΠΎΠ΄Π½ΠΎΠΉ Π°ΡƒΠ΄ΠΈΡ‚ΠΎΡ€ΠΈΠΈ?

β€” Участники ЕНВ Π½Π΅ Π·Π° сСмь Π΄Π½Π΅ΠΉ Π±ΡƒΠ΄ΡƒΡ‚ ΡΠ΄Π°Π²Π°Ρ‚ΡŒ тСстированиС, ΠΊΠ°ΠΊ это Π±Ρ‹Π»ΠΎ Ρ€Π°Π½ΡŒΡˆΠ΅, Π° Π² Ρ‚Π΅Ρ‡Π΅Π½ΠΈΠ΅ Ρ‚Ρ€Π΅Ρ… мСсяцСв. ΠŸΠΎΡΡ‚ΠΎΠΌΡƒ ΠΏΠΎ заполняСмости Π°ΡƒΠ΄ΠΈΡ‚ΠΎΡ€ΠΈΠΈ вопросов Π½Π΅ Π±ΡƒΠ΄Π΅Ρ‚.

β€” Π‘ΡƒΠ΄ΡƒΡ‚ Π»ΠΈ уТСсточСны трСбования ΠΏΠΎ дисциплинС, Π·Π°ΠΏΡ€Π΅Ρ‰Π΅Π½Π½Ρ‹ΠΌ ΠΏΡ€Π΅Π΄ΠΌΠ΅Ρ‚Π°ΠΌ?

β€” ΠœΡ‹ удСляСм большоС Π²Π½ΠΈΠΌΠ°Π½ΠΈΠ΅ акадСмичСской чСстности. На Π²Ρ…ΠΎΠ΄Π΅ Π² Ρ†Π΅Π½Ρ‚Ρ€Ρ‹ тСстирования, ΠΊΠ°ΠΊ ΠΈ Π² ΠΏΡ€Π΅Π΄Ρ‹Π΄ΡƒΡ‰ΠΈΠ΅ Π³ΠΎΠ΄Ρ‹, Π±ΡƒΠ΄ΡƒΡ‚ ΡΡ‚ΠΎΡΡ‚ΡŒ мСталлоискатСли. ΠŸΠ΅Ρ€Π΅Ρ‡Π΅Π½ΡŒ Π·Π°ΠΏΡ€Π΅Ρ‰Π΅Π½Π½Ρ‹Ρ… ΠΏΡ€Π΅Π΄ΠΌΠ΅Ρ‚ΠΎΠ² остаСтся ΠΏΡ€Π΅ΠΆΠ½ΠΈΠΌ β€” Ρ‚Π΅Π»Π΅Ρ„ΠΎΠ½Ρ‹, ΡˆΠΏΠ°Ρ€Π³Π°Π»ΠΊΠΈ ΠΈ ΠΏΡ€ΠΎΡ‡Π΅Π΅. Но, ΠΏΠΎΠΌΠΈΠΌΠΎ Ρ„Ρ€ΠΎΠ½Ρ‚Π°Π»ΡŒΠ½ΠΎΠΉ ΠΊΠ°ΠΌΠ΅Ρ€Ρ‹, которая Π±ΡƒΠ΄Π΅Ρ‚ Ρ‚Ρ€Π°Π½ΡΠ»ΠΈΡ€ΠΎΠ²Π°Ρ‚ΡŒ происходящСС Π² Π°ΡƒΠ΄ΠΈΡ‚ΠΎΡ€ΠΈΠΈ, Π½Π°Π΄ ΠΊΠ°ΠΆΠ΄Ρ‹ΠΌ столом Π±ΡƒΠ΄Π΅Ρ‚ установлСна Π΅Ρ‰Π΅ ΠΎΠ΄Π½Π° ΠΊΠ°ΠΌΠ΅Ρ€Π°. Она ΠΆΠ΅ Π±ΡƒΠ΄Π΅Ρ‚ ΠΈΡΠΏΠΎΠ»ΡŒΠ·ΠΎΠ²Π°Ρ‚ΡŒΡΡ Π² качСствС ΠΈΠ΄Π΅Π½Ρ‚ΠΈΡ„ΠΈΠΊΠ°Ρ†ΠΈΠΈ школьника β€” ΠΊΠ°ΠΊ Face ID. Π‘Π΅Π», зарСгистрировался ΠΈ приступил ΠΊ заданиям. ΠœΡ‹ примСнСям систСму ΠΏΡ€ΠΎΠΊΡ‚ΠΎΡ€ΠΈΠ½Π³Π°.

ΠŸΠΎΠ½ΡΡ‚Π½ΠΎ, Ρ‡Ρ‚ΠΎ ΠΊΠ°ΠΆΠ΄ΠΎΠ΅ Π΄Π²ΠΈΠΆΠ΅Π½ΠΈΠ΅ Π°Π±ΠΈΡ‚ΡƒΡ€ΠΈΠ΅Π½Ρ‚Π° Π½Π°ΠΌ Π±ΡƒΠ΄Π΅Ρ‚ Π²ΠΈΠ΄Π½ΠΎ. Если Π²ΠΎ врСмя сдачи ЕНВ ΠΎΠ±Π½Π°Ρ€ΡƒΠΆΠΈΠΌ, Ρ‡Ρ‚ΠΎ ΡΠ΄Π°ΡŽΡ‰ΠΈΠΉ использовал Ρ‚Π΅Π»Π΅Ρ„ΠΎΠ½ ΠΈΠ»ΠΈ ΡˆΠΏΠ°Ρ€Π³Π°Π»ΠΊΡƒ, Ρ‚ΠΎ тСстированиС автоматичСски Π±ΡƒΠ΄Π΅Ρ‚ ΠΏΡ€Π΅ΠΊΡ€Π°Ρ‰Π΅Π½ΠΎ, систСма ΠΎΡ‚ΠΊΠ»ΡŽΡ‡ΠΈΡ‚ΡΡ.

β€” А Π½Π°Π±Π»ΡŽΠ΄Π°Ρ‚Π΅Π»ΠΈ Π±ΡƒΠ΄ΡƒΡ‚ ΠΏΡ€ΠΈΡΡƒΡ‚ΡΡ‚Π²ΠΎΠ²Π°Ρ‚ΡŒ Π²ΠΎ врСмя сдачи тСстирования?

β€” Когда Π² Π±ΡƒΠΌΠ°ΠΆΠ½ΠΎΠΌ Ρ„ΠΎΡ€ΠΌΠ°Ρ‚Π΅ ΠΏΡ€ΠΎΠ²ΠΎΠ΄ΠΈΠ»ΠΈ ЕНВ, ΠΌΡ‹ ΠΏΡ€ΠΈΠ²Π»Π΅ΠΊΠ°Π»ΠΈ ΠΎΡ‡Π΅Π½ΡŒ ΠΌΠ½ΠΎΠ³ΠΎ Π΄Π΅ΠΆΡƒΡ€Π½Ρ‹Ρ…. Π’ ΠΎΠ΄Π½ΠΎΠΉ Π°ΡƒΠ΄ΠΈΡ‚ΠΎΡ€ΠΈΠΈ Π±Ρ‹Π»ΠΎ ΠΏΠΎ 3-4 Ρ‡Π΅Π»ΠΎΠ²Π΅ΠΊΠ°. ΠŸΡ€ΠΈ элСктронной сдачС Ρ‚Π°ΠΊΠΎΠ³ΠΎ Π½Π΅ Π±ΡƒΠ΄Π΅Ρ‚, максимум ΠΎΠ΄ΠΈΠ½ Π½Π°Π±Π»ΡŽΠ΄Π°Ρ‚Π΅Π»ΡŒ, ΠΏΠΎΡ‚ΠΎΠΌΡƒ Ρ‡Ρ‚ΠΎ всС Π±ΡƒΠ΄Π΅Ρ‚ Π²ΠΈΠ΄Π½ΠΎ ΠΏΠΎ ΠΊΠ°ΠΌΠ΅Ρ€Π°ΠΌ.

β€” По вашим наблюдСниям школьники стали мСньшС ΠΈΡΠΏΠΎΠ»ΡŒΠ·ΠΎΠ²Π°Ρ‚ΡŒ Π·Π°ΠΏΡ€Π΅Ρ‰Π΅Π½Π½Ρ‹Π΅ ΠΏΡ€Π΅Π΄ΠΌΠ΅Ρ‚Ρ‹, ΠΊ ΠΏΡ€ΠΈΠΌΠ΅Ρ€Ρƒ, ΠΏΠΎΠ»ΡŒΠ·ΠΎΠ²Π°Ρ‚ΡŒΡΡ Ρ‚Π΅Π»Π΅Ρ„ΠΎΠ½Π°ΠΌΠΈ?

β€” ΠŸΡ€Π°ΠΊΡ‚ΠΈΠΊΠ° ΠΏΠΎΠΊΠ°Π·Ρ‹Π²Π°Π΅Ρ‚, Ρ‡Ρ‚ΠΎ школьники стали отвСтствСннСС ΠΎΡ‚Π½ΠΎΡΠΈΡ‚ΡŒΡΡ ΠΊ ЕНВ. Если Π² 2019 Π³ΠΎΠ΄Ρƒ Π½Π° 120 тыс. школьников ΠΌΡ‹ изъяли 120 тыс. Π·Π°ΠΏΡ€Π΅Ρ‰Π΅Π½Π½Ρ‹Ρ… ΠΏΡ€Π΅Π΄ΠΌΠ΅Ρ‚ΠΎΠ², ΠΏΠΎ сути Ρƒ ΠΊΠ°ΠΆΠ΄ΠΎΠ³ΠΎ ΡΠ΄Π°ΡŽΡ‰Π΅Π³ΠΎ Π±Ρ‹Π» Ρ‚Π΅Π»Π΅Ρ„ΠΎΠ½. Π’ΠΎ Π² ΠΏΡ€ΠΎΡˆΠ»ΠΎΠΌ Π³ΠΎΠ΄Ρƒ ΠΌΡ‹ Π½Π° 120 тыс. школьников ΠΎΠ±Π½Π°Ρ€ΡƒΠΆΠΈΠ»ΠΈ всСго 2,5 тыс. Ρ‚Π΅Π»Π΅Ρ„ΠΎΠ½ΠΎΠ², ΠΈ Ρƒ всСх Π±Ρ‹Π»ΠΈ Π°Π½Π½ΡƒΠ»ΠΈΡ€ΠΎΠ²Π°Π½Ρ‹ Ρ€Π΅Π·ΡƒΠ»ΡŒΡ‚Π°Ρ‚Ρ‹.

Напомню, Ρ‡Ρ‚ΠΎ Π² 2020 Π³ΠΎΠ΄Ρƒ ΠΌΡ‹ Ρ‚Π°ΠΊΠΆΠ΅ Π½Π°Ρ‡Π°Π»ΠΈ ΠΈΡΠΏΠΎΠ»ΡŒΠ·ΠΎΠ²Π°Ρ‚ΡŒ систСму искусствСнного ΠΈΠ½Ρ‚Π΅Π»Π»Π΅ΠΊΡ‚Π°. Π­Ρ‚ΠΎ Π°Π½Π°Π»ΠΈΠ· видСозаписСй, ΠΊΠΎΡ‚ΠΎΡ€Ρ‹ΠΉ проводится послС тСстирования. Π’Π°ΠΊ, Π² ΠΏΡ€ΠΎΡˆΠ»ΠΎΠΌ Π³ΠΎΠ΄Ρƒ 100 Π°Π±ΠΈΡ‚ΡƒΡ€ΠΈΠ΅Π½Ρ‚ΠΎΠ² лишились Π³Ρ€Π°Π½Ρ‚ΠΎΠ² Π·Π° Ρ‚ΠΎ, Ρ‡Ρ‚ΠΎ Π²ΠΎ врСмя сдачи ЕНВ использовали Π·Π°ΠΏΡ€Π΅Ρ‰Π΅Π½Π½Ρ‹Π΅ ΠΏΡ€Π΅Π΄ΠΌΠ΅Ρ‚Ρ‹.

β€” Бколько срСдств Π²Ρ‹Π΄Π΅Π»Π΅Π½ΠΎ Π½Π° ΠΏΡ€ΠΎΠ²Π΅Π΄Π΅Π½ΠΈΠ΅ ЕНВ Π² этом Π³ΠΎΠ΄Ρƒ?

Если Ρ€Π°Π½ΡŒΡˆΠ΅ Π½Π° ЕНВ Ρ‚Ρ€Π΅Π±ΠΎΠ²Π°Π»ΠΎΡΡŒ 1,5 ΠΌΠ»Ρ€Π΄ Ρ‚Π΅Π½Π³Π΅ ΠΈΠ·-Π·Π° распСчатки ΠΊΠ½ΠΈΠΆΠ΅ΠΊ ΠΈ листов ΠΎΡ‚Π²Π΅Ρ‚ΠΎΠ², Ρ‚ΠΎ сСйчас расходы Π·Π½Π°Ρ‡ΠΈΡ‚Π΅Π»ΡŒΠ½ΠΎ сокращСны Π·Π° счСт ΠΏΠ΅Ρ€Π΅Ρ…ΠΎΠ΄Π° Π½Π° элСктронный Ρ„ΠΎΡ€ΠΌΠ°Ρ‚. Они Π±ΡƒΠ΄ΡƒΡ‚, Π½ΠΎ нСсущСствСнныС.

β€” ВсС-Ρ‚Π°ΠΊΠΈ ΠΏΠΎΡ‡Π΅ΠΌΡƒ ΠΈΠΌΠ΅Π½Π½ΠΎ Π² 2021 Π³ΠΎΠ΄Ρƒ Π±Ρ‹Π»ΠΎ принято Ρ€Π΅ΡˆΠ΅Π½ΠΈΠ΅ ΠΏΡ€ΠΎΠ²ΠΎΠ΄ΠΈΡ‚ΡŒ ЕНВ Π² элСктронном Ρ„ΠΎΡ€ΠΌΠ°Ρ‚Π΅. Π­Ρ‚ΠΎ ΠΊΠ°ΠΊ-Ρ‚ΠΎ связано с ΠΏΠ°Π½Π΄Π΅ΠΌΠΈΠ΅ΠΉ?

β€” Π­Ρ‚ΠΎ Π½Π΅ связано с ΠΏΠ°Π½Π΄Π΅ΠΌΠΈΠ΅ΠΉ. ΠŸΡ€ΠΎΡΡ‚ΠΎ Π½ΡƒΠΆΠ½ΠΎ ΠΏΠ΅Ρ€Π΅Ρ…ΠΎΠ΄ΠΈΡ‚ΡŒ Π½Π° качСствСнно Π½ΠΎΠ²Ρ‹ΠΉ ΡƒΡ€ΠΎΠ²Π΅Π½ΡŒ. ΠœΡ‹ Π°ΠΏΡ€ΠΎΠ±ΠΈΡ€ΠΎΠ²Π°Π»ΠΈ Π΄Π°Π½Π½Ρ‹ΠΉ Ρ„ΠΎΡ€ΠΌΠ°Ρ‚ Π½Π° ΠΏΠ΅Π΄Π°Π³ΠΎΠ³Π°Ρ… школ, Π²Ρ‹ Π·Π½Π°Π΅Ρ‚Π΅, Ρ‡Ρ‚ΠΎ ΠΎΠ½ΠΈ ΡΠ΄Π°ΡŽΡ‚ ΠΊΠ²Π°Π»ΠΈΡ„ΠΈΠΊΠ°Ρ†ΠΈΠΎΠ½Π½Ρ‹ΠΉ тСст, Π½Π° магистрантах, Ρ‚Π°ΠΊ ΠΏΠΎΡ‡Π΅ΠΌΡƒ Π±Ρ‹ Π½Π΅ ΠΈΡΠΏΠΎΠ»ΡŒΠ·ΠΎΠ²Π°Ρ‚ΡŒ этот ΠΆΠ΅ Ρ„ΠΎΡ€ΠΌΠ°Ρ‚ ΠΏΡ€ΠΈ сдачС ЕНВ. Π’Π΅ΠΌ Π±ΠΎΠ»Π΅Π΅, Ρ‡Ρ‚ΠΎ это ΡƒΠ΄ΠΎΠ±Π½ΠΎ, ΠΈ для школьников Ρ‚Π΅ΠΏΠ΅Ρ€ΡŒ Π±ΡƒΠ΄Π΅Ρ‚ ΠΌΠ½ΠΎΠ³ΠΎ плюсов.

Π£Ρ€ΠΎΠΊ Ρ€Π΅ΡˆΠ΅Π½ΠΈΡ Π·Π°Π΄Π°Ρ‡ ΠΏΠΎ Ρ‚Π΅ΠΌΠ΅: “Π‘Π»ΠΎΠΊΠΈ. Наклонная ΠΏΠ»ΠΎΡΠΊΠΎΡΡ‚ΡŒ”

Π˜Ρ€ΠΈΠ½Π° Павловна НСрода,

ΡƒΡ‡ΠΈΡ‚Π΅Π»ΡŒ Ρ„ΠΈΠ·ΠΈΠΊΠΈ ΠœΠ‘ΠžΠ£ Β«Π‘Π¨ β„–16Β»

Π³. Ачинск

Π’Π΅ΠΌΠ° ΡƒΡ€ΠΎΠΊΠ°: РСшСниС Π·Π°Π΄Π°Ρ‡ ΠΏΠΎ Ρ‚Π΅ΠΌΠ΅: Β«Π‘Π»ΠΎΠΊΠΈ. Наклонная ΠΏΠ»ΠΎΡΠΊΠΎΡΡ‚ΡŒΒ»Β»

ЦСль ΡƒΡ€ΠΎΠΊΠ°: ΠΏΡ€ΠΈΠΌΠ΅Π½ΡΡ‚ΡŒ знания Π½Π° ΠΏΡ€Π°Π²ΠΈΠ»ΠΎ ΠΌΠΎΠΌΠ΅Π½Ρ‚ΠΎΠ² сил, Π½Π°Ρ…ΠΎΠΆΠ΄Π΅Π½ΠΈΠ΅ Π²Ρ‹ΠΈΠ³Ρ€Ρ‹ΡˆΠ° с силС.

Π—Π°Π΄Π°Ρ‡ΠΈ ΡƒΡ€ΠΎΠΊΠ°: Π·Π°ΠΊΡ€Π΅ΠΏΠΈΡ‚ΡŒ практичСскиС Π½Π°Π²Ρ‹ΠΊΠΈ ΠΈ умСния ΠΏΠΎ ΠΎΠΏΡ€Π΅Π΄Π΅Π»Π΅Π½ΠΈΡŽ Π²Ρ‹ΠΈΠ³Ρ€Ρ‹ΡˆΠ° Π² силС, нахоТдСния ΠΏΡ€ΠΈΠ»ΠΎΠΆΠ΅Π½Π½ΠΎΠΉ силы ΠΏΠΎ ΠΏΡ€Π°Π²ΠΈΠ»Ρƒ ΠΌΠΎΠΌΠ΅Π½Ρ‚ΠΎΠ² сил.

ΠŸΠΎΠ·Π½Π°Π²Π°Ρ‚Π΅Π»ΡŒΠ½Ρ‹Π΅ ΡƒΡ‡Π΅Π±Π½Ρ‹Π΅ дСйствия:Β 

  • Π²Ρ‹Π±ΠΎΡ€ Π½Π°ΠΈΠ±ΠΎΠ»Π΅Π΅ эффСктивных способов Ρ€Π΅ΡˆΠ΅Π½ΠΈΡ Π·Π°Π΄Π°Ρ‡ Π² зависимости ΠΎΡ‚ ΠΊΠΎΠ½ΠΊΡ€Π΅Ρ‚Π½Ρ‹Ρ… условий;

  • рСфлСксия способов ΠΈ условий дСйствия, ΠΊΠΎΠ½Ρ‚Ρ€ΠΎΠ»ΡŒ ΠΈ ΠΎΡ†Π΅Π½ΠΊΡƒ процСсса ΠΈ Ρ€Π΅Π·ΡƒΠ»ΡŒΡ‚Π°Ρ‚ΠΎΠ² Π΄Π΅ΡΡ‚Π΅Π»ΡŒΠ½ΠΎΡΡ‚ΠΈ;

  • ΡƒΠΌΠ΅Π½ΠΈΠ΅ ΡΡ‚Ρ€ΠΎΠΈΡ‚ΡŒ Ρ€Π΅Ρ‡Π΅Π²Ρ‹Π΅ высказывания Π² устной ΠΈ письмСнной Ρ€Π΅Ρ‡ΠΈ.

ЛичностныС ΡƒΡ‡Π΅Π±Π½Ρ‹Π΅ дСйствия:

РСгулятивныС ΡƒΡ‡Π΅Π±Π½Ρ‹Π΅ дСйствия:Β 

  • Ρ†Π΅Π»Π΅ΠΏΠΎΠ»Π°Π³Π°Π½ΠΈΠ΅, ΠΊΠ°ΠΊ постановка ΡƒΡ‡Π΅Π±Π½ΠΎΠΉ Π·Π°Π΄Π°Ρ‡ΠΈ Π½Π° основС соотнСсСния Ρ‚ΠΎΠ³ΠΎ, Ρ‡Ρ‚ΠΎ ΡƒΠΆΠ΅ извСстно ΠΈ усвоСно учащимися, ΠΈ Ρ‚ΠΎΠ³ΠΎ, Ρ‡Ρ‚ΠΎ Π΅Ρ‰Π΅ нСизвСстно;

  • составлСниС ΠΏΠ»Π°Π½Π° ΠΈ ΠΏΠΎΡΠ»Π΅Π΄ΠΎΠ²Π°Ρ‚Π΅Π»ΡŒΠ½ΠΎΡΡ‚ΠΈ дСйствий;

  • ΠΊΠΎΠ½Ρ‚Ρ€ΠΎΠ»ΡŒ Π² Ρ„ΠΎΡ€ΠΌΠ΅ сравнСния способа дСйствия ΠΈ Π΅Π³ΠΎ Ρ€Π΅Π·ΡƒΠ»ΡŒΡ‚Π°Ρ‚Π° с Π·Π°Π΄Π°Π½Π½Ρ‹ΠΌ эталоном с Ρ†Π΅Π»ΡŒΡŽ обнаруТСния ΠΎΡ‚ΠΊΠ»ΠΎΠ½Π΅Π½ΠΈΠΉ ΠΈ ΠΎΡ‚Π»ΠΈΡ‡ΠΈΠΉ ΠΎΡ‚ эталона;

  • ΠΎΡ†Π΅Π½ΠΊΠ° – Π²Ρ‹Π΄Π΅Π»Π΅Π½ΠΈΠ΅ ΠΈ осознаниС учащимися Ρ‚ΠΎΠ³ΠΎ, Ρ‡Ρ‚ΠΎ ΡƒΠΆΠ΅ усвоСно ΠΈ Ρ‡Ρ‚ΠΎ Π΅Ρ‰Π΅ ΠΏΠΎΠ΄Π»Π΅ΠΆΠΈΡ‚ ΡƒΡΠ²ΠΎΠ΅Π½ΠΈΡŽ, осознаниС качСства ΠΈ уровня усвоСния;

  • волСвая саморСгуляция, ΠΊΠ°ΠΊ ΡΠΏΠΎΡΠΎΠ±Π½ΠΎΡΡ‚ΡŒ ΠΊ Π²ΠΎΠ»Π΅Π²ΠΎΠΌΡƒ ΡƒΡΠΈΠ»ΠΈΡŽ, ΠΊ ΠΏΡ€Π΅ΠΎΠ΄ΠΎΠ»Π΅Π½ΠΈΡŽ прСпятствий.

Π£Π½ΠΈΠ²Π΅Ρ€ΡΠ°Π»ΡŒΠ½Ρ‹Π΅ ΡƒΡ‡Π΅Π±Π½Ρ‹Π΅ дСйствия:Β 

  • Π°Π½Π°Π»ΠΈΠ·;Β 
    синтСз;

  • Π΄ΠΎΠΊΠ°Π·Π°Ρ‚Π΅Π»ΡŒΡΡ‚Π²ΠΎ, установлСниС ΠΏΡ€ΠΈΡ‡ΠΈΠ½Π½ΠΎ – слСдствСнных связСй, построСниС логичСской Ρ†Π΅ΠΏΠΈ рассуТдСний.

ΠšΠ»ΡŽΡ‡Π΅Π²Ρ‹Π΅ понятия: Π±Π»ΠΎΠΊ, наклонная ΠΏΠ»ΠΎΡΠΊΠΎΡΡ‚ΡŒ, сила, ΠΌΠΎΠΌΠ΅Π½Ρ‚ сил, Π²Ρ‹ΠΈΠ³Ρ€Ρ‹Ρˆ Π² силС.

Π£Ρ‡Π΅Π±Π½ΠΈΠΊ: Π€ΠΈΠ·ΠΈΠΊΠ° 7 Π›.Π­ Π“Π΅Π½Π΄Π΅Π½ΡˆΡ‚Π΅ΠΉΠ½, А.А. Π‘ΡƒΠ»Π°Ρ‚ΠΎΠ²Π°, И.Н.ΠšΠΎΡ€Π½ΠΈΠ»ΡŒΠ΅Π², А.Π’.Кошкина

План ΡƒΡ€ΠΎΠΊΠ°

Π Π°Π±ΠΎΡ‚Π° Π² ΠΏΠ°Ρ€Π°Ρ… ΠΈ ΠΏΠΎΡΠ»Π΅Π΄ΡƒΡŽΡ‰Π΅Π΅ Ρ„Ρ€ΠΎΠ½Ρ‚Π°Π»ΡŒΠ½ΠΎΠ΅ обсуТдСниС вопросов:
  1. НазовитС Π²ΠΈΠ΄Ρ‹ простых ΠΌΠ΅Ρ…Π°Π½ΠΈΠ·ΠΌΠΎΠ².

  2. ΠŸΡ€ΠΈΠ²Π΅Π΄ΠΈΡ‚Π΅ ΠΏΡ€ΠΈΠΌΠ΅Ρ€Ρ‹ простых ΠΌΠ΅Ρ…Π°Π½ΠΈΠ·ΠΌΠΎΠ² ΠΈ Π½Π°Π·ΠΎΠ²ΠΈΡ‚Π΅, для Ρ‡Π΅Π³ΠΎ ΠΈΡ… ΠΈΡΠΏΠΎΠ»ΡŒΠ·ΡƒΡŽΡ‚?

  3. Π§Ρ‚ΠΎ называСтся Π±Π»ΠΎΠΊΠΎΠΌ?

  4. КакиС Π±Ρ‹Π²Π°ΡŽΡ‚ Π±Π»ΠΎΠΊΠΈ? ВсС Π»ΠΈ ΠΎΠ½ΠΈ Π΄Π°ΡŽΡ‚ Π²Ρ‹ΠΈΠ³Ρ€Ρ‹Ρˆ Π² силС?

2

Π˜Π·ΡƒΡ‡Π΅Π½ΠΈΠ΅ Π½ΠΎΠ²ΠΎΠ³ΠΎ ΠΌΠ°Ρ‚Π΅Ρ€ΠΈΠ°Π»Π°

РСшСниС Π·Π°Π΄Π°Ρ‡ΠΈ Ρƒ доски ΠΈΠ· ΡƒΡ‡Π΅Π±Π½ΠΈΠΊΠ° Β§24 β„–5

На рисунках ΠΈΠ·ΠΎΠ±Ρ€Π°ΠΆΠ΅Π½ подъСм Π³Ρ€ΡƒΠ·Π° силой F с ΠΏΠΎΠΌΠΎΡ‰ΡŒΡŽ Π½Π΅ΠΏΠΎΠ΄Π²ΠΈΠΆΠ½ΠΎΠ³ΠΎ ΠΈ ΠΏΠΎΠ΄Π²ΠΈΠΆΠ½ΠΎΠ³ΠΎ Π±Π»ΠΎΠΊΠΎΠ². Π’ ΠΎΠ±ΠΎΠΈΡ… случаях Π³Ρ€ΡƒΠ· ΠΏΠΎΠ΄Π½ΠΈΠΌΠ°ΡŽΡ‚ Π½Π° 50см. На ΠΊΠ°ΠΊΠΎΠ΅ расстояниС ΠΏΠ΅Ρ€Π΅ΠΌΠ΅Ρ‰Π°ΡŽΡ‚ Π² ΠΊΠ°ΠΆΠ΄ΠΎΠΌ случаС ΠΊΠΎΠ½Π΅Ρ† троса, ΠΊ ΠΊΠΎΡ‚ΠΎΡ€ΠΎΠΌΡƒ ΠΏΡ€ΠΈΠ»ΠΎΠΆΠ΅Π½Π° сила F?

3

Π­Ρ‚Π°ΠΏ закрСплСния ΠΈ ΠΏΠ΅Ρ€Π²ΠΈΡ‡Π½ΠΎΠΉ ΠΏΡ€ΠΎΠ²Π΅Ρ€ΠΊΠΈ

РСшСниС ΠΏΠΎΡ…ΠΎΠΆΠΈΡ… Π·Π°Π΄Π°Ρ‡ ΡΠ°ΠΌΠΎΡΡ‚ΠΎΡΡ‚Π΅Π»ΡŒΠ½ΠΎ ΠΈ ΠΏΠΎΡΠ»Π΅Π΄ΡƒΡŽΡ‰Π°Ρ ΠΏΡ€ΠΎΠ²Π΅Ρ€ΠΊΠ°:

Β§24 β„–6

На рисункС ΠΈΠ·ΠΎΠ±Ρ€Π°ΠΆΠ΅Π½ подъСм Π³Ρ€ΡƒΠ·Π° с ΠΏΠΎΠΌΠΎΡ‰ΡŒΡŽ систСмы Π±Π»ΠΎΠΊΠΎΠ².

А) Бколько Π² этой систСмС ΠΏΠΎΠ΄Π²ΠΈΠΆΠ½Ρ‹Ρ… Π±Π»ΠΎΠΊΠΎΠ² ΠΈ сколько Π½Π΅ΠΏΠΎΠ΄Π²ΠΈΠΆΠ½Ρ‹Ρ…;

Π‘) ΠŸΠΎΠ»ΡƒΡ‡Π°Π΅Ρ‚ΡΡ Π»ΠΈ ΠΏΡ€ΠΈ этом Π²Ρ‹ΠΈΠ³Ρ€Ρ‹Ρˆ Π² силС, ΠΈ Ссли Π΄Π°, Ρ‚ΠΎ ΠΊΠ°ΠΊΠΎΠΉ?

3

Π˜Π·ΡƒΡ‡Π΅Π½ΠΈΠ΅ Π½ΠΎΠ²ΠΎΠ³ΠΎ ΠΌΠ°Ρ‚Π΅Ρ€ΠΈΠ°Π»Π°

РСшСниС Π·Π°Π΄Π°Ρ‡ΠΈ Ρƒ доски ΠΈΠ· ΡƒΡ‡Π΅Π±Π½ΠΈΠΊΠ° Β§24 β„–8

1. Π’Π΅Π»Π΅ΠΆΠΊΡƒ массой 1 ΠΊΠ³ Ρ€Π°Π²Π½ΠΎΠΌΠ΅Ρ€Π½ΠΎ ΠΏΠΎΠ΄Π½ΠΈΠΌΠ°ΡŽΡ‚ ΠΏΠΎ Π½Π°ΠΊΠ»ΠΎΠ½Π½ΠΎΠΉ плоскости Π΄Π»ΠΈΠ½ΠΎΠΉ 2 ΠΌ, прикладывая силу F. Высота Π½Π°ΠΊΠ»ΠΎΠ½Π½ΠΎΠΉ плоскости 1 ΠΌ. Π’Ρ€Π΅Π½ΠΈΠ΅ΠΌ ΠΏΡ€Π΅Π½Π΅Π±Ρ€Π΅Ρ‡ΡŒ.

А) Какой Π²Ρ‹ΠΈΠ³Ρ€Ρ‹Ρˆ Π² силС получаСтся с ΠΏΠΎΠΌΠΎΡ‰ΡŒΡŽ Π΄Π°Π½Π½ΠΎΠΉ Π½Π°ΠΊΠ»ΠΎΠ½Π½ΠΎΠΉ плоскости?

Π‘) Π”Π°Π΅Ρ‚ Π»ΠΈ Π²Ρ‹ΠΈΠ³Ρ€Ρ‹Ρˆ использованиС Π±Π»ΠΎΠΊΠ°, ΠΈ Ссли Π΄Π°, Ρ‚ΠΎ ΠΊΠ°ΠΊΠΎΠΉ?

Π’) ΠšΠ°ΠΊΡƒΡŽ силу Π½Π°Π΄ΠΎ ΠΏΡ€ΠΈΠ»ΠΎΠΆΠΈΡ‚ΡŒ ΠΊ свободному ΠΊΠΎΠ½Ρ†Ρƒ троса, Ρ‡Ρ‚ΠΎΠ±Ρ‹ Ρ€Π°Π²Π½ΠΎΠΌΠ΅Ρ€Π½ΠΎ ΠΏΠΎΠ΄Π½ΠΈΠΌΠ°Ρ‚ΡŒ Ρ‚Π΅Π»Π΅ΠΆΠΊΡƒ ΠΏΠΎ Π½Π°ΠΊΠ»ΠΎΠ½Π½ΠΎΠΉ плоскости?

Π“) На ΠΊΠ°ΠΊΠΎΠ΅ расстояниС Π½ΡƒΠΆΠ½ΠΎ ΠΎΠΏΡƒΡΡ‚ΠΈΡ‚ΡŒ свободный ΠΊΠΎΠ½Π΅Ρ† троса. Π§Ρ‚ΠΎΠ±Ρ‹ ΠΏΠΎΠ΄Π½ΡΡ‚ΡŒ Ρ‚Π΅Π»Π΅ΠΆΠΊΡƒ вдоль всСй плоскости?

2. На рисункС схСматично ΠΈΠ·ΠΎΠ±Ρ€Π°ΠΆΠ΅Π½Ρ‹ Π½Π°ΠΊΠ»ΠΎΠ½Π½Ρ‹Π΅ плоскости. Какая ΠΈΠ· Π½ΠΈΡ… Π΄Π°Π΅Ρ‚ наибольший Π²Ρ‹ΠΈΠ³Ρ€Ρ‹Ρˆ Π² силС?

3

Π­Ρ‚Π°ΠΏ закрСплСния ΠΈ ΠΏΠ΅Ρ€Π²ΠΈΡ‡Π½ΠΎΠΉ ΠΏΡ€ΠΎΠ²Π΅Ρ€ΠΊΠΈ

РСшСниС ΠΏΠΎΡ…ΠΎΠΆΠΈΡ… Π·Π°Π΄Π°Ρ‡ ΡΠ°ΠΌΠΎΡΡ‚ΠΎΡΡ‚Π΅Π»ΡŒΠ½ΠΎ ΠΈ ΠΏΠΎΡΠ»Π΅Π΄ΡƒΡŽΡ‰Π°Ρ ΠΏΡ€ΠΎΠ²Π΅Ρ€ΠΊΠ°:

Β§24 β„–9

Π’Π΅Π»Π΅ΠΆΠΊΡƒ Ρ€Π°Π²Π½ΠΎΠΌΠ΅Ρ€Π½ΠΎ ΠΏΠΎΠ΄Π½ΠΈΠΌΠ°ΡŽΡ‚ ΠΏΠΎ Π½Π°ΠΊΠ»ΠΎΠ½Π½ΠΎΠΉ плоскости, прикладывая ΠΊ свободному ΠΊΠΎΠ½Ρ†Ρƒ тороса силу F, Ρ€Π°Π²Π½ΡƒΡŽ ΠΏΠΎ ΠΌΠΎΠ΄ΡƒΠ»ΡŽ 10Н. Π”Π»ΠΈΠ½Π° Π½Π°ΠΊΠ»ΠΎΠ½Π½ΠΎΠΉ плоскости 3 ΠΌ, Π° высота-75см. Π§Π΅ΠΌΡƒ Ρ€Π°Π²Π½Π° масса Ρ‚Π΅Π»Π΅ΠΆΠΊΠΈ? Π’Ρ€Π΅Π½ΠΈΠ΅ΠΌ ΠΈ массой Π±Π»ΠΎΠΊΠ° ΠΏΡ€Π΅Π½Π΅Π±Ρ€Π΅Ρ‡ΡŒ.

6

РСфлСксия

ВыставлСниС ΠΎΡ†Π΅Π½ΠΎΠΊ. ΠžΠ±ΡΡƒΠΆΠ΄Π΅Π½ΠΈΠ΅ Ρ‚ΠΎΠ³ΠΎ, Ρ‡Ρ‚ΠΎ Π²Ρ‹Π·Π²Π°Π»ΠΎ затруднСния Π½Π° ΡƒΡ€ΠΎΠΊΠ΅.

7

Π”ΠΎΠΌΠ°ΡˆΠ½Π΅Π΅ Π·Π°Π΄Π°Π½ΠΈΠ΅

  • Β§24 ,

  • НарисуйтС схСмы, с ΠΏΠΎΠΌΠΎΡ‰ΡŒΡŽ ΠΊΠΎΡ‚ΠΎΡ€Ρ‹Ρ… ΠΌΠΎΠΆΠ½ΠΎ ΠΏΠΎΠ»ΡƒΡ‡ΠΈΡ‚ΡŒ Π²Ρ‹ΠΈΠ³Ρ€Ρ‹Ρˆ Π² силС Π² 5 Ρ€Π°Π·, состоящиС ΠΈΠ· ΠΏΠΎΠ΄Π²ΠΈΠΆΠ½Ρ‹Ρ… ΠΈ Π½Π΅ΠΏΠΎΠ΄Π²ΠΈΠΆΠ½Ρ‹Ρ… Π±Π»ΠΎΠΊΠΎΠ². ΠŸΡ€ΠΈΠ΄ΡƒΠΌΠ°ΠΉΡ‚Π΅, ΠΊΠ°ΠΊ с ΠΏΠΎΠΌΠΎΡ‰ΡŒΡŽ ΠΈΠΌΠ΅ΡŽΡ‰ΠΈΡ…ΡΡ Π΄ΠΎΠΌΠ° ΠΏΡ€Π΅Π΄ΠΌΠ΅Ρ‚ΠΎΠ² ΠΈΠ·Π³ΠΎΡ‚ΠΎΠ²ΠΈΡ‚ΡŒ Π΄Π°Π½Π½Ρ‹Π΅ схСмы (дСтский конструктор, ΠΊΠ°Ρ‚ΡƒΡˆΠΊΠΈ Π½ΠΈΡ‚ΠΎΠΊ).

Π›ΠΈΡ‚Π΅Ρ€Π°Ρ‚ΡƒΡ€Π°:

  1. Π€ΠΎΡ€ΠΌΠΈΡ€ΠΎΠ²Π°Π½ΠΈΠ΅Β Π£Π£Π”Β Π½Π° уроках физики согласно Π€Π“ΠžΠ‘

( kopilkaurokov.ru)

  1. Π€ΠΈΠ·ΠΈΠΊΠ° 7 Π›.Π­ Π“Π΅Π½Π΄Π΅Π½ΡˆΡ‚Π΅ΠΉΠ½, А.А. Π‘ΡƒΠ»Π°Ρ‚ΠΎΠ²Π°, И.Н.ΠšΠΎΡ€Π½ΠΈΠ»ΡŒΠ΅Π², А.Π’.Кошкина

Π€ΠΈΠ·ΠΈΠΊΠ° 7 класс. ΠŸΡ€ΠΎΡΡ‚Ρ‹Π΅ ΠΌΠ΅Ρ…Π°Π½ΠΈΠ·ΠΌΡ‹. Π‘Π»ΠΎΠΊ :: Класс!ная Ρ„ΠΈΠ·ΠΈΠΊΠ°

Π€ΠΈΠ·ΠΈΠΊΠ° 7 класс. ΠŸΠ ΠžΠ‘Π’Π«Π• ΠœΠ•Π₯ΠΠΠ˜Π—ΠœΠ«

Π’ соврСмСнной Ρ‚Π΅Ρ…Π½ΠΈΠΊΠ΅ для пСрСноса Π³Ρ€ΡƒΠ·ΠΎΠ² Π½Π° стройках ΠΈ прСдприятиях ΡˆΠΈΡ€ΠΎΠΊΠΎ ΠΈΡΠΏΠΎΠ»ΡŒΠ·ΡƒΡŽΡ‚ΡΡ Π³Ρ€ΡƒΠ·ΠΎΠΏΠΎΠ΄ΡŠΠ΅ΠΌΠ½Ρ‹Π΅ ΠΌΠ΅Ρ…Π°Π½ΠΈΠ·ΠΌΡ‹, Π½Π΅Π·Π°ΠΌΠ΅Π½ΠΈΠΌΡ‹ΠΌΠΈ составными частями ΠΊΠΎΡ‚ΠΎΡ€Ρ‹Ρ… ΠΌΠΎΠΆΠ½ΠΎ Π½Π°Π·Π²Π°Ρ‚ΡŒ простыС ΠΌΠ΅Ρ…Π°Π½ΠΈΠ·ΠΌΡ‹. Π‘Ρ€Π΅Π΄ΠΈ Π½ΠΈΡ… Π΄Ρ€Π΅Π²Π½Π΅ΠΉΡˆΠΈΠ΅ изобрСтСния чСловСчСства: Π±Π»ΠΎΠΊ ΠΈ Ρ€Ρ‹Ρ‡Π°Π³. ДрСвнСгрСчСский ΡƒΡ‡Π΅Π½Ρ‹ΠΉ АрхимСд ΠΎΠ±Π»Π΅Π³Ρ‡ΠΈΠ» Ρ‚Ρ€ΡƒΠ΄ Ρ‡Π΅Π»ΠΎΠ²Π΅ΠΊΠ°, Π΄Π°Π² Π΅ΠΌΡƒ ΠΏΡ€ΠΈ использовании своСго изобрСтСния Π²Ρ‹ΠΈΠ³Ρ€Ρ‹Ρˆ Π² силС, ΠΈ Π½Π°ΡƒΡ‡ΠΈΠ» ΠΌΠ΅Π½ΡΡ‚ΡŒ Π½Π°ΠΏΡ€Π°Π²Π»Π΅Π½ΠΈΠ΅ дСйствия силы.

Π‘Π»ΠΎΠΊ – это колСсо с ΠΆΠ΅Π»ΠΎΠ±ΠΎΠΌ ΠΏΠΎ окруТности для ΠΊΠ°Π½Π°Ρ‚Π° ΠΈΠ»ΠΈ Ρ†Π΅ΠΏΠΈ, ось ΠΊΠΎΡ‚ΠΎΡ€ΠΎΠ³ΠΎ ТСстко ΠΏΡ€ΠΈΠΊΡ€Π΅ΠΏΠ»Π΅Π½Π° ΠΊ стСнС ΠΈΠ»ΠΈ ΠΏΠΎΡ‚ΠΎΠ»ΠΎΡ‡Π½ΠΎΠΉ Π±Π°Π»ΠΊΠ΅. Π“Ρ€ΡƒΠ·ΠΎΠΏΠΎΠ΄ΡŠΠ΅ΠΌΠ½Ρ‹Π΅ устройства ΠΎΠ±Ρ‹Ρ‡Π½ΠΎ ΠΈΡΠΏΠΎΠ»ΡŒΠ·ΡƒΡŽΡ‚ Π½Π΅ ΠΎΠ΄ΠΈΠ½, Π° нСсколько Π±Π»ΠΎΠΊΠΎΠ². БистСма Π±Π»ΠΎΠΊΠΎΠ² ΠΈ тросов, прСдназначСнная для ΠΏΠΎΠ²Ρ‹ΡˆΠ΅Π½ΠΈΡ Π³Ρ€ΡƒΠ·ΠΎΠΏΠΎΠ΄ΡŠΠ΅ΠΌΠ½ΠΎΡΡ‚ΠΈ, называСтся полиспаст.

ΠŸΠΎΠ΄Π²ΠΈΠΆΠ½Ρ‹ΠΉ ΠΈ Π½Π΅ΠΏΠΎΠ΄Π²ΠΈΠΆΠ½Ρ‹ΠΉ Π±Π»ΠΎΠΊ – Ρ‚Π°ΠΊΠΈΠ΅ ΠΆΠ΅ Π΄Ρ€Π΅Π²Π½Π΅ΠΉΡˆΠΈΠ΅ простыС ΠΌΠ΅Ρ…Π°Π½ΠΈΠ·ΠΌΡ‹, ΠΊΠ°ΠΊ ΠΈ Ρ€Ρ‹Ρ‡Π°Π³. Π£ΠΆΠ΅ Π² 212 Π³.Π΄ΠΎ Π½.эры с ΠΏΠΎΠΌΠΎΡ‰ΡŒΡŽ ΠΊΡ€ΡŽΠΊΠΎΠ² ΠΈ Π·Π°Ρ…Π²Π°Ρ‚ΠΎΠ², соСдинСнных с Π±Π»ΠΎΠΊΠ°ΠΌΠΈ, сиракузцы Π·Π°Ρ…Π²Π°Ρ‚Ρ‹Π²Π°Π»ΠΈ Ρƒ римлян срСдства осады. Π‘ΠΎΠΎΡ€ΡƒΠΆΠ΅Π½ΠΈΠ΅ΠΌ Π²ΠΎΠ΅Π½Π½Ρ‹Ρ… машин ΠΈ ΠΎΠ±ΠΎΡ€ΠΎΠ½ΠΎΠΉ Π³ΠΎΡ€ΠΎΠ΄Π° Ρ€ΡƒΠΊΠΎΠ²ΠΎΠ΄ΠΈΠ» АрхимСд.

НСподвиТный Π±Π»ΠΎΠΊ АрхимСд рассматривал ΠΊΠ°ΠΊ Ρ€Π°Π²Π½ΠΎΠΏΠ»Π΅Ρ‡ΠΈΠΉ Ρ€Ρ‹Ρ‡Π°Π³.
ΠœΠΎΠΌΠ΅Π½Ρ‚ силы, Π΄Π΅ΠΉΡΡ‚Π²ΡƒΡŽΡ‰Π΅ΠΉ с ΠΎΠ΄Π½ΠΎΠΉ стороны Π±Π»ΠΎΠΊΠ°, Ρ€Π°Π²Π΅Π½ ΠΌΠΎΠΌΠ΅Π½Ρ‚Ρƒ силы, ΠΏΡ€ΠΈΠ»ΠΎΠΆΠ΅Π½Π½ΠΎΠΉ с Π΄Ρ€ΡƒΠ³ΠΎΠΉ стороны Π±Π»ΠΎΠΊΠ°. ΠžΠ΄ΠΈΠ½Π°ΠΊΠΎΠ²Ρ‹ ΠΈ силы, ΡΠΎΠ·Π΄Π°ΡŽΡ‰ΠΈΠ΅ эти ΠΌΠΎΠΌΠ΅Π½Ρ‚Ρ‹.
Π’Ρ‹ΠΈΠ³Ρ€Ρ‹Ρˆ Π² силС ΠΏΡ€ΠΈ этом отсутствуСт, Π½ΠΎ Ρ‚Π°ΠΊΠΎΠΉ Π±Π»ΠΎΠΊ позволяСт ΠΈΠ·ΠΌΠ΅Π½ΠΈΡ‚ΡŒ Π½Π°ΠΏΡ€Π°Π²Π»Π΅Π½ΠΈΠ΅ дСйствия силы, Ρ‡Ρ‚ΠΎ ΠΈΠ½ΠΎΠ³Π΄Π° Π½Π΅ΠΎΠ±Ρ…ΠΎΠ΄ΠΈΠΌΠΎ.

ΠŸΠΎΠ΄Π²ΠΈΠΆΠ½Ρ‹ΠΉ Π±Π»ΠΎΠΊ АрхимСд ΠΏΡ€ΠΈΠ½ΠΈΠΌΠ°Π» Π·Π° Π½Π΅Ρ€Π°Π²Π½ΠΎΠΏΠ»Π΅Ρ‡ΠΈΠΉ Ρ€Ρ‹Ρ‡Π°Π³, Π΄Π°ΡŽΡ‰ΠΈΠΉ Π²Ρ‹ΠΈΠ³Ρ€Ρ‹Ρˆ Π² силС Π² 2 Ρ€Π°Π·Π°. ΠžΡ‚Π½ΠΎΡΠΈΡ‚Π΅Π»ΡŒΠ½ΠΎ Ρ†Π΅Π½Ρ‚Ρ€Π° вращСния Π΄Π΅ΠΉΡΡ‚Π²ΡƒΡŽΡ‚ ΠΌΠΎΠΌΠ΅Π½Ρ‚Ρ‹ сил, ΠΊΠΎΡ‚ΠΎΡ€Ρ‹Π΅ ΠΏΡ€ΠΈ равновСсии Π΄ΠΎΠ»ΠΆΠ½Ρ‹ Π±Ρ‹Ρ‚ΡŒ Ρ€Π°Π²Π½Ρ‹.

АрхимСд ΠΈΠ·ΡƒΡ‡ΠΈΠ» мСханичСскиС свойства ΠΏΠΎΠ΄Π²ΠΈΠΆΠ½ΠΎΠ³ΠΎ Π±Π»ΠΎΠΊΠ° ΠΈ ΠΏΡ€ΠΈΠΌΠ΅Π½ΠΈΠ» Π΅Π³ΠΎ Π½Π° ΠΏΡ€Π°ΠΊΡ‚ΠΈΠΊΠ΅. По ΡΠ²ΠΈΠ΄Π΅Ρ‚Π΅Π»ΡŒΡΡ‚Π²Ρƒ АфинСя, “для спуска Π½Π° Π²ΠΎΠ΄Ρƒ исполинского корабля, построСнного сиракузским Ρ‚ΠΈΡ€Π°Π½ΠΎΠΌ Π“ΠΈΠ΅Ρ€ΠΎΠ½ΠΎΠΌ, ΠΏΡ€ΠΈΠ΄ΡƒΠΌΡ‹Π²Π°Π»ΠΈ ΠΌΠ½ΠΎΠ³ΠΎ способов, Π½ΠΎ ΠΌΠ΅Ρ…Π°Π½ΠΈΠΊ АрхимСд, ΠΏΡ€ΠΈΠΌΠ΅Π½ΠΈΠ² простыС ΠΌΠ΅Ρ…Π°Π½ΠΈΠ·ΠΌΡ‹, ΠΎΠ΄ΠΈΠ½ сумСл ΡΠ΄Π²ΠΈΠ½ΡƒΡ‚ΡŒ ΠΊΠΎΡ€Π°Π±Π»ΡŒ с ΠΏΠΎΠΌΠΎΡ‰ΡŒΡŽ Π½Π΅ΠΌΠ½ΠΎΠ³ΠΈΡ… людСй. АрхимСд ΠΏΡ€ΠΈΠ΄ΡƒΠΌΠ°Π» Π±Π»ΠΎΠΊ ΠΈ посрСдством Π½Π΅Π³ΠΎ спустил Π½Π° Π²ΠΎΠ΄Ρƒ Π³Ρ€ΠΎΠΌΠ°Π΄Π½Ρ‹ΠΉ ΠΊΠΎΡ€Π°Π±Π»ΡŒ”.

Π‘Π»ΠΎΠΊ Π½Π΅ Π΄Π°Π΅Ρ‚ Π²Ρ‹ΠΈΠ³Ρ€Ρ‹ΡˆΠ° Π² Ρ€Π°Π±ΠΎΡ‚Π΅, подтвСрТдая “Π·ΠΎΠ»ΠΎΡ‚ΠΎΠ΅ ΠΏΡ€Π°Π²ΠΈΠ»ΠΎ” ΠΌΠ΅Ρ…Π°Π½ΠΈΠΊΠΈ. Π’ этом Π»Π΅Π³ΠΊΠΎ ΡƒΠ±Π΅Π΄ΠΈΡ‚ΡŒΡΡ, ΠΎΠ±Ρ€Π°Ρ‚ΠΈΠ² Π²Π½ΠΈΠΌΠ°Π½ΠΈΠ΅ Π½Π° расстояния, ΠΏΡ€ΠΎΠΉΠ΄Π΅Π½Π½Ρ‹Π΅ Ρ€ΡƒΠΊΠΎΠΉ ΠΈ Π³ΠΈΡ€Π΅ΠΉ.

Π‘ΠΏΠΎΡ€Ρ‚ΠΈΠ²Π½Ρ‹Π΅ парусныС суда, ΠΊΠ°ΠΊ ΠΈ парусники ΠΏΡ€ΠΎΡˆΠ»ΠΎΠ³ΠΎ,


Устали? – ΠžΡ‚Π΄Ρ‹Ρ…Π°Π΅ΠΌ!

Π‘Ρ‚Ρ€Π°Π½ΠΈΡ†Π° Π½Π΅ Π½Π°ΠΉΠ΄Π΅Π½Π° | MIT

ΠŸΠ΅Ρ€Π΅ΠΉΡ‚ΠΈ ΠΊ ΡΠΎΠ΄Π΅Ρ€ΠΆΠ°Π½ΠΈΡŽ ↓
  • ΠžΠ±Ρ€Π°Π·ΠΎΠ²Π°Π½ΠΈΠ΅
  • Π˜ΡΡΠ»Π΅Π΄ΠΎΠ²Π°Ρ‚ΡŒ
  • Π˜Π½Π½ΠΎΠ²Π°Ρ†ΠΈΠΈ
  • ΠŸΡ€ΠΈΠ΅ΠΌ + ΠΏΠΎΠΌΠΎΡ‰ΡŒ
  • БтудСнчСская Тизнь
  • Новости
  • Выпускников
  • О ΠœΠ°ΡΡΠ°Ρ‡ΡƒΡΠ΅Ρ‚ΡΠΊΠΎΠΌ тСхнологичСском институтС
  • ΠŸΠΎΠ΄Ρ€ΠΎΠ±Π½Π΅Π΅ ↓
    • ΠŸΡ€ΠΈΠ΅ΠΌ + ΠΏΠΎΠΌΠΎΡ‰ΡŒ
    • БтудСнчСская Тизнь
    • Новости
    • Выпускников
    • О ΠœΠ°ΡΡΠ°Ρ‡ΡƒΡΠ΅Ρ‚ΡΠΊΠΎΠΌ тСхнологичСском институтС
МСню ↓ Поиск МСню Ой, ΠΏΠΎΡ…ΠΎΠΆΠ΅, ΠΌΡ‹ Π½Π΅ смогли Π½Π°ΠΉΡ‚ΠΈ Ρ‚ΠΎ, Ρ‡Ρ‚ΠΎ Π²Ρ‹ искали!
ΠŸΠΎΠΏΡ€ΠΎΠ±ΡƒΠΉΡ‚Π΅ ΠΏΠΎΠΈΡΠΊΠ°Ρ‚ΡŒ Ρ‡Ρ‚ΠΎ-Π½ΠΈΠ±ΡƒΠ΄ΡŒ Π΅Ρ‰Π΅! Π§Ρ‚ΠΎ Π²Ρ‹ ΠΈΡ‰Π΅Ρ‚Π΅? Π£Π²ΠΈΠ΄Π΅Ρ‚ΡŒ большС Ρ€Π΅Π·ΡƒΠ»ΡŒΡ‚Π°Ρ‚ΠΎΠ²

ΠŸΡ€Π΅Π΄Π»ΠΎΠΆΠ΅Π½ΠΈΡ ΠΈΠ»ΠΈ ΠΎΡ‚Π·Ρ‹Π²Ρ‹?

Π½ΡŒΡŽΡ‚ΠΎΠ½ΠΎΠ²ΡΠΊΠΎΠΉ ΠΌΠ΅Ρ…Π°Π½ΠΈΠΊΠΈ – Π—Π°Π΄Π°Ρ‡Π° Π±Π»ΠΎΠΊΠ° Π½Π° Π±Π»ΠΎΠΊΠ΅ с Ρ‚Ρ€Π΅Π½ΠΈΠ΅ΠΌ

ΠŸΡ€ΠΈΡ‡ΠΈΠ½Π°, ΠΏΠΎ ΠΊΠΎΡ‚ΠΎΡ€ΠΎΠΉ Π²Ρ‹ Π½Π°Ρ…ΠΎΠ΄ΠΈΡ‚Π΅ это Π½Π΅ ΠΈΠ½Ρ‚ΡƒΠΈΡ‚ΠΈΠ²Π½ΠΎ понятным, Π·Π°ΠΊΠ»ΡŽΡ‡Π°Π΅Ρ‚ΡΡ Π² Ρ‚ΠΎΠΌ, Ρ‡Ρ‚ΠΎ Π²Ρ‹ ΠΏΡ€Π΅Π΄ΠΏΠΎΠ»Π°Π³Π°Π΅Ρ‚Π΅, Ρ‡Ρ‚ΠΎ прилоТСнная сила ΠΊ Π²Π΅Ρ€Ρ…Π½Π΅ΠΌΡƒ Π±Π»ΠΎΠΊΡƒ заставит Π½ΠΈΠΆΠ½ΠΈΠΉ Π±Π»ΠΎΠΊ ΠΈΡΠΏΡ‹Ρ‚Ρ‹Π²Π°Ρ‚ΡŒ Ρ‚Ρƒ ΠΆΠ΅ силу ΠΈΠ·-Π·Π° трСния, Ρ‡Ρ‚ΠΎ Π½Π΅Π²Π΅Ρ€Π½ΠΎ.

НапримСр, рассмотрим Π΄Π²Π° Π±Π»ΠΎΠΊΠ° A, ΠΈ B , ΠΊΠΎΡ‚ΠΎΡ€Ρ‹Π΅ сидят Π±ΠΎΠΊ ΠΎ Π±ΠΎΠΊ Π½Π° Π³Π»Π°Π΄ΠΊΠΎΠΌ столС. Если Π²Ρ‹ ΠΏΡ€ΠΈΠ»ΠΎΠΆΠΈΡ‚Π΅ силу F ΠΊ A , Ρ‚ΠΎ ΠΌΠΎΠΆΠ½ΠΎ ΠΎΡˆΠΈΠ±ΠΎΡ‡Π½ΠΎ ΡΠΊΠ°Π·Π°Ρ‚ΡŒ, Ρ‡Ρ‚ΠΎ сила, ΠΊΠΎΡ‚ΠΎΡ€ΡƒΡŽ испытываСт B , Ρ‚Π°ΠΊΠΆΠ΅ Ρ€Π°Π²Π½Π° F . Но это Π±Ρ‹Π»ΠΎ Π±Ρ‹ Π½Π΅ΠΏΡ€Π°Π²ΠΈΠ»ΡŒΠ½ΠΎ. На самом Π΄Π΅Π»Π΅ это мСньшС, Ρ‡Π΅ΠΌ F .

Π’ΠΎ ΠΆΠ΅ Π²Π΅Ρ€Π½ΠΎ ΠΈ для вашСго ΠΏΡ€ΠΈΠΌΠ΅Ρ€Π°. Π‘ΠΈΠ»Π° трСния, испытываСмая Π½ΠΈΠΆΠ½ΠΈΠΌ Π±Π»ΠΎΠΊΠΎΠΌ, мСньшС Π΄Π΅ΠΉΡΡ‚Π²ΠΈΡ‚Π΅Π»ΡŒΠ½ΠΎΠΉ силы, ΠΏΡ€ΠΈΠ»ΠΎΠΆΠ΅Π½Π½ΠΎΠΉ ΠΊ Π²Π΅Ρ€Ρ…Π½Π΅ΠΌΡƒ Π±Π»ΠΎΠΊΡƒ.Π‘Π»Π΅Π΄ΠΎΠ²Π°Ρ‚Π΅Π»ΡŒΠ½ΠΎ, Ρ‡Ρ‚ΠΎΠ±Ρ‹ Π½ΠΈΠΆΠ½ΠΈΠΉ Π±Π»ΠΎΠΊ испытывал ΠΌΠ°ΠΊΡΠΈΠΌΠ°Π»ΡŒΠ½ΡƒΡŽ силу статичСского трСния, Π½Π΅ΠΎΠ±Ρ…ΠΎΠ΄ΠΈΠΌΠΎ ΠΏΡ€ΠΈΠ»ΠΎΠΆΠΈΡ‚ΡŒ силу, ΠΏΡ€Π΅Π²Ρ‹ΡˆΠ°ΡŽΡ‰ΡƒΡŽ эту. ΠžΠ±Ρ€Π°Ρ‚ΠΈΡ‚Π΅ Π²Π½ΠΈΠΌΠ°Π½ΠΈΠ΅, Ρ‡Ρ‚ΠΎ эта «сила трСния» Π²ΠΎΠ·Π½ΠΈΠΊΠ°Π΅Ρ‚ ΠΈΠ·-Π·Π° самого Π²Π΅Ρ€Ρ…Π½Π΅Π³ΠΎ Π±Π»ΠΎΠΊΠ°, ΠΈ, ΡΠ»Π΅Π΄ΠΎΠ²Π°Ρ‚Π΅Π»ΡŒΠ½ΠΎ, ваш ΠΏΡ€ΠΈΠΌΠ΅Ρ€ Π°Π½Π°Π»ΠΎΠ³ΠΈΡ‡Π΅Π½ ΠΏΡ€ΠΈΠ²Π΅Π΄Π΅Π½Π½ΠΎΠΌΡƒ Π²Ρ‹ΡˆΠ΅ ΠΏΡ€ΠΈΠΌΠ΅Ρ€Ρƒ.

Но ΠΏΠΎΡ‡Π΅ΠΌΡƒ Π½ΠΈΠΆΠ½ΠΈΠΉ Π±Π»ΠΎΠΊ Π²ΠΎΠΎΠ±Ρ‰Π΅ испытываСт ΠΌΠ΅Π½ΡŒΡˆΡƒΡŽ силу? Π­Ρ‚ΠΎ ΠΏΠΎΡ‚ΠΎΠΌΡƒ, Ρ‡Ρ‚ΠΎ Β«ΠΎΡΡ‚Π°Π»ΡŒΠ½Π°ΡΒ» сила (ΠΈΠ»ΠΈ, Ρ‚ΠΎΡ‡Π½Π΅Π΅ говоря: ΠΎΡΡ‚Π°Π»ΡŒΠ½Π°Ρ Ρ‡Π°ΡΡ‚ΡŒ ΠΈΠΌΠΏΡƒΠ»ΡŒΡΠ°) ΠΈΡΠΏΠΎΠ»ΡŒΠ·ΡƒΠ΅Ρ‚ΡΡ для пСрСмСщСния самого Π²Π΅Ρ€Ρ…Π½Π΅Π³ΠΎ Π±Π»ΠΎΠΊΠ°. Если Π²Π΅Ρ€Ρ…Π½ΠΈΠΉ Π±Π»ΠΎΠΊ массивный, Π²Π°ΠΌ Π½ΡƒΠΆΠ½ΠΎ Π±ΡƒΠ΄Π΅Ρ‚ ΠΏΡ€ΠΈΠ»ΠΎΠΆΠΈΡ‚ΡŒ Π΅Ρ‰Π΅ Π±ΠΎΠ»ΡŒΡˆΡƒΡŽ силу, Ρ‡Ρ‚ΠΎΠ±Ρ‹ Π·Π°ΡΡ‚Π°Π²ΠΈΡ‚ΡŒ Π²Π΅Ρ€Ρ…Π½ΠΈΠΉ Π±Π»ΠΎΠΊ ΡƒΡΠΊΠΎΡ€ΡΡ‚ΡŒΡΡ с Ρ‚ΠΎΠΉ ΠΆΠ΅ ΡΠΊΠΎΡ€ΠΎΡΡ‚ΡŒΡŽ, Ρ‡Ρ‚ΠΎ ΠΈ Π½ΠΈΠΆΠ½ΠΈΠΉ Π±Π»ΠΎΠΊ.Π‘ΠΎΠ»ΡŒΡˆΠ°Ρ Ρ‡Π°ΡΡ‚ΡŒ ΠΏΠ΅Ρ€Π΅Π΄Π°Π½Π½ΠΎΠ³ΠΎ ΠΈΠΌΠΏΡƒΠ»ΡŒΡΠ° (силы) здСсь пСрСдаСтся Π²Π΅Ρ€Ρ…Π½Π΅ΠΌΡƒ Π±Π»ΠΎΠΊΡƒ, Π° ΠΎΡΡ‚Π°Π»ΡŒΠ½Π°Ρ Ρ‡Π°ΡΡ‚ΡŒ – Π½ΠΈΠΆΠ½Π΅ΠΌΡƒ Π±Π»ΠΎΠΊΡƒ. Если Π²Π΅Ρ€Ρ…Π½ΠΈΠΉ Π±Π»ΠΎΠΊ Π»Π΅Π³ΠΊΠΈΠΉ, Ρ‚ΠΎ ΠΎΡ‡Π΅Π½ΡŒ нСбольшая Ρ‡Π°ΡΡ‚ΡŒ ΠΏΠ΅Ρ€Π΅Π΄Π°Π²Π°Π΅ΠΌΠΎΠ³ΠΎ ΠΈΠΌΠΏΡƒΠ»ΡŒΡΠ° (силы) ΠΈΡΠΏΠΎΠ»ΡŒΠ·ΡƒΠ΅Ρ‚ΡΡ для пСрСмСщСния Π²Π΅Ρ€Ρ…Π½Π΅Π³ΠΎ Π±Π»ΠΎΠΊΠ°, Π° ΠΎΡΡ‚Π°Π»ΡŒΠ½Π°Ρ Ρ‡Π°ΡΡ‚ΡŒ пСрСдаСтся Π½ΠΈΠΆΠ½Π΅ΠΌΡƒ Π±Π»ΠΎΠΊΡƒ. Π‘Π»Π΅Π΄ΠΎΠ²Π°Ρ‚Π΅Π»ΡŒΠ½ΠΎ, Π² этом случаС трСбуСмая сила Π±ΡƒΠ΄Π΅Ρ‚ Π±Π»ΠΈΠ·ΠΊΠ° ΠΊ максимальной силС статичСского трСния.

Π’Ρ‹ ΠΌΠΎΠΆΠ΅Ρ‚Π΅ ΡƒΠ²ΠΈΠ΄Π΅Ρ‚ΡŒ это Π² своСм Π²Ρ‹Ρ€Π°ΠΆΠ΅Π½ΠΈΠΈ: $ F = \ frac {(m_1 + m_2) F_ {sf}} {m_2} $

ΠΏΡ€ΠΈ $ m_1 \ to 0 $ трСбуСмая сила приблиТаСтся ΠΊ силС трСния.

Однако Π² этом Π²Ρ‹Ρ€Π°ΠΆΠ΅Π½ΠΈΠΈ: $ F = \ frac {m_1} {m_2} \ mu_s (m_1 + m_2) g $

ΠΊΠ°ΠΊ $ m_1 \ to 0 $, сила Ρ‚Π°ΠΊΠΆΠ΅ приблиТаСтся ΠΊ $ 0 $. Π­Ρ‚ΠΎ ΠΏΠΎΡ‚ΠΎΠΌΡƒ, Ρ‡Ρ‚ΠΎ это Π²Ρ‹Ρ€Π°ΠΆΠ΅Π½ΠΈΠ΅ Π²ΠΊΠ»ΡŽΡ‡Π°Π΅Ρ‚ Ρ‚ΠΎΡ‚ Ρ„Π°ΠΊΡ‚, Ρ‡Ρ‚ΠΎ максимальная сила трСния ΡƒΠΌΠ΅Π½ΡŒΡˆΠ°Π΅Ρ‚ΡΡ ΠΏΠΎ ΠΌΠ΅Ρ€Π΅ ΡƒΠΌΠ΅Π½ΡŒΡˆΠ΅Π½ΠΈΡ массы Π²Π΅Ρ€Ρ…Π½Π΅Π³ΠΎ Π±Π»ΠΎΠΊΠ° (Ρ‡Π΅ΠΌ Π»Π΅Π³Ρ‡Π΅ Π²Π΅Ρ€Ρ…Π½ΠΈΠΉ Π±Π»ΠΎΠΊ, Ρ‚Π΅ΠΌ мСньшС Π½ΠΎΡ€ΠΌΠ°Π»ΡŒΠ½Π°Ρ сила ΠΌΠ΅ΠΆΠ΄Ρƒ двумя Π±Π»ΠΎΠΊΠ°ΠΌΠΈ). Π‘Π»Π΅Π΄ΠΎΠ²Π°Ρ‚Π΅Π»ΡŒΠ½ΠΎ, ΠΊΠΎΠ³Π΄Π° масса Π²Π΅Ρ€Ρ…Π½Π΅Π³ΠΎ Π±Π»ΠΎΠΊΠ° приблиТаСтся ΠΊ $ 0 $, максимальная сила, ΠΊΠΎΡ‚ΠΎΡ€ΡƒΡŽ Π²Ρ‹ ΠΌΠΎΠΆΠ΅Ρ‚Π΅ ΠΏΡ€ΠΈΠ»ΠΎΠΆΠΈΡ‚ΡŒ, Ρ‚Π°ΠΊΠΆΠ΅ приблиТаСтся ΠΊ $ 0 $.

Π΄ΠΎΠΌΠ°ΡˆΠ½ΠΈΡ… Π·Π°Π΄Π°Π½ΠΈΠΉ ΠΈ ΡƒΠΏΡ€Π°ΠΆΠ½Π΅Π½ΠΈΠΉ – ΠŸΠ°Ρ€Π°Π΄ΠΎΠΊΡ Π² Π·Π°Π΄Π°Ρ‡Π΅ Π΄Π²ΡƒΡ… Π±Π»ΠΎΠΊΠΎΠ²

Π—Π°ΠΊΡ€Ρ‹Ρ‚ΠΎ. Π­Ρ‚ΠΎ вопрос Π½Π΅ ΠΏΠΎ Ρ‚Π΅ΠΌΠ΅. Π’ настоящСС врСмя ΠΎΠ½ Π½Π΅ ΠΏΡ€ΠΈΠ½ΠΈΠΌΠ°Π΅Ρ‚ ΠΎΡ‚Π²Π΅Ρ‚Ρ‹.

Π₯ΠΎΡ‚ΠΈΡ‚Π΅ ΡƒΠ»ΡƒΡ‡ΡˆΠΈΡ‚ΡŒ этот вопрос? ΠžΠ±Π½ΠΎΠ²ΠΈΡ‚Π΅ вопрос, Ρ‡Ρ‚ΠΎΠ±Ρ‹ ΠΎΠ½ соотвСтствовал Ρ‚Π΅ΠΌΠ΅ Physics Stack Exchange.

Π—Π°ΠΊΡ€Ρ‹Ρ‚ 6 Π»Π΅Ρ‚ Π½Π°Π·Π°Π΄.

ΠžΠ‘ΠΠžΠ’Π›Π•ΠΠ˜Π•

(ΠΎΡ‚Π½ΠΎΡΠΈΡ‚Π΅Π»ΡŒΠ½ΠΎ Π΄ΡƒΠ±Π»ΠΈΠΊΠ°Ρ‚Π°):

Π­Ρ‚ΠΎΡ‚ вопрос Π½Π΅ Π΄ΡƒΠ±Π»ΠΈΡ€ΡƒΠ΅Ρ‚ Π΄Ρ€ΡƒΠ³ΠΎΠΉ вопрос.ΠšΠΎΠ½Π΅Ρ‡Π½ΠΎ, ситуация Π² ΠΎΠ±ΠΎΠΈΡ… вопросах ΠΎΠ΄ΠΈΠ½Π°ΠΊΠΎΠ²Π°, ΠΈ, Π΄Π°, ΠΎΠ±Π° вопроса Π² ΠΊΠΎΠ½Π΅Ρ‡Π½ΠΎΠΌ ΠΈΡ‚ΠΎΠ³Π΅ ΠΏΡ€Π΅Π΄ΠΎΡΡ‚Π°Π²Π»ΡΡŽΡ‚ ΠΌΠ΅Ρ‚ΠΎΠ΄ΠΎΠ»ΠΎΠ³ΠΈΡŽ для Ρ€Π΅ΡˆΠ΅Π½ΠΈΡ ΠΏΡ€ΠΎΠ±Π»Π΅ΠΌΡ‹ ΠΈ нахоТдСния ΠΏΡ€Π°Π²ΠΈΠ»ΡŒΠ½ΠΎΠ³ΠΎ значСния трСния, Π½ΠΎ ΠΌΠΎΠ΄Π΅Ρ€Π°Ρ‚ΠΎΡ€ Π΄ΠΎΠ»ΠΆΠ΅Π½ ΠΏΠΎΠ½ΠΈΠΌΠ°Ρ‚ΡŒ, Ρ‡Ρ‚ΠΎ здСсь я НЕ просил ΠΎ ΠΌΠ΅Ρ‚ΠΎΠ΄Π΅ Ρ€Π΅ΡˆΠ΅Π½ΠΈΡ для Π’Ρ€Π΅Π½ΠΈΠ΅ . ΠŸΡ€ΠΎΠ±Π»Π΅ΠΌΠ° Π·Π°ΠΊΠ»ΡŽΡ‡Π°Π»Π°ΡΡŒ Π² Ρ‚ΠΎΠΌ, Ρ‡Ρ‚ΠΎ я Π΄Π°ΠΆΠ΅ Π½Π΅ ΠΏΠΎΠ½ΠΈΠΌΠ°Π», Ρ‡Ρ‚ΠΎ придСтся Ρ€Π΅ΡˆΠ°Ρ‚ΡŒ ΠΏΠΎ Ρ‚Ρ€Π΅Π½ΠΈΡŽ.

Π― Ρ€Π΅ΡˆΠΈΠ» ΠΌΠ½ΠΎΠ³ΠΎ Ρ‚Π°ΠΊΠΈΡ… Π·Π°Π΄Π°Ρ‡ Π΅Ρ‰Π΅ Π² школС, ΠΈ Ρƒ мСня появилась ΠΏΡ€ΠΈΠ²Ρ‹Ρ‡ΠΊΠ° ΡΡ‡ΠΈΡ‚Π°Ρ‚ΡŒ статичСскоС Ρ‚Ρ€Π΅Π½ΠΈΠ΅ Ρ€Π°Π²Π½Ρ‹ΠΌ ΠΈ ΠΏΡ€ΠΎΡ‚ΠΈΠ²ΠΎΠΏΠΎΠ»ΠΎΠΆΠ½Ρ‹ΠΌ любой силС, ΠΏΡ€ΠΈΠ»Π°Π³Π°Π΅ΠΌΠΎΠΉ ΠΊ Ρ‚Ρ€Π΅Π½ΠΈΡŽ (Π²ΠΏΠ»ΠΎΡ‚ΡŒ Π΄ΠΎ максимального ΠΏΡ€Π΅Π΄Π΅Π»Π° трСния).Π­Ρ‚ΠΎ Π±Ρ‹Π»Π° грубая ошибка, ΠΊΠΎΡ‚ΠΎΡ€ΡƒΡŽ я сдСлал, ΠΈ ΠΈΠΌΠ΅Π½Π½ΠΎ поэтому ΠΎΠ½Π° выглядСла ΠΊΠ°ΠΊ «парадокс». Π‘ΠΎΠ»Π΅Π΅ Ρ‚ΠΎΠ³ΠΎ, ΠΊΠ°ΠΊ оказалось, я Π·Π°Π΄Π°Π» Ρ‚Ρƒ ΠΆΠ΅ Π·Π°Π΄Π°Ρ‡Ρƒ нСскольким своим Π΄Ρ€ΡƒΠ·ΡŒΡΠΌ, ΠΈ ΠΌΠ½ΠΎΠ³ΠΈΠ΅ ΠΈΠ· Π½ΠΈΡ… ΡΠΎΠ²Π΅Ρ€ΡˆΠΈΠ»ΠΈ Ρ‚Ρƒ ΠΆΠ΅ ΠΎΡˆΠΈΠ±ΠΊΡƒ.

Π˜Ρ‚Π°ΠΊ, ΠΏΠΎ сути, «другая» ΠΏΡ€ΠΎΠ±Π»Π΅ΠΌΠ° – это просто запрос ΠΎΠ±Ρ‰Π΅ΠΉ ΠΌΠ΅Ρ‚ΠΎΠ΄ΠΎΠ»ΠΎΠ³ΠΈΠΈ для Ρ€Π΅ΡˆΠ΅Π½ΠΈΡ Ρ‚Π°ΠΊΠΈΡ… ΠΏΡ€ΠΎΠ±Π»Π΅ΠΌ, Π² Ρ‚ΠΎ врСмя ΠΊΠ°ΠΊ эта ΠΏΡ€ΠΎΠ±Π»Π΅ΠΌΠ° ΠΏΠΎΡ…ΠΎΠΆΠ° Π½Π° Π³ΠΎΠ»ΠΎΠ²ΠΎΠ»ΠΎΠΌΠΊΡƒ, которая прСдставляСт ΠΏΠΎΠ»ΡŒΠ·ΠΎΠ²Π°Ρ‚Π΅Π»ΡŽ ΠΌΠ΅Ρ‚ΠΎΠ΄ΠΎΠ»ΠΎΠ³ΠΈΡŽ Ρ€Π΅ΡˆΠ΅Π½ΠΈΡ ΠΏΡ€ΠΎΠ±Π»Π΅ΠΌΡ‹ ΠΏΡƒΡ‚Π΅ΠΌ рассмотрСния Ρ€Π°Π·Π»ΠΈΡ‡Π½Ρ‹Ρ… систСм ΠΈ ΠΏΡ€ΠΎΡ‚ΠΈΠ²ΠΎΡ€Π΅Ρ‡ΠΈΠΉ, ΠΊΠΎΡ‚ΠΎΡ€Ρ‹Π΅ Π²ΠΎΠ·Π½ΠΈΠΊΠ°ΡŽΡ‚ ΠΈΠ·-Π·Π° Π½ΠΈΡ…. . Π― ΡΡ‡ΠΈΡ‚Π°ΡŽ, Ρ‡Ρ‚ΠΎ ΠΏΠΎΠ»ΡŒΠ·ΠΎΠ²Π°Ρ‚Π΅Π»ΡŒ, ΠΊΠΎΡ‚ΠΎΡ€Ρ‹ΠΉ Π·Π½Π°Π΅Ρ‚ ΠΎΠ±Ρ‰ΡƒΡŽ ΠΌΠ΅Ρ‚ΠΎΠ΄ΠΎΠ»ΠΎΠ³ΠΈΡŽ, ΠΏΡ€Π΅Π΄ΡΡ‚Π°Π²Π»Π΅Π½Π½ΡƒΡŽ Π² Π΄Ρ€ΡƒΠ³ΠΎΠΌ вопросС, ΠΏΠΎΠ΄Π²Π΅Ρ€ΠΆΠ΅Π½ ΠΏΡƒΡ‚Π°Π½ΠΈΡ†Π΅ / парадоксу, ΠΊΠΎΡ‚ΠΎΡ€Ρ‹ΠΉ прСдставляСт эта ΠΏΡ€ΠΎΠ±Π»Π΅ΠΌΠ°.2 $$

Π­Ρ‚ΠΎ ΠΏΠ°Ρ€Π°Π΄ΠΎΠΊΡΠ°Π»ΡŒΠ½ΠΎ. ΠŸΠΎΡ‡Π΅ΠΌΡƒ Π·Π½Π°Ρ‡Π΅Π½ΠΈΠ΅ ускорСния оказываСтся Π΄Ρ€ΡƒΠ³ΠΈΠΌ, Ссли ΠΌΡ‹ рассматриваСм Ρ‚ΠΎΠ»ΡŒΠΊΠΎ Π²Ρ‚ΠΎΡ€ΠΎΠΉ Π±Π»ΠΎΠΊ Π² нашСй Π·Π°Π΄Π°Ρ‡Π΅?

2.4: РСшСниС ΠΏΡ€ΠΎΠ±Π»Π΅ΠΌ – Π€ΠΈΠ·ΠΈΠΊΠ° LibreTexts

Π”Π°Π»Π΅Π΅ ΠΌΡ‹ сосрСдоточимся Π½Π° элСмСнтах Ρ€Π΅ΡˆΠ΅Π½ΠΈΡ ΠΏΡ€ΠΎΠ±Π»Π΅ΠΌ. Π£ нас Π΅ΡΡ‚ΡŒ всС Π½Π΅ΠΎΠ±Ρ…ΠΎΠ΄ΠΈΠΌΡ‹Π΅ инструмСнты, Ρ‚Π°ΠΊ Ρ‡Ρ‚ΠΎ это Π½Π΅ Π±ΡƒΠ΄Π΅Ρ‚ связано с ΠΊΠ°ΠΊΠΎΠΉ-Π»ΠΈΠ±ΠΎ Π½ΠΎΠ²ΠΎΠΉ Ρ„ΠΈΠ·ΠΈΠΊΠΎΠΉ, Π½ΠΎ идСя состоит Π² Ρ‚ΠΎΠΌ, Ρ‡Ρ‚ΠΎΠ±Ρ‹ ΠΏΠΎΠ·Π½Π°ΠΊΠΎΠΌΠΈΡ‚ΡŒ вас с Π½Π΅ΠΊΠΎΡ‚ΠΎΡ€Ρ‹ΠΌΠΈ ΠΎΠ±Ρ‰ΠΈΠΌΠΈ Ρ‚Π΅ΠΌΠ°ΠΌΠΈ, ΠΊΠΎΡ‚ΠΎΡ€Ρ‹Π΅ Π²ΠΎΠ·Π½ΠΈΠΊΠ°ΡŽΡ‚ Π² Π·Π°Π΄Π°Ρ‡Π°Ρ… Ρ„ΠΈΠ·ΠΈΠΊΠΈ ΠΈ ΠΌΠ΅Ρ…Π°Π½ΠΈΠΊΠΈ.

Π¨ΠΊΠΈΠ²Ρ‹

Π¨ΠΊΠΈΠ² – ΠΎΠ΄Π½ΠΎ ΠΈΠ· Π»ΡŽΠ±ΠΈΠΌΡ‹Ρ… устройств для Ρ€Π΅ΡˆΠ΅Π½ΠΈΡ физичСских Π·Π°Π΄Π°Ρ‡.Как Π±Ρ‹Π»ΠΎ сказано Π² описании силы натяТСния, для Π½Π°Ρ‡Π°Π»Π° ΠΌΡ‹ ΠΈΡΠΏΠΎΠ»ΡŒΠ·ΡƒΠ΅ΠΌ ΠΏΡ€ΠΎΡΡ‚Π΅ΠΉΡˆΡƒΡŽ модСль, Ρ‡Ρ‚ΠΎ ΠΎΠ·Π½Π°Ρ‡Π°Π΅Ρ‚, Ρ‡Ρ‚ΠΎ ΠΌΡ‹ Π±ΡƒΠ΄Π΅ΠΌ ΠΏΡ€Π΅Π΄ΠΏΠΎΠ»Π°Π³Π°Ρ‚ΡŒ, Ρ‡Ρ‚ΠΎ ΡˆΠΊΠΈΠ²Ρ‹ бСзмассовыС ΠΈ Π±Π΅Π· трСния. Π¨ΠΊΠΈΠ²Ρ‹ становятся особСнно интСрСсными Π² ситуациях, ΠΏΠΎΠ΄ΠΎΠ±Π½Ρ‹Ρ… ΡΠ»Π΅Π΄ΡƒΡŽΡ‰Π΅ΠΌΡƒ ΠΏΡ€ΠΈΠΌΠ΅Ρ€Ρƒ, ΠΊΠΎΠ³Π΄Π° хотя Π±Ρ‹ ΠΎΠ΄ΠΈΠ½ ΠΈΠ· шкивов ΠΌΠΎΠΆΠ΅Ρ‚ Π΄Π²ΠΈΠ³Π°Ρ‚ΡŒΡΡ. Π”Π²Π° Π±Π»ΠΎΠΊΠ° ΠΎΡΡ‚Π°ΡŽΡ‚ΡΡ Π½Π΅ΠΏΠΎΠ΄Π²ΠΈΠΆΠ½Ρ‹ΠΌΠΈ Π² систСмС тросов ΠΈ шкивов, ΠΏΠΎΠΊΠ°Π·Π°Π½Π½ΠΎΠΉ Π½Π° схСмС. Учитывая эту ΠΈΠ½Ρ„ΠΎΡ€ΠΌΠ°Ρ†ΠΈΡŽ, ΠΌΠΎΠΆΠ΅Ρ‚Π΅ Π»ΠΈ Π²Ρ‹ ΡΠ΄Π΅Π»Π°Ρ‚ΡŒ Π²Ρ‹Π²ΠΎΠ΄, ΠΊΠ°ΠΊ ΡΡ€Π°Π²Π½ΠΈΠ²Π°ΡŽΡ‚ΡΡ Π΄Π²Π΅ массы?

Рисунок 2.4.1 – Π‘Π»ΠΎΠΊΠΈ, ΠΏΠΎΠ΄Π²Π΅ΡˆΠ΅Π½Π½Ρ‹Π΅ Π½Π° Π½Π΅ΡΠΊΠΎΠ»ΡŒΠΊΠΈΡ… ΡˆΠΊΠΈΠ²Π°Ρ…

К настоящСму Π²Ρ€Π΅ΠΌΠ΅Π½ΠΈ ΠΌΡ‹ Π·Π½Π°Π΅ΠΌ, Ρ‡Ρ‚ΠΎ ΠΊΠΎΠ³Π΄Π° Π΄Π΅Π»ΠΎ Π΄ΠΎΡ…ΠΎΠ΄ΠΈΡ‚ Π΄ΠΎ Π°Π½Π°Π»ΠΈΠ·Π° сил, ΠΏΡ€ΠΈΡΡƒΡ‚ΡΡ‚Π²ΡƒΡŽΡ‰ΠΈΡ… Π² систСмС, Π½Π΅Ρ‚ Π»ΡƒΡ‡ΡˆΠ΅Π³ΠΎ инструмСнта, Ρ‡Π΅ΠΌ FBD.НачнСм Ρ‚Π°ΠΌ:

Рисунок 2.4.2 – FBD Π±Π»ΠΎΠΊΠ° ΠΈ шкива

[ΠœΡ‹ взяли Π½Π° сСбя ΡΠΌΠ΅Π»ΠΎΡΡ‚ΡŒ ΠΎΠΏΡ€Π΅Π΄Π΅Π»ΡΡ‚ΡŒ систСмы ΠΊΠΎΠΎΡ€Π΄ΠΈΠ½Π°Ρ‚ Π² Π½Π°ΡˆΠΈΡ… FBD – Π²Π²Π΅Ρ€Ρ… это Π½Π°ΠΏΡ€Π°Π²Π»Π΅Π½ΠΈΠ΅ \ (+ y \) – для ΠΎΠ±ΠΎΠΈΡ… – Ρ‡Ρ‚ΠΎ Π½Π°ΠΌ понадобится Π² блиТайшСС врСмя.]

ΠœΠΎΠΆΠ΅Ρ‚ Π²ΠΎΠ·Π½ΠΈΠΊΠ½ΡƒΡ‚ΡŒ вопрос, ΠΏΠΎΡ‡Π΅ΠΌΡƒ для шкива нарисовано Π΄Π²Π° Π²Π΅ΠΊΡ‚ΠΎΡ€Π° силы натяТСния. Π‘Π°ΠΌΡ‹ΠΉ простой ΠΎΡ‚Π²Π΅Ρ‚ – ΠΏΠΎΠ΄ΡƒΠΌΠ°Ρ‚ΡŒ ΠΎ Ρ‚ΠΎΠΌ, Ρ‡Ρ‚ΠΎ Π²Ρ‹ почувствуСтС, Ссли ΠΏΠ΅Ρ€Π΅Ρ€Π΅ΠΆΠ΅Ρ‚Π΅ Π²Π΅Ρ€Π΅Π²ΠΊΡƒ с ΠΎΠ±Π΅ΠΈΡ… сторон шкива ΠΈ Π΄Π΅Ρ€ΠΆΠΈΡ‚Π΅ ΠΏΠΎ ΠΎΠ΄Π½ΠΎΠΌΡƒ ΠΊΠΎΠ½Ρ†Ρƒ Π² ΠΊΠ°ΠΆΠ΄ΠΎΠΉ Ρ€ΡƒΠΊΠ΅. ΠžΡ‡Π΅Π²ΠΈΠ΄Π½ΠΎ, Π²Ρ‹ почувствуСтС, ΠΊΠ°ΠΊ ΠΎΠ±Π° ΠΊΠΎΠ½Ρ†Π° Π²Π΅Ρ€Π΅Π²ΠΊΠΈ тянут Π²Π½ΠΈΠ·.Π‘Π»Π΅Π΄ΠΎΠ²Π°Ρ‚Π΅Π»ΡŒΠ½ΠΎ, согласно Ρ‚Ρ€Π΅Ρ‚ΡŒΠ΅ΠΌΡƒ Π·Π°ΠΊΠΎΠ½Ρƒ ΠΡŒΡŽΡ‚ΠΎΠ½Π°, ΠΎΠ±Π° ΠΊΠΎΠ½Ρ†Π° Π²Π΅Ρ€Π΅Π²ΠΊΠΈ Π½Π°Ρ‚ΡΠ³ΠΈΠ²Π°ΡŽΡ‚ΡΡ Π½Π° шкивС. Для шкива Π±Π΅Π· массы ΠΈ Π±Π΅Π· трСния эти Π΄Π²Π΅ силы натяТСния Ρ‚Π°ΠΊΠΆΠ΅ Π΄ΠΎΠ»ΠΆΠ½Ρ‹ Π±Ρ‹Ρ‚ΡŒ Ρ€Π°Π²Π½Ρ‹, Ρ‡Ρ‚ΠΎ ΠΎΠ±ΡŠΡΡΠ½ΡΠ΅Ρ‚, ΠΏΠΎΡ‡Π΅ΠΌΡƒ ΠΎΠ½ΠΈ ΠΎΠ±ΠΎΠ·Π½Π°Ρ‡Π΅Π½Ρ‹ ΠΎΠ΄ΠΈΠ½Π°ΠΊΠΎΠ²ΠΎ. ΠžΠ±Ρ€Π°Ρ‚ΠΈΡ‚Π΅ Π²Π½ΠΈΠΌΠ°Π½ΠΈΠ΅, Ρ‡Ρ‚ΠΎ Π²Π΅ΠΊΡ‚ΠΎΡ€ натяТСния Π½Π° Π±Π»ΠΎΠΊΠ΅ Ρ‚Π°ΠΊΠΆΠ΅ ΠΏΠΎΠΌΠ΅Ρ‡Π΅Π½ Ρ‚Π΅ΠΌ ΠΆΠ΅ ΠΈΠΌΠ΅Π½Π΅ΠΌ ΠΏΠ΅Ρ€Π΅ΠΌΠ΅Π½Π½ΠΎΠΉ. Π­Ρ‚ΠΎ ΠΏΠΎΡ‚ΠΎΠΌΡƒ, Ρ‡Ρ‚ΠΎ это Ρ‚Π° ΠΆΠ΅ Π²Π΅Ρ€Π΅Π²ΠΊΠ° , , ΠΈ нашС ΠΏΡ€Π΅Π΄ΠΏΠΎΠ»ΠΎΠΆΠ΅Π½ΠΈΠ΅ ΠΎ бСзмассовых ΡˆΠΊΠΈΠ²Π°Ρ… Π±Π΅Π· трСния Π³Π°Ρ€Π°Π½Ρ‚ΠΈΡ€ΡƒΠ΅Ρ‚, Ρ‡Ρ‚ΠΎ Π²Π΅Π·Π΄Π΅, Π³Π΄Π΅ ΠΌΡ‹ измСряСм натяТСниС ΠΎΠ΄Π½ΠΎΠ³ΠΎ куска Π²Π΅Ρ€Π΅Π²ΠΊΠΈ, ΠΎΠ½ΠΎ Π±ΡƒΠ΄Π΅Ρ‚ ΠΎΠ΄ΠΈΠ½Π°ΠΊΠΎΠ²Ρ‹ΠΌ.

ΠŸΡ€Π΅Π΄ΡƒΠΏΡ€Π΅ΠΆΠ΄Π΅Π½ΠΈΠ΅

Если Π±Ρ‹ ΠΌΡ‹ ΠΈΠ·ΠΎΠ±Ρ€Π°Π·ΠΈΠ»ΠΈ Π²Π΅ΠΊΡ‚ΠΎΡ€ силы натяТСния, тянущий Π²Π²Π΅Ρ€Ρ… Π½Π° ΠΏΡ€Π°Π²ΠΎΠΌ шкивС, ΠΌΡ‹ Π½Π΅ смогли Π±Ρ‹ ΠΎΠ±ΠΎΠ·Π½Π°Ρ‡ΠΈΡ‚ΡŒ Π΅Π³ΠΎ Ρ‚Π°ΠΊ ΠΆΠ΅. НС всС Π²Π΅ΠΊΡ‚ΠΎΡ€Ρ‹ натяТСния Π² ΠΎΠ΄Π½ΠΎΠΉ физичСской систСмС Ρ€Π°Π²Π½Ρ‹, Ρ‚ΠΎΠ»ΡŒΠΊΠΎ Π²Π΅Π»ΠΈΡ‡ΠΈΠ½Ρ‹ всСх Π²Π΅ΠΊΡ‚ΠΎΡ€ΠΎΠ² натяТСния, ΠΏΠΎΠ»ΡƒΡ‡Π΅Π½Π½Ρ‹Π΅ ΠΎΡ‚ ΠΎΠ΄Π½ΠΎΠΉ ΠΈ Ρ‚ΠΎΠΉ ΠΆΠ΅ Π²Π΅Ρ€Π΅Π²ΠΊΠΈ.

Π•Ρ‰Π΅ ΠΎΠ΄ΠΈΠ½ Π»ΡŽΠ±ΠΎΠΏΡ‹Ρ‚Π½Ρ‹ΠΉ аспСкт этого FBD – это вСсовая этикСтка Π»Π΅Π²ΠΎΠ³ΠΎ шкива. ВСхничСски эта сила дСйствуСт Π½Π° Π±Π»ΠΎΠΊ, ΠΈ Π±Π»ΠΎΠΊ тянСт Π·Π° шкив. Π’ этом случаС сила натяТСния шкива Π±Π»ΠΎΠΊΠΎΠΌ Ρ€Π°Π²Π½Π° вСсу Π±Π»ΠΎΠΊΠ°, Π° сам шкив Π½Π΅ ΠΈΠΌΠ΅Π΅Ρ‚ собствСнного вСса, Ρ‚Π°ΠΊ Ρ‡Ρ‚ΠΎ ΠΌΡ‹ Π²ΠΏΡ€Π°Π²Π΅ ΠΏΠΎΠΉΡ‚ΠΈ ΠΏΠΎ этому ΠΏΡƒΡ‚ΠΈ.Π”Ρ€ΡƒΠ³ΠΎΠΉ способ ΠΎΠ±ΠΎΡΠ½ΠΎΠ²Π°Ρ‚ΡŒ это – Ρ€Π°ΡΡΠΌΠ°Ρ‚Ρ€ΠΈΠ²Π°Ρ‚ΡŒ Π±Π»ΠΎΠΊ + шкив ΠΊΠ°ΠΊ Π΅Π΄ΠΈΠ½ΡƒΡŽ систСму, Π° сила тяТСсти, Π΄Π΅ΠΉΡΡ‚Π²ΡƒΡŽΡ‰Π°Ρ Π½Π° систСму, – это ΠΏΠΎΠΊΠ°Π·Π°Π½Π½Ρ‹ΠΉ Π²Π΅ΠΊΡ‚ΠΎΡ€ силы.

Π‘Π»Π΅Π΄ΡƒΡŽΡ‰ΠΈΠΌ шагом Π² нашСм Π°Π½Π°Π»ΠΈΠ·Π΅ являСтся суммированиС сил для ΠΊΠ°ΠΆΠ΄ΠΎΠ³ΠΎ ΠΎΠ±ΡŠΠ΅ΠΊΡ‚Π° ΠΈ ΠΏΡ€ΠΈΠΌΠ΅Π½Π΅Π½ΠΈΠ΅ Π²Ρ‚ΠΎΡ€ΠΎΠ³ΠΎ Π·Π°ΠΊΠΎΠ½Π° ΠΡŒΡŽΡ‚ΠΎΠ½Π°, ΠΊΠΎΡ‚ΠΎΡ€Ρ‹ΠΉ Π² Π΄Π°Π½Π½ΠΎΠΌ случаС ΠΏΡ€Π΅Π΄ΠΏΠΎΠ»Π°Π³Π°Π΅Ρ‚ Π½ΡƒΠ»Π΅Π²ΠΎΠ΅ ускорСниС. ΠŸΡ€ΠΈ вычислСнии суммы сил ΠΌΡ‹ Π΄ΠΎΠ»ΠΆΠ½Ρ‹ ΠΏΠΎΠ·Π°Π±ΠΎΡ‚ΠΈΡ‚ΡŒΡΡ ΠΎ Ρ‚ΠΎΠΌ, Ρ‡Ρ‚ΠΎΠ±Ρ‹ ΠΏΡ€Π°Π²ΠΈΠ»ΡŒΠ½ΠΎ ΠΈΡΠΏΠΎΠ»ΡŒΠ·ΠΎΠ²Π°Ρ‚ΡŒ Π½Π°ΡˆΡƒ систСму ΠΊΠΎΠΎΡ€Π΄ΠΈΠ½Π°Ρ‚:

\ [\ left. \ begin {array} {l} 0 = a_1 = \ dfrac {F_ {net \; 1}} {m_1} = \ dfrac {2T – m_1 g} {m_1} \; \; \; \ΠŸΡ€Π°Π²Π°Ρ стрСлка \;\;\; T = \ dfrac {m_1 g} {2} \\ 0 = a_2 = \ dfrac {F_ {net \; 2}} {m_2} = \ dfrac {T – m_2 g} {m_2} \; \; \; \; \ΠŸΡ€Π°Π²Π°Ρ стрСлка \;\;\; T = m_2 g \\ \ end {array} \ right \} \; \; \; \ΠŸΡ€Π°Π²Π°Ρ стрСлка \;\;\; ΠΌ_1 = 2 ΠΌ_2 \]

ΠžΠ±Ρ€Π°Ρ‚ΠΈΡ‚Π΅ Π²Π½ΠΈΠΌΠ°Π½ΠΈΠ΅, Ρ‡Ρ‚ΠΎ Π»Π΅Π³ΠΊΠΈΠΉ \ (m_2 \) Π²Ρ‹Π΄Π΅Ρ€ΠΆΠΈΠ²Π°Π΅Ρ‚ Π±ΠΎΠ»Π΅Π΅ тяТСлый, ΠΏΠΎΡ‚ΠΎΠΌΡƒ Ρ‡Ρ‚ΠΎ Ρ€Π°Π·ΠΌΠ΅Ρ‰Π΅Π½ΠΈΠ΅ шкива позволяСт Π½Π°ΠΌ ΠΈΡΠΏΠΎΠ»ΡŒΠ·ΠΎΠ²Π°Ρ‚ΡŒ натяТСниС ΠΎΠ΄Π½ΠΎΠΉ ΠΈ Ρ‚ΠΎΠΉ ΠΆΠ΅ Π²Π΅Ρ€Π΅Π²ΠΊΠΈ Π΄Π²Π°ΠΆΠ΄Ρ‹ Π½Π° Π±ΠΎΠ»Π΅Π΅ Ρ‚ΡΠΆΠ΅Π»ΡƒΡŽ массу.Π­Ρ‚ΠΎΡ‚ Ρ‚Ρ€ΡŽΠΊ ΠΌΠΎΠΆΠ½ΠΎ ΠΏΠΎΠ²Ρ‚ΠΎΡ€ΡΡ‚ΡŒ сколько ΡƒΠ³ΠΎΠ΄Π½ΠΎ Ρ€Π°Π· (Π½Π° шкивС ΠΌΠΎΠΆΠ΅Ρ‚ Π±Ρ‹Ρ‚ΡŒ нСсколько Π΄ΠΎΡ€ΠΎΠΆΠ΅ΠΊ), ΠΈ это позволяСт Π½Π°ΠΌ ΠΏΠΎΠ΄Π½ΠΈΠΌΠ°Ρ‚ΡŒ ΠΎΡ‡Π΅Π½ΡŒ тяТСлыС вСса с ΠΎΡ‡Π΅Π½ΡŒ нСбольшой силой. Π”Π°Π½Π½ΠΎΠ΅ ΠΈΠ·ΠΎΠ±Ρ€Π΅Ρ‚Π΅Π½ΠΈΠ΅ называСтся Π±Π»ΠΎΠΊΠΎΠΌ ΠΈ Π·Π°Ρ…Π²Π°Ρ‚ΠΎΠΌ . Они ΠΈΡΠΏΠΎΠ»ΡŒΠ·ΡƒΡŽΡ‚ΡΡ для плавания Π½Π° парусных судах (тяТСлыС паруса ΠΈ Π³ΠΈΠΊΠ° ΠΌΠΎΠΆΠ½ΠΎ Π·Π°Ρ‚ΡΠ½ΡƒΡ‚ΡŒ сильнСС), подъСма Π±Π»ΠΎΠΊΠΎΠ² Π΄Π²ΠΈΠ³Π°Ρ‚Π΅Π»Π΅ΠΉ ΠΈ ΠΌΠ½ΠΎΠ³ΠΈΡ… Π΄Ρ€ΡƒΠ³ΠΈΡ… Π·Π°Π΄Π°Ρ‡.

ΠžΠ³Ρ€Π°Π½ΠΈΡ‡Π΅Π½ΠΈΡ

Π—Π°Ρ‚Π΅ΠΌ ΠΌΡ‹ рассмотрим ΡΠ»ΠΎΠΆΠ½ΡƒΡŽ ΠΊΠΎΠ½Ρ†Π΅ΠΏΡ†ΠΈΡŽ, ΠΈΠ·Π²Π΅ΡΡ‚Π½ΡƒΡŽ ΠΊΠ°ΠΊ ΠΎΠ³Ρ€Π°Π½ΠΈΡ‡Π΅Π½ΠΈΠ΅ s. ΠžΠ³Ρ€Π°Π½ΠΈΡ‡Π΅Π½ΠΈΠ΅ – это условиС, ΠΊΠΎΡ‚ΠΎΡ€ΠΎΠ΅ сущСствуСт для физичСской систСмы, ΠΊΠΎΡ‚ΠΎΡ€ΠΎΠ΅ ΠΎΠ³Ρ€Π°Π½ΠΈΡ‡ΠΈΠ²Π°Π΅Ρ‚ Π΅Π΅ ΠΏΠΎΠ²Π΅Π΄Π΅Π½ΠΈΠ΅.Π§Ρ‚ΠΎ Π΄Π΅Π»Π°Π΅Ρ‚ ΠΊΠΎΠ½Ρ†Π΅ΠΏΡ†ΠΈΡŽ «слоТной», Ρ‚Π°ΠΊ это Ρ‚ΠΎ, Ρ‡Ρ‚ΠΎ ΠΎΠ½ΠΈ, Π² ΠΊΠΎΠ½Π΅Ρ‡Π½ΠΎΠΌ счСтС, ΠΈΠ³Ρ€Π°ΡŽΡ‚ ΠΌΠ°Ρ‚Π΅ΠΌΠ°Ρ‚ΠΈΡ‡Π΅ΡΠΊΡƒΡŽ Ρ€ΠΎΠ»ΡŒ Π² Ρ€Π΅ΡˆΠ΅Π½ΠΈΠΈ ΠΏΡ€ΠΎΠ±Π»Π΅ΠΌΡ‹, Π½ΠΎ эту Ρ€ΠΎΠ»ΡŒ часто Ρ‚Ρ€ΡƒΠ΄Π½ΠΎ ΠΈΠ·Π²Π»Π΅Ρ‡ΡŒ ΠΈΠ· постановки Π·Π°Π΄Π°Ρ‡ΠΈ. ΠŸΡ€ΠΎΡ‰Π΅ говоря, ограничСния ΡΠ²ΡΠ·Ρ‹Π²Π°ΡŽΡ‚ ΠΏΠ΅Ρ€Π΅ΠΌΠ΅Π½Π½Ρ‹Π΅ Π² Π·Π°Π΄Π°Ρ‡Π΅ Π΄Ρ€ΡƒΠ³ с Π΄Ρ€ΡƒΠ³ΠΎΠΌ, обСспСчивая Π΄ΠΎΠΏΠΎΠ»Π½ΠΈΡ‚Π΅Π»ΡŒΠ½Ρ‹Π΅ уравнСния (ΠΏΠΎΠΌΠΈΠΌΠΎ Π²Ρ‚ΠΎΡ€ΠΎΠ³ΠΎ Π·Π°ΠΊΠΎΠ½Π° ΠΡŒΡŽΡ‚ΠΎΠ½Π°), с ΠΊΠΎΡ‚ΠΎΡ€Ρ‹ΠΌΠΈ ΠΌΠΎΠΆΠ½ΠΎ Ρ€Π°Π±ΠΎΡ‚Π°Ρ‚ΡŒ. ΠœΡ‹ ΡƒΠΆΠ΅ Π²ΠΈΠ΄Π΅Π»ΠΈ ΠΏΡ€ΠΈΠΌΠ΅Ρ€ ограничСния. Π­Ρ‚ΠΎ ΡΠΎΠΎΡ‚Π½ΠΎΡˆΠ΅Π½ΠΈΠ΅ ΠΌΠ΅ΠΆΠ΄Ρƒ силой трСния ΠΈ Π½ΠΎΡ€ΠΌΠ°Π»ΡŒΠ½ΠΎΠΉ силой. Для кинСтичСского трСния это Π΄Π°Π΅Ρ‚ ΡƒΡ€Π°Π²Π½Π΅Π½ΠΈΠ΅, ΠΊΠΎΡ‚ΠΎΡ€ΠΎΠ΅ связываСт эти Π΄Π²Π΅ силы, Π² Ρ‚ΠΎ врСмя ΠΊΠ°ΠΊ для статичСского трСния ΠΎΠ½ устанавливаСт Π²Π΅Ρ€Ρ…Π½ΠΈΠΉ ΠΏΡ€Π΅Π΄Π΅Π» Π²Π΅Π»ΠΈΡ‡ΠΈΠ½Ρ‹ трСния для Π΄Π°Π½Π½ΠΎΠΉ Π½ΠΎΡ€ΠΌΠ°Π»ΡŒΠ½ΠΎΠΉ силы.

Один ΠΈΠ· Π½Π°ΠΈΠ±ΠΎΠ»Π΅Π΅ распространСнных ΠΏΡ€ΠΈΠΌΠ΅Ρ€ΠΎΠ² ограничСния относится ΠΊ ΠΊΠ°Π½Π°Ρ‚Π°ΠΌ, двиТущимся Ρ‡Π΅Ρ€Π΅Π· ΡˆΠΊΠΈΠ²Ρ‹. Π­Ρ‚ΠΎ ΠΎΠ³Ρ€Π°Π½ΠΈΡ‡Π΅Π½ΠΈΠ΅ связываСт Π΄Π²ΠΈΠΆΠ΅Π½ΠΈΠ΅ ΠΎΠ΄Π½ΠΎΠ³ΠΎ ΠΎΠ±ΡŠΠ΅ΠΊΡ‚Π° с Π΄Π²ΠΈΠΆΠ΅Π½ΠΈΠ΅ΠΌ Π΄Ρ€ΡƒΠ³ΠΎΠ³ΠΎ, ΠΊΠΎΠ³Π΄Π° ΠΎΠ½ΠΈ соСдинСны систСмой шкивов. ВСрнСмся ΠΊ систСмС, ΠΏΠΎΠΊΠ°Π·Π°Π½Π½ΠΎΠΉ Π½Π° рисункС 2.4.1, ΠΈ Π·Π°Π΄Π°Π΄ΠΈΠΌ ΡΠ»Π΅Π΄ΡƒΡŽΡ‰ΠΈΠΉ вопрос: Ссли Π±Π»ΠΎΠΊ \ (m_2 \) ΠΏΠ°Π΄Π°Π΅Ρ‚ Π½Π° расстояниС \ (\ Delta y \), Ρ‡Ρ‚ΠΎ происходит с Π±Π»ΠΎΠΊΠΎΠΌ \ (m_1 \)?

ΠŸΡ€Π΅ΠΆΠ΄Π΅ всСго, Π΄ΠΎΠ»ΠΆΠ½ΠΎ Π±Ρ‹Ρ‚ΡŒ ясно, Ρ‡Ρ‚ΠΎ \ (m_1 \) возрастаСт ΠΏΠΎ ΠΌΠ΅Ρ€Π΅ Ρ‚ΠΎΠ³ΠΎ, ΠΊΠ°ΠΊ \ (m_2 \) ΠΏΠ°Π΄Π°Π΅Ρ‚, поэтому вопрос Ρ‚ΠΎΠ»ΡŒΠΊΠΎ Π² Ρ‚ΠΎΠΌ, ΠΊΠ°ΠΊ Π΄Π°Π»Π΅ΠΊΠΎ? Π‘Π½Π°Ρ‡Π°Π»Π° это ΠΌΠΎΠΆΠ΅Ρ‚ Π±Ρ‹Ρ‚ΡŒ Π½Π΅ΠΎΡ‡Π΅Π²ΠΈΠ΄Π½Ρ‹ΠΌ, Π½ΠΎ ΠΏΠΎΠ΄ΡƒΠΌΠ°ΠΉΡ‚Π΅ ΠΎΠ± этом Ρ‚Π°ΠΊ: ΠΊΠΎΠ³Π΄Π° шкив, ΡƒΠ΄Π΅Ρ€ΠΆΠΈΠ²Π°ΡŽΡ‰ΠΈΠΉ \ (m_1 \), поднимаСтся Π½Π° 1 Π΅Π΄ΠΈΠ½ΠΈΡ†Ρƒ, ΠΎΠ±Π° сСгмСнта струны, ΠΈΠ΄ΡƒΡ‰ΠΈΠ΅ Π²Π²Π΅Ρ€Ρ… ΠΎΡ‚ шкива, становятся ΠΊΠΎΡ€ΠΎΡ‡Π΅ Π½Π° 1 Π΅Π΄ΠΈΠ½ΠΈΡ†Ρƒ.Π­Ρ‚ΠΈ Π΄Π²Π΅ Π΅Π΄ΠΈΠ½ΠΈΡ†Ρ‹ строки Π½Π΅ просто ΠΈΡΡ‡Π΅Π·Π°ΡŽΡ‚, ΠΈ фактичСски ΠΎΠ½ΠΈ ΠΏΡ€ΠΈΠ½ΠΈΠΌΠ°ΡŽΡ‚ΡΡ свободным ΠΊΠΎΠ½Ρ†ΠΎΠΌ строки, ΠΊΠΎΡ‚ΠΎΡ€Ρ‹ΠΉ ΠΏΡ€ΠΈΠΊΡ€Π΅ΠΏΠ»Π΅Π½ ΠΊ \ (m_2 \). Π­Ρ‚ΠΎ ΠΎΠ·Π½Π°Ρ‡Π°Π΅Ρ‚, Ρ‡Ρ‚ΠΎ ΠΊΠΎΠ³Π΄Π° \ (m_2 \) ΠΏΠ°Π΄Π°Π΅Ρ‚ Π½Π° расстояниС \ (\ Delta y \), \ (m_1 \) Π΄ΠΎΠ»ΠΆΠ΅Π½ ΠΏΠΎΠ΄Π½ΠΈΠΌΠ°Ρ‚ΡŒΡΡ Ρ‚ΠΎΠ»ΡŒΠΊΠΎ Π½Π° ΠΏΠΎΠ»ΠΎΠ²ΠΈΠ½Ρƒ этого расстояния.

Π§Ρ‚ΠΎ это Π³ΠΎΠ²ΠΎΡ€ΠΈΡ‚ ΠΎ сравнСнии скоростСй ΠΈ ускорСний Π΄Π²ΡƒΡ… Π±Π»ΠΎΠΊΠΎΠ²? Π§Ρ‚ΠΎ ΠΆ, ΠΎΠ½ΠΈ Π΄ΠΎΠ»ΠΆΠ½Ρ‹ Π΄Π²ΠΈΠ³Π°Ρ‚ΡŒΡΡ ΠΎΠ΄Π½ΠΎΠ²Ρ€Π΅ΠΌΠ΅Π½Π½ΠΎ, поэтому каТдая Π΅Π΄ΠΈΠ½ΠΈΡ†Π° Π΄Π»ΠΈΠ½Ρ‹, ΠΎΡ‚Π±Ρ€ΠΎΡˆΠ΅Π½Π½Π°Ρ Π½Π° \ (m_2 \), соотвСтствуСт ΡƒΠ²Π΅Π»ΠΈΡ‡Π΅Π½ΠΈΡŽ \ (m_1 \) Π½Π° ΠΏΠΎΠ»ΠΎΠ²ΠΈΠ½Ρƒ, Π° это ΠΎΠ·Π½Π°Ρ‡Π°Π΅Ρ‚, Ρ‡Ρ‚ΠΎ \ (m_1 \) всСгда пСрСмСщаСтся Π½Π° ΠΏΠΎΠ»ΠΎΠ²ΠΈΠ½Ρƒ ΡΠΊΠΎΡ€ΠΎΡΡ‚ΡŒ ΠΈ разгоняСтся Π²Π΄Π²ΠΎΠ΅ мСньшС, Ρ‡Π΅ΠΌ \ (m_2 \).Если эта систСма Π½Π΅ сбалансирована (ΠΊΠ°ΠΊ это Π±Ρ‹Π»ΠΎ Π²Ρ‹ΡˆΠ΅), Ρ‚ΠΎ ΠΏΡ€ΠΈΠΌΠ΅Π½Π΅Π½ΠΈΠ΅ Π²Ρ‚ΠΎΡ€ΠΎΠ³ΠΎ Π·Π°ΠΊΠΎΠ½Π° ΠΡŒΡŽΡ‚ΠΎΠ½Π° ΠΊ ΠΎΠ±ΠΎΠΈΠΌ Π±Π»ΠΎΠΊΠ°ΠΌ Π²ΠΊΠ»ΡŽΡ‡Π°Π΅Ρ‚ Π² сСбя Π΄Π²Π° ускорСния, Π½ΠΎ ΠΎΠ½ΠΈ ΠΎΠ³Ρ€Π°Π½ΠΈΡ‡Π΅Π½Ρ‹, связаны Π΄Ρ€ΡƒΠ³ с Π΄Ρ€ΡƒΠ³ΠΎΠΌ Π² Π΄Π²Π° Ρ€Π°Π·Π°, Ρ‡Ρ‚ΠΎ Π΄Π°Π΅Ρ‚ Π½Π°ΠΌ Π΄ΠΎΠΏΠΎΠ»Π½ΠΈΡ‚Π΅Π»ΡŒΠ½ΠΎΠ΅ ΠΎΠ³Ρ€Π°Π½ΠΈΡ‡Π΅Π½ΠΈΠ΅ ΡƒΡ€Π°Π²Π½Π΅Π½ΠΈΠ΅ :

\ [2 \ ΠΎΡΡ‚Π°Π»ΠΎΡΡŒ | a_1 \ right | = \ left | a_2 \ справа | \]

Π§Ρ‚ΠΎ с Π°Π±ΡΠΎΠ»ΡŽΡ‚Π½Ρ‹ΠΌΠΈ значСниями, спроситС Π²Ρ‹? Π§Ρ‚ΠΎ ΠΆ, эти ΠΏΠ΅Ρ€Π΅ΠΌΠ΅Π½Π½Ρ‹Π΅ ΠΌΠΎΠ³ΡƒΡ‚ ΠΈΠΌΠ΅Ρ‚ΡŒ ΠΏΠΎΠ»ΠΎΠΆΠΈΡ‚Π΅Π»ΡŒΠ½Ρ‹Π΅ ΠΈΠ»ΠΈ ΠΎΡ‚Ρ€ΠΈΡ†Π°Ρ‚Π΅Π»ΡŒΠ½Ρ‹Π΅ значСния, ΠΈ ΠΌΡ‹ Π΄ΠΎΠ»ΠΆΠ½Ρ‹ Π±Ρ‹Ρ‚ΡŒ остороТны, ΠΊΠΎΠ³Π΄Π° Π΄Π΅Π»ΠΎ касаСтся Π·Π½Π°ΠΊΠΎΠ².Π² частности, ΠΌΡ‹ Π΄ΠΎΠ»ΠΆΠ½Ρ‹ ΠΏΠΎΡΠΌΠΎΡ‚Ρ€Π΅Ρ‚ΡŒ, ΠΊΠ°ΠΊ нашС ΠΎΠ³Ρ€Π°Π½ΠΈΡ‡Π΅Π½ΠΈΠ΅ связано с нашим Π²Ρ‹Π±ΠΎΡ€ΠΎΠΌ систСм ΠΊΠΎΠΎΡ€Π΄ΠΈΠ½Π°Ρ‚ для Π΄Π²ΡƒΡ… Π±Π»ΠΎΠΊΠΎΠ². На рисункС 2.4.2 ΠΌΡ‹ Π²Ρ‹Π±Ρ€Π°Π»ΠΈ Β«Π²Π²Π΅Ρ€Ρ…Β» Π² качСствС ΠΏΠΎΠ»ΠΎΠΆΠΈΡ‚Π΅Π»ΡŒΠ½ΠΎΠ³ΠΎ направлСния для ΠΎΠ±ΠΎΠΈΡ… Π±Π»ΠΎΠΊΠΎΠ². ΠŸΠΎΡΡ‚ΠΎΠΌΡƒ Π½Π°ΠΌ Π½ΡƒΠΆΠ½ΠΎ ΡΠΏΡ€ΠΎΡΠΈΡ‚ΡŒ сСбя: «Если ΠΎΠ΄ΠΈΠ½ Π±Π»ΠΎΠΊ испытываСт ΠΏΠΎΠ»ΠΎΠΆΠΈΡ‚Π΅Π»ΡŒΠ½ΠΎΠ΅ смСщСниС, ΠΊΠ°ΠΊΠΎΠ² Π·Π½Π°ΠΊ смСщСния Π΄Ρ€ΡƒΠ³ΠΎΠ³ΠΎ Π±Π»ΠΎΠΊΠ°?Β» Π’ этом случаС ясно, Ρ‡Ρ‚ΠΎ смСщСния Π΄Π²ΡƒΡ… Π±Π»ΠΎΠΊΠΎΠ² ΠΈΠΌΠ΅ΡŽΡ‚ ΠΏΡ€ΠΎΡ‚ΠΈΠ²ΠΎΠΏΠΎΠ»ΠΎΠΆΠ½Ρ‹Π΅ Π·Π½Π°ΠΊΠΈ. Π‘Π»Π΅Π΄ΠΎΠ²Π°Ρ‚Π΅Π»ΡŒΠ½ΠΎ, ΡƒΡ€Π°Π²Π½Π΅Π½ΠΈΠ΅ ограничСния для ускорСний Π±Π»ΠΎΠΊΠ°:

\ [2a_1 = -a_2 \]

ΠžΠ±Ρ€Π°Ρ‚ΠΈΡ‚Π΅ Π²Π½ΠΈΠΌΠ°Π½ΠΈΠ΅, Ρ‡Ρ‚ΠΎ ΡΠΎΠ²Π΅Ρ€ΡˆΠ΅Π½Π½ΠΎ Π½ΠΎΡ€ΠΌΠ°Π»ΡŒΠ½ΠΎ ΡƒΡΡ‚Π°Π½ΠΎΠ²ΠΈΡ‚ΡŒ Ρ€Π°Π·Π½Ρ‹Π΅ систСмы ΠΊΠΎΠΎΡ€Π΄ΠΈΠ½Π°Ρ‚ для Π΄Π²ΡƒΡ… Π±Π»ΠΎΠΊΠΎΠ² – ΠΊΠ°ΠΆΠ΄Ρ‹ΠΉ FBD ΠΈΠΌΠ΅Π΅Ρ‚ ΠΏΡ€Π°Π²ΠΎ Π½Π° свою ΡΠΎΠ±ΡΡ‚Π²Π΅Π½Π½ΡƒΡŽ ΠΈΠ½Π΄ΠΈΠ²ΠΈΠ΄ΡƒΠ°Π»ΡŒΠ½ΡƒΡŽ систСму ΠΊΠΎΠΎΡ€Π΄ΠΈΠ½Π°Ρ‚.Π’ΠΎ, ΠΊΠ°ΠΊ систСмы ΠΊΠΎΠΎΡ€Π΄ΠΈΠ½Π°Ρ‚ соотносятся Π΄Ρ€ΡƒΠ³ с Π΄Ρ€ΡƒΠ³ΠΎΠΌ, влияСт Π½Π° ΡƒΡ€Π°Π²Π½Π΅Π½ΠΈΠ΅ связи. Π’Π°ΠΊ, Π½Π°ΠΏΡ€ΠΈΠΌΠ΅Ρ€, Ссли Π±Ρ‹ ΠΌΡ‹ вмСсто этого Π²Ρ‹Π±Ρ€Π°Π»ΠΈ нисходящСС Π½Π°ΠΏΡ€Π°Π²Π»Π΅Π½ΠΈΠ΅ + \ (y \) – для Π±Π»ΠΎΠΊΠ° β„– 2 (Π½ΠΎ оставили Π²Π²Π΅Ρ€Ρ… ΠΊΠ°ΠΊ ΠΏΠΎΠ»ΠΎΠΆΠΈΡ‚Π΅Π»ΡŒΠ½ΠΎΠ΅ Π·Π½Π°Ρ‡Π΅Π½ΠΈΠ΅ для Π΄Ρ€ΡƒΠ³ΠΎΠ³ΠΎ Π±Π»ΠΎΠΊΠ°), Ρ‚ΠΎΠ³Π΄Π° Π½Π΅ Π±Ρ‹Π»ΠΎ Π±Ρ‹ нСобходимости Π² Π·Π½Π°ΠΊΠ΅ минус Π² ΠΎΠ³Ρ€Π°Π½ΠΈΡ‡Π΅Π½ΠΈΠΈ ΡƒΡ€Π°Π²Π½Π΅Π½ΠΈΠ΅ – ΠΏΠΎΠ»ΠΎΠΆΠΈΡ‚Π΅Π»ΡŒΠ½Ρ‹Π΅ смСщСния ΠΎΠ΄Π½ΠΎΠ³ΠΎ Π±Π»ΠΎΠΊΠ° ΡΠΎΠΎΡ‚Π²Π΅Ρ‚ΡΡ‚Π²ΡƒΡŽΡ‚ ΠΏΠΎΠ»ΠΎΠΆΠΈΡ‚Π΅Π»ΡŒΠ½Ρ‹ΠΌ смСщСниям Π΄Ρ€ΡƒΠ³ΠΎΠ³ΠΎ Π±Π»ΠΎΠΊΠ°. ΠœΡ‹ Π²ΠΈΠ΄ΠΈΠΌ, Ρ‡Ρ‚ΠΎ, ΡΠ»Π΅Π΄ΠΎΠ²Π°Ρ‚Π΅Π»ΡŒΠ½ΠΎ, Π½Π΅Ρ‚ Β«ΠΏΡ€Π°Π²ΠΈΠ»ΡŒΠ½ΠΎΠ³ΠΎΒ» Π²Ρ‹Π±ΠΎΡ€Π° систСмы ΠΊΠΎΠΎΡ€Π΄ΠΈΠ½Π°Ρ‚, Π½ΠΎ ΠΌΡ‹ Π΄ΠΎΠ»ΠΆΠ½Ρ‹ ΠΏΠΎΠ·Π°Π±ΠΎΡ‚ΠΈΡ‚ΡŒΡΡ ΠΎ Ρ‚ΠΎΠΌ, Ρ‡Ρ‚ΠΎΠ±Ρ‹ ΠΊΠΎΠ³Π΄Π° ΠΏΡ€ΠΈΠ΄Π΅Ρ‚ врСмя ΠΎΠ±ΡŠΠ΅Π΄ΠΈΠ½ΠΈΡ‚ΡŒ уравнСния ΠΈΠ· Π΄Π²ΡƒΡ… FBD, Ρ‡Ρ‚ΠΎΠ±Ρ‹ уравнСния ΠΎΠ³Ρ€Π°Π½ΠΈΡ‡Π΅Π½ΠΈΠΉ ΠΏΡ€Π°Π²ΠΈΠ»ΡŒΠ½ΠΎ связывали ΠΏΠ΅Ρ€Π΅ΠΌΠ΅Π½Π½Ρ‹Π΅.Π’ ΡΠ»Π΅Π΄ΡƒΡŽΡ‰Π΅ΠΌ ΠΏΡ€ΠΈΠΌΠ΅Ρ€Π΅ ΠΌΡ‹ ΡƒΠ²ΠΈΠ΄ΠΈΠΌ Π΅Ρ‰Π΅ Π±ΠΎΠ»Π΅Π΅ яркий ΠΏΡ€ΠΈΠΌΠ΅Ρ€ Π²Ρ‹Π±ΠΎΡ€Π° систСмы ΠΊΠΎΠΎΡ€Π΄ΠΈΠ½Π°Ρ‚.

ΠŸΡ€ΠΈΠΌΠ΅Ρ€ \ (\ PageIndex {1} \)

Высота Π΄Π²ΡƒΡ… Π±Π»ΠΎΠΊΠΎΠ² Π½Π° Π΄ΠΈΠ°Π³Ρ€Π°ΠΌΠΌΠ΅ Π½ΠΈΠΆΠ΅ отличаСтся Π½Π° 36 см. Когда ΠΎΠ½ΠΈ выходят ΠΈΠ· состояния покоя, Π²Π΅Ρ€Ρ…Π½ΠΈΠΉ Π±Π»ΠΎΠΊ ΠΏΠ°Π΄Π°Π΅Ρ‚, Π° Π½ΠΈΠΆΠ½ΠΈΠΉ Π±Π»ΠΎΠΊ поднимаСтся. Один ΠΈΠ· Π±Π»ΠΎΠΊΠΎΠ² ΠΈΠΌΠ΅Π΅Ρ‚ массу, Π² Ρ‚Ρ€ΠΈ Ρ€Π°Π·Π° ΠΏΡ€Π΅Π²Ρ‹ΡˆΠ°ΡŽΡ‰ΡƒΡŽ массу Π΄Ρ€ΡƒΠ³ΠΎΠ³ΠΎ Π±Π»ΠΎΠΊΠ°, ΡˆΠΊΠΈΠ²Ρ‹ бСзмассовыС ΠΈ Π±Π΅Π· трСния, Π° струна Π½Π΅ растягиваСтся.

  1. Какой Π±Π»ΠΎΠΊ тяТСлСС? ΠžΠ±ΡŠΡΡΠ½ΡΡ‚ΡŒ.
  2. НайдитС расстояниС, Π½Π° ΠΊΠΎΡ‚ΠΎΡ€ΠΎΠ΅ поднимаСтся Π½ΠΈΠΆΠ½ΠΈΠΉ Π±Π»ΠΎΠΊ, ΠΊΠΎΠ³Π΄Π° Π΄Π²Π° Π±Π»ΠΎΠΊΠ° Π²Ρ‹Ρ€ΠΎΠ²Π½Π΅Π½Ρ‹.
  3. НайдитС врСмя, Π½Π΅ΠΎΠ±Ρ…ΠΎΠ΄ΠΈΠΌΠΎΠ΅ для выравнивания Π΄Π²ΡƒΡ… Π±Π»ΠΎΠΊΠΎΠ².
    РСшСниС

    Π°. НатяТСниС струны, ΠΏΡ€ΠΎΠ΄Π΅Ρ‚ΠΎΠΉ Ρ‡Π΅Ρ€Π΅Π· ΡˆΠΊΠΈΠ²Ρ‹, Π²Π΅Π·Π΄Π΅ ΠΎΠ΄ΠΈΠ½Π°ΠΊΠΎΠ²ΠΎ, поэтому Π½Π° Π½ΠΈΠΆΠ½ΠΈΠΉ Π±Π»ΠΎΠΊ дСйствуСт Π² Ρ‚Ρ€ΠΈ Ρ€Π°Π·Π° большС силы натяТСния, Ρ‡Π΅ΠΌ Π½Π° Π²Π΅Ρ€Ρ…Π½ΠΈΠΉ. Если Π±Ρ‹ Π½ΠΈΠΆΠ½ΠΈΠΉ Π±Π»ΠΎΠΊ Π±Ρ‹Π» Π² Ρ‚Ρ€ΠΈ Ρ€Π°Π·Π° тяТСлСС Π²Π΅Ρ€Ρ…Π½Π΅Π³ΠΎ Π±Π»ΠΎΠΊΠ°, Ρ‚ΠΎΠ³Π΄Π° систСма Π±Ρ‹Π»Π° Π±Ρ‹ сбалансирована, ΠΈ Π½ΠΈ ΠΎΠ΄Π½Π° масса Π½Π΅ ΠΌΠΎΠ³Π»Π° Π±Ρ‹ ΡƒΡΠΊΠΎΡ€ΡΡ‚ΡŒΡΡ.[Π’Ρ‹ Π΄ΠΎΠ»ΠΆΠ½Ρ‹ ΠΏΠΎΠΏΡ€ΠΎΠ±ΠΎΠ²Π°Ρ‚ΡŒ Π²Ρ‹ΠΏΠΎΠ»Π½ΠΈΡ‚ΡŒ ΠΌΠ°Ρ‚Π΅ΠΌΠ°Ρ‚ΠΈΠΊΡƒ части (c) с массами Π΄Ρ€ΡƒΠ³ΠΈΠΌ способом ΠΈ ΠΏΡ€ΠΎΠ΄Π΅ΠΌΠΎΠ½ΡΡ‚Ρ€ΠΈΡ€ΠΎΠ²Π°Ρ‚ΡŒ сСбС, Ρ‡Ρ‚ΠΎ это ускорСниС Π΄ΠΎΠ»ΠΆΠ½ΠΎ Π±Ρ‹Ρ‚ΡŒ Ρ€Π°Π²Π½ΠΎ Π½ΡƒΠ»ΡŽ.] Учитывая, Ρ‡Ρ‚ΠΎ систСма ускоряСтся, это Π΄ΠΎΠ»ΠΆΠ΅Π½ Π±Ρ‹Ρ‚ΡŒ Π±ΠΎΠ»Π΅Π΅ высокий Π±Π»ΠΎΠΊ с большим масса.

    Π³. Когда Π²Π΅Ρ€Ρ…Π½ΠΈΠΉ Π±Π»ΠΎΠΊ тянСт струну Π²Π½ΠΈΠ· с ΠΎΠ΄Π½ΠΎΠΉ стороны большого шкива, Ρ‚ΠΎ ΠΆΠ΅ количСство струны, ΠΊΠΎΡ‚ΠΎΡ€ΠΎΠ΅ набираСтся с Π»Π΅Π²ΠΎΠΉ стороны шкива, тСряСтся с ΠΏΡ€Π°Π²ΠΎΠΉ стороны. Π‘Ρ‚Ρ€ΡƒΠ½Π° с ΠΏΡ€Π°Π²ΠΎΠΉ стороны большого шкива Ρ€Π°Π·Π΄Π΅Π»Π΅Π½Π° ΠΌΠ΅ΠΆΠ΄Ρƒ трСмя сСгмСнтами, ΡƒΠ΄Π΅Ρ€ΠΆΠΈΠ²Π°ΡŽΡ‰ΠΈΠΌΠΈ Π΄Ρ€ΡƒΠ³ΠΎΠΉ Π±Π»ΠΎΠΊ.Π‘Π»Π΅Π΄ΠΎΠ²Π°Ρ‚Π΅Π»ΡŒΠ½ΠΎ, Π½ΠΈΠΆΠ½ΠΈΠΉ Π±Π»ΠΎΠΊ пСрСмСщаСтся Π½Π° ΠΎΠ΄Π½Ρƒ Ρ‚Ρ€Π΅Ρ‚ΡŒ Π²Π²Π΅Ρ€Ρ… ΠΏΠΎ ΠΌΠ΅Ρ€Π΅ Ρ‚ΠΎΠ³ΠΎ, ΠΊΠ°ΠΊ Π²Π΅Ρ€Ρ…Π½ΠΈΠΉ Π±Π»ΠΎΠΊ пСрСмСщаСтся Π²Π½ΠΈΠ·. Если Π½ΠΈΠΆΠ½ΠΈΠΉ Π±Π»ΠΎΠΊ поднимаСтся Π½Π° расстояниС \ (y \), Ρ‚ΠΎ Π²Π΅Ρ€Ρ…Π½ΠΈΠΉ Π±Π»ΠΎΠΊ опускаСтся Π½Π° расстояниС \ (3y \), ΠΈ, ΠΏΠΎΡΠΊΠΎΠ»ΡŒΠΊΡƒ ΠΎΠ½ΠΈ Π΄ΠΎΡΡ‚ΠΈΠ³Π°ΡŽΡ‚ Ρ‚ΠΎΠΉ ΠΆΠ΅ высоты, сумма этих ΠΈΠ·ΠΌΠ΅Π½Π΅Π½ΠΈΠΉ составляСт 36 см, Ρ‡Ρ‚ΠΎ ΠΎΠ·Π½Π°Ρ‡Π°Π΅Ρ‚, Ρ‡Ρ‚ΠΎ Π½ΠΈΠΆΠ½ΠΈΠΉ Π±Π»ΠΎΠΊ поднимаСтся Π½Π° расстояниС Π΄Π»ΠΈΠ½Ρ‹ \ (y = 9 \) см.

    Π³. Π§Ρ‚ΠΎΠ±Ρ‹ ΡƒΠ·Π½Π°Ρ‚ΡŒ врСмя, Π½Π΅ΠΎΠ±Ρ…ΠΎΠ΄ΠΈΠΌΠΎΠ΅ ΠΈΠΌ для выравнивания, Π½Π°ΠΌ Π½ΡƒΠΆΠ½ΠΎ ΠΈΡΠΏΠΎΠ»ΡŒΠ·ΠΎΠ²Π°Ρ‚ΡŒ ΠΈΡ… ускорСния. ΠœΡ‹ ΡƒΠΆΠ΅ Π·Π½Π°Π΅ΠΌ ΠΈΡ… ΠΎΡ‚Π½ΠΎΡΠΈΡ‚Π΅Π»ΡŒΠ½Ρ‹Π΅ ускорСния: Π½ΠΈΠΆΠ½ΠΈΠΉ Π±Π»ΠΎΠΊ ускоряСтся Π½Π° ΠΎΠ΄Π½Ρƒ Ρ‚Ρ€Π΅Ρ‚ΡŒ скорости Π²Π΅Ρ€Ρ…Π½Π΅Π³ΠΎ Π±Π»ΠΎΠΊΠ°.ΠŸΠΎΡΡ‚ΠΎΠΌΡƒ ΠΌΡ‹ Π±ΡƒΠ΄Π΅ΠΌ Π½Π°Π·Ρ‹Π²Π°Ρ‚ΡŒ ускорСниС Π²Π΅Ρ€Ρ…Π½Π΅Π³ΠΎ Π±Π»ΠΎΠΊΠ° Β«\ (3a \)Β», дСлая ускорСниС Π½ΠΈΠΆΠ½Π΅Π³ΠΎ Π±Π»ΠΎΠΊΠ° Ρ€Π°Π²Π½Ρ‹ΠΌ \ (a \). Но Π½Π°ΠΌ Π½ΡƒΠΆΠ½Ρ‹ Π·Π°ΠΊΠΎΠ½Ρ‹ ΠΡŒΡŽΡ‚ΠΎΠ½Π°, Ρ‡Ρ‚ΠΎΠ±Ρ‹ ΠΏΠΎΠΉΡ‚ΠΈ дальшС. FBD Π΄Π²ΡƒΡ… задСйствованных систСм выглядит Ρ‚Π°ΠΊ:

    Π‘ΠΎΠ»Π΅Π΅ высокая масса Π² Ρ‚Ρ€ΠΈ Ρ€Π°Π·Π° большС мСньшСй массы, поэтому ΠΌΡ‹ Π±ΡƒΠ΄Π΅ΠΌ Π½Π°Π·Ρ‹Π²Π°Ρ‚ΡŒ \ (m_2 \) просто Β«\ (m \)Β», Ρ‡Ρ‚ΠΎ Π΄Π΅Π»Π°Π΅Ρ‚ \ (m_1 \) Ρ€Π°Π²Π½Ρ‹ΠΌ \ (3m \). Π’ΠΊΠ»ΡŽΡ‡Π΅Π½ΠΈΠ΅ всСго Π²ΠΎ Π²Ρ‚ΠΎΡ€ΠΎΠΉ Π·Π°ΠΊΠΎΠ½ ΠΡŒΡŽΡ‚ΠΎΠ½Π° для ΠΎΠ±ΠΎΠΈΡ… FBD Π΄Π°Π΅Ρ‚ ΡΠ»Π΅Π΄ΡƒΡŽΡ‰ΠΈΠ΅ уравнСния:

    \ [\ left.\ begin {array} {l} a_1 = \ dfrac {F_ {net \; 1}} {m_1} \; \; \; \ΠŸΡ€Π°Π²Π°Ρ стрСлка \;\;\; 3a = \ dfrac {-T + 3mg} {3m} \\ a_2 = \ dfrac {F_ {net \; 2}} {m_2} \; \; \; \ΠŸΡ€Π°Π²Π°Ρ стрСлка \;\;\; a = \ dfrac {3T – mg} {m} \\ \ end {array} \ right \} \; \; \; \ΠŸΡ€Π°Π²Π°Ρ стрСлка \;\;\; a = \ frac {2} {7} g \ nonumber \]

    Π‘ ΠΏΠΎΠΌΠΎΡ‰ΡŒΡŽ ускорСния Π½ΠΈΠΆΠ½Π΅Π³ΠΎ Π±Π»ΠΎΠΊΠ°, расстояния, ΠΊΠΎΡ‚ΠΎΡ€ΠΎΠ΅ ΠΎΠ½ ΠΏΡ€ΠΎΡ…ΠΎΠ΄ΠΈΡ‚, ΠΈ Ρ‚ΠΎΠ³ΠΎ Ρ„Π°ΠΊΡ‚Π°, Ρ‡Ρ‚ΠΎ ΠΎΠ½ начинаСтся Π² состоянии покоя, ΠΌΡ‹ ΠΌΠΎΠΆΠ΅ΠΌ Π²Ρ‹Ρ‡ΠΈΡΠ»ΠΈΡ‚ΡŒ врСмя, Π½Π΅ΠΎΠ±Ρ…ΠΎΠ΄ΠΈΠΌΠΎΠ΅ для ΡΠΎΠ²Π΅Ρ€ΡˆΠ΅Π½ΠΈΡ ΠΏΠΎΠ΅Π·Π΄ΠΊΠΈ:

    \ [y = v_o t + \ frac {1} {2} at ^ 2 \; \; \; \ΠŸΡ€Π°Π²Π°Ρ стрСлка \;\;\; t = \ sqrt {\ dfrac {7y} {g}} = \ Π² ΠΊΠΎΡ€ΠΎΠ±ΠΊΠ΅ {0.25s} \ nonumber \]

    НаклонныС плоскости

    Π¨ΠΊΠΈΠ²Ρ‹

    (ΠΈ, Π² частности, Π±Π»ΠΎΠΊΠΈΡ€Π°Ρ‚ΠΎΡ€ ΠΈ Π·Π°Ρ…Π²Π°Ρ‚) ΡΠ²Π»ΡΡŽΡ‚ΡΡ ΠΏΡ€ΠΈΠΌΠ΅Ρ€ΠΎΠΌ Ρ‚ΠΎΠ³ΠΎ, Ρ‡Ρ‚ΠΎ часто Π½Π°Π·Ρ‹Π²Π°ΡŽΡ‚ простым ΠΌΠ΅Ρ…Π°Π½ΠΈΠ·ΠΌΠΎΠΌ . Π­Ρ‚ΠΎ ΠΏΠΎΡ‚ΠΎΠΌΡƒ, Ρ‡Ρ‚ΠΎ ΠΌΡ‹ ΠΌΠΎΠΆΠ΅ΠΌ ΠΈΡΠΏΠΎΠ»ΡŒΠ·ΠΎΠ²Π°Ρ‚ΡŒ Ρ‚Π°ΠΊΠΎΠ΅ устройство для подъСма тяТСлого вСса с силой, мСньшСй, Ρ‡Π΅ΠΌ сам вСс. Π”Ρ€ΡƒΠ³ΠΎΠΉ ΠΏΡ€ΠΈΠΌΠ΅Ρ€ простой ΠΌΠ°ΡˆΠΈΠ½Ρ‹ – наклонная ΠΏΠ»ΠΎΡΠΊΠΎΡΡ‚ΡŒ . Π­Ρ‚ΠΈ устройства Ρ‚Π°ΠΊΠΆΠ΅ ΠΏΠΎΠ·Π²ΠΎΠ»ΡΡŽΡ‚ Π²Ρ‹ΠΏΠΎΠ»Π½ΡΡ‚ΡŒ Π·Π°Π΄Π°Ρ‡Ρƒ ΠΏΠΎ ΠΏΠΎΠ΄ΡŠΠ΅ΠΌΡƒ тяТСлого ΠΎΠ±ΡŠΠ΅ΠΊΡ‚Π° Π½Π° Π±ΠΎΠ»Π΅Π΅ высокоС ΠΏΠΎΠ»ΠΎΠΆΠ΅Π½ΠΈΠ΅ с использованиСм мСньшСго усилия, Ρ‡Π΅ΠΌ Π±Ρ‹Π»ΠΎ Π±Ρ‹ Π½Π΅ΠΎΠ±Ρ…ΠΎΠ΄ΠΈΠΌΠΎ ΠΏΡ€ΠΈ прямом подъСмС.Основная ΠΎΡΠΎΠ±Π΅Π½Π½ΠΎΡΡ‚ΡŒ Π·Π°Π΄Π°Ρ‡, связанных с Π½Π°ΠΊΠ»ΠΎΠ½Π½Ρ‹ΠΌΠΈ плоскостями, – это систСма ΠΊΠΎΠΎΡ€Π΄ΠΈΠ½Π°Ρ‚, ΠΈΡΠΏΠΎΠ»ΡŒΠ·ΡƒΠ΅ΠΌΠ°Ρ для ΠΎΠ±ΡŠΠ΅ΠΊΡ‚Π° Π½Π° Π½Π°ΠΊΠ»ΠΎΠ½Π½ΠΎΠΉ плоскости. Π”Π°Π²Π°ΠΉΡ‚Π΅ посмотрим Π½Π° ΠΏΡ€ΠΈΠΌΠ΅Ρ€. Как ΠΎΠ±Ρ‹Ρ‡Π½ΠΎ, ΠΌΡ‹ Π½Π°Ρ‡ΠΈΠ½Π°Π΅ΠΌ с самого простого ΠΏΡ€ΠΈΠΌΠ΅Ρ€Π° ΠΈ услоТняСм Π΅Π³ΠΎ ΠΏΠΎ Ρ…ΠΎΠ΄Ρƒ Π΄Π΅Π»Π° – Π±Π»ΠΎΠΊ массы \ (M \) Π½Π° плоскости Π±Π΅Π· трСния, Π½Π°ΠΊΠ»ΠΎΠ½Π΅Π½Π½ΠΎΠΉ ΠΏΠΎΠ΄ ΡƒΠ³Π»ΠΎΠΌ \ (\ theta \):

    Рисунок 2.4.3 – Π‘Π»ΠΎΠΊ Π½Π° Π½Π°ΠΊΠ»ΠΎΠ½Π½ΠΎΠΉ плоскости

    НачнСм наш Π°Π½Π°Π»ΠΈΠ·, ΠΊΠ°ΠΊ всСгда, с Π΄ΠΈΠ°Π³Ρ€Π°ΠΌΠΌΡ‹ свободного Ρ‚Π΅Π»Π°.Ни ΠΎΠ΄ΠΈΠ½ FBD Π½Π΅ Π±ΡƒΠ΄Π΅Ρ‚ ΠΏΠΎΠ»Π½Ρ‹ΠΌ Π±Π΅Π· Π²Ρ‹Π±ΠΎΡ€Π° систСмы ΠΊΠΎΠΎΡ€Π΄ΠΈΠ½Π°Ρ‚, поэтому ΠΌΡ‹ Π΄ΠΎΠ»ΠΆΠ½Ρ‹ Π²Ρ‹Π±Ρ€Π°Ρ‚ΡŒ Π΅Π΅ здСсь. Если ΠΌΡ‹ Π²Ρ‹Π±Π΅Ρ€Π΅ΠΌ Π½Π°ΡˆΡƒ систСму ΠΊΠΎΠΎΡ€Π΄ΠΈΠ½Π°Ρ‚ Π³ΠΎΡ€ΠΈΠ·ΠΎΠ½Ρ‚Π°Π»ΡŒΠ½ΠΎΠΉ ΠΈ Π²Π΅Ρ€Ρ‚ΠΈΠΊΠ°Π»ΡŒΠ½ΠΎΠΉ, ΠΊΠ°ΠΊ ΠΌΡ‹ ΠΎΠ±Ρ‹Ρ‡Π½ΠΎ Π΄Π΅Π»Π°Π΅ΠΌ, Ρ‚ΠΎΠ³Π΄Π°, ΠΊΠΎΠ³Π΄Π° Π±Π»ΠΎΠΊ ΡΠΊΠΎΠ»ΡŒΠ·ΠΈΡ‚ ΠΏΠΎ плоскости, Π΅Π³ΠΎ ускорСниС Π±ΡƒΠ΄Π΅Ρ‚ ΠΈΠΌΠ΅Ρ‚ΡŒ ΠΊΠ°ΠΊ ΠΊΠΎΠΌΠΏΠΎΠ½Π΅Π½Ρ‚Ρ‹ \ (x \), Ρ‚Π°ΠΊ ΠΈ \ (y \). Π­Ρ‚ΠΎ, ΠΊΠΎΠ½Π΅Ρ‡Π½ΠΎ, Π½ΠΎΡ€ΠΌΠ°Π»ΡŒΠ½ΠΎ, Π½ΠΎ Ρ€Π°Π±ΠΎΡ‚Π°Ρ‚ΡŒ с этим ΠΌΠΎΠΆΠ΅Ρ‚ Π±Ρ‹Ρ‚ΡŒ Π½Π΅ΠΌΠ½ΠΎΠ³ΠΎ Π½Π΅ΡƒΠ΄ΠΎΠ±Π½ΠΎ. Π’ Π³ΠΎΡ€ΠΈΠ·ΠΎΠ½Ρ‚Π°Π»ΡŒΠ½ΠΎΠΌ ΠΈ Π²Π΅Ρ€Ρ‚ΠΈΠΊΠ°Π»ΡŒΠ½ΠΎΠΌ направлСниях Π½Π΅Ρ‚ Π½ΠΈΡ‡Π΅Π³ΠΎ ΡΠ°ΠΊΡ€Π°Π»ΡŒΠ½ΠΎΠ³ΠΎ, Ρ‚Π°ΠΊ ΠΏΠΎΡ‡Π΅ΠΌΡƒ Π±Ρ‹ Π½Π΅ Π²Ρ‹Π±Ρ€Π°Ρ‚ΡŒ систСму ΠΊΠΎΠΎΡ€Π΄ΠΈΠ½Π°Ρ‚, ΠΏΠ°Ρ€Π°Π»Π»Π΅Π»ΡŒΠ½ΡƒΡŽ ΠΈ пСрпСндикулярной плоскости?

    Рисунок 2.4.4 – БистСма ΠΊΠΎΠΎΡ€Π΄ΠΈΠ½Π°Ρ‚ Π½Π°ΠΊΠ»ΠΎΠ½Π½ΠΎΠΉ плоскости

    На срСднСй Π΄ΠΈΠ°Π³Ρ€Π°ΠΌΠΌΠ΅ Π½Π° рисункС 2.4.4 ΠΏΠΎΠΊΠ°Π·Π°Π½Π° сумма Π½ΠΎΡ€ΠΌΠ°Π»ΡŒΠ½Ρ‹Ρ… ΠΈ Π³Ρ€Π°Π²ΠΈΡ‚Π°Ρ†ΠΈΠΎΠ½Π½Ρ‹Ρ… сил ΠΌΠ΅ΠΆΠ΄Ρƒ хвостом ΠΈ Π³ΠΎΠ»ΠΎΠ²ΠΎΠΉ, приводящая ΠΊ Ρ€Π΅Π·ΡƒΠ»ΡŒΡ‚ΠΈΡ€ΡƒΡŽΡ‰Π΅ΠΉ силС, ΠΏΠ°Ρ€Π°Π»Π»Π΅Π»ΡŒΠ½ΠΎΠΉ плоскости ΠΈ Π²Π½ΠΈΠ· ΠΏΠΎ плоскости, ΠΊΠ°ΠΊ ΠΈ Π΄ΠΎΠ»ΠΆΠ½ΠΎ Π±Ρ‹Ρ‚ΡŒ, ΠΏΠΎΡΠΊΠΎΠ»ΡŒΠΊΡƒ ΠΌΡ‹ Π·Π½Π°Π΅ΠΌ, Π² ΠΊΠ°ΠΊΠΎΠΌ Π½Π°ΠΏΡ€Π°Π²Π»Π΅Π½ΠΈΠΈ Π±ΡƒΠ΄Π΅Ρ‚ Π΄Π²ΠΈΠ³Π°Ρ‚ΡŒΡΡ Π±Π»ΠΎΠΊ. ΡƒΡΠΊΠΎΡ€ΡΡ‚ΡŒΡΡ. На ΠΏΡ€Π°Π²ΠΎΠΉ Π΄ΠΈΠ°Π³Ρ€Π°ΠΌΠΌΠ΅ ΠΏΠΎΠΊΠ°Π·Π°Π½Π° сила тяТСсти, разбитая Π½Π° Π΅Π΅ ΠΊΠΎΠΌΠΏΠΎΠ½Π΅Π½Ρ‚Ρ‹ \ (x \) ΠΈ \ (y \) Π² Π²Ρ‹Π±Ρ€Π°Π½Π½ΠΎΠΉ систСмС ΠΊΠΎΠΎΡ€Π΄ΠΈΠ½Π°Ρ‚ (Π²Ρ‹ Π΄ΠΎΠ»ΠΆΠ½Ρ‹ сами ΠΏΡ€ΠΎΠ²Π΅Ρ€ΠΈΡ‚ΡŒ Π³Π΅ΠΎΠΌΠ΅Ρ‚Ρ€ΠΈΡŽ, которая ΠΏΡ€ΠΈΠ²ΠΎΠ΄ΠΈΡ‚ ΠΊ Π·Π°ΠΊΠ»ΡŽΡ‡Π΅Π½ΠΈΡŽ, Ρ‡Ρ‚ΠΎ ΡƒΠ³ΠΎΠ» Π½Π° этой Π΄ΠΈΠ°Π³Ρ€Π°ΠΌΠΌΠ΅ совпадаСт с ΡƒΠ³Π»ΠΎΠΌ Π½Π°ΠΊΠ»ΠΎΠ½ составляСт Π³ΠΎΡ€ΠΈΠ·ΠΎΠ½Ρ‚Π°Π»ΡŒ).ΠžΠ±Ρ€Π°Ρ‚ΠΈΡ‚Π΅ Π²Π½ΠΈΠΌΠ°Π½ΠΈΠ΅: ΠΏΠΎΡΠΊΠΎΠ»ΡŒΠΊΡƒ Π±Π»ΠΎΠΊ Π½Π΅ ускоряСтся пСрпСндикулярно плоскости, ΠΌΡ‹ ΠΌΠΎΠΆΠ΅ΠΌ ΡΠ΄Π΅Π»Π°Ρ‚ΡŒ Π²Ρ‹Π²ΠΎΠ΄, Ρ‡Ρ‚ΠΎ \ (N = Mg \ cos {\ theta} \). Π’Π°ΠΊΠΆΠ΅ ясно, Ρ‡Ρ‚ΠΎ чистая сила – это \ (x \) – ΠΊΠΎΠΌΠΏΠΎΠ½Π΅Π½Ρ‚ силы тяТСсти, Π² Ρ€Π΅Π·ΡƒΠ»ΡŒΡ‚Π°Ρ‚Π΅ Ρ‡Π΅Π³ΠΎ ускорСниС Π²Π½ΠΈΠ· ΠΏΠΎ плоскости Ρ€Π°Π²Π½ΠΎ \ (a = g \ sin {\ theta} \).

    ΠŸΡ€Π΅Π΄ΠΏΠΎΠ»ΠΎΠΆΠΈΠΌ, ΠΌΡ‹ Ρ…ΠΎΡ‚ΠΈΠΌ ΠΏΠΎΠ΄Π½ΡΡ‚ΡŒ этот Π±Π»ΠΎΠΊ Π½Π° Π½ΠΎΠ²ΡƒΡŽ высоту. Π§Ρ‚ΠΎΠ±Ρ‹ Π·Π°ΡΡ‚Π°Π²ΠΈΡ‚ΡŒ Π΅Π³ΠΎ Π΄Π²ΠΈΠ³Π°Ρ‚ΡŒΡΡ Π²Π²Π΅Ρ€Ρ… ΠΏΠΎ плоскости, Π½Π°ΠΌ Π½ΡƒΠΆΠ½ΠΎ ΠΏΡ€ΠΈΠ»ΠΎΠΆΠΈΡ‚ΡŒ силу, ΠΏΡ€Π΅Π²Ρ‹ΡˆΠ°ΡŽΡ‰ΡƒΡŽ Ρ‡ΠΈΡΡ‚ΡƒΡŽ силу, ΠΏΠΎΠΊΠ°Π·Π°Π½Π½ΡƒΡŽ Π²Ρ‹ΡˆΠ΅, которая мСньшС силы, ΠΊΠΎΡ‚ΠΎΡ€ΡƒΡŽ Π½Π°ΠΌ ΠΏΡ€ΠΈΡˆΠ»ΠΎΡΡŒ Π±Ρ‹ ΠΏΡ€ΠΈΠ»ΠΎΠΆΠΈΡ‚ΡŒ, Ρ‡Ρ‚ΠΎΠ±Ρ‹ ΠΏΠΎΠ΄Π½ΡΡ‚ΡŒ Π΅Π³ΠΎ прямо Π²Π²Π΅Ρ€Ρ….Как ΠΈ Π² случаС с Π±Π»ΠΎΠΊΠΎΠΌ ΠΈ ΡΠ½Π°ΡΡ‚ΡŒΡŽ, эта «простая машина» ΠΏΠΎΠΌΠΎΠ³Π°Π΅Ρ‚ Π½Π°ΠΌ Π²Ρ‹ΠΏΠΎΠ»Π½ΡΡ‚ΡŒ Ρ€Π°Π±ΠΎΡ‚Ρƒ с мСньшими усилиями, Ρ‡Π΅ΠΌ трСбуСтся, Ссли ΠΎΠ½Π° выполняСтся Π±ΠΎΠ»Π΅Π΅ нСпосрСдствСнно. Π’ случаС Π±Π»ΠΎΠΊΠ° ΠΈ снасти, Ρ‡Ρ‚ΠΎΠ±Ρ‹ ΠΏΠΎΠ΄Π½ΡΡ‚ΡŒ Π³Ρ€ΡƒΠ· Π½Π° Π½Π΅ΠΊΠΎΡ‚ΠΎΡ€ΠΎΠ΅ расстояниС, Π΄Ρ€ΡƒΠ³ΠΎΠΉ ΠΊΠΎΠ½Π΅Ρ† Π²Π΅Ρ€Π΅Π²ΠΊΠΈ Π½ΡƒΠΆΠ½ΠΎ Π±Ρ‹Π»ΠΎ ΠΏΡ€ΠΎΡ‚ΡΠ½ΡƒΡ‚ΡŒ Π½Π° большСС расстояниС. Π’ этом случаС ΠΌΡ‹ Π²ΠΈΠ΄ΠΈΠΌ Π°Π½Π°Π»ΠΎΠ³ΠΈΡ‡Π½ΡƒΡŽ Π²Π΅Ρ‰ΡŒ – расстояниС, Π½Π° ΠΊΠΎΡ‚ΠΎΡ€ΠΎΠ΅ ΠΌΡ‹ Π΄ΠΎΠ»ΠΆΠ½Ρ‹ Ρ‚ΠΎΠ»ΠΊΠ½ΡƒΡ‚ΡŒ Π±Π»ΠΎΠΊ, являСтся Π³ΠΈΠΏΠΎΡ‚Π΅Π½ΡƒΠ·ΠΎΠΉ Ρ‚Ρ€Π΅ΡƒΠ³ΠΎΠ»ΡŒΠ½ΠΈΠΊΠ°, Ρ‡Ρ‚ΠΎΠ±Ρ‹ ΠΏΠΎΠ΄Π½ΡΡ‚ΡŒ Π΅Π³ΠΎ Π½Π° высоту Π²Π΅Ρ€Ρ‚ΠΈΠΊΠ°Π»ΡŒΠ½ΠΎΠ³ΠΎ ΠΊΠ°Ρ‚Π΅Ρ‚Π° Ρ‚Ρ€Π΅ΡƒΠ³ΠΎΠ»ΡŒΠ½ΠΈΠΊΠ°. Π­Ρ‚ΠΎ обычная Ρ‚Π΅ΠΌΠ° для простых машин – трСбуСтся мСньшСС усилиС, Π½ΠΎ ΠΎΠ½ΠΎ Π΄ΠΎΠ»ΠΆΠ½ΠΎ ΠΏΡ€ΠΈΠΌΠ΅Π½ΡΡ‚ΡŒΡΡ Π½Π° большСм расстоянии.Π’ ΡΠ»Π΅Π΄ΡƒΡŽΡ‰Π΅ΠΉ Π³Π»Π°Π²Π΅ ΠΌΡ‹ ΡƒΠ²ΠΈΠ΄ΠΈΠΌ, ΠΏΠΎΡ‡Π΅ΠΌΡƒ это Ρ‚Π°ΠΊ.

    ΠœΡ‹ ΠΌΠΎΠΆΠ΅ΠΌ Π·Π½Π°Ρ‡ΠΈΡ‚Π΅Π»ΡŒΠ½ΠΎ ΡƒΡΠ»ΠΎΠΆΠ½ΠΈΡ‚ΡŒ этот простой ΠΏΡ€ΠΈΠΌΠ΅Ρ€. Бамая СстСствСнная Ρ€Π΅Π³ΡƒΠ»ΠΈΡ€ΠΎΠ²ΠΊΠ° – Π²ΠΊΠ»ΡŽΡ‡ΠΈΡ‚ΡŒ Ρ‚Ρ€Π΅Π½ΠΈΠ΅. Из-Π·Π° ΠΏΡ€ΠΈΡ€ΠΎΠ΄Ρ‹ Π΄Π²ΡƒΡ… Ρ‚ΠΈΠΏΠΎΠ² трСния Π΄ΠΎΠ±Π°Π²Π»Π΅Π½ΠΈΠ΅ статичСского трСния (ΠΊΠΎΡ‚ΠΎΡ€ΠΎΠ΅ Π²Ρ…ΠΎΠ΄ΠΈΡ‚ Π² нСравСнство) ΠΌΠΎΠΆΠ΅Ρ‚ Π±Ρ‹Ρ‚ΡŒ особСнно ΠΏΡ€ΠΎΠ±Π»Π΅ΠΌΠ°Ρ‚ΠΈΡ‡Π½Ρ‹ΠΌ. Π§Ρ‚ΠΎΠ±Ρ‹ ΠΏΠΎΠ½ΡΡ‚ΡŒ, ΠΏΠΎΡ‡Π΅ΠΌΡƒ, Π΄Π°Π²Π°ΠΉΡ‚Π΅ рассмотрим ΡΠ»Π΅Π΄ΡƒΡŽΡ‰ΡƒΡŽ ΠΏΡ€ΠΎΠ±Π»Π΅ΠΌΡƒ:

    БистСма, изобраТСнная Π½Π° рисункС 2.4.5, ΠΏΠΎΠΊΠ°Π·Ρ‹Π²Π°Π΅Ρ‚ Π΄Π²Π° Π±Π»ΠΎΠΊΠ°, ΠΊΠΎΡ‚ΠΎΡ€Ρ‹Π΅ ΠΎΡΡ‚Π°ΡŽΡ‚ΡΡ Π² состоянии покоя, ΠΊΠΎΡ‚ΠΎΡ€Ρ‹Π΅ ΠΏΡ€ΠΈΠΊΡ€Π΅ΠΏΠ»Π΅Π½Ρ‹ бСзмассовой струной ΠΊ бСзмассовому ΡˆΠΊΠΈΠ²Ρƒ Π±Π΅Π· трСния.ΠŸΠ»ΠΎΡΠΊΠΎΡΡ‚ΡŒ Π½Π°ΠΊΠ»ΠΎΠ½Π΅Π½Π° ΠΏΠΎΠ΄ ΡƒΠ³Π»ΠΎΠΌ \ (Ρ‚Π΅Ρ‚Π° \) Π²Π²Π΅Ρ€Ρ… ΠΎΡ‚ Π³ΠΎΡ€ΠΈΠ·ΠΎΠ½Ρ‚Π°Π»ΠΈ, Π° Π΅Π΅ ΠΏΠΎΠ²Π΅Ρ€Ρ…Π½ΠΎΡΡ‚ΡŒ ΡˆΠ΅Ρ€ΠΎΡ…ΠΎΠ²Π°Ρ‚Π°Ρ (Ρ‚.Π΅.Π½Π΅ ΠΈΠΌΠ΅Π΅Ρ‚ трСния). ΠŸΡ€ΠΈΠ²Π΅Π΄Π΅Π½Ρ‹ масса подвСсного Π±Π»ΠΎΠΊΠ°, ΡƒΠ³ΠΎΠ» Π½Π°ΠΊΠ»ΠΎΠ½Π° ΠΈ коэффициСнт статичСского трСния. По этим Π²Π΅Π»ΠΈΡ‡ΠΈΠ½Π°ΠΌ ΠΎΠΏΡ€Π΅Π΄Π΅Π»ΠΈΡ‚Π΅ минимально Π²ΠΎΠ·ΠΌΠΎΠΆΠ½ΠΎΠ΅ Π·Π½Π°Ρ‡Π΅Π½ΠΈΠ΅ массы Π±Π»ΠΎΠΊΠ° Π½Π° плоскости.

    Рисунок 2.4.5 – ΠœΠ΅Ρ…Π°Π½ΠΈΡ‡Π΅ΡΠΊΠ°Ρ систСма

    Начиная (ΠΊΠ°ΠΊ всСгда) с FBD (Π²ΠΊΠ»ΡŽΡ‡Π°Ρ систСму ΠΊΠΎΠΎΡ€Π΄ΠΈΠ½Π°Ρ‚) для ΠΊΠ°ΠΆΠ΄ΠΎΠ³ΠΎ Π±Π»ΠΎΠΊΠ°, ΠΌΡ‹ ΠΈΠΌΠ΅Π΅ΠΌ:

    Рисунок 2.4.6 – Π‘Ρ…Π΅ΠΌΡ‹ Π±Π»ΠΎΠΊΠΎΠ² со свободным Ρ‚Π΅Π»ΠΎΠΌ

    Π”Π°Π²Π°ΠΉΡ‚Π΅ ΠΏΡ€ΠΎΠΊΠΎΠΌΠΌΠ΅Π½Ρ‚ΠΈΡ€ΡƒΠ΅ΠΌ Π½Π°ΠΏΡ€Π°Π²Π»Π΅Π½ΠΈΠ΅ силы статичСского трСния. Напомним, Ρ‡Ρ‚ΠΎ сила статичСского трСния просто Ρ€Π΅Π°Π³ΠΈΡ€ΡƒΠ΅Ρ‚ Π½Π° Β«ΠΏΠΎΠΏΡ‹Ρ‚ΠΊΡƒΒ» двиТСния ΠΎΠ±ΡŠΠ΅ΠΊΡ‚Π° ΠΏΠΎ повСрхности. Π’ этом случаС, Ссли Π±Π»ΠΎΠΊ «пытался» ΡΠΊΠΎΠ»ΡŒΠ·ΠΈΡ‚ΡŒ ΠΏΠΎ плоскости, Ρ‚ΠΎ сила статичСского трСния Π΄ΠΎΠ»ΠΆΠ½Π° Π±Ρ‹Ρ‚ΡŒ Π²Π²Π΅Ρ€Ρ… ΠΏΠΎ плоскости. Π—Π΄Π΅ΡΡŒ ΠΎΠ½ Π½Π°ΠΏΡ€Π°Π²Π»Π΅Π½ Π²Π½ΠΈΠ· ΠΏΠΎ плоскости, Ρ‡Ρ‚ΠΎ ΠΎΠ·Π½Π°Ρ‡Π°Π΅Ρ‚, Ρ‡Ρ‚ΠΎ Π΄Ρ€ΡƒΠ³ΠΈΠ΅ силы Π΄ΠΎΠ»ΠΆΠ½Ρ‹ Π±Ρ‹Ρ‚ΡŒ Ρ‚Π°ΠΊΠΈΠΌΠΈ, Ρ‡Ρ‚ΠΎΠ±Ρ‹ ΡƒΡΠΊΠΎΡ€ΡΡ‚ΡŒ Π΅Π³ΠΎ Π²Π²Π΅Ρ€Ρ… ΠΏΠΎ плоскости… ΠžΡ‚ΠΊΡƒΠ΄Π° ΠΌΡ‹ Π·Π½Π°Π΅ΠΌ, Ρ‡Ρ‚ΠΎ это Ρ‚Π°ΠΊ? ΠžΡ‚Π²Π΅Ρ‚ Π·Π°ΠΊΠ»ΡŽΡ‡Π°Π΅Ρ‚ΡΡ Π² постановкС вопроса: ΠΌΡ‹ ΠΈΡ‰Π΅ΠΌ ΠΌΠΈΠ½ΠΈΠΌΠ°Π»ΡŒΠ½ΡƒΡŽ массу Π±Π»ΠΎΠΊΠ° Π½Π° плоскости. ΠŸΡ€Π΅Π΄ΡΡ‚Π°Π²ΡŒΡ‚Π΅ сСбС Π±Π»ΠΎΠΊ, масса ΠΊΠΎΡ‚ΠΎΡ€ΠΎΠ³ΠΎ ΡƒΡ€Π°Π²Π½ΠΎΠ²Π΅ΡˆΠΈΠ²Π°Π΅Ρ‚ систСму. Ссли ΠΊ Π±Π»ΠΎΠΊΡƒ добавляСтся ΠΈΠ»ΠΈ вычитаСтся нСбольшая масса, систСма ΠΌΠΎΠΆΠ΅Ρ‚ ΠΎΡΡ‚Π°Π²Π°Ρ‚ΡŒΡΡ Π² ΠΏΠΎΠΊΠΎΠ΅, ΠΏΠΎΡΠΊΠΎΠ»ΡŒΠΊΡƒ статичСскоС Ρ‚Ρ€Π΅Π½ΠΈΠ΅ ΡƒΠ΄Π΅Ρ€ΠΆΠΈΠ²Π°Π΅Ρ‚ баланс. Если ΠΌΡ‹ Π΄ΠΎΠ±Π°Π²ΠΈΠΌ этому Π±Π»ΠΎΠΊΡƒ слишком ΠΌΠ½ΠΎΠ³ΠΎ массы, статичСскоС Ρ‚Ρ€Π΅Π½ΠΈΠ΅ достигнСт своСго ΠΏΡ€Π΅Π΄Π΅Π»Π°, ΠΈ Π±Π»ΠΎΠΊ Π½Π°Ρ‡Π½Π΅Ρ‚ ΡΠΊΠΎΠ»ΡŒΠ·ΠΈΡ‚ΡŒ Π²Π½ΠΈΠ·, Π° Ссли ΠΌΡ‹ ΡƒΠ±Π΅Ρ€Π΅ΠΌ слишком ΠΌΠ½ΠΎΠ³ΠΎ массы, ΠΎΠ½ Π±ΡƒΠ΄Π΅Ρ‚ ΡΠΊΠΎΠ»ΡŒΠ·ΠΈΡ‚ΡŒ Π²Π²Π΅Ρ€Ρ… ΠΏΠΎ плоскости.БтатичСскоС Ρ‚Ρ€Π΅Π½ΠΈΠ΅ Π±ΡƒΠ΄Π΅Ρ‚ ΠΏΡ€ΠΎΡ‚ΠΈΠ²ΠΎΠ΄Π΅ΠΉΡΡ‚Π²ΠΎΠ²Π°Ρ‚ΡŒ Π½Π°ΠΌΠ΅Ρ‡Π΅Π½Π½ΠΎΠΌΡƒ двиТСнию, поэтому для минимальной массы сила статичСского трСния Π΄ΠΎΠ»ΠΆΠ½Π° Π±Ρ‹Ρ‚ΡŒ Π½Π°ΠΏΡ€Π°Π²Π»Π΅Π½Π° ​​вниз ΠΏΠΎ плоскости.

    ΠŸΡ€Π΅Π΄ΡƒΠΏΡ€Π΅ΠΆΠ΄Π΅Π½ΠΈΠ΅

    Π₯отя ΠΌΡ‹ ΠΌΠΎΠΆΠ΅ΠΌ ΠΎΠΏΡ€Π΅Π΄Π΅Π»ΠΈΡ‚ΡŒ Π½Π°ΠΏΡ€Π°Π²Π»Π΅Π½ΠΈΠ΅ статичСского трСния для этой Π·Π°Π΄Π°Ρ‡ΠΈ, Π²ΠΎ ΠΌΠ½ΠΎΠ³ΠΈΡ… Π·Π°Π΄Π°Ρ‡Π°Ρ… это Π½Π΅Π²ΠΎΠ·ΠΌΠΎΠΆΠ½ΠΎ. Если Π²Ρ‹ Π·Π½Π°Π΅Ρ‚Π΅, Ρ‡Ρ‚ΠΎ сила статичСского трСния присутствуСт (ΠΈΠ»ΠΈ ΠΌΠΎΠΆΠ΅Ρ‚ Π±Ρ‹Ρ‚ΡŒ), просто втянитС Π΅Π΅ Π² любом Π½Π°ΠΏΡ€Π°Π²Π»Π΅Π½ΠΈΠΈ. Когда ΠΏΡ€ΠΎΠ±Π»Π΅ΠΌΠ° Ρ€Π΅ΡˆΠ΅Π½Π°, Ссли Π²Ρ‹ Ρ€Π΅ΡˆΠΈΡ‚Π΅ Π·Π½Π°Ρ‡Π΅Π½ΠΈΠ΅ этой силы, ΠΎΠ½Π° Π±ΡƒΠ΄Π΅Ρ‚ ΠΏΠΎΠ»ΠΎΠΆΠΈΡ‚Π΅Π»ΡŒΠ½ΠΎΠΉ, Ссли Π²Ρ‹ Π²Ρ‹Π±Ρ€Π°Π»ΠΈ ΠΏΡ€Π°Π²ΠΈΠ»ΡŒΠ½ΠΎΠ΅ Π½Π°ΠΏΡ€Π°Π²Π»Π΅Π½ΠΈΠ΅, ΠΈ ΠΎΡ‚Ρ€ΠΈΡ†Π°Ρ‚Π΅Π»ΡŒΠ½ΠΎΠΉ, Ссли Π²Ρ‹ этого Π½Π΅ сдСлали.FBD – это просто инструмСнт, ΠΈ Π² ΠΊΠΎΠ½Π΅Ρ‡Π½ΠΎΠΌ ΠΈΡ‚ΠΎΠ³Π΅ Π²Ρ‹ ΠΏΠΎΠ»ΡƒΡ‡ΠΈΡ‚Π΅ ΠΎΡ‚Π²Π΅Ρ‚, поэтому Π½Π΅ Ρ‚Ρ€Π°Ρ‚ΡŒΡ‚Π΅ ΡΠ½Π΅Ρ€Π³ΠΈΡŽ Π½Π° Ρ‚ΠΎ, Ρ‡Ρ‚ΠΎΠ±Ρ‹ ΠΏΠΎΠ»ΡƒΡ‡ΠΈΡ‚ΡŒ ΠΏΡ€Π°Π²ΠΈΠ»ΡŒΠ½ΠΎΠ΅ Π½Π°ΠΏΡ€Π°Π²Π»Π΅Π½ΠΈΠ΅ Π½Π° Π΄ΠΈΠ°Π³Ρ€Π°ΠΌΠΌΠ΅.

    Π Π°Π·Π±ΠΈΠ΅Π½ΠΈΠ΅ Π²Π΅ΠΊΡ‚ΠΎΡ€ΠΎΠ² Π½Π° ΠΊΠΎΠΌΠΏΠΎΠ½Π΅Π½Ρ‚Ρ‹ Π² Π²Ρ‹Π±Ρ€Π°Π½Π½Ρ‹Ρ… Π½Π°ΠΌΠΈ систСмах ΠΊΠΎΠΎΡ€Π΄ΠΈΠ½Π°Ρ‚ ΠΈ ΠΏΡ€ΠΈΠΌΠ΅Π½Π΅Π½ΠΈΠ΅ Π²Ρ‚ΠΎΡ€ΠΎΠ³ΠΎ Π·Π°ΠΊΠΎΠ½Π° ΠΡŒΡŽΡ‚ΠΎΠ½Π° (для Π½ΡƒΠ»Π΅Π²ΠΎΠ³ΠΎ ускорСния) Π΄Π°Π΅Ρ‚:

    \ [\ begin {array} {l} block \; Π½Π° \; плоскости: \; \; \ begin {array} {l} x – Π½Π°ΠΏΡ€Π°Π²Π»Π΅Π½ΠΈΠ΅ \; силы: \; \; 0 = a_x = \ dfrac {f – T + Mg \ sin {\ theta}} {M} \\ y – Π½Π°ΠΏΡ€Π°Π²Π»Π΅Π½ΠΈΠ΅ \; силы: \; \; 0 = {a_y} = \ dfrac {N – Mg \ cos {\ theta}} {M} \ end {array} \\ висит \; Π±Π»ΠΎΠΊ: \; \; \; y – Π½Π°ΠΏΡ€Π°Π²Π»Π΅Π½ΠΈΠ΅ \; силы: \; \; 0 = {a_y} = \ dfrac {T – mg} {m} \ end {array} \]

    Π—Π°Ρ‚Π΅ΠΌ ΠΏΡ€ΠΈΠΌΠ΅Π½ΠΈΡ‚Π΅ ΠΎΠ³Ρ€Π°Π½ΠΈΡ‡Π΅Π½ΠΈΠ΅, ΠΊΠΎΡ‚ΠΎΡ€ΠΎΠ΅ связываСт ΠΌΠ°ΠΊΡΠΈΠΌΠ°Π»ΡŒΠ½ΡƒΡŽ силу трСния покоя (которая Π²ΠΎΠ·Π½ΠΈΠΊΠ°Π΅Ρ‚, ΠΊΠΎΠ³Π΄Π° минимальная масса находится Π½Π° плоскости) ΠΈ Π½ΠΎΡ€ΠΌΠ°Π»ΡŒΠ½ΡƒΡŽ силу:

    \ [f \ le \ mu_S N \; \; \; \ΠŸΡ€Π°Π²Π°Ρ стрСлка \;\;\; (максимум \; f \; для \; ΠΌΠΈΠ½ΠΈΠΌΡƒΠΌ \; M) \; \; \; \ΠŸΡ€Π°Π²Π°Ρ стрСлка \;\;\; f = \ mu_S N \]

    ΠžΡΡ‚Π°Π»ΡŒΠ½ΠΎΠ΅ – Π°Π»Π³Π΅Π±Ρ€Π° с Ρ‡Π΅Ρ‚Ρ‹Ρ€ΡŒΠΌΡ систСмными уравнСниями, Ρ€Π΅Π·ΡƒΠ»ΡŒΡ‚Π°Ρ‚ ΠΊΠΎΡ‚ΠΎΡ€Ρ‹Ρ…:

    \ [M_ {min} = \ frac {m} {\ mu _S \ cos \ theta + \ sin \ theta} \]

    Π’Π΅ΠΏΠ΅Ρ€ΡŒ ΠΌΡ‹ Π΄ΠΎΠ»ΠΆΠ½Ρ‹ ΠΏΡ€ΠΎΠ²Π΅Ρ€ΠΈΡ‚ΡŒ, ΠΈΠΌΠ΅Π΅Ρ‚ Π»ΠΈ этот ΠΎΡ‚Π²Π΅Ρ‚ смысл.o \) случаС (Π³ΠΎΡ€ΠΈΠ·ΠΎΠ½Ρ‚Π°Π»ΡŒΠ½Π°Ρ ΠΏΠΎΠ²Π΅Ρ€Ρ…Π½ΠΎΡΡ‚ΡŒ, Π³Π΄Π΅ Π½ΠΎΡ€ΠΌΠ°Π»ΡŒΠ½Π°Ρ сила Ρ€Π°Π²Π½Π° вСсу Π±Π»ΠΎΠΊΠ° Π½Π° повСрхности) ΠΏΠΎΡ‚Ρ€Π΅Π±ΡƒΠ΅Ρ‚, Ρ‡Ρ‚ΠΎΠ±Ρ‹ сила трСния Ρ€Π°Π²Π½ΡΠ»Π°ΡΡŒ вСсу подвСсного Π±Π»ΠΎΠΊΠ°. Π’ΠΎ Π΅ΡΡ‚ΡŒ ΠΌΡ‹ Π΄ΠΎΠ»ΠΆΠ½Ρ‹ ΠΈΠΌΠ΅Ρ‚ΡŒ \ (mg = f = \ mu_S N = \ mu_S Mg \; \; \ Rightarrow \; \; m = \ mu_S M \), Ρ‡Ρ‚ΠΎ ΠΌΡ‹ ΠΏΠΎΠ»ΡƒΡ‡Π°Π΅ΠΌ, ΠΊΠΎΠ³Π΄Π° подставляСм ноль для \ ( \ Ρ‚Π΅Ρ‚Π° \).

    Π­Ρ‚Π° Π·Π°Π΄Π°Ρ‡Π° ΠΌΠΎΠ³Π»Π° Π±Ρ‹ Ρ‚Π°ΠΊ ΠΆΠ΅ Π»Π΅Π³ΠΊΠΎ Π·Π°ΠΏΡ€ΠΎΡΠΈΡ‚ΡŒ максимальноС Π²ΠΎΠ·ΠΌΠΎΠΆΠ½ΠΎΠ΅ Π·Π½Π°Ρ‡Π΅Π½ΠΈΠ΅ для \ (M \). Π§ΠΈΡ‚Π°Ρ‚Π΅Π»ΡŽ прСдлагаСтся ΠΏΠΎΠΏΡ€ΠΎΠ±ΠΎΠ²Π°Ρ‚ΡŒ это Π² качСствС упраТнСния.

    Π‘ этой ΠΏΡ€ΠΎΠ±Π»Π΅ΠΌΠΎΠΉ Π±Ρ‹Π»ΠΎ связано ΠΌΠ½ΠΎΠ³ΠΎ всСго, Π½ΠΎ Π³Π»Π°Π²Π½ΠΎΠ΅ – Π΄Π΅Π»Π°Ρ‚ΡŒ это шаг Π·Π° шагом ΠΈ Π²Ρ‹ΠΏΠΎΠ»Π½ΡΡ‚ΡŒ ΡΠ»Π΅Π΄ΡƒΡŽΡ‰ΠΈΠ΅ прСдписанныС шаги:

    1. Π½Π°Ρ€ΠΈΡΠΎΠ²Π°Ρ‚ΡŒ схСму
    2. Π²Ρ‹Π΄Π΅Π»ΠΈΡ‚Π΅ ΡΠΎΠΎΡ‚Π²Π΅Ρ‚ΡΡ‚Π²ΡƒΡŽΡ‰ΠΈΠ΅ ΠΎΠ±ΡŠΠ΅ΠΊΡ‚Ρ‹ ΠΈ Π½Π°Ρ‡Π΅Ρ€Ρ‚ΠΈΡ‚Π΅ для Π½ΠΈΡ… Π΄ΠΈΠ°Π³Ρ€Π°ΠΌΠΌΡ‹ свободного Ρ‚Π΅Π»Π°
    3. Π²Ρ‹Π±Π΅Ρ€ΠΈΡ‚Π΅ систСмы ΠΊΠΎΠΎΡ€Π΄ΠΈΠ½Π°Ρ‚ для схСм, ΠΊΠΎΡ‚ΠΎΡ€Ρ‹Π΅ ΡƒΠ΄ΠΎΠ±Π½Ρ‹
    4. Ρ€Π°Π·Π±ΠΈΠ²Π°Π΅Ρ‚ силы Π½Π° ΠΊΠΎΠΌΠΏΠΎΠ½Π΅Π½Ρ‚Ρ‹ Π² Π²Ρ‹Π±Ρ€Π°Π½Π½ΠΎΠΉ систСмС ΠΊΠΎΠΎΡ€Π΄ΠΈΠ½Π°Ρ‚
    5. суммируйтС силы Π² направлСниях \ (x \) ΠΈ \ (y \) ΠΈ ΠΏΡ€ΠΈΠΌΠ΅Π½ΠΈΡ‚Π΅ Π²Ρ‚ΠΎΡ€ΠΎΠΉ Π·Π°ΠΊΠΎΠ½ ΠΡŒΡŽΡ‚ΠΎΠ½Π° для ΠΎΠ±ΠΎΠΈΡ… Π½Π°ΠΏΡ€Π°Π²Π»Π΅Π½ΠΈΠΉ
    6. ΠΏΡ€ΠΈΠΌΠ΅Π½ΠΈΡ‚ΡŒ ограничСния
    7. Ρ€Π΅ΡˆΠΈΡ‚ΡŒ Π°Π»Π³Π΅Π±Ρ€Ρƒ

    ΠŸΡ€ΠΈΠΌΠ΅Ρ€ \ (\ PageIndex {2} \)

    Врос Π·Π°ΠΊΡ€Π΅ΠΏΠ»Π΅Π½ Π½Π° 50.Π‘Π»ΠΎΠΊ 0 ΠΊΠ³ Π² Π΄Π²ΡƒΡ… мСстах ΠΈ ​​проходит Ρ‡Π΅Ρ€Π΅Π· систСму ΠΈΠ· Π΄Π²ΡƒΡ… шкивов, ΠΊΠ°ΠΊ ΠΏΠΎΠΊΠ°Π·Π°Π½ΠΎ Π½Π° схСмС Π½ΠΈΠΆΠ΅. Π‘Π»ΠΎΠΊ опираСтся Π½Π° ΡˆΠ΅Ρ€ΠΎΡ…ΠΎΠ²Π°Ρ‚ΡƒΡŽ (коэффициСнт трСния покоя 0,400) Π³ΠΎΡ€ΠΈΠ·ΠΎΠ½Ρ‚Π°Π»ΡŒΠ½ΡƒΡŽ ΠΏΠΎΠ²Π΅Ρ€Ρ…Π½ΠΎΡΡ‚ΡŒ. Π—Π°Ρ‚Π΅ΠΌ больший шкив тянСт Π²Π²Π΅Ρ€Ρ… с постСпСнно ΡƒΠ²Π΅Π»ΠΈΡ‡ΠΈΠ²Π°ΡŽΡ‰Π΅ΠΉΡΡ силой. Оба шкива бСзмассовыС ΠΈ Π±Π΅Π· трСния, ΠΊΠ°Π½Π°Ρ‚ Ρ‚Π°ΠΊΠΆΠ΅ бСзмассовый. МСньший шкив ΠΏΡ€ΠΈΠΊΡ€Π΅ΠΏΠ»Π΅Π½ ΠΊ ΠΏΠΎΠ»Ρƒ, ΠΈ ΠΎΠ±Π° шкива располоТСны Ρ‚Π°ΠΊ, Ρ‡Ρ‚ΠΎ Π²Π΅Ρ€Π΅Π²ΠΊΠ° пСрпСндикулярна ΠΏΠΎΠ»Ρƒ Π½Π° ΠΎΠ΄Π½ΠΎΠΌ ΠΊΠΎΠ½Ρ†Π΅ ΠΈ ΠΏΠ°Ρ€Π°Π»Π»Π΅Π»ΡŒΠ½Π° Π΅ΠΌΡƒ Π½Π° Π΄Ρ€ΡƒΠ³ΠΎΠΌ.Когда тяговоС усилиС достигаСт ΠΎΠΏΡ€Π΅Π΄Π΅Π»Π΅Π½Π½ΠΎΠΉ Π²Π΅Π»ΠΈΡ‡ΠΈΠ½Ρ‹, Π±Π»ΠΎΠΊ Π΅Π΄Π²Π° Π½Π°Ρ‡ΠΈΠ½Π°Π΅Ρ‚ ΡΠΊΠΎΠ»ΡŒΠ·ΠΈΡ‚ΡŒ Π²ΠΏΡ€Π°Π²ΠΎ. ВычислитС Π²Π΅Π»ΠΈΡ‡ΠΈΠ½Ρƒ этой тянущСй силы.

      РСшСниС

      Π’ этой Π·Π°Π΄Π°Ρ‡Π΅ Π½Π΅Ρ‚ Π½Π°ΠΊΠ»ΠΎΠ½Π½ΠΎΠΉ плоскости (хотя ΠΎΠ½Π° ΠΌΠΎΠΆΠ΅Ρ‚ Π±Ρ‹Ρ‚ΡŒ!), Но это Ρ…ΠΎΡ€ΠΎΡˆΠΈΠΉ ΠΏΡ€ΠΈΠΌΠ΅Ρ€ ваТности слСдования ΡƒΠΊΠ°Π·Π°Π½Π½Ρ‹ΠΌ Π²Ρ‹ΡˆΠ΅ прСдписаниям. НачнитС с Π΄ΠΈΠ°Π³Ρ€Π°ΠΌΠΌ свободного Ρ‚Π΅Π»Π° ΠΈ систСм ΠΊΠΎΠΎΡ€Π΄ΠΈΠ½Π°Ρ‚. FBD мСньшСго шкива Π½Π΅ принСсСт Π½Π°ΠΌ Π½ΠΈΡ‡Π΅Π³ΠΎ ΠΏΠΎΠ»Π΅Π·Π½ΠΎΠ³ΠΎ, поэтому Π½ΡƒΠΆΠ½ΠΎ Π½Π°Ρ€ΠΈΡΠΎΠ²Π°Ρ‚ΡŒ Ρ‚ΠΎΠ»ΡŒΠΊΠΎ Π΄Π²Π° FBD.ΠžΠ±Ρ€Π°Ρ‚ΠΈΡ‚Π΅ Π²Π½ΠΈΠΌΠ°Π½ΠΈΠ΅, Ρ‡Ρ‚ΠΎ натяТСниС Π½Π° сторонС Π±Π»ΠΎΠΊΠ° происходит ΠΎΡ‚ Ρ‚ΠΎΠΉ ΠΆΠ΅ Π²Π΅Ρ€Π΅Π²ΠΊΠΈ, Ρ‡Ρ‚ΠΎ ΠΈ натяТСниС Π½Π° Π²Π΅Ρ€Ρ…Π½Π΅ΠΉ части Π±Π»ΠΎΠΊΠ°, поэтому ΠΎΠ½ΠΈ Ρ€Π°Π²Π½Ρ‹:

      Π‘Π»ΠΎΠΊ Π²ΠΎΠΎΠ±Ρ‰Π΅ Π½Π΅ ускоряСтся (ΠΊΠ°ΠΊ ΠΈ шкив), поэтому сумма сил Π² ΠΊΠ°ΠΆΠ΄ΠΎΠΌ ΠΈΠ· Π½Π°ΠΏΡ€Π°Π²Π»Π΅Π½ΠΈΠΉ \ (x \) ΠΈ \ (y \) Ρ€Π°Π²Π½Π° Π½ΡƒΠ»ΡŽ.

      Π‘Π»ΠΎΠΊ \ [\ begin {array} {l}: \; \; \ begin {array} {l} x – Π½Π°ΠΏΡ€Π°Π²Π»Π΅Π½ΠΈΠ΅ \; силы: \; \; 0 = T – f \\ y – Π½Π°ΠΏΡ€Π°Π²Π»Π΅Π½ΠΈΠ΅ \; силы: \; \; 0 = T + N-mg \ end {array} \\ шкив: \; \; y – Π½Π°ΠΏΡ€Π°Π²Π»Π΅Π½ΠΈΠ΅ \; силы: \; \; 0 = Ρ‚ΡΠ½ΡƒΡ‚ΡŒ – 2T \ end {array} \ nonumber \]

      Если Π½Π°ΠΌ Π½ΡƒΠΆΠ½ΠΎ Ρ‚ΡΠ½ΡƒΡ‚ΡŒ «достаточно сильно», Ρ‡Ρ‚ΠΎΠ±Ρ‹ Π·Π°ΡΡ‚Π°Π²ΠΈΡ‚ΡŒ Π±Π»ΠΎΠΊ Π΄Π²ΠΈΠ³Π°Ρ‚ΡŒΡΡ, Ρ‚ΠΎ это происходит, ΠΊΠΎΠ³Π΄Π° Π³ΠΎΡ€ΠΈΠ·ΠΎΠ½Ρ‚Π°Π»ΡŒΠ½ΠΎΠ΅ усилиС Ρ€Π°Π²Π½ΠΎ максимальной силС статичСского трСния, Ρ‡Ρ‚ΠΎ Π΄Π°Π΅Ρ‚ Π½Π°ΠΌ ΡƒΡ€Π°Π²Π½Π΅Π½ΠΈΠ΅ ограничСния:

      \ [f = \ mu_S N \ nonumber \]

      ΠžΠ±Ρ€Π°Ρ‚ΠΈΡ‚Π΅ Π²Π½ΠΈΠΌΠ°Π½ΠΈΠ΅, Ρ‡Ρ‚ΠΎ Π±Π»ΠΎΠΊ Π΄ΠΎΠ»ΠΆΠ΅Π½ Π½Π°Ρ‡Π°Ρ‚ΡŒ ΡΠΊΠΎΠ»ΡŒΠ·ΠΈΡ‚ΡŒ, ΠΏΡ€Π΅ΠΆΠ΄Π΅ Ρ‡Π΅ΠΌ ΠΎΠ½ Π½Π°Ρ‡Π½Π΅Ρ‚ ΠΏΠΎΠ΄Π½ΠΈΠΌΠ°Ρ‚ΡŒΡΡ, ΠΏΠΎΡ‚ΠΎΠΌΡƒ Ρ‡Ρ‚ΠΎ для подъСма трСбуСтся, Ρ‡Ρ‚ΠΎΠ±Ρ‹ Π½ΠΎΡ€ΠΌΠ°Π»ΡŒΠ½Π°Ρ сила Π±Ρ‹Π»Π° Ρ€Π°Π²Π½Π° Π½ΡƒΠ»ΡŽ, ΠΈ ΠΎΠ½ Π±ΡƒΠ΄Π΅Ρ‚ ΡΠΊΠΎΠ»ΡŒΠ·ΠΈΡ‚ΡŒ, ΠΊΠΎΠ³Π΄Π° сила статичСского трСния ΠΌΠ°Π»Π°, Π½ΠΎ Π½Π΅ Ρ€Π°Π²Π½Π° Π½ΡƒΠ»ΡŽ.Π’Π΅ΠΏΠ΅Ρ€ΡŒ Ρ€Π΅ΡˆΠΈΡ‚Π΅ уравнСния ΠΎΠ΄Π½ΠΎΠ²Ρ€Π΅ΠΌΠ΅Π½Π½ΠΎ, Ρ‡Ρ‚ΠΎΠ±Ρ‹ ΠΏΠΎΠ»ΡƒΡ‡ΠΈΡ‚ΡŒ:

      \ [pull = \ boxed {280N} \ nonumber \]

      Π’ΠΊΠ»ΡŽΡ‡Π°Ρ Π΄Π²ΠΈΠΆΠ΅Π½ΠΈΠ΅

      ΠœΡ‹ сдСлали Π΄Π²Π° ΠΏΡ€ΠΈΠΌΠ΅Ρ€Π° с систСмами, для ΠΊΠΎΡ‚ΠΎΡ€Ρ‹Ρ… ускорСниС Ρ€Π°Π²Π½ΠΎ Π½ΡƒΠ»ΡŽ. Но, ΠΊΠΎΠ½Π΅Ρ‡Π½ΠΎ, Π²ΠΎΠ·ΠΌΠΎΠΆΠ½ΠΎ, Ρ‡Ρ‚ΠΎ ΠΏΡ€ΠΎΠ±Π»Π΅ΠΌΠ° Π΄Π΅ΠΉΡΡ‚Π²ΠΈΡ‚Π΅Π»ΡŒΠ½ΠΎ связана с ускорСниСм ΠΎΠ±ΡŠΠ΅ΠΊΡ‚ΠΎΠ². Иногда нас просят Π½Π°ΠΉΡ‚ΠΈ это ускорСниС, Π° ΠΈΠ½ΠΎΠ³Π΄Π° ускорСниС – это Ρ‡Π°ΡΡ‚ΡŒ ΠΈΠ½Ρ„ΠΎΡ€ΠΌΠ°Ρ†ΠΈΠΈ, которая даСтся. УскорСниС ΠΌΠΎΠΆΠ΅Ρ‚ Π±Ρ‹Ρ‚ΡŒ Π·Π°Π΄Π°Π½ΠΎ Π½Π°ΠΏΡ€ΡΠΌΡƒΡŽ, ΠΈΠ»ΠΈ, Π²ΠΎΠ·ΠΌΠΎΠΆΠ½ΠΎ, ΠΎΠ½ΠΎ ΠΌΠΎΠΆΠ΅Ρ‚ Π±Ρ‹Ρ‚ΡŒ вычислСно Π΄Ρ€ΡƒΠ³ΠΈΠΌ способом, Π²ΠΎΠ·ΠΌΠΎΠΆΠ½ΠΎ, исходя ΠΈΠ· ΠΊΠΈΠ½Π΅ΠΌΠ°Ρ‚ΠΈΠΊΠΈ ΠΈΠ»ΠΈ Ссли Π΄Π²ΠΈΠΆΠ΅Π½ΠΈΠ΅ являСтся ΠΊΡ€ΡƒΠ³ΠΎΠ²Ρ‹ΠΌ.Π—Π½Π°Π½ΠΈΠ΅ Ρ‡Π΅Π³ΠΎ-Π»ΠΈΠ±ΠΎ ΠΎ Π΄Π²ΠΈΠΆΠ΅Π½ΠΈΠΈ ΠΎΠ±ΡŠΠ΅ΠΊΡ‚Π° ΠΏΠΎΠ΄ΠΏΠ°Π΄Π°Π΅Ρ‚ ΠΏΠΎΠ΄ ΠΊΠ°Ρ‚Π΅Π³ΠΎΡ€ΠΈΡŽ Β«ΠΎΠ³Ρ€Π°Π½ΠΈΡ‡Π΅Π½ΠΈΠΉΒ», ΠΏΠΎΡ‚ΠΎΠΌΡƒ Ρ‡Ρ‚ΠΎ Π΄Π²ΠΈΠΆΠ΅Π½ΠΈΠ΅ Π·Π°Π΄Π°Π½ΠΎ (ΠΎΠ³Ρ€Π°Π½ΠΈΡ‡Π΅Π½ΠΎ), Ρ‡Ρ‚ΠΎ ΠΏΡ€ΠΈΠ²ΠΎΠ΄ΠΈΡ‚ ΠΊ уравнСниям, ΠΊΠΎΡ‚ΠΎΡ€Ρ‹Π΅ Π½Π΅ ΡΠ²Π»ΡΡŽΡ‚ΡΡ Ρ€Π΅Π·ΡƒΠ»ΡŒΡ‚Π°Ρ‚ΠΎΠΌ Π²Ρ‚ΠΎΡ€ΠΎΠ³ΠΎ Π·Π°ΠΊΠΎΠ½Π° ΠΡŒΡŽΡ‚ΠΎΠ½Π°. ΠœΡ‹ рассмотрим ΠΏΡ€ΠΈΠΌΠ΅Ρ€, Π² ΠΊΠΎΡ‚ΠΎΡ€ΠΎΠΌ ΠΈΡΠΏΠΎΠ»ΡŒΠ·ΡƒΡŽΡ‚ΡΡ шаги для Ρ€Π΅ΡˆΠ΅Π½ΠΈΡ Π·Π°Π΄Π°Ρ‡ ΠΌΠ΅Ρ…Π°Π½ΠΈΠΊΠΈ, ΠΊΠΎΡ‚ΠΎΡ€Ρ‹Π΅ Ρ‚Π°ΠΊΠΆΠ΅ Π²ΠΊΠ»ΡŽΡ‡Π°ΡŽΡ‚ Π΄ΠΎΠΏΠΎΠ»Π½ΠΈΡ‚Π΅Π»ΡŒΠ½ΠΎΠ΅ ΠΎΠ³Ρ€Π°Π½ΠΈΡ‡Π΅Π½ΠΈΠ΅ ΠΊΡ€ΡƒΠ³ΠΎΠ²ΠΎΠ³ΠΎ двиТСния. ΠŸΡ€Π΅ΠΆΠ΄Π΅ Ρ‡Π΅ΠΌ ΠΌΡ‹ это сдСлаСм, ΠΏΠΎΠΏΡ€ΠΎΠ±ΡƒΠΉΡ‚Π΅ ΡΠ»Π΅Π΄ΡƒΡŽΡ‰ΠΈΠΉ ΠΏΡ€ΠΈΠΌΠ΅Ρ€:

        ΠŸΡ€ΠΈΠΌΠ΅Ρ€ \ (\ PageIndex {3} \)

        Π‘Π»ΠΎΠΊ ΠΏΡ€ΠΈΠΊΡ€Π΅ΠΏΠ»Π΅Π½ ΠΊ ΠΎΠ΄Π½ΠΎΠΌΡƒ ΠΊΠΎΠ½Ρ†Ρƒ бСзмассовой ΠΏΡ€ΡƒΠΆΠΈΠ½Ρ‹, Π΄Ρ€ΡƒΠ³ΠΎΠΉ ΠΊΠΎΠ½Π΅Ρ† ΠΊΠΎΡ‚ΠΎΡ€ΠΎΠΉ ΠΏΡ€ΠΈΠΊΡ€Π΅ΠΏΠ»Π΅Π½ ΠΊ Π²Π΅Ρ€Ρ‚ΠΈΠΊΠ°Π»ΡŒΠ½ΠΎΠΌΡƒ Π½Π΅ΠΏΠΎΠ΄Π²ΠΈΠΆΠ½ΠΎΠΌΡƒ ΡˆΡ‚ΠΈΡ„Ρ‚Ρƒ Π½Π° Π³ΠΎΡ€ΠΈΠ·ΠΎΠ½Ρ‚Π°Π»ΡŒΠ½ΠΎΠΉ повСрхности Π±Π΅Π· трСния.Π‘Π»ΠΎΠΊ вращаСтся ΠΏΠΎ ΠΊΡ€ΡƒΠ³Ρƒ, ΠΈ Π² Ρ€Π΅Π·ΡƒΠ»ΡŒΡ‚Π°Ρ‚Π΅ этого двиТСния ΠΏΡ€ΡƒΠΆΠΈΠ½Π° растягиваСтся. ЀактичСски, Ρ‡Π΅ΠΌ быстрСС Π΄Π²ΠΈΠΆΠ΅Π½ΠΈΠ΅, Ρ‚Π΅ΠΌ большС растягиваСтся ΠΏΡ€ΡƒΠΆΠΈΠ½Π°. Π§Ρ‚ΠΎΠ±Ρ‹ Ρ€Π°ΡΡ‚ΡΠ½ΡƒΡ‚ΡŒ Π»ΡŽΠ±ΡƒΡŽ ΠΏΡ€ΡƒΠΆΠΈΠ½Ρƒ, Π½ΡƒΠΆΠ½ΠΎ ΠΎΠ΄Π½ΠΎΠ²Ρ€Π΅ΠΌΠ΅Π½Π½ΠΎ Ρ‚ΡΠ½ΡƒΡ‚ΡŒ Π·Π° ΠΎΠ±Π° ΠΊΠΎΠ½Ρ†Π°. Ясно, Ρ‡Ρ‚ΠΎ ΠΊΠΎΠ»Ρ‹ΡˆΠ΅ΠΊ натягиваСт ΠΎΠ΄ΠΈΠ½ ΠΊΠΎΠ½Π΅Ρ† ΠΏΡ€ΡƒΠΆΠΈΠ½Ρ‹, ΠΊΠΎΠ³Π΄Π° Π±Π»ΠΎΠΊ двиТСтся ΠΏΠΎ ΠΊΡ€ΡƒΠ³Ρƒ, Π½ΠΎ какая сила тянСт Π±Π»ΠΎΠΊ Π½Π°Ρ€ΡƒΠΆΡƒ, Ρ‡Ρ‚ΠΎΠ±Ρ‹ Ρ€Π°ΡΡ‚ΡΠ½ΡƒΡ‚ΡŒ ΠΏΡ€ΡƒΠΆΠΈΠ½Ρƒ?

          РСшСниС

          Π‘Π»ΠΎΠΊ Π½Π΅ Π²Ρ‹Ρ‚Π°Ρ‰ΠΈΠ» Π½Π°Ρ€ΡƒΠΆΡƒ ! Он Ρ‚ΠΎΠ»ΡŒΠΊΠΎ втягиваСтся Π²Π½ΡƒΡ‚Ρ€ΡŒ (ΠΏΡ€ΡƒΠΆΠΈΠ½ΠΎΠΉ).Π§Ρ‚ΠΎΠ±Ρ‹ Ρ€Π°ΡΡ‚ΡΠ½ΡƒΡ‚ΡŒ ΠΏΡ€ΡƒΠΆΠΈΠ½Ρƒ, Π½ΡƒΠΆΠ½ΠΎ Π²Ρ‹Ρ‚ΡΠ½ΡƒΡ‚ΡŒ Π½Π°Ρ€ΡƒΠΆΡƒ Π½Π΅ Π±Π»ΠΎΠΊ, Π° ΠΏΡ€ΡƒΠΆΠΈΠ½Ρƒ, ΠΊΠΎΡ‚ΠΎΡ€ΡƒΡŽ Π½ΡƒΠΆΠ½ΠΎ ΠΏΠΎΡ‚ΡΠ½ΡƒΡ‚ΡŒ Ρ‚Π°ΠΊΠΈΠΌ ΠΎΠ±Ρ€Π°Π·ΠΎΠΌ. ΠŸΡ€ΡƒΠΆΠΈΠ½Π° втягиваСт Π±Π»ΠΎΠΊ Π²Π½ΡƒΡ‚Ρ€ΡŒ (поддСрТивая Π΅Π³ΠΎ Ρ†Π΅Π½Ρ‚Ρ€ΠΎΡΡ‚Ρ€Π΅ΠΌΠΈΡ‚Π΅Π»ΡŒΠ½ΠΎΠ΅ ускорСниС), ΠΈ сила Ρ‚Ρ€Π΅Ρ‚ΡŒΠ΅Π³ΠΎ Π·Π°ΠΊΠΎΠ½Π° ΠΏΠ°Ρ€Ρ‹ Π±Π»ΠΎΠΊΠ° Π½Π° ΠΏΡ€ΡƒΠΆΠΈΠ½Π΅ – это Ρ‚ΠΎ, Ρ‡Ρ‚ΠΎ тянСт ΠΏΡ€ΡƒΠΆΠΈΠ½Ρƒ Π½Π°Ρ€ΡƒΠΆΡƒ.

          Π’ΠΎΠ·ΠΌΠΎΠΆΠ½ΠΎ, это Π»ΡƒΡ‡ΡˆΠ΅, Ρ‡Π΅ΠΌ любой Π΄Ρ€ΡƒΠ³ΠΎΠΉ ΠΏΡ€ΠΈΠΌΠ΅Ρ€, ΡƒΠΊΠ°Π·Ρ‹Π²Π°Π΅Ρ‚ Π½Π° Π²Π°ΠΆΠ½ΠΎΡΡ‚ΡŒ изоляции ΠΎΠ±ΡŠΠ΅ΠΊΡ‚ΠΎΠ² с ΠΏΠΎΠΌΠΎΡ‰ΡŒΡŽ силовых Π΄ΠΈΠ°Π³Ρ€Π°ΠΌΠΌ. Π‘Π»ΠΎΠΊ здСсь Π½Π΅ являСтся ΠΏΡ€ΠΎΠ²ΠΎΠ΄Π½ΠΈΠΊΠΎΠΌ для ΠΊΠ°ΠΊΠΎΠΉ-Ρ‚ΠΎ таинствСнной силы, Π²Ρ‹Ρ‚ΡΠ³ΠΈΠ²Π°ΡŽΡ‰Π΅ΠΉ ΠΏΡ€ΡƒΠΆΠΈΠ½Ρƒ – это ΠΎΠ±ΡŠΠ΅ΠΊΡ‚, Π²Ρ‹Ρ‚ΡΠ³ΠΈΠ²Π°ΡŽΡ‰ΠΈΠΉ ΠΏΡ€ΡƒΠΆΠΈΠ½Ρƒ.Π’Π°ΠΌ Π½ΡƒΠΆΠ½ΠΎ ΠΏΠΎΠ»Π½ΠΎΡΡ‚ΡŒΡŽ Π΄ΠΎΠ²Π΅Ρ€ΡΡ‚ΡŒ Ρ‚Ρ€Π΅Ρ‚ΡŒΠ΅ΠΌΡƒ Π·Π°ΠΊΠΎΠ½Ρƒ, Ρ‡Ρ‚ΠΎΠ±Ρ‹ ΠΏΠΎΠ»ΡƒΡ‡ΠΈΡ‚ΡŒ силу ΠΌΠ΅ΠΆΠ΄Ρƒ ΠΏΡ€ΡƒΠΆΠΈΠ½ΠΎΠΉ ΠΈ Π±Π»ΠΎΠΊΠΎΠΌ, ΠΈ Π²Π°ΠΌ Π½ΡƒΠΆΠ½ΠΎ ΠΏΠΎΠ»Π½ΠΎΡΡ‚ΡŒΡŽ Π΄ΠΎΠ²Π΅Ρ€ΡΡ‚ΡŒ Π²Ρ‚ΠΎΡ€ΠΎΠΌΡƒ Π·Π°ΠΊΠΎΠ½Ρƒ, Ρ‡Ρ‚ΠΎΠ±Ρ‹ ΠΏΠΎΠ½ΡΡ‚ΡŒ, Ρ‡Ρ‚ΠΎ Π±Π»ΠΎΠΊ Π½Π΅ Ρ‚Ρ€Π΅Π±ΡƒΠ΅Ρ‚ Π΄ΠΎΠΏΠΎΠ»Π½ΠΈΡ‚Π΅Π»ΡŒΠ½ΠΎΠΉ силы, Π²ΠΎΠ·Π΄Π΅ΠΉΡΡ‚Π²ΡƒΡŽΡ‰Π΅ΠΉ Π½Π° Π½Π΅Π³ΠΎ Π½Π°Ρ€ΡƒΠΆΡƒ, Ρ‡Ρ‚ΠΎΠ±Ρ‹ ΡƒΡ€Π°Π²Π½ΠΎΠ²Π΅ΡΠΈΡ‚ΡŒ силу ΠΏΡ€ΡƒΠΆΠΈΠ½Ρ‹, ΠΏΠΎΡ‚ΠΎΠΌΡƒ Ρ‡Ρ‚ΠΎ это ускоряСтся.

          Π’Π΅ΠΏΠ΅Ρ€ΡŒ ΠΎΠ± ΠΎΠ±Π΅Ρ‰Π°Π½Π½ΠΎΠΌ ΠΏΡ€ΠΈΠΌΠ΅Ρ€Π΅, ΠΊΠΎΡ‚ΠΎΡ€Ρ‹ΠΉ Π²ΠΊΠ»ΡŽΡ‡Π°Π΅Ρ‚ Π΄Π²ΠΈΠΆΠ΅Π½ΠΈΠ΅ Π² соотвСтствии с ΠΏΠΎΡˆΠ°Π³ΠΎΠ²Ρ‹ΠΌΠΈ инструкциями, ΠΏΡ€ΠΈΠ²Π΅Π΄Π΅Π½Π½Ρ‹ΠΌΠΈ Ρ€Π°Π½Π΅Π΅. Π§Ρ‚ΠΎ Π΄Π΅Π»Π°Π΅Ρ‚ эту ΠΏΡ€ΠΎΠ±Π»Π΅ΠΌΡƒ интСрСсной, Ρ‚Π°ΠΊ это информация, которая скрыта Π²Π½ΡƒΡ‚Ρ€ΠΈ Ρ„ΠΎΡ€ΠΌΡƒΠ»ΠΈΡ€ΠΎΠ²ΠΊΠΈ …

          КамСнь Π½Π° Ρ‚Π΅Ρ‚ΠΈΠ²Π΅ Π»Π΅Ρ‚Π°Π΅Ρ‚ ΠΏΠΎ ΠΊΡ€ΡƒΠ³Ρƒ Π² Π²Π΅Ρ€Ρ‚ΠΈΠΊΠ°Π»ΡŒΠ½ΠΎΠΉ плоскости (ΠΏΡ€ΠΈ Π½Π°Π»ΠΈΡ‡ΠΈΠΈ Π·Π΅ΠΌΠ½ΠΎΠΉ силы тяТСсти) Ρ‚Π°ΠΊ, Ρ‡Ρ‚ΠΎ Π΅Π΄Π²Π° достигаСт Π²Π΅Ρ€ΡˆΠΈΠ½Ρ‹ (струна остаСтся натянутой Π½Π° всю Π΄Π»ΠΈΠ½Ρƒ, Π½ΠΎ Π½Π΅Ρ‚ натяТСния) продолТая свой ΠΊΡ€ΡƒΠ³ΠΎΠ²ΠΎΠΉ ΠΏΡƒΡ‚ΡŒ.НайдитС ΡΠΊΠΎΡ€ΠΎΡΡ‚ΡŒ камня ΠΏΠΎ Π΄Π»ΠΈΠ½Π΅ струны.

          • Π²Ρ‹Π΄Π΅Π»ΠΈΡ‚ΡŒ ΡΠΎΠΎΡ‚Π²Π΅Ρ‚ΡΡ‚Π²ΡƒΡŽΡ‰ΠΈΠ΅ ΠΎΠ±ΡŠΠ΅ΠΊΡ‚Ρ‹ ΠΈ Π½Π°Ρ€ΠΈΡΠΎΠ²Π°Ρ‚ΡŒ для Π½ΠΈΡ… Π΄ΠΈΠ°Π³Ρ€Π°ΠΌΠΌΡ‹ свободного Ρ‚Π΅Π»Π°

          • Π²Ρ‹Π±Ρ€Π°Ρ‚ΡŒ систСмы ΠΊΠΎΠΎΡ€Π΄ΠΈΠ½Π°Ρ‚ для схСм, ΠΊΠΎΡ‚ΠΎΡ€Ρ‹Π΅ ΡƒΠ΄ΠΎΠ±Π½Ρ‹

          Π’ качСствС ΠΏΠΎΠ»ΠΎΠΆΠΈΡ‚Π΅Π»ΡŒΠ½ΠΎΠ³ΠΎ направлСния \ (y \) Π²Ρ‹Π±Π΅Ρ€Π΅ΠΌ Π²Π½ΠΈΠ·.

          • силы Ρ€Π°Π·Ρ€Ρ‹Π²Π° Π½Π° ΠΊΠΎΠΌΠΏΠΎΠ½Π΅Π½Ρ‚Ρ‹ Π² Π²Ρ‹Π±Ρ€Π°Π½Π½ΠΎΠΉ систСмС ΠΊΠΎΠΎΡ€Π΄ΠΈΠ½Π°Ρ‚

          Π—Π΄Π΅ΡΡŒ Π½Π΅Ρ‚ нСобходимости.

          • суммируйтС силы Π² направлСниях \ (x \) ΠΈ \ (y \) ΠΈ ΠΏΡ€ΠΈΠΌΠ΅Π½ΠΈΡ‚Π΅ Π²Ρ‚ΠΎΡ€ΠΎΠΉ Π·Π°ΠΊΠΎΠ½ ΠΡŒΡŽΡ‚ΠΎΠ½Π° Π² ΠΎΠ±ΠΎΠΈΡ… направлСниях

          \ [a = \ dfrac {\ sum {F_y}} {m} = \ dfrac {T + mg} {m} \]

          ΠŸΠ΅Ρ€Π²ΠΎΠ΅ ΠΎΠ³Ρ€Π°Π½ΠΈΡ‡Π΅Π½ΠΈΠ΅ состоит Π² Ρ‚ΠΎΠΌ, Ρ‡Ρ‚ΠΎ скала ΠΏΠΎΡ‡Ρ‚ΠΈ Π½Π΅ пСрСмСщаСтся. Π§Ρ‚ΠΎ это Π·Π½Π°Ρ‡ΠΈΡ‚? Π§Ρ‚ΠΎΠ±Ρ‹ ΠΎΡ‚Π²Π΅Ρ‚ΠΈΡ‚ΡŒ Π½Π° этот вопрос, ΠΏΠΎΠ΄ΡƒΠΌΠ°ΠΉΡ‚Π΅, Ρ‡Ρ‚ΠΎ Π±Ρ‹ ΠΏΡ€ΠΎΠΈΠ·ΠΎΡˆΠ»ΠΎ, Ссли Π±Ρ‹ камСнь двигался ΠΌΠ΅Π΄Π»Π΅Π½Π½Π΅Π΅ … Он Π²Ρ‹ΠΏΠ°Π» Π±Ρ‹ ΠΈΠ· ΠΊΡ€ΡƒΠ³Π°, Π° это Π·Π½Π°Ρ‡ΠΈΡ‚, Ρ‡Ρ‚ΠΎ струна Π½Π΅ останСтся прямой. Π­Ρ‚ΠΎ ΠΎΠ·Π½Π°Ρ‡Π°Π»ΠΎ Π±Ρ‹, Ρ‡Ρ‚ΠΎ напряТСниС Ρ€Π°Π²Π½ΠΎ Π½ΡƒΠ»ΡŽ.2} {l} \]

          Достаточно просто:

          \ [v = \ sqrt {gl} \]

          Π₯отя это довольно простой ΠΏΡ€ΠΈΠΌΠ΅Ρ€ с Ρ‚ΠΎΡ‡ΠΊΠΈ зрСния прСдпринятых шагов, ΠΎΠ½ ΡƒΠΊΠ°Π·Ρ‹Π²Π°Π΅Ρ‚ Π½Π° ΠΎΠ΄ΠΈΠ½ ΠΎΡ‡Π΅Π½ΡŒ Π²Π°ΠΆΠ½Ρ‹ΠΉ аспСкт этих ΠΏΡ€ΠΎΠ±Π»Π΅ΠΌ. Если Π²Ρ‹ Π½Π΅ Ρ‚Ρ€Π°Ρ‚ΠΈΡ‚Π΅ Π½Π΅ΠΊΠΎΡ‚ΠΎΡ€ΠΎΠ΅ врСмя Π½Π° Ρ€Π°Π·ΠΌΡ‹ΡˆΠ»Π΅Π½ΠΈΡ ΠΎ Ρ‚ΠΎΠΌ, Ρ‡Ρ‚ΠΎ происходит физичСски, Π²Ρ‹, скорСС всСго, упуститС ΠΈΠ· Π²ΠΈΠ΄Ρƒ Β«ΡΠΊΡ€Ρ‹Ρ‚ΡƒΡŽΒ» ΠΈΠ½Ρ„ΠΎΡ€ΠΌΠ°Ρ†ΠΈΡŽ Π² Ρ„ΠΎΡ€ΠΌΡƒΠ»ΠΈΡ€ΠΎΠ²ΠΊΠ΅ ΠΏΡ€ΠΎΠ±Π»Π΅ΠΌΡ‹. Π­Ρ‚ΠΎ Π½Π΅ ΡƒΠ»ΠΎΠ²ΠΊΠ°, Ρ‡Ρ‚ΠΎΠ±Ρ‹ ΡΠ±ΠΈΡ‚ΡŒ вас с Ρ‚ΠΎΠ»ΠΊΡƒ – это ΠΈΠΌΠ΅Π½Π½ΠΎ Ρ‚ΠΎ, с Ρ‡Π΅ΠΌ Π²Ρ‹ ΡΡ‚Π°Π»ΠΊΠΈΠ²Π°Π΅Ρ‚Π΅ΡΡŒ Π² Ρ€Π΅Π°Π»ΡŒΠ½ΠΎΠΌ ΠΌΠΈΡ€Π΅, ΠΊΠΎΠ³Π΄Π° Π²Π°ΠΌ Π½ΡƒΠΆΠ½ΠΎ Ρ€Π΅ΡˆΠΈΡ‚ΡŒ Ρ€Π΅Π°Π»ΡŒΠ½ΡƒΡŽ ΠΏΡ€ΠΎΠ±Π»Π΅ΠΌΡƒ.Π’Π°ΠΌ Π½Π΅ΠΎΠ±Ρ…ΠΎΠ΄ΠΈΠΌΠΎ ΡƒΠΌΠ΅Ρ‚ΡŒ ΠΏΡ€Π΅ΠΎΠ±Ρ€Π°Π·ΠΎΠ²Ρ‹Π²Π°Ρ‚ΡŒ ΠΎΠΏΠΈΡΠ°Ρ‚Π΅Π»ΡŒΠ½Ρ‹Π΅ аспСкты систСмы Π² матСматичСски Π°Π½Π°Π»ΠΈΠ·ΠΈΡ€ΡƒΠ΅ΠΌΡ‹Π΅ Π²Π΅Π»ΠΈΡ‡ΠΈΠ½Ρ‹.

          ΠŸΡ€ΠΈΠΌΠ΅Ρ€ \ (\ PageIndex {4} \)

          Π’Π΅Ρ‚Π΅Ρ€Π±ΠΎΠ» вращаСтся Π²ΠΎΠΊΡ€ΡƒΠ³ ΡˆΠ΅ΡΡ‚Π°, дСлая ΠΏΠΎΠ»Π½Ρ‹ΠΉ ΠΊΡ€ΡƒΠ³ ΠΊΠ°ΠΆΠ΄Ρ‹Π΅ 1,5 сСкунды. ΠžΠ±Ρ‰Π°Ρ Π΄Π»ΠΈΠ½Π° Π²Π΅Ρ€Π΅Π²ΠΊΠΈ – 2,4 ΠΌ. ВычислитС ΡƒΠ³ΠΎΠ» ΞΈ, ΠΊΠΎΡ‚ΠΎΡ€Ρ‹ΠΉ Π²Π΅Ρ€Π΅Π²ΠΊΠ° ΠΎΠ±Ρ€Π°Π·ΡƒΠ΅Ρ‚ с ΡˆΠ΅ΡΡ‚ΠΎΠΌ. Π’Π΅Ρ€Π΅Π²ΠΊΠ° ΠΈΠΌΠ΅Π΅Ρ‚ Π½Π΅Π·Π½Π°Ρ‡ΠΈΡ‚Π΅Π»ΡŒΠ½ΡƒΡŽ массу.

            РСшСниС

            НачнитС с Π΄ΠΈΠ°Π³Ρ€Π°ΠΌΠΌΡ‹ силы мяча, Π²ΠΊΠ»ΡŽΡ‡Π°Ρ систСму ΠΊΠΎΠΎΡ€Π΄ΠΈΠ½Π°Ρ‚:

            Π—Π°Ρ‚Π΅ΠΌ слоТитС силы ΠΏΠΎ осям \ (x \) ΠΈ \ (y \) ΠΈ ΠΏΡ€ΠΈΠΌΠ΅Π½ΠΈΡ‚Π΅ Π²Ρ‚ΠΎΡ€ΠΎΠΉ Π·Π°ΠΊΠΎΠ½ ΠΡŒΡŽΡ‚ΠΎΠ½Π°:

            \ [\ begin {array} {l} a_x = \ dfrac {T \ sin \ theta} {m} \\ a_y = \ dfrac {T \ cos \ theta – mg} {m} \ end {array} \ nonumber \]

            УскорСниС Π² Π½Π°ΠΏΡ€Π°Π²Π»Π΅Π½ΠΈΠΈ \ (x \) являСтся Ρ†Π΅Π½Ρ‚Ρ€ΠΎΡΡ‚Ρ€Π΅ΠΌΠΈΡ‚Π΅Π»ΡŒΠ½Ρ‹ΠΌ, с радиусом Π΅Π³ΠΎ ΠΊΡ€ΡƒΠ³ΠΎΠ²ΠΎΠ³ΠΎ двиТСния, Ρ€Π°Π²Π½Ρ‹ΠΌ Π³ΠΎΡ€ΠΈΠ·ΠΎΠ½Ρ‚Π°Π»ΡŒΠ½ΠΎΠΌΡƒ участку ΠΏΡ€ΡΠΌΠΎΡƒΠ³ΠΎΠ»ΡŒΠ½ΠΎΠ³ΠΎ Ρ‚Ρ€Π΅ΡƒΠ³ΠΎΠ»ΡŒΠ½ΠΈΠΊΠ°, Π³ΠΈΠΏΠΎΡ‚Π΅Π½ΡƒΠ·Π° ΠΊΠΎΡ‚ΠΎΡ€ΠΎΠ³ΠΎ Ρ€Π°Π²Π½Π° Π΄Π»ΠΈΠ½Π΅ струны.o} \ nonumber \]

            Π—Π°Π΄Π°Ρ‡ΠΈ Π΄Π²ΡƒΡ… Ρ‚Π΅Π»

            Π’ Π±Π»ΠΎΠΊΠ΅ Π·Π°ΠΊΠΎΠ½ΠΎΠ² ΠΡŒΡŽΡ‚ΠΎΠ½Π° Π²Π²Π΅Π΄Π΅Π½Π° Ρ‚Π΅ΠΌΠ° Π·Π°Π΄Π°Ρ‡ Π΄Π²ΡƒΡ… Ρ‚Π΅Π». Π‘Ρ‹Π»Π° обсуТдСна ΠΏΠ°Ρ€Π° стратСгий Ρ€Π΅ΡˆΠ΅Π½ΠΈΡ ΠΏΡ€ΠΎΠ±Π»Π΅ΠΌ, ΠΊΠΎΡ‚ΠΎΡ€Ρ‹Π΅ Π±Ρ‹Π»ΠΈ ΠΏΡ€ΠΈΠΌΠ΅Π½Π΅Π½Ρ‹ для Ρ€Π΅ΡˆΠ΅Π½ΠΈΡ Ρ‚Ρ€Π΅Ρ… ΠΏΡ€ΠΈΠΌΠ΅Ρ€ΠΎΠ² ΠΏΡ€ΠΎΠ±Π»Π΅ΠΌ. Π’Π°ΠΊΠΈΠ΅ ΠΏΡ€ΠΎΠ±Π»Π΅ΠΌΡ‹ с двумя Ρ‚Π΅Π»Π°ΠΌΠΈ ΠΎΠ±Ρ‹Ρ‡Π½ΠΎ Π²ΠΊΠ»ΡŽΡ‡Π°ΡŽΡ‚ Ρ€Π΅ΡˆΠ΅Π½ΠΈΠ΅ для ускорСния ΠΎΠ±ΡŠΠ΅ΠΊΡ‚ΠΎΠ² ΠΈ силы, Π΄Π΅ΠΉΡΡ‚Π²ΡƒΡŽΡ‰Π΅ΠΉ ΠΌΠ΅ΠΆΠ΄Ρƒ ΠΎΠ±ΡŠΠ΅ΠΊΡ‚Π°ΠΌΠΈ. Одна ΠΈΠ· стратСгий Ρ€Π΅ΡˆΠ΅Π½ΠΈΡ Π·Π°Π΄Π°Ρ‡ Π΄Π²ΡƒΡ… Ρ‚Π΅Π» Π²ΠΊΠ»ΡŽΡ‡Π°Π΅Ρ‚ использованиС систСмного Π°Π½Π°Π»ΠΈΠ·Π° для опрСдСлСния ускорСния Π² сочСтании с Π°Π½Π°Π»ΠΈΠ·ΠΎΠΌ ΠΎΡ‚Π΄Π΅Π»ΡŒΠ½ΠΎΠ³ΠΎ ΠΎΠ±ΡŠΠ΅ΠΊΡ‚Π° для опрСдСлСния силы, ΠΏΠ΅Ρ€Π΅Π΄Π°Π²Π°Π΅ΠΌΠΎΠΉ ΠΌΠ΅ΠΆΠ΄Ρƒ ΠΎΠ±ΡŠΠ΅ΠΊΡ‚Π°ΠΌΠΈ.Вторая стратСгия Π·Π°ΠΊΠ»ΡŽΡ‡Π°Π»Π°ΡΡŒ Π² использовании Π°Π½Π°Π»ΠΈΠ·Π° Π΄Π²ΡƒΡ… ΠΎΡ‚Π΄Π΅Π»ΡŒΠ½Ρ‹Ρ… ΠΎΠ±ΡŠΠ΅ΠΊΡ‚ΠΎΠ² с Ρ†Π΅Π»ΡŒΡŽ Ρ€Π°Π·Ρ€Π°Π±ΠΎΡ‚ΠΊΠΈ систСмы ΠΈΠ· Π΄Π²ΡƒΡ… ΡƒΡ€Π°Π²Π½Π΅Π½ΠΈΠΉ для Ρ€Π΅ΡˆΠ΅Π½ΠΈΡ Π΄Π²ΡƒΡ… нСизвСстных Π²Π΅Π»ΠΈΡ‡ΠΈΠ½. ΠŸΡ€ΠΈ нСобходимости Π½Π°ΠΉΠ΄ΠΈΡ‚Π΅ врСмя, Ρ‡Ρ‚ΠΎΠ±Ρ‹ ΠΏΡ€ΠΎΡΠΌΠΎΡ‚Ρ€Π΅Ρ‚ΡŒ страницу ΠΎ Ρ€Π΅ΡˆΠ΅Π½ΠΈΠΈ Π·Π°Π΄Π°Ρ‡ Π΄Π²ΡƒΡ… Ρ‚Π΅Π». Π­Ρ‚Π° страница Π±ΡƒΠ΄Π΅Ρ‚ ΠΎΡΠ½ΠΎΠ²Ρ‹Π²Π°Ρ‚ΡŒΡΡ Π½Π° ΡƒΡ€ΠΎΠΊΠ°Ρ…, ΠΈΠ·Π²Π»Π΅Ρ‡Π΅Π½Π½Ρ‹Ρ… Ρ€Π°Π½Π΅Π΅ Π² Ρ€Π°Π·Π΄Π΅Π»Π΅ Β«Π—Π°ΠΊΠΎΠ½Ρ‹ ΠΡŒΡŽΡ‚ΠΎΠ½Π°Β».

            Π’ этом ΡƒΡ€ΠΎΠΊΠ΅ ΠΌΡ‹ ΠΏΡ€ΠΎΠ°Π½Π°Π»ΠΈΠ·ΠΈΡ€ΡƒΠ΅ΠΌ Π·Π°Π΄Π°Ρ‡ΠΈ ΠΎ Π΄Π²ΡƒΡ… Ρ‚Π΅Π»Π°Ρ…, Π² ΠΊΠΎΡ‚ΠΎΡ€Ρ‹Ρ… ΠΎΠ±ΡŠΠ΅ΠΊΡ‚Ρ‹ двиТутся Π² Ρ€Π°Π·Π½Ρ‹Ρ… направлСниях. Π’ этих Π·Π°Π΄Π°Ρ‡Π°Ρ… Π΄Π²Π° ΠΎΠ±ΡŠΠ΅ΠΊΡ‚Π° связаны Ρ†Π΅ΠΏΠΎΡ‡ΠΊΠΎΠΉ, которая ΠΏΠ΅Ρ€Π΅Π΄Π°Π΅Ρ‚ силу ΠΎΠ΄Π½ΠΎΠ³ΠΎ ΠΎΠ±ΡŠΠ΅ΠΊΡ‚Π° Π΄Ρ€ΡƒΠ³ΠΎΠΌΡƒ.Π‘Ρ‚Ρ€ΡƒΠ½Π° наматываСтся Π½Π° шкив, ΠΊΠΎΡ‚ΠΎΡ€Ρ‹ΠΉ измСняСт Π½Π°ΠΏΡ€Π°Π²Π»Π΅Π½ΠΈΠ΅ прилоТСния силы Π±Π΅Π· измСнСния Π²Π΅Π»ΠΈΡ‡ΠΈΠ½Ρ‹. Π’ качСствС ΠΈΠ»Π»ΡŽΡΡ‚Ρ€Π°Ρ†ΠΈΠΈ Ρ‚ΠΎΠ³ΠΎ, ΠΊΠ°ΠΊ Ρ€Π°Π±ΠΎΡ‚Π°Π΅Ρ‚ шкив, рассмотрим схСму справа. ΠžΠ±ΡŠΠ΅ΠΊΡ‚ A связан с ΠΎΠ±ΡŠΠ΅ΠΊΡ‚ΠΎΠΌ B строкой. Π’Π΅Ρ€Π΅Π²ΠΊΠ° наматываСтся Π½Π° шкив Π² ΠΊΠΎΠ½Ρ†Π΅ стола. ΠžΠ±ΡŠΠ΅ΠΊΡ‚ A подвСшСн Π² Π²ΠΎΠ·Π΄ΡƒΡ…Π΅, Π° ΠΎΠ±ΡŠΠ΅ΠΊΡ‚ B Π»Π΅ΠΆΠΈΡ‚ Π½Π° столС. Π’ этой ситуации ΠΎΠ±ΡŠΠ΅ΠΊΡ‚ А ΡƒΠΏΠ°Π΄Π΅Ρ‚ Π²Π½ΠΈΠ· ΠΏΠΎΠ΄ дСйствиСм силы тяТСсти, потянув Π²Π½ΠΈΠ· ΠΎΠ΄ΠΈΠ½ ΠΊΠΎΠ½Π΅Ρ† струны, ΠΊ ΠΊΠΎΡ‚ΠΎΡ€ΠΎΠΉ ΠΎΠ½ подсоСдинСн.Богласно Π·Π°ΠΊΠΎΠ½Ρƒ дСйствия-противодСйствия ΠΡŒΡŽΡ‚ΠΎΠ½Π°, этот Π½ΠΈΠΆΠ½ΠΈΠΉ ΠΊΠΎΠ½Π΅Ρ† струны Π±ΡƒΠ΄Π΅Ρ‚ Ρ‚ΡΠ½ΡƒΡ‚ΡŒ Π²Π²Π΅Ρ€Ρ… Π½Π° ΠΎΠ±ΡŠΠ΅ΠΊΡ‚ A. ΠŸΡ€ΠΎΡ‚ΠΈΠ²ΠΎΠΏΠΎΠ»ΠΎΠΆΠ½Ρ‹ΠΉ ΠΊΠΎΠ½Π΅Ρ† струны соСдинСн с ΠΎΠ±ΡŠΠ΅ΠΊΡ‚ΠΎΠΌ B. Π­Ρ‚ΠΎΡ‚ ΠΊΠΎΠ½Π΅Ρ† струны тянСт Π²ΠΏΡ€Π°Π²ΠΎ Π½Π° ΠΎΠ±ΡŠΠ΅ΠΊΡ‚ B. соСдинСниС Π΄Π²ΡƒΡ… ΠΎΠ±ΡŠΠ΅ΠΊΡ‚ΠΎΠ² тянСт ΠΎΠ±Π° ΠΎΠ±ΡŠΠ΅ΠΊΡ‚Π° с ΠΎΠ΄ΠΈΠ½Π°ΠΊΠΎΠ²ΠΎΠΉ силой, Π½ΠΎ Π² Ρ€Π°Π·Π½Ρ‹Ρ… направлСниях. Врос тянСт Π²Π²Π΅Ρ€Ρ… Π½Π° ΠΎΠ±ΡŠΠ΅ΠΊΡ‚ A ΠΈ Π²ΠΏΡ€Π°Π²ΠΎ Π½Π° ΠΎΠ±ΡŠΠ΅ΠΊΡ‚ B. Π¨ΠΊΠΈΠ² ΠΈΠ·ΠΌΠ΅Π½ΠΈΠ» Π½Π°ΠΏΡ€Π°Π²Π»Π΅Π½ΠΈΠ΅ прилоТСния силы.

            ΠŸΡ€ΠΎΠ±Π»Π΅ΠΌΡ‹, связанныС с двумя ΠΎΠ±ΡŠΠ΅ΠΊΡ‚Π°ΠΌΠΈ, ΡΠΎΠ΅Π΄ΠΈΠ½ΠΈΡ‚Π΅Π»ΡŒΠ½Ρ‹ΠΌΠΈ струнами ΠΈ шкивами, Ρ…Π°Ρ€Π°ΠΊΡ‚Π΅Ρ€ΠΈΠ·ΡƒΡŽΡ‚ΡΡ ΠΎΠ±ΡŠΠ΅ΠΊΡ‚Π°ΠΌΠΈ, ΠΊΠΎΡ‚ΠΎΡ€Ρ‹Π΅ двиТутся (ΠΈΠ»ΠΈ Π΄Π°ΠΆΠ΅ ΡƒΡΠΊΠΎΡ€ΡΡŽΡ‚ΡΡ) Π² Ρ€Π°Π·Π½Ρ‹Ρ… направлСниях.Они двиТутся ΠΈΠ»ΠΈ ΡƒΡΠΊΠΎΡ€ΡΡŽΡ‚ΡΡ с ΠΎΠ΄ΠΈΠ½Π°ΠΊΠΎΠ²ΠΎΠΉ ΡΠΊΠΎΡ€ΠΎΡΡ‚ΡŒΡŽ, Π½ΠΎ Π² Ρ€Π°Π·Π½Ρ‹Ρ… направлСниях. Π’Π°ΠΊΠΈΠΌ ΠΎΠ±Ρ€Π°Π·ΠΎΠΌ, ΠΏΡ€ΠΈ Ρ€Π΅ΡˆΠ΅Π½ΠΈΠΈ Ρ‚Π°ΠΊΠΈΡ… Π·Π°Π΄Π°Ρ‡ становится Π²Π°ΠΆΠ½Ρ‹ΠΌ Π²Ρ‹Π±Ρ€Π°Ρ‚ΡŒ Π΄Ρ€ΡƒΠ³ΡƒΡŽ систСму отсчСта ΠΈ систСму осСй для ΠΊΠ°ΠΆΠ΄ΠΎΠ³ΠΎ ΠΎΠ±ΡŠΠ΅ΠΊΡ‚Π°. Π‘Π»Π΅Π΄ΡƒΠ΅Ρ‚ ΠΎΠ±Ρ€Π°Ρ‚ΠΈΡ‚ΡŒ Π²Π½ΠΈΠΌΠ°Π½ΠΈΠ΅ Π½Π° Ρ‚Π°ΠΊΠΎΠΉ Π²Ρ‹Π±ΠΎΡ€ систСмы осСй, Ρ‡Ρ‚ΠΎΠ±Ρ‹ ΠΎΠ±Π° ΠΎΠ±ΡŠΠ΅ΠΊΡ‚Π° ΡƒΡΠΊΠΎΡ€ΡΠ»ΠΈΡΡŒ вдоль оси Π² ΠΏΠΎΠ»ΠΎΠΆΠΈΡ‚Π΅Π»ΡŒΠ½ΠΎΠΌ Π½Π°ΠΏΡ€Π°Π²Π»Π΅Π½ΠΈΠΈ. Π‘ осями, ΠΏΡ€Π°Π²ΠΈΠ»ΡŒΠ½ΠΎ ΠΎΠΏΡ€Π΅Π΄Π΅Π»Π΅Π½Π½Ρ‹ΠΌΠΈ для ΠΊΠ°ΠΆΠ΄ΠΎΠ³ΠΎ ΠΎΡ‚Π΄Π΅Π»ΡŒΠ½ΠΎΠ³ΠΎ ΠΎΠ±ΡŠΠ΅ΠΊΡ‚Π°, ΠΌΠΎΠΆΠ½ΠΎ ΠΏΠΎΡΡ‚Ρ€ΠΎΠΈΡ‚ΡŒ Π΄ΠΈΠ°Π³Ρ€Π°ΠΌΠΌΡƒ свободного Ρ‚Π΅Π»Π°. Π—Π°Ρ‚Π΅ΠΌ ΠΊ ΠΊΠ°ΠΆΠ΄ΠΎΠΉ Π΄ΠΈΠ°Π³Ρ€Π°ΠΌΠΌΠ΅ ΠΌΠΎΠΆΠ½ΠΎ ΠΏΡ€ΠΈΠΌΠ΅Π½ΠΈΡ‚ΡŒ Π·Π°ΠΊΠΎΠ½Ρ‹ ΠΡŒΡŽΡ‚ΠΎΠ½Π°, Ρ‡Ρ‚ΠΎΠ±Ρ‹ ΠΏΠΎΠ»ΡƒΡ‡ΠΈΡ‚ΡŒ систСму ΠΈΠ· Π΄Π²ΡƒΡ… ΡƒΡ€Π°Π²Π½Π΅Π½ΠΈΠΉ для Ρ€Π΅ΡˆΠ΅Π½ΠΈΡ Π΄Π²ΡƒΡ… нСизвСстных.Π­Ρ‚ΠΎΡ‚ процСсс Ρ€Π΅ΡˆΠ΅Π½ΠΈΡ ΠΏΡ€ΠΎΠ±Π»Π΅ΠΌ Π±ΡƒΠ΄Π΅Ρ‚ продСмонстрирован Π½Π° Ρ‚Ρ€Π΅Ρ… Ρ€Π°Π·Π»ΠΈΡ‡Π½Ρ‹Ρ… ΠΏΡ€ΠΈΠΌΠ΅Ρ€Π°Ρ… Π·Π°Π΄Π°Ρ‡.

            ΠŸΡ€ΠΈΠΌΠ΅Ρ€ Π·Π°Π΄Π°Ρ‡ΠΈ 1

            Масса 200,0 Π³Ρ€Π°ΠΌΠΌΠ° (m 1 ) ΠΈ масса 50,0 Π³Ρ€Π°ΠΌΠΌΠ° (m 2 ) соСдинСны Π²Π΅Ρ€Π΅Π²ΠΊΠΎΠΉ. Π‘Ρ‚Ρ€ΡƒΠ½Π° натянута Π½Π° шкив. ΠžΠΏΡ€Π΅Π΄Π΅Π»ΠΈΡ‚Π΅ ускорСниС масс ΠΈ натяТСниС струны.

            Как это часто Π±Ρ‹Π²Π°Π΅Ρ‚, Π² этом ΠΏΡ€ΠΈΠΌΠ΅Ρ€Π΅ ΠΏΡ€ΠΎΠ±Π»Π΅ΠΌΠ° Π·Π°ΠΏΡ€Π°ΡˆΠΈΠ²Π°Π΅Ρ‚ ΠΈΠ½Ρ„ΠΎΡ€ΠΌΠ°Ρ†ΠΈΡŽ ΠΎ Π΄Π²ΡƒΡ… нСизвСстных – ускорСнии ΠΎΠ±ΡŠΠ΅ΠΊΡ‚ΠΎΠ² ΠΈ силС, Π΄Π΅ΠΉΡΡ‚Π²ΡƒΡŽΡ‰Π΅ΠΉ ΠΌΠ΅ΠΆΠ΄Ρƒ ΠΎΠ±ΡŠΠ΅ΠΊΡ‚Π°ΠΌΠΈ.Π’ Ρ‚Π°ΠΊΠΎΠΉ ситуации, ΠΊΠ°ΠΊ эта, ΠΊΠΎΠ³Π΄Π° Π΄Π²Π° ΠΎΠ±ΡŠΠ΅ΠΊΡ‚Π° ΠΏΠΎΠ΄Π²Π΅ΡˆΠ΅Π½Ρ‹ Π½Π° шкивС, Π±ΠΎΠ»Π΅Π΅ массивный ΠΎΠ±ΡŠΠ΅ΠΊΡ‚ Π±ΡƒΠ΄Π΅Ρ‚ ΡƒΡΠΊΠΎΡ€ΡΡ‚ΡŒΡΡ Π²Π½ΠΈΠ·, Π° Π½Π°ΠΈΠΌΠ΅Π½Π΅Π΅ массивный ΠΎΠ±ΡŠΠ΅ΠΊΡ‚ Π±ΡƒΠ΄Π΅Ρ‚ ΡƒΡΠΊΠΎΡ€ΡΡ‚ΡŒΡΡ Π²Π²Π΅Ρ€Ρ…. Π’Π΅Π»ΠΈΡ‡ΠΈΠ½Π° ускорСния Π±ΡƒΠ΄Π΅Ρ‚ ΠΎΠ΄ΠΈΠ½Π°ΠΊΠΎΠ²ΠΎΠΉ для ΠΊΠ°ΠΆΠ΄ΠΎΠ³ΠΎ ΠΎΠ±ΡŠΠ΅ΠΊΡ‚Π°. БистСма ΠΊΠΎΠΎΡ€Π΄ΠΈΠ½Π°Ρ‚, выбранная для m 1 , ΠΈΠΌΠ΅Π΅Ρ‚ ΠΏΠΎΠ»ΠΎΠΆΠΈΡ‚Π΅Π»ΡŒΠ½ΡƒΡŽ ось y, Π½Π°ΠΏΡ€Π°Π²Π»Π΅Π½Π½ΡƒΡŽ Π²Π½ΠΈΠ·; систСма ΠΊΠΎΠΎΡ€Π΄ΠΈΠ½Π°Ρ‚, выбранная для m 2 , ΠΈΠΌΠ΅Π΅Ρ‚ ΠΏΠΎΠ»ΠΎΠΆΠΈΡ‚Π΅Π»ΡŒΠ½ΡƒΡŽ ось y, Π½Π°ΠΏΡ€Π°Π²Π»Π΅Π½Π½ΡƒΡŽ Π²Π²Π΅Ρ€Ρ…. ΠŸΡ€ΠΈ Ρ‚Π°ΠΊΠΎΠΌ Π²Ρ‹Π±ΠΎΡ€Π΅ осСй Π½Π°ΠΏΡ€Π°Π²Π»Π΅Π½ΠΈΠ΅ ускорСния Π±ΡƒΠ΄Π΅Ρ‚ ΠΏΠΎΠ»ΠΎΠΆΠΈΡ‚Π΅Π»ΡŒΠ½Ρ‹ΠΌ для ΠΊΠ°ΠΆΠ΄ΠΎΠ³ΠΎ ΠΎΠ±ΡŠΠ΅ΠΊΡ‚Π°.Π”ΠΈΠ°Π³Ρ€Π°ΠΌΠΌΡ‹ свободного Ρ‚Π΅Π»Π° для ΠΊΠ°ΠΆΠ΄ΠΎΠΉ ΠΈΠ½Π΄ΠΈΠ²ΠΈΠ΄ΡƒΠ°Π»ΡŒΠ½ΠΎΠΉ массы ΠΏΠΎΠΊΠ°Π·Π°Π½Ρ‹ Π½ΠΈΠΆΠ΅. ΠšΠ°ΠΆΠ΄Ρ‹ΠΉ ΠΎΠ±ΡŠΠ΅ΠΊΡ‚ испытываСт Π½ΠΈΡΡ…ΠΎΠ΄ΡΡ‰ΡƒΡŽ силу тяТСсти, которая рассчитываСтся ΠΊΠ°ΠΊ 1 β€’ g ΠΈ 2 β€’ g соотвСтствСнно. ΠšΠ°ΠΆΠ΄Ρ‹ΠΉ ΠΎΠ±ΡŠΠ΅ΠΊΡ‚ Ρ‚Π°ΠΊΠΆΠ΅ испытываСт Π²ΠΎΡΡ…ΠΎΠ΄ΡΡ‰ΡƒΡŽ силу натяТСния, которая притягиваСт Π΄Π²Π° ΠΎΠ±ΡŠΠ΅ΠΊΡ‚Π° Π΄Ρ€ΡƒΠ³ ΠΊ Π΄Ρ€ΡƒΠ³Ρƒ.

            Π£Ρ€Π°Π²Π½Π΅Π½ΠΈΠ΅ Π²Ρ‚ΠΎΡ€ΠΎΠ³ΠΎ Π·Π°ΠΊΠΎΠ½Π°

            ΠΡŒΡŽΡ‚ΠΎΠ½Π° (F net = m β€’ a) ΠΌΠΎΠΆΠ½ΠΎ ΠΏΡ€ΠΈΠΌΠ΅Π½ΠΈΡ‚ΡŒ ΠΊ ΠΎΠ±Π΅ΠΈΠΌ Π΄ΠΈΠ°Π³Ρ€Π°ΠΌΠΌΠ°ΠΌ, Ρ‡Ρ‚ΠΎΠ±Ρ‹ Π½Π°ΠΏΠΈΡΠ°Ρ‚ΡŒ Π΄Π²Π° уравнСния для Π΄Π²ΡƒΡ… нСизвСстных. F net Π±ΡƒΠ΄Π΅Ρ‚ Π²Ρ‹Ρ€Π°ΠΆΠ°Ρ‚ΡŒΡΡ ΠΊΠ°ΠΊ сила Π² Π½Π°ΠΏΡ€Π°Π²Π»Π΅Π½ΠΈΠΈ ускорСния минус сила, которая Π΅ΠΌΡƒ противодСйствуСт.Π’Π°ΠΊΠΈΠΌ ΠΎΠ±Ρ€Π°Π·ΠΎΠΌ, для массы 200,0 Π³Ρ€Π°ΠΌΠΌ F net записываСтся ΠΊΠ°ΠΊ 1,960 N – F дСсятки . Для массы 50,0 Π³Ρ€Π°ΠΌΠΌ F net записываСтся ΠΊΠ°ΠΊ F дСсятки – 0,490 Н. УравнСния 1 ΠΈ 2 ΡΠ²Π»ΡΡŽΡ‚ΡΡ Ρ€Π΅Π·ΡƒΠ»ΡŒΡ‚Π°Ρ‚ΠΎΠΌ примСнСния уравнСния Π²Ρ‚ΠΎΡ€ΠΎΠ³ΠΎ Π·Π°ΠΊΠΎΠ½Π° ΠΡŒΡŽΡ‚ΠΎΠ½Π° ΠΊ массам 200,0 ΠΈ 50,0 Π³Ρ€Π°ΠΌΠΌΠΎΠ². (ΠžΠ±Ρ€Π°Ρ‚ΠΈΡ‚Π΅ Π²Π½ΠΈΠΌΠ°Π½ΠΈΠ΅, Ρ‡Ρ‚ΠΎ значСния массы ΠΏΡ€Π΅ΠΎΠ±Ρ€Π°Π·ΡƒΡŽΡ‚ΡΡ Π² стандартныС ΠΊΠΈΠ»ΠΎΠ³Ρ€Π°ΠΌΠΌΡ‹ ΠΏΠ΅Ρ€Π΅Π΄ использованиСм Π² уравнСниях. Π’Π°ΠΊΠΆΠ΅ ΠΎΠ±Ρ€Π°Ρ‚ΠΈΡ‚Π΅ Π²Π½ΠΈΠΌΠ°Π½ΠΈΠ΅, Ρ‡Ρ‚ΠΎ Π΅Π΄ΠΈΠ½ΠΈΡ†Ρ‹ измСрСния Π±Ρ‹Π»ΠΈ ΠΎΠΏΡƒΡ‰Π΅Π½Ρ‹, Ρ‡Ρ‚ΠΎΠ±Ρ‹ уравнСния Ρ‡ΠΈΡ‚Π°Π»ΠΈΡΡŒ Π±ΠΎΠ»Π΅Π΅ Ρ‡Π΅Ρ‚ΠΊΠΎ.)

            1.960 – F дСсятки = 0,2000 β€’ a

            F дСсятки – 0,490 = 0,0500 β€’ a

            Π‘ этого ΠΌΠΎΠΌΠ΅Π½Ρ‚Π° нСсколько шагов ΠΏΠΎ Π°Π»Π³Π΅Π±Ρ€Π΅ ΠΏΡ€ΠΈΠ²Π΅Π΄ΡƒΡ‚ ΠΊ ΠΎΡ‚Π²Π΅Ρ‚Π°ΠΌ Π½Π° ΠΏΡ€ΠΎΠ±Π»Π΅ΠΌΡƒ. Π£Ρ€Π°Π²Π½Π΅Π½ΠΈΠ΅ 2 ΠΌΠΎΠΆΠ½ΠΎ ΠΏΠ΅Ρ€Π΅Ρ„ΠΎΡ€ΠΌΡƒΠ»ΠΈΡ€ΠΎΠ²Π°Ρ‚ΡŒ, Ρ‡Ρ‚ΠΎΠ±Ρ‹ ΠΏΠΎΠ»ΡƒΡ‡ΠΈΡ‚ΡŒ Π²Ρ‹Ρ€Π°ΠΆΠ΅Π½ΠΈΠ΅ для F дСсятков , записанноС Π² Ρ‚Π΅Ρ€ΠΌΠΈΠ½Π°Ρ… ускорСния.

            F дСсятки = 0,0500 β€’ a + 0,490

            Π­Ρ‚ΠΎ Π²Ρ‹Ρ€Π°ΠΆΠ΅Π½ΠΈΠ΅ для F дСсятков Ρ‚Π΅ΠΏΠ΅Ρ€ΡŒ ΠΌΠΎΠΆΠ½ΠΎ ΠΏΠΎΠ΄ΡΡ‚Π°Π²ΠΈΡ‚ΡŒ Π² ΡƒΡ€Π°Π²Π½Π΅Π½ΠΈΠ΅ 1, Ρ‡Ρ‚ΠΎΠ±Ρ‹ ΠΏΡ€Π΅ΠΎΠ±Ρ€Π°Π·ΠΎΠ²Π°Ρ‚ΡŒ Π΅Π³ΠΎ Π² ΡƒΡ€Π°Π²Π½Π΅Π½ΠΈΠ΅ с ΠΎΠ΄Π½ΠΈΠΌ нСизвСстным.Π­Ρ‚ΠΎ ΡƒΡ€Π°Π²Π½Π΅Π½ΠΈΠ΅ ΠΈ ΠΏΠΎΡΠ»Π΅Π΄ΡƒΡŽΡ‰ΠΈΠ΅ шаги Π°Π»Π³Π΅Π±Ρ€Ρ‹, приводящиС ΠΊ Π·Π½Π°Ρ‡Π΅Π½ΠΈΡŽ ускорСния, ΠΏΠΎΠΊΠ°Π·Π°Π½Ρ‹ Π½ΠΈΠΆΠ΅.

            1,96 – (0,0500 β€’ a + 0,490) = 0,2000 β€’ a
            1,96 – 0,0500 β€’ Π° – 0,490 = 0,2000 β€’ Π°
            1,47 = 0,2500 β€’
            a = 1,47 / 0,2500 = 5,88 м / с 2

            Π’Π΅ΠΏΠ΅Ρ€ΡŒ, ΠΊΠΎΠ³Π΄Π° ускорСниС Π±Ρ‹Π»ΠΎ Π½Π°ΠΉΠ΄Π΅Π½ΠΎ ΠΈΠ· уравнСния 1, Π΅Π³ΠΎ Π·Π½Π°Ρ‡Π΅Π½ΠΈΠ΅ ΠΌΠΎΠΆΠ½ΠΎ ΠΏΠΎΠ΄ΡΡ‚Π°Π²ΠΈΡ‚ΡŒ Π² ΡƒΡ€Π°Π²Π½Π΅Π½ΠΈΠ΅ 3, Ρ‡Ρ‚ΠΎΠ±Ρ‹ ΠΎΠΏΡ€Π΅Π΄Π΅Π»ΠΈΡ‚ΡŒ натяТСниС.

            F дСсятков = 0.0500 β€’ (5,88) + 0,490
            F дСсятков = 0,784 Н

            ΠŸΡ€ΠΎΠ°Π½Π°Π»ΠΈΠ·ΠΈΡ€ΠΎΠ²Π°Π½Π½ΡƒΡŽ здСсь ΡˆΠΊΠΈΠ²Π½ΡƒΡŽ систСму ΠΈΠ½ΠΎΠ³Π΄Π° Π½Π°Π·Ρ‹Π²Π°ΡŽΡ‚ машиной Атвуда. ΠŸΠΎΠ΄Ρ…ΠΎΠ΄ ΠΊ Ρ€Π΅ΡˆΠ΅Π½ΠΈΡŽ ΠΏΡ€ΠΎΠ±Π»Π΅ΠΌ – это стандартный ΠΏΠΎΠ΄Ρ…ΠΎΠ΄, ΠΊΠΎΡ‚ΠΎΡ€Ρ‹ΠΉ Π±ΡƒΠ΄Π΅Ρ‚ ΠΈΡΠΏΠΎΠ»ΡŒΠ·ΠΎΠ²Π°Ρ‚ΡŒΡΡ Π½Π° этой страницС для Ρ€Π΅ΡˆΠ΅Π½ΠΈΡ Π΄Π²ΡƒΡ… нСизвСстных. Он Π±ΡƒΠ΄Π΅Ρ‚ ΠΏΠΎΠ²Ρ‚ΠΎΡ€Π΅Π½ Π² ΠΏΡ€ΠΈΠΌΠ΅Ρ€Π΅ ΠΏΡ€ΠΎΠ±Π»Π΅ΠΌΡ‹ 2, Ρ‡Ρ‚ΠΎΠ±Ρ‹ Ρ€Π΅ΡˆΠΈΡ‚ΡŒ Ρ‚ΠΎ, Ρ‡Ρ‚ΠΎ ΠΎΠ±Ρ‹Ρ‡Π½ΠΎ Π½Π°Π·Ρ‹Π²Π°ΡŽΡ‚ ΠΌΠΎΠ΄ΠΈΡ„ΠΈΡ†ΠΈΡ€ΠΎΠ²Π°Π½Π½ΠΎΠΉ машинной ΠΏΡ€ΠΎΠ±Π»Π΅ΠΌΠΎΠΉ Π­Ρ‚Π²ΡƒΠ΄Π°.


            ΠŸΡ€ΠΈΠΌΠ΅Ρ€ Π·Π°Π΄Π°Ρ‡ΠΈ 2

            Рассмотрим ΡΠΈΡ‚ΡƒΠ°Ρ†ΠΈΡŽ с двумя Ρ‚Π΅Π»Π°ΠΌΠΈ справа.20,0-Π³Ρ€Π°ΠΌΠΌΠΎΠ²Ρ‹ΠΉ подвСсной Π³Ρ€ΡƒΠ· (m 2 ) ΠΏΡ€ΠΈΠΊΡ€Π΅ΠΏΠ»Π΅Π½ ΠΊ 250-Π³Ρ€Π°ΠΌΠΌΠΎΠ²ΠΎΠΌΡƒ ΠΏΠ»Π°Π½Π΅Ρ€Ρƒ с Π²ΠΎΠ·Π΄ΡƒΡˆΠ½Ρ‹ΠΌ гусСничным Ρ…ΠΎΠ΄ΠΎΠΌ (m 1 ). ΠžΠΏΡ€Π΅Π΄Π΅Π»ΠΈΡ‚Π΅ ускорСниС систСмы ΠΈ натяТСниС струны.

            Как ΠΈ Π² ΠΏΡ€ΠΈΠΌΠ΅Ρ€Π΅ Π·Π°Π΄Π°Ρ‡ΠΈ 1, эту систСму Π½Π΅ΠΎΠ±Ρ…ΠΎΠ΄ΠΈΠΌΠΎ сначала ΠΏΡ€ΠΎΠ°Π½Π°Π»ΠΈΠ·ΠΈΡ€ΠΎΠ²Π°Ρ‚ΡŒ ΠΊΠΎΠ½Ρ†Π΅ΠΏΡ‚ΡƒΠ°Π»ΡŒΠ½ΠΎ, Ρ‡Ρ‚ΠΎΠ±Ρ‹ ΠΎΠΏΡ€Π΅Π΄Π΅Π»ΠΈΡ‚ΡŒ Π½Π°ΠΏΡ€Π°Π²Π»Π΅Π½ΠΈΠ΅ ускорСния Π΄Π²ΡƒΡ… ΠΎΠ±ΡŠΠ΅ΠΊΡ‚ΠΎΠ². Π­Ρ‚ΠΎ ΠΏΠΎΠ·Π²ΠΎΠ»ΠΈΡ‚ Π½Π°Π·Π½Π°Ρ‡ΠΈΡ‚ΡŒ оси ΠΊΠΎΠΎΡ€Π΄ΠΈΠ½Π°Ρ‚ для ΠΊΠ°ΠΆΠ΄ΠΎΠ³ΠΎ ΠΎΠ±ΡŠΠ΅ΠΊΡ‚Π°. ΠŸΠΎΡΠΊΠΎΠ»ΡŒΠΊΡƒ Π½ΠΈΡ‡Ρ‚ΠΎ Π½Π΅ Ρ‚ΠΎΠ»ΠΊΠ°Π΅Ρ‚ m 1 Π²Π»Π΅Π²ΠΎ, ΠΌΡ‹ ΠΌΠΎΠ³Π»ΠΈ Π±Ρ‹ ΠΏΡ€Π΅Π΄ΠΏΠΎΠ»ΠΎΠΆΠΈΡ‚ΡŒ, Ρ‡Ρ‚ΠΎ ΠΎΠ½ Π±ΡƒΠ΄Π΅Ρ‚ ΡƒΡΠΊΠΎΡ€ΡΡ‚ΡŒΡΡ Π²ΠΏΡ€Π°Π²ΠΎ ΠΈΠ·-Π·Π° натяТСния струны.Висящая масса (m 2 ) Π±ΡƒΠ΄Π΅Ρ‚ явно ΡƒΡΠΊΠΎΡ€ΡΡ‚ΡŒΡΡ Π²Π½ΠΈΠ· ΠΏΠΎΠ΄ дСйствиСм силы тяТСсти. Π’Π°ΠΊΠΈΠΌ ΠΎΠ±Ρ€Π°Π·ΠΎΠΌ, систСма ΠΊΠΎΠΎΡ€Π΄ΠΈΠ½Π°Ρ‚ Π²Ρ‹Π±Ρ€Π°Π½Π° для m 2 ΠΈΠΌΠ΅Π΅Ρ‚ ΠΏΠΎΠ»ΠΎΠΆΠΈΡ‚Π΅Π»ΡŒΠ½ΡƒΡŽ ось y, Π½Π°ΠΏΡ€Π°Π²Π»Π΅Π½Π½ΡƒΡŽ Π²Π½ΠΈΠ·; систСма ΠΊΠΎΠΎΡ€Π΄ΠΈΠ½Π°Ρ‚, выбранная для m 1 , ΠΈΠΌΠ΅Π΅Ρ‚ ΠΏΠΎΠ»ΠΎΠΆΠΈΡ‚Π΅Π»ΡŒΠ½ΡƒΡŽ ось x, Π½Π°ΠΏΡ€Π°Π²Π»Π΅Π½Π½ΡƒΡŽ Π²ΠΏΡ€Π°Π²ΠΎ. ΠŸΡ€ΠΈ Ρ‚Π°ΠΊΠΎΠΌ Π²Ρ‹Π±ΠΎΡ€Π΅ осСй Π½Π°ΠΏΡ€Π°Π²Π»Π΅Π½ΠΈΠ΅ ускорСния Π±ΡƒΠ΄Π΅Ρ‚ ΠΏΠΎΠ»ΠΎΠΆΠΈΡ‚Π΅Π»ΡŒΠ½Ρ‹ΠΌ для ΠΊΠ°ΠΆΠ΄ΠΎΠ³ΠΎ ΠΎΠ±ΡŠΠ΅ΠΊΡ‚Π°.

            Π”ΠΈΠ°Π³Ρ€Π°ΠΌΠΌΠ° свободного Ρ‚Π΅Π»Π° для ΠΊΠ°ΠΆΠ΄ΠΎΠΉ ΠΎΡ‚Π΄Π΅Π»ΡŒΠ½ΠΎΠΉ массы ΠΏΠΎΠΊΠ°Π·Π°Π½Π° Π½ΠΈΠΆΠ΅. ΠšΠ°ΠΆΠ΄Ρ‹ΠΉ ΠΎΠ±ΡŠΠ΅ΠΊΡ‚ испытываСт Π½ΠΈΡΡ…ΠΎΠ΄ΡΡ‰ΡƒΡŽ силу тяТСсти (F grav ), Ρ€Π°ΡΡΡ‡ΠΈΡ‚Ρ‹Π²Π°Π΅ΠΌΡƒΡŽ ΠΊΠ°ΠΊ m 1 β€’ g ΠΈ m 2 β€’ g соотвСтствСнно.ΠŸΠ»Π°Π½Π΅Ρ€ (m 1 ) испытываСт Π²ΠΎΡΡ…ΠΎΠ΄ΡΡ‰ΡƒΡŽ ΠΎΠΏΠΎΡ€Π½ΡƒΡŽ силу (Π²ΠΎΠ·Π΄ΡƒΡ… Ρ‚ΠΎΠ»ΠΊΠ°Π΅Ρ‚ Π΅Π³ΠΎ Π²Π²Π΅Ρ€Ρ…), Ρ‡Ρ‚ΠΎΠ±Ρ‹ ΡƒΡ€Π°Π²Π½ΠΎΠ²Π΅ΡΠΈΡ‚ΡŒ силу тяТСсти. ΠŸΠ»Π°Π½Π΅Ρ€ Ρ‚Π°ΠΊΠΆΠ΅ испытываСт Π³ΠΎΡ€ΠΈΠ·ΠΎΠ½Ρ‚Π°Π»ΡŒΠ½ΡƒΡŽ силу – силу натяТСния (F дСсятки ) Π²ΠΏΡ€Π°Π²ΠΎ. Висящая масса (m 2 ) испытываСт Π²ΠΎΡΡ…ΠΎΠ΄ΡΡ‰ΡƒΡŽ силу натяТСния (F дСсятки ), которая ΠΎΠΊΠ°Π·Ρ‹Π²Π°Π΅Ρ‚ Π½Π΅ΠΊΠΎΡ‚ΠΎΡ€ΠΎΠ΅ сопротивлСниС нисходящСй силС тяТСсти.

            Π£Ρ€Π°Π²Π½Π΅Π½ΠΈΠ΅ Π²Ρ‚ΠΎΡ€ΠΎΠ³ΠΎ Π·Π°ΠΊΠΎΠ½Π°

            ΠΡŒΡŽΡ‚ΠΎΠ½Π° (F net = m β€’ a) ΠΌΠΎΠΆΠ½ΠΎ ΠΏΡ€ΠΈΠΌΠ΅Π½ΠΈΡ‚ΡŒ ΠΊ ΠΎΠ±Π΅ΠΈΠΌ Π΄ΠΈΠ°Π³Ρ€Π°ΠΌΠΌΠ°ΠΌ свободного Ρ‚Π΅Π»Π°, Ρ‡Ρ‚ΠΎΠ±Ρ‹ Π½Π°ΠΏΠΈΡΠ°Ρ‚ΡŒ Π΄Π²Π° уравнСния для Π΄Π²ΡƒΡ… нСизвСстных.F net Π±ΡƒΠ΄Π΅Ρ‚ Π²Ρ‹Ρ€Π°ΠΆΠ°Ρ‚ΡŒΡΡ ΠΊΠ°ΠΊ сила Π² Π½Π°ΠΏΡ€Π°Π²Π»Π΅Π½ΠΈΠΈ ускорСния минус всС, Ρ‡Ρ‚ΠΎ Π΅ΠΌΡƒ противодСйствуСт. Для ΠΏΠ°Ρ€Π°ΠΏΠ»Π°Π½Π° массой 250,0 Π³Ρ€Π°ΠΌΠΌ (0,250 ΠΊΠ³) F net – это просто Π½Π΅ΡƒΡ€Π°Π²Π½ΠΎΠ²Π΅ΡˆΠ΅Π½Π½Π°Ρ сила натяТСния (F дСсятки ). Для подвСшСнной массы 20,0 Π³Ρ€Π°ΠΌΠΌ (0,020 ΠΊΠ³) F net записываСтся ΠΊΠ°ΠΊ 0,196 N – F дСсятки . УравнСния 4 ΠΈ 5 ΡΠ²Π»ΡΡŽΡ‚ΡΡ Ρ€Π΅Π·ΡƒΠ»ΡŒΡ‚Π°Ρ‚ΠΎΠΌ примСнСния уравнСния Π²Ρ‚ΠΎΡ€ΠΎΠ³ΠΎ Π·Π°ΠΊΠΎΠ½Π° ΠΡŒΡŽΡ‚ΠΎΠ½Π° ΠΊ 250,0-Π³Ρ€Π°ΠΌΠΌΠΎΠ²ΠΎΠΌΡƒ ΠΏΠ»Π°Π½Π΅Ρ€Ρƒ ΠΈ 20,0-Π³Ρ€Π°ΠΌΠΌΠΎΠ²ΠΎΠΉ висящСй массС. (ΠžΠ±Ρ€Π°Ρ‚ΠΈΡ‚Π΅ Π²Π½ΠΈΠΌΠ°Π½ΠΈΠ΅, Ρ‡Ρ‚ΠΎ значСния массы ΠΏΡ€Π΅ΠΎΠ±Ρ€Π°Π·ΡƒΡŽΡ‚ΡΡ Π² стандартныС ΠΊΠΈΠ»ΠΎΠ³Ρ€Π°ΠΌΠΌΡ‹ ΠΏΠ΅Ρ€Π΅Π΄ использованиСм Π² уравнСниях.Π’Π°ΠΊΠΆΠ΅ ΠΎΠ±Ρ€Π°Ρ‚ΠΈΡ‚Π΅ Π²Π½ΠΈΠΌΠ°Π½ΠΈΠ΅, Ρ‡Ρ‚ΠΎ Π΅Π΄ΠΈΠ½ΠΈΡ†Ρ‹ измСрСния Π±Ρ‹Π»ΠΈ ΠΎΠΏΡƒΡ‰Π΅Π½Ρ‹, Ρ‡Ρ‚ΠΎΠ±Ρ‹ уравнСния Ρ‡ΠΈΡ‚Π°Π»ΠΈΡΡŒ Π±ΠΎΠ»Π΅Π΅ Ρ‡Π΅Ρ‚ΠΊΠΎ.)

            F дСсятки = 0,2500 β€’ a

            0,196 – F дСсятки = 0,0200 β€’ a

            Π‘ этого ΠΌΠΎΠΌΠ΅Π½Ρ‚Π° нСсколько шагов ΠΏΠΎ Π°Π»Π³Π΅Π±Ρ€Π΅ ΠΏΡ€ΠΈΠ²Π΅Π΄ΡƒΡ‚ ΠΊ ΠΎΡ‚Π²Π΅Ρ‚Π°ΠΌ Π½Π° ΠΏΡ€ΠΎΠ±Π»Π΅ΠΌΡƒ. Π£Ρ€Π°Π²Π½Π΅Π½ΠΈΠ΅ 4 Π²Ρ‹Ρ€Π°ΠΆΠ°Π΅Ρ‚ Π·Π½Π°Ρ‡Π΅Π½ΠΈΠ΅ F дСсятков с Ρ‚ΠΎΡ‡ΠΊΠΈ зрСния ускорСния. Π­Ρ‚ΠΎ Π²Ρ‹Ρ€Π°ΠΆΠ΅Π½ΠΈΠ΅ для F tens ΠΌΠΎΠΆΠ½ΠΎ ΠΏΠΎΠ΄ΡΡ‚Π°Π²ΠΈΡ‚ΡŒ Π² ΡƒΡ€Π°Π²Π½Π΅Π½ΠΈΠ΅ 5, Ρ‡Ρ‚ΠΎΠ±Ρ‹ ΠΏΡ€Π΅ΠΎΠ±Ρ€Π°Π·ΠΎΠ²Π°Ρ‚ΡŒ Π΅Π³ΠΎ Π² ΡƒΡ€Π°Π²Π½Π΅Π½ΠΈΠ΅ с ΠΎΠ΄Π½ΠΈΠΌ нСизвСстным.Π­Ρ‚ΠΎ ΡƒΡ€Π°Π²Π½Π΅Π½ΠΈΠ΅ ΠΈ ΠΏΠΎΡΠ»Π΅Π΄ΡƒΡŽΡ‰ΠΈΠ΅ шаги Π°Π»Π³Π΅Π±Ρ€Ρ‹, приводящиС ΠΊ Π·Π½Π°Ρ‡Π΅Π½ΠΈΡŽ ускорСния, ΠΏΠΎΠΊΠ°Π·Π°Π½Ρ‹ Π½ΠΈΠΆΠ΅.

            0,196 – 0,2500 β€’ Π° = 0,0200 β€’ Π°
            0,196 = 0,2700 β€’
            a = 0,196 / 0,2700 = 0,72593 м / с 2
            а = ~ 0,726 м / с 2

            Π’Π΅ΠΏΠ΅Ρ€ΡŒ, ΠΊΠΎΠ³Π΄Π° ускорСниС Π±Ρ‹Π»ΠΎ Π½Π°ΠΉΠ΄Π΅Π½ΠΎ ΠΈΠ· уравнСния 5, Π΅Π³ΠΎ Π·Π½Π°Ρ‡Π΅Π½ΠΈΠ΅ ΠΌΠΎΠΆΠ½ΠΎ ΠΏΠΎΠ΄ΡΡ‚Π°Π²ΠΈΡ‚ΡŒ Π² ΡƒΡ€Π°Π²Π½Π΅Π½ΠΈΠ΅ 4, Ρ‡Ρ‚ΠΎΠ±Ρ‹ ΠΎΠΏΡ€Π΅Π΄Π΅Π»ΠΈΡ‚ΡŒ натяТСниС.

            F дСсятки = 0,2500 β€’ (0.72593) = 0,18148
            F дСсятков = ~ 0,181 Н

            БистСма шкивов, проанализированная Π² ΠΏΡ€ΠΈΠΌΠ΅Ρ€Π΅ Π·Π°Π΄Π°Ρ‡ΠΈ 2, ΠΈΠ½ΠΎΠ³Π΄Π° упоминаСтся ΠΊΠ°ΠΊ модифицированная машина Атвуда. Анализ Π½Π΅ΠΌΠ½ΠΎΠ³ΠΎ слоТнСС, Ρ‡Π΅ΠΌ машина Атвуда Π² ΠΏΡ€ΠΈΠΌΠ΅Ρ€Π΅ Π·Π°Π΄Π°Ρ‡ΠΈ 1. ПослСдний ΠΏΡ€ΠΈΠΌΠ΅Ρ€ Π·Π°Π΄Π°Ρ‡ΠΈ Π±ΡƒΠ΄Π΅Ρ‚ ΠΏΡ€Π΅Π΄ΡΡ‚Π°Π²Π»ΡΡ‚ΡŒ собой случай ΠΌΠΎΠ΄ΠΈΡ„ΠΈΡ†ΠΈΡ€ΠΎΠ²Π°Π½Π½ΠΎΠΉ ΠΌΠ°ΡˆΠΈΠ½Ρ‹ Атвуда с ΠΏΠΎΠ²Π΅Ρ€Ρ…Π½ΠΎΡΡ‚ΡŒΡŽ, Π½Π°ΠΊΠ»ΠΎΠ½Π΅Π½Π½ΠΎΠΉ, ΠΊΠ°ΠΊ ΠΏΠΎΠΊΠ°Π·Π°Π½ΠΎ Π½ΠΈΠΆΠ΅. ΠŸΠΎΠ΄Ρ…ΠΎΠ΄ ΠΊ Ρ€Π΅ΡˆΠ΅Π½ΠΈΡŽ ΠΏΡ€ΠΎΠ±Π»Π΅ΠΌ Π±ΡƒΠ΄Π΅Ρ‚ Ρ‚Π°ΠΊΠΈΠΌ ΠΆΠ΅.


            ΠŸΡ€ΠΈΠΌΠ΅Ρ€ Π·Π°Π΄Π°Ρ‡ΠΈ 3

            Рассмотрим ΡΠΈΡ‚ΡƒΠ°Ρ†ΠΈΡŽ с двумя Ρ‚Π΅Π»Π°ΠΌΠΈ справа.Π―Ρ‰ΠΈΠΊ 2,50×10 3 ΠΊΠ³ (m 1 ) стоит Π½Π° Π½Π°ΠΊΠ»ΠΎΠ½Π½ΠΎΠΉ плоскости ΠΈ соСдинСн ΠΊΠ°Π±Π΅Π»Π΅ΠΌ с массой 4,00×10 3 ΠΊΠ³ (m 2 ). Π­Ρ‚Π° вторая масса (m 2 ) подвСшСна Π½Π° шкивС. Π£Π³ΠΎΠ» Π½Π°ΠΊΠ»ΠΎΠ½Π° 30,0 Β°, ΠΏΠΎΠ²Π΅Ρ€Ρ…Π½ΠΎΡΡ‚ΡŒ Π½Π΅ ΠΈΠΌΠ΅Π΅Ρ‚ трСния. ΠžΠΏΡ€Π΅Π΄Π΅Π»ΠΈΡ‚Π΅ ускорСниС систСмы ΠΈ натяТСниС троса.

            Как ΠΈ прСдыдущая Π·Π°Π΄Π°Ρ‡Π°, пСрвая Π·Π°Π΄Π°Ρ‡Π° Π²ΠΊΠ»ΡŽΡ‡Π°Π΅Ρ‚ Π°Π½Π°Π»ΠΈΠ· ситуации, Ρ‡Ρ‚ΠΎΠ±Ρ‹ ΠΎΠΏΡ€Π΅Π΄Π΅Π»ΠΈΡ‚ΡŒ, Π² ΠΊΠ°ΠΊΠΎΠΌ Π½Π°ΠΏΡ€Π°Π²Π»Π΅Π½ΠΈΠΈ ΠΎΠ±ΡŠΠ΅ΠΊΡ‚Ρ‹ Π±ΡƒΠ΄ΡƒΡ‚ ΡƒΡΠΊΠΎΡ€ΡΡ‚ΡŒΡΡ.Π’Π°ΠΊΠΎΠΉ Π°Π½Π°Π»ΠΈΠ· ΠΏΠΎΠ·Π²ΠΎΠ»ΠΈΡ‚ ΠΏΡ€ΠΈΡΠ²ΠΎΠΈΡ‚ΡŒ ΠΊΠ°ΠΆΠ΄ΠΎΠΌΡƒ ΠΎΠ±ΡŠΠ΅ΠΊΡ‚Ρƒ систСму ΠΊΠΎΠΎΡ€Π΄ΠΈΠ½Π°Ρ‚Π½Ρ‹Ρ… осСй. Π’ этом случаС висящая масса (m 2 ) ΠΌΠΎΠ³Π»Π° ΡƒΡΠΊΠΎΡ€ΡΡ‚ΡŒΡΡ Π²Π²Π΅Ρ€Ρ… ΠΈΠ»ΠΈ Π²Π½ΠΈΠ·. НаправлСниС Π΅Π³ΠΎ ускорСния зависит ΠΎΡ‚ сравнСния Π΅Π³ΠΎ вСса (силы тяТСсти) с ΠΏΡ€ΠΎΡ‚ΠΈΠ²ΠΎΠΏΠΎΠ»ΠΎΠΆΠ½ΠΎΠΉ силой, Π΄Π΅ΠΉΡΡ‚Π²ΡƒΡŽΡ‰Π΅ΠΉ Π½Π° Π΄Ρ€ΡƒΠ³ΡƒΡŽ массу (m 1 ). Масса Π½Π° Π½Π°ΠΊΠ»ΠΎΠ½Π½ΠΎΠΉ плоскости сталкиваСтся с трСмя силами – силой тяТСсти, Π½ΠΎΡ€ΠΌΠ°Π»ΡŒΠ½ΠΎΠΉ силой ΠΈ силой натяТСния. Π‘ΠΈΠ»Π° тяТСсти Π½Π°ΠΏΡ€Π°Π²Π»Π΅Π½Π° ​​вниз (ΠΊΠ°ΠΊ ΠΎΠ±Ρ‹Ρ‡Π½ΠΎ) ΠΈ рассчитываСтся ΠΊΠ°ΠΊ m 1 β€’ g.ΠΠΎΡ€ΠΌΠ°Π»ΡŒΠ½Π°Ρ сила Π½Π°ΠΏΡ€Π°Π²Π»Π΅Π½Π° ​​пСрпСндикулярно повСрхности (ΠΊΠ°ΠΊ ΠΎΠ±Ρ‹Ρ‡Π½ΠΎ). Π‘ΠΈΠ»Π° натяТСния Π½Π°ΠΏΡ€Π°Π²Π»Π΅Π½Π° ​​ввСрх ΠΈ Π²ΠΏΡ€Π°Π²ΠΎ – ΠΏΠ°Ρ€Π°Π»Π»Π΅Π»ΡŒΠ½ΠΎ Π½Π°ΠΊΠ»ΠΎΠ½Π½ΠΎΠΉ плоскости ΠΈ Π² Ρ‚ΠΎΠΉ ΠΆΠ΅ ΠΎΡ€ΠΈΠ΅Π½Ρ‚Π°Ρ†ΠΈΠΈ, Ρ‡Ρ‚ΠΎ ΠΈ струна, ΠΎΠ±Π΅ΡΠΏΠ΅Ρ‡ΠΈΠ²Π°ΡŽΡ‰Π°Ρ эту силу. Как ΠΎΠ±ΡΡƒΠΆΠ΄Π°Π»ΠΎΡΡŒ Π½Π° ΠΏΡ€Π΅Π΄Ρ‹Π΄ΡƒΡ‰Π΅ΠΉ страницС, ΠΎΠ±ΡŠΠ΅ΠΊΡ‚Ρ‹, Ρ€Π°Π·ΠΌΠ΅Ρ‰Π΅Π½Π½Ρ‹Π΅ Π½Π° Π½Π°ΠΊΠ»ΠΎΠ½Π½Ρ‹Ρ… плоскостях, Π°Π½Π°Π»ΠΈΠ·ΠΈΡ€ΡƒΡŽΡ‚ΡΡ ΠΏΡƒΡ‚Π΅ΠΌ раздСлСния силы тяТСсти Π½Π° Π΄Π²Π° ΠΊΠΎΠΌΠΏΠΎΠ½Π΅Π½Ρ‚Π°. Один ΠΊΠΎΠΌΠΏΠΎΠ½Π΅Π½Ρ‚ Π½Π°ΠΏΡ€Π°Π²Π»Π΅Π½ ΠΏΠ°Ρ€Π°Π»Π»Π΅Π»ΡŒΠ½ΠΎ плоскости (ΠΈ Π²Π½ΠΈΠ· ΠΏΠΎΠ΄ этим ΡƒΠ³Π»ΠΎΠΌ), Π° Π΄Ρ€ΡƒΠ³ΠΎΠΉ ΠΊΠΎΠΌΠΏΠΎΠ½Π΅Π½Ρ‚ Π½Π°ΠΏΡ€Π°Π²Π»Π΅Π½ пСрпСндикулярно плоскости (ΠΈ Π²Π²Π΅Ρ€Ρ… ΠΏΠΎΠ΄ этим ΡƒΠ³Π»ΠΎΠΌ).Π­Ρ‚ΠΎ ΠΏΠ°Ρ€Π°Π»Π»Π΅Π»ΡŒΠ½Π°Ρ ΡΠΎΡΡ‚Π°Π²Π»ΡΡŽΡ‰Π°Ρ силы тяТСсти, которая пытаСтся увСсти m 1 Π²Π½ΠΈΠ· ΠΏΠΎ Π½Π°ΠΊΠ»ΠΎΠ½Π½ΠΎΠΉ плоскости. Как ΡƒΠΏΠΎΠΌΠΈΠ½Π°Π»ΠΎΡΡŒ Ρ€Π°Π½Π΅Π΅, этот ΠΊΠΎΠΌΠΏΠΎΠ½Π΅Π½Ρ‚ ΠΌΠΎΠΆΠ΅Ρ‚ Π±Ρ‹Ρ‚ΡŒ вычислСн ΠΏΡƒΡ‚Π΅ΠΌ умноТСния вСса ΠΎΠ±ΡŠΠ΅ΠΊΡ‚Π° (m 1 β€’ g) Π½Π° синус ΡƒΠ³Π»Π° Π½Π°ΠΊΠ»ΠΎΠ½Π° (30 Β°). Π—Π½Π°Ρ‡Π΅Π½ΠΈΠ΅ для F parallel составляСт

            .

            F ΠΏΠ°Ρ€Π°Π»Π»Π΅Π»ΡŒΠ½ΠΎ = ΠΌ 1 β€’ g β€’ синус (ΞΈ) = (2500 ΠΊΠ³) β€’ (9,8 Н / ΠΊΠ³) β€’ синус (30 Β°)
            F ΠΏΠ°Ρ€Π°Π»Π»Π΅Π»ΡŒΠ½ΠΎ = 12250 Н

            Π­Ρ‚ΠΎΡ‚ ΠΏΠ°Ρ€Π°Π»Π»Π΅Π»ΡŒΠ½Ρ‹ΠΉ ΠΊΠΎΠΌΠΏΠΎΠ½Π΅Π½Ρ‚ силы тяТСсти пытаСтся Ρ‚ΡΠ½ΡƒΡ‚ΡŒ m 1 Π²Π½ΠΈΠ· ΠΏΠΎ Π½Π°ΠΊΠ»ΠΎΠ½Π½ΠΎΠΉ плоскости.Π’Π°ΠΊ ΠΊΠ°ΠΊ m 1 ΠΏΡ€ΠΈΠΊΡ€Π΅ΠΏΠ»Π΅Π½ ΠΊΠ°Π±Π΅Π»Π΅ΠΌ ΠΊ m 2 , подвСсная масса Π±ΡƒΠ΄Π΅Ρ‚ Ρ‚ΡΠ½ΡƒΡ‚ΡŒΡΡ вмСстС с Π½ΠΈΠΌ. Однако Π΅ΡΡ‚ΡŒ ΠΏΡ€ΠΎΡ‚ΠΈΠ²ΠΎΠΏΠΎΠ»ΠΎΠΆΠ½ΠΎΠ΅ дСйствиС силы тяТСсти, тянущСС m 2 Π²Π½ΠΈΠ·; это ΠΏΡ€ΠΎΡ‚ΠΈΠ²ΠΎΠΏΠΎΠ»ΠΎΠΆΠ½ΠΎΠ΅ дСйствиС, Ссли ΠΎΠ½ΠΎ Π±ΡƒΠ΄Π΅Ρ‚ Π΄ΠΎΠΌΠΈΠ½ΠΈΡ€ΡƒΡŽΡ‰ΠΈΠΌ, ΠΏΠ΅Ρ€Π΅Ρ‚Π°Ρ‰ΠΈΡ‚ ΠΎΠ±ΡŠΠ΅ΠΊΡ‚ m 1 Π²Π²Π΅Ρ€Ρ… ΠΏΠΎ Π½Π°ΠΊΠ»ΠΎΠ½Π½ΠΎΠΉ плоскости. Π‘ΠΈΠ»Π° тяТСсти Π½Π° ΠΌ 2 составляСт

            F grav-2 = m 2 β€’ g = (4000 ΠΊΠ³) β€’ (9,8 Н / ΠΊΠ³) = 39200 Н

            Π­Ρ‚Π° сила тяТСсти Π½Π° m 2 являСтся Π΄ΠΎΠΌΠΈΠ½ΠΈΡ€ΡƒΡŽΡ‰Π΅ΠΉ силой.Π’Π°ΠΊΠΈΠΌ ΠΎΠ±Ρ€Π°Π·ΠΎΠΌ, m 1 Π±ΡƒΠ΄Π΅Ρ‚ ΡƒΡΠΊΠΎΡ€ΡΡ‚ΡŒΡΡ Π²Π²Π΅Ρ€Ρ… ΠΏΠΎ Π½Π°ΠΊΠ»ΠΎΠ½Π½ΠΎΠΉ плоскости, Π° m 2 Π±ΡƒΠ΄Π΅Ρ‚ ΡƒΡΠΊΠΎΡ€ΡΡ‚ΡŒΡΡ Π²Π½ΠΈΠ·. Оси ΠΊΠΎΠΎΡ€Π΄ΠΈΠ½Π°Ρ‚ Π½Π°Π·Π½Π°Ρ‡Π°ΡŽΡ‚ΡΡ соотвСтствСнно Ρ‚Π°ΠΊ, Ρ‡Ρ‚ΠΎΠ±Ρ‹ ΠΊΠ°ΠΆΠ΄Ρ‹ΠΉ ΠΎΠ±ΡŠΠ΅ΠΊΡ‚ ΠΈΠΌΠ΅Π» ΠΏΠΎΠ»ΠΎΠΆΠΈΡ‚Π΅Π»ΡŒΠ½ΠΎΠ΅ ускорСниС.

            На схСмах Π½ΠΈΠΆΠ΅ ΠΏΠΎΠΊΠ°Π·Π°Π½Ρ‹ эти оси ΠΊΠΎΠΎΡ€Π΄ΠΈΠ½Π°Ρ‚ ΠΈ силы, Π΄Π΅ΠΉΡΡ‚Π²ΡƒΡŽΡ‰ΠΈΠ΅ Π½Π° Π΄Π²Π° ΠΎΠ±ΡŠΠ΅ΠΊΡ‚Π°. Π’Ρ€ΠΈ силы Π½Π° m 1 ΡƒΠΆΠ΅ ΠΎΠ±ΡΡƒΠΆΠ΄Π°Π»ΠΈΡΡŒ. На схСмС ΠΏΠΎΠΊΠ°Π·Π°Π½Ρ‹ Π΄Π²Π° ΠΊΠΎΠΌΠΏΠΎΠ½Π΅Π½Ρ‚Π° F grav . Как ΡƒΠΏΠΎΠΌΠΈΠ½Π°Π»ΠΎΡΡŒ Π½Π° ΠΏΡ€Π΅Π΄Ρ‹Π΄ΡƒΡ‰Π΅ΠΉ страницС, пСрпСндикулярная ΡΠΎΡΡ‚Π°Π²Π»ΡΡŽΡ‰Π°Ρ силы тяТСсти рассчитываСтся ΠΊΠ°ΠΊ

            F пСрпСндикуляр = m 1 β€’ g β€’ cosΞΈ = (2500 ΠΊΠ³) β€’ (9.8 Н / ΠΊΠ³) β€’ cos (30 Β°)
            F пСрпСндикуляр = 21218 Н

            ΠΠΎΡ€ΠΌΠ°Π»ΡŒΠ½Π°Ρ сила (F norm ), Π΄Π΅ΠΉΡΡ‚Π²ΡƒΡŽΡ‰Π°Ρ Π½Π° m 1 , ΡƒΡ€Π°Π²Π½ΠΎΠ²Π΅ΡˆΠΈΠ²Π°Π΅Ρ‚ F пСрпСндикуляр , Ρ‚Π°ΠΊ Ρ‡Ρ‚ΠΎ Π½Π΅Ρ‚ ускорСния, пСрпСндикулярного Π½Π°ΠΊΠ»ΠΎΠ½Π½ΠΎΠΉ плоскости. Π—Π½Π°Ρ‡Π΅Π½ΠΈΠ΅ Π½ΠΎΡ€ΠΌΡ‹ для F Ρ‚Π°ΠΊΠΆΠ΅ составляСт 21218 Н. Висящая масса (m 2 ) испытываСт Ρ‚ΠΎΠ»ΡŒΠΊΠΎ Π΄Π²Π΅ силы – силу тяТСсти, Π½Π°ΠΏΡ€Π°Π²Π»Π΅Π½Π½ΡƒΡŽ Π²Π½ΠΈΠ·, ΠΈ силу натяТСния Π²Π²Π΅Ρ€Ρ….

            Π’Π΅ΠΏΠ΅Ρ€ΡŒ ΡƒΡ€Π°Π²Π½Π΅Π½ΠΈΠ΅ Π²Ρ‚ΠΎΡ€ΠΎΠ³ΠΎ Π·Π°ΠΊΠΎΠ½Π° ΠΡŒΡŽΡ‚ΠΎΠ½Π° (F net = m β€’ a) ΠΌΠΎΠΆΠ½ΠΎ ΠΏΡ€ΠΈΠΌΠ΅Π½ΠΈΡ‚ΡŒ ΠΊ ΠΎΠ±Π΅ΠΈΠΌ Π΄ΠΈΠ°Π³Ρ€Π°ΠΌΠΌΠ°ΠΌ свободного Ρ‚Π΅Π»Π°, Ρ‡Ρ‚ΠΎΠ±Ρ‹ Π½Π°ΠΏΠΈΡΠ°Ρ‚ΡŒ Π΄Π²Π° уравнСния для Π΄Π²ΡƒΡ… нСизвСстных.F net выраТаСтся ΠΊΠ°ΠΊ сила Π² Π½Π°ΠΏΡ€Π°Π²Π»Π΅Π½ΠΈΠΈ ускорСния Π·Π° Π²Ρ‹Ρ‡Π΅Ρ‚ΠΎΠΌ любой силы, которая Π΅ΠΌΡƒ противодСйствуСт. Для Π³Ρ€ΡƒΠ·Π° массой 2500 ΠΊΠ³ Π½Π° ΡƒΠΊΠ»ΠΎΠ½Π΅ (m 1 ), F net – это просто сила натяТСния (F дСсятки ) Π·Π° Π²Ρ‹Ρ‡Π΅Ρ‚ΠΎΠΌ ΠΏΠ°Ρ€Π°Π»Π»Π΅Π»ΡŒΠ½ΠΎΠΉ ΡΠΎΡΡ‚Π°Π²Π»ΡΡŽΡ‰Π΅ΠΉ силы тяТСсти. Для подвСшСнной массы 4000 ΠΊΠ³ (m 2 ) F net – это сила тяТСсти (39200 Н) Π·Π° Π²Ρ‹Ρ‡Π΅Ρ‚ΠΎΠΌ силы натяТСния (F дСсятки ). УравнСния 6 ΠΈ 7 ΡΠ²Π»ΡΡŽΡ‚ΡΡ Ρ€Π΅Π·ΡƒΠ»ΡŒΡ‚Π°Ρ‚ΠΎΠΌ примСнСния уравнСния Π²Ρ‚ΠΎΡ€ΠΎΠ³ΠΎ Π·Π°ΠΊΠΎΠ½Π° ΠΡŒΡŽΡ‚ΠΎΠ½Π° ΠΊ m 1 ΠΈ m 2 .(ΠžΠ±Ρ€Π°Ρ‚ΠΈΡ‚Π΅ Π²Π½ΠΈΠΌΠ°Π½ΠΈΠ΅, Ρ‡Ρ‚ΠΎ Π΅Π΄ΠΈΠ½ΠΈΡ†Ρ‹ измСрСния Π±Ρ‹Π»ΠΈ ΠΎΠΏΡƒΡ‰Π΅Π½Ρ‹, Ρ‡Ρ‚ΠΎΠ±Ρ‹ уравнСния Ρ‡ΠΈΡ‚Π°Π»ΠΈΡΡŒ Π±ΠΎΠ»Π΅Π΅ Ρ‡Π΅Ρ‚ΠΊΠΎ.)

            F дСсятки – 12250 = 2500 β€’ a

            39200 – F дСсятки = 4000 β€’ a

            Π‘ этого ΠΌΠΎΠΌΠ΅Π½Ρ‚Π° нСсколько шагов ΠΏΠΎ Π°Π»Π³Π΅Π±Ρ€Π΅ ΠΏΡ€ΠΈΠ²Π΅Π΄ΡƒΡ‚ ΠΊ ΠΎΡ‚Π²Π΅Ρ‚Π°ΠΌ Π½Π° ΠΏΡ€ΠΎΠ±Π»Π΅ΠΌΡƒ. Π£Ρ€Π°Π²Π½Π΅Π½ΠΈΠ΅ 6 ΠΌΠΎΠΆΠ½ΠΎ ΠΈΠ·ΠΌΠ΅Π½ΠΈΡ‚ΡŒ, Ρ‡Ρ‚ΠΎΠ±Ρ‹ ΠΏΠΎΠ»ΡƒΡ‡ΠΈΡ‚ΡŒ Π²Ρ‹Ρ€Π°ΠΆΠ΅Π½ΠΈΠ΅ для F дСсятков , Π²Ρ‹Ρ€Π°ΠΆΠ΅Π½Π½ΠΎΠ΅ Π² Π΅Π΄ΠΈΠ½ΠΈΡ†Π°Ρ… ускорСния.

            F дСсятки = 2500 β€’ a + 12250

            Π­Ρ‚ΠΎ Π²Ρ‹Ρ€Π°ΠΆΠ΅Π½ΠΈΠ΅ для F tens ΠΌΠΎΠΆΠ½ΠΎ ΠΏΠΎΠ΄ΡΡ‚Π°Π²ΠΈΡ‚ΡŒ Π² ΡƒΡ€Π°Π²Π½Π΅Π½ΠΈΠ΅ 7, Ρ‡Ρ‚ΠΎΠ±Ρ‹ ΠΏΡ€Π΅ΠΎΠ±Ρ€Π°Π·ΠΎΠ²Π°Ρ‚ΡŒ Π΅Π³ΠΎ Π² ΡƒΡ€Π°Π²Π½Π΅Π½ΠΈΠ΅ с ΠΎΠ΄Π½ΠΈΠΌ нСизвСстным.Π­Ρ‚ΠΎ ΡƒΡ€Π°Π²Π½Π΅Π½ΠΈΠ΅ ΠΈ ΠΏΠΎΡΠ»Π΅Π΄ΡƒΡŽΡ‰ΠΈΠ΅ шаги Π°Π»Π³Π΅Π±Ρ€Ρ‹, приводящиС ΠΊ Π·Π½Π°Ρ‡Π΅Π½ΠΈΡŽ ускорСния, ΠΏΠΎΠΊΠ°Π·Π°Π½Ρ‹ Π½ΠΈΠΆΠ΅.

            39200 – (2500 β€’ a + 12250) = 4000 β€’ a
            39200 – 2500 β€’ a – 12250 = 4000 β€’ a
            26950 = 6500 β€’
            a = 26950/6500 = 4,1462 м / с 2
            а = ~ 4,15 м / с 2

            Π’Π΅ΠΏΠ΅Ρ€ΡŒ, ΠΊΠΎΠ³Π΄Π° ускорСниС Π±Ρ‹Π»ΠΎ Π½Π°ΠΉΠ΄Π΅Π½ΠΎ ΠΈΠ· уравнСния 7, Π΅Π³ΠΎ Π·Π½Π°Ρ‡Π΅Π½ΠΈΠ΅ ΠΌΠΎΠΆΠ½ΠΎ ΠΏΠΎΠ΄ΡΡ‚Π°Π²ΠΈΡ‚ΡŒ Π² ΡƒΡ€Π°Π²Π½Π΅Π½ΠΈΠ΅ 8, Ρ‡Ρ‚ΠΎΠ±Ρ‹ ΠΎΠΏΡ€Π΅Π΄Π΅Π»ΠΈΡ‚ΡŒ силу натяТСния (F дСсятки ).

            F дСсятков = 2500 β€’ a + 12250 = 2500 β€’ (4,1462) + 12250 = 22615 N
            F дСсятков = ~ 2,26 x 10 4 N

            ΠŸΡ€ΠΎΠ±Π»Π΅ΠΌΡ‹ с двумя Ρ‚Π΅Π»Π°ΠΌΠΈ, ΠΏΠΎΠ΄ΠΎΠ±Π½Ρ‹Π΅ этим Ρ‚Ρ€Π΅ΠΌ ΠΏΡ€ΠΈΠΌΠ΅Ρ€Π°ΠΌ Π·Π°Π΄Π°Ρ‡, ΠΌΠΎΠ³ΡƒΡ‚ Π±Ρ‹Ρ‚ΡŒ довольно слоТной Π·Π°Π΄Π°Ρ‡Π΅ΠΉ. БистСмный ΠΏΠΎΠ΄Ρ…ΠΎΠ΄, примСняСмый ΠΊ ΠΊΠ°ΠΆΠ΄ΠΎΠΉ ΠΏΡ€ΠΎΠ±Π»Π΅ΠΌΠ΅, ΡƒΠΏΡ€ΠΎΡ‰Π°Π΅Ρ‚ Π°Π½Π°Π»ΠΈΠ·. Π₯ΠΎΡ€ΠΎΡˆΠ΅Π΅ ΠΊΠΎΠ½Ρ†Π΅ΠΏΡ‚ΡƒΠ°Π»ΡŒΠ½ΠΎΠ΅ ΠΏΠΎΠ½ΠΈΠΌΠ°Π½ΠΈΠ΅, ΠΏΡ€ΠΈΠ²Π΅Ρ€ΠΆΠ΅Π½Π½ΠΎΡΡ‚ΡŒ использованию Π΄ΠΈΠ°Π³Ρ€Π°ΠΌΠΌ свободного Ρ‚Π΅Π»Π° ΠΈ Ρ‚Π²Π΅Ρ€Π΄ΠΎΠ΅ ΠΏΠΎΠ½ΠΈΠΌΠ°Π½ΠΈΠ΅ Π²Ρ‚ΠΎΡ€ΠΎΠ³ΠΎ Π·Π°ΠΊΠΎΠ½Π° ΠΡŒΡŽΡ‚ΠΎΠ½Π° – Π²ΠΎΡ‚ основныС ΡΠΎΡΡ‚Π°Π²Π»ΡΡŽΡ‰ΠΈΠ΅ успСха.

            ΠœΡ‹ Ρ…ΠΎΡ‚Π΅Π»ΠΈ Π±Ρ‹ ΠΏΡ€Π΅Π΄Π»ΠΎΠΆΠΈΡ‚ΡŒ … Иногда просто ΠΏΡ€ΠΎΡ‡ΠΈΡ‚Π°Ρ‚ΡŒ ΠΎΠ± этом нСдостаточно. Π’Ρ‹ Π΄ΠΎΠ»ΠΆΠ½Ρ‹ Π²Π·Π°ΠΈΠΌΠΎΠ΄Π΅ΠΉΡΡ‚Π²ΠΎΠ²Π°Ρ‚ΡŒ с Π½ΠΈΠΌ! И это ΠΈΠΌΠ΅Π½Π½ΠΎ Ρ‚ΠΎ, Ρ‡Ρ‚ΠΎ Π²Ρ‹ Π΄Π΅Π»Π°Π΅Ρ‚Π΅, ΠΊΠΎΠ³Π΄Π° ΠΈΡΠΏΠΎΠ»ΡŒΠ·ΡƒΠ΅Ρ‚Π΅ ΠΎΠ΄ΠΈΠ½ ΠΈΠ· ΠΈΠ½Ρ‚Π΅Ρ€Π°ΠΊΡ‚ΠΈΠ²Π½Ρ‹Ρ… ΠΌΠ°Ρ‚Π΅Ρ€ΠΈΠ°Π»ΠΎΠ² The Physics Classroom. ΠœΡ‹ Ρ…ΠΎΡ‚Π΅Π»ΠΈ Π±Ρ‹ ΠΏΡ€Π΅Π΄Π»ΠΎΠΆΠΈΡ‚ΡŒ Π²Π°ΠΌ ΡΠΎΠ²ΠΌΠ΅ΡΡ‚ΠΈΡ‚ΡŒ Ρ‡Ρ‚Π΅Π½ΠΈΠ΅ этой страницы с использованиСм нашСго симулятора ΠΌΠ°ΡˆΠΈΠ½Ρ‹ Π­Ρ‚Π²ΡƒΠ΄Π°. Π’Ρ‹ ΠΌΠΎΠΆΠ΅Ρ‚Π΅ Π½Π°ΠΉΡ‚ΠΈ Π΅Π³ΠΎ Π² Ρ€Π°Π·Π΄Π΅Π»Π΅ Physics Interactives Π½Π° нашСм сайтС. Π’Ρ€Π΅Π½Π°ΠΆΠ΅Ρ€ позволяСт ΠΈΡΡΠ»Π΅Π΄ΠΎΠ²Π°Ρ‚ΡŒ двухмассовыС систСмы, ускоряСмыС подвСшСнной массой.


            ΠŸΡ€ΠΎΠ²Π΅Ρ€ΡŒΡ‚Π΅ своС ΠΏΠΎΠ½ΠΈΠΌΠ°Π½ΠΈΠ΅

            1. Рассмотрим ΡΠΈΡ‚ΡƒΠ°Ρ†ΠΈΡŽ с двумя Ρ‚Π΅Π»Π°ΠΌΠΈ справа. 100,0-Π³Ρ€Π°ΠΌΠΌΠΎΠ²Ρ‹ΠΉ ΠΏΠΎΠ΄Π²Π΅ΡˆΠ΅Π½Π½Ρ‹ΠΉ Π³Ρ€ΡƒΠ· (m2) ΠΏΡ€ΠΈΠΊΡ€Π΅ΠΏΠ»ΡΡŽΡ‚ ΠΊ 325-Π³Ρ€Π°ΠΌΠΌΠΎΠ²ΠΎΠΉ массС (m1), покоящСйся Π½Π° столС. ΠšΠΎΡΡ„Ρ„ΠΈΡ†ΠΈΠ΅Π½Ρ‚ трСния ΠΌΠ΅ΠΆΠ΄Ρƒ 325,0-Π³Ρ€Π°ΠΌΠΌΠΎΠ²ΠΎΠΉ массой ΠΈ столом составляСт 0,215. ΠžΠΏΡ€Π΅Π΄Π΅Π»ΠΈΡ‚Π΅ ускорСниС систСмы ΠΈ натяТСниС струны.

            2. Рассмотрим ΡΠΈΡ‚ΡƒΠ°Ρ†ΠΈΡŽ с двумя Ρ‚Π΅Π»Π°ΠΌΠΈ справа.Π―Ρ‰ΠΈΠΊ 3,50×10 3 ΠΊΠ³ (m 1 ) стоит Π½Π° Π½Π°ΠΊΠ»ΠΎΠ½Π½ΠΎΠΉ плоскости ΠΈ соСдинСн ΠΊΠ°Π±Π΅Π»Π΅ΠΌ с массой 1,00×10 3 ΠΊΠ³ (m 2 ). Π­Ρ‚Π° вторая масса (m 2 ) подвСшСна Π½Π° шкивС. Π£Π³ΠΎΠ» Π½Π°ΠΊΠ»ΠΎΠ½Π° составляСт 30,0 Β°, Π° ΠΏΠΎΠ²Π΅Ρ€Ρ…Π½ΠΎΡΡ‚ΡŒ ΠΈΠΌΠ΅Π΅Ρ‚ коэффициСнт трСния 0,210. ΠžΠΏΡ€Π΅Π΄Π΅Π»ΠΈΡ‚Π΅ ускорСниС систСмы ΠΈ натяТСниС троса.

            Π”Π²ΠΎΠΉΠ½Ρ‹Π΅ ΠΏΡ€ΠΎΠ±Π»Π΅ΠΌΡ‹: Π΄Π²Π΅ ΠΏΡ€ΠΎΠ±Π»Π΅ΠΌΡ‹ Ρ‚Π΅Π»Π°

            НашС исслСдованиС Π΄ΠΎ сих ΠΏΠΎΡ€ ΠΎΠ³Ρ€Π°Π½ΠΈΡ‡ΠΈΠ²Π°Π»ΠΎΡΡŒ Π°Π½Π°Π»ΠΈΠ·ΠΎΠΌ ΠΎΡ‚Π΄Π΅Π»ΡŒΠ½Ρ‹Ρ… ΠΎΠ±ΡŠΠ΅ΠΊΡ‚ΠΎΠ², двиТущихся ΠΏΠΎΠ΄ дСйствиСм Π·Π°ΠΊΠΎΠ½ΠΎΠ² ΠΡŒΡŽΡ‚ΠΎΠ½Π°.Но Ρ‡Ρ‚ΠΎ ΠΏΡ€ΠΎΠΈΠ·ΠΎΠΉΠ΄Π΅Ρ‚, Ссли Π΄Π²Π° ΠΎΠ±ΡŠΠ΅ΠΊΡ‚Π° Ρ‚Π°ΠΊ ΠΈΠ»ΠΈ ΠΈΠ½Π°Ρ‡Π΅ связаны вмСстС? НапримСр, эвакуатор ΠΌΠΎΠΆΠ΅Ρ‚ Ρ‚ΡΠ½ΡƒΡ‚ΡŒ ΠΌΠ°ΡˆΠΈΠ½Ρƒ ΠΏΠΎ шоссС. Как проводится Ρ‚Π°ΠΊΠΎΠΉ Π°Π½Π°Π»ΠΈΠ·? Как опрСдСляСтся ускорСниС эвакуатора ΠΈ автомобиля? А ΠΊΠ°ΠΊ насчСт силы, Π΄Π΅ΠΉΡΡ‚Π²ΡƒΡŽΡ‰Π΅ΠΉ ΠΌΠ΅ΠΆΠ΄Ρƒ эвакуатором ΠΈ Π°Π²Ρ‚ΠΎΠΌΠΎΠ±ΠΈΠ»Π΅ΠΌ? Π’ этой части Π£Ρ€ΠΎΠΊΠ° 3 ΠΌΡ‹ попытаСмся ΠΏΡ€ΠΎΠ°Π½Π°Π»ΠΈΠ·ΠΈΡ€ΠΎΠ²Π°Ρ‚ΡŒ Ρ‚Π°ΠΊΠΈΠ΅ ситуации. ΠœΡ‹ ΠΎΠ±Π½Π°Ρ€ΡƒΠΆΠΈΠΌ, Ρ‡Ρ‚ΠΎ Π°Π½Π°Π»ΠΈΠ· проводится Ρ‚Π°ΠΊΠΈΠΌ ΠΆΠ΅ ΠΎΠ±Ρ‰ΠΈΠΌ ΠΎΠ±Ρ€Π°Π·ΠΎΠΌ, ΠΊΠ°ΠΊ ΠΈ ΠΏΡ€ΠΈ Π½Π°Π»ΠΈΡ‡ΠΈΠΈ ΠΎΠ΄Π½ΠΎΠ³ΠΎ ΠΎΠ±ΡŠΠ΅ΠΊΡ‚Π° – с использованиСм Π΄ΠΈΠ°Π³Ρ€Π°ΠΌΠΌ свободного Ρ‚Π΅Π»Π° ΠΈ Π·Π°ΠΊΠΎΠ½ΠΎΠ² ΠΡŒΡŽΡ‚ΠΎΠ½Π°.

            Π‘Π°Π·ΠΎΠ²Ρ‹ΠΉ ΠΏΠΎΠ΄Ρ…ΠΎΠ΄

            Π‘ΠΈΡ‚ΡƒΠ°Ρ†ΠΈΠΈ с двумя ΠΎΠ±ΡŠΠ΅ΠΊΡ‚Π°ΠΌΠΈ часто Π½Π°Π·Ρ‹Π²Π°ΡŽΡ‚ ситуациями Π΄Π²ΡƒΡ… Ρ‚Π΅Π». Π‘ΡƒΠ΄ΡƒΡ‡ΠΈ физичСскими Π·Π°Π΄Π°Ρ‡Π°ΠΌΠΈ, Π·Π°Π΄Π°Ρ‡ΠΈ Π΄Π²ΡƒΡ… Ρ‚Π΅Π» Ρ…Π°Ρ€Π°ΠΊΡ‚Π΅Ρ€ΠΈΠ·ΡƒΡŽΡ‚ΡΡ Π½Π°Π±ΠΎΡ€ΠΎΠΌ Π΄Π²ΡƒΡ… нСизвСстных Π²Π΅Π»ΠΈΡ‡ΠΈΠ½. Π§Π°Ρ‰Π΅ всСго (хотя ΠΈ Π½Π΅ всСгда) двумя нСизвСстными ΡΠ²Π»ΡΡŽΡ‚ΡΡ ускорСниС Π΄Π²ΡƒΡ… ΠΎΠ±ΡŠΠ΅ΠΊΡ‚ΠΎΠ² ΠΈ сила, пСрСдаваСмая ΠΌΠ΅ΠΆΠ΄Ρƒ двумя ΠΎΠ±ΡŠΠ΅ΠΊΡ‚Π°ΠΌΠΈ. К Π΄Π²ΡƒΠΌ тСлСсным ΠΏΡ€ΠΎΠ±Π»Π΅ΠΌΠ°ΠΌ ΠΎΠ±Ρ‹Ρ‡Π½ΠΎ ΠΌΠΎΠΆΠ½ΠΎ ΠΏΠΎΠ΄ΠΎΠΉΡ‚ΠΈ, ΠΈΡΠΏΠΎΠ»ΡŒΠ·ΡƒΡ ΠΎΠ΄ΠΈΠ½ ΠΈΠ· Π΄Π²ΡƒΡ… основных ΠΏΠΎΠ΄Ρ…ΠΎΠ΄ΠΎΠ².Один ΠΈΠ· ΠΏΠΎΠ΄Ρ…ΠΎΠ΄ΠΎΠ² Π²ΠΊΠ»ΡŽΡ‡Π°Π΅Ρ‚ ΠΊΠΎΠΌΠ±ΠΈΠ½Π°Ρ†ΠΈΡŽ систСмного Π°Π½Π°Π»ΠΈΠ·Π° ΠΈ ΠΈΠ½Π΄ΠΈΠ²ΠΈΠ΄ΡƒΠ°Π»ΡŒΠ½ΠΎΠ³ΠΎ Π°Π½Π°Π»ΠΈΠ·Π° Ρ‚Π΅Π»Π°. Π’ систСмном Π°Π½Π°Π»ΠΈΠ·Π΅ Π΄Π²Π° ΠΎΠ±ΡŠΠ΅ΠΊΡ‚Π° Ρ€Π°ΡΡΠΌΠ°Ρ‚Ρ€ΠΈΠ²Π°ΡŽΡ‚ΡΡ ΠΊΠ°ΠΊ ΠΎΠ΄ΠΈΠ½ ΠΎΠ±ΡŠΠ΅ΠΊΡ‚, двиТущийся (ΠΈΠ»ΠΈ ΡƒΡΠΊΠΎΡ€ΡΡŽΡ‰ΠΈΠΉΡΡ) вмСстС ΠΊΠ°ΠΊ Π΅Π΄ΠΈΠ½ΠΎΠ΅ Ρ†Π΅Π»ΠΎΠ΅. Масса систСмы – это сумма масс Π΄Π²ΡƒΡ… ΠΎΡ‚Π΄Π΅Π»ΡŒΠ½Ρ‹Ρ… ΠΎΠ±ΡŠΠ΅ΠΊΡ‚ΠΎΠ². Если задСйствовано ускорСниС, ускорСниС систСмы Ρ‚Π°ΠΊΠΎΠ΅ ΠΆΠ΅, ΠΊΠ°ΠΊ ΠΈ Ρƒ ΠΎΡ‚Π΄Π΅Π»ΡŒΠ½Ρ‹Ρ… ΠΎΠ±ΡŠΠ΅ΠΊΡ‚ΠΎΠ². БистСмный Π°Π½Π°Π»ΠΈΠ· ΠΎΠ±Ρ‹Ρ‡Π½ΠΎ выполняСтся для опрСдСлСния ускорСния систСмы. БистСмный Π°Π½Π°Π»ΠΈΠ· совмСщСн с Π°Π½Π°Π»ΠΈΠ·ΠΎΠΌ ΠΎΡ‚Π΄Π΅Π»ΡŒΠ½ΠΎΠ³ΠΎ ΠΎΠ±ΡŠΠ΅ΠΊΡ‚Π°.ΠŸΡ€ΠΈ Π°Π½Π°Π»ΠΈΠ·Π΅ ΠΎΡ‚Π΄Π΅Π»ΡŒΠ½Ρ‹Ρ… ΠΎΠ±ΡŠΠ΅ΠΊΡ‚ΠΎΠ² ΠΎΠ΄ΠΈΠ½ ΠΈΠ· Π΄Π²ΡƒΡ… ΠΎΠ±ΡŠΠ΅ΠΊΡ‚ΠΎΠ² ΠΈΠ·ΠΎΠ»ΠΈΡ€ΠΎΠ²Π°Π½ ΠΈ рассматриваСтся ΠΊΠ°ΠΊ ΠΎΡ‚Π΄Π΅Π»ΡŒΠ½Ρ‹ΠΉ нСзависимый ΠΎΠ±ΡŠΠ΅ΠΊΡ‚. Бтроится Π΄ΠΈΠ°Π³Ρ€Π°ΠΌΠΌΠ° свободного Ρ‚Π΅Π»Π°, ΠΎΠΏΡ€Π΅Π΄Π΅Π»ΡΡŽΡ‚ΡΡ ΠΈ Π²Ρ‹Ρ‡ΠΈΡΠ»ΡΡŽΡ‚ΡΡ ΠΎΡ‚Π΄Π΅Π»ΡŒΠ½Ρ‹Π΅ силы, Π΄Π΅ΠΉΡΡ‚Π²ΡƒΡŽΡ‰ΠΈΠ΅ Π½Π° ΠΎΠ±ΡŠΠ΅ΠΊΡ‚. Анализ ΠΎΡ‚Π΄Π΅Π»ΡŒΠ½ΠΎΠ³ΠΎ ΠΎΠ±ΡŠΠ΅ΠΊΡ‚Π° ΠΎΠ±Ρ‹Ρ‡Π½ΠΎ выполняСтся для опрСдСлСния значСния любой силы, Π΄Π΅ΠΉΡΡ‚Π²ΡƒΡŽΡ‰Π΅ΠΉ ΠΌΠ΅ΠΆΠ΄Ρƒ двумя ΠΎΠ±ΡŠΠ΅ΠΊΡ‚Π°ΠΌΠΈ, Π½Π°ΠΏΡ€ΠΈΠΌΠ΅Ρ€, ΠΊΠΎΠ½Ρ‚Π°ΠΊΡ‚Π½Ρ‹Ρ… сил ΠΈΠ»ΠΈ сил натяТСния.

            Двойная комбинация систСмного Π°Π½Π°Π»ΠΈΠ·Π° ΠΈ Π°Π½Π°Π»ΠΈΠ·Π° ΠΎΡ‚Π΄Π΅Π»ΡŒΠ½ΠΎΠ³ΠΎ ΠΎΠ±ΡŠΠ΅ΠΊΡ‚Π° – это ΠΎΠ΄ΠΈΠ½ ΠΈΠ· Π΄Π²ΡƒΡ… ΠΏΠΎΠ΄Ρ…ΠΎΠ΄ΠΎΠ², ΠΊΠΎΡ‚ΠΎΡ€Ρ‹Π΅ ΠΎΠ±Ρ‹Ρ‡Π½ΠΎ ΠΈΡΠΏΠΎΠ»ΡŒΠ·ΡƒΡŽΡ‚ΡΡ для Π°Π½Π°Π»ΠΈΠ·Π° Π·Π°Π΄Π°Ρ‡ Π΄Π²ΡƒΡ… Ρ‚Π΅Π».Π’Ρ‚ΠΎΡ€ΠΎΠΉ ΠΏΠΎΠ΄Ρ…ΠΎΠ΄ ΠΏΡ€Π΅Π΄ΠΏΠΎΠ»Π°Π³Π°Π΅Ρ‚ использованиС Π΄Π²ΡƒΡ… ΠΎΡ‚Π΄Π΅Π»ΡŒΠ½Ρ‹Ρ… Π°Π½Π°Π»ΠΈΠ·ΠΎΠ² ΠΎΡ‚Π΄Π΅Π»ΡŒΠ½Ρ‹Ρ… ΠΎΠ±ΡŠΠ΅ΠΊΡ‚ΠΎΠ². ΠŸΡ€ΠΈ Ρ‚Π°ΠΊΠΎΠΌ ΠΏΠΎΠ΄Ρ…ΠΎΠ΄Π΅ Π΄ΠΈΠ°Π³Ρ€Π°ΠΌΠΌΡ‹ свободного Ρ‚Π΅Π»Π° строятся нСзависимо для ΠΊΠ°ΠΆΠ΄ΠΎΠ³ΠΎ ΠΎΠ±ΡŠΠ΅ΠΊΡ‚Π°, ΠΈ Π²Ρ‚ΠΎΡ€ΠΎΠΉ Π·Π°ΠΊΠΎΠ½ ΠΡŒΡŽΡ‚ΠΎΠ½Π° ΠΈΡΠΏΠΎΠ»ΡŒΠ·ΡƒΠ΅Ρ‚ΡΡ для связи ΠΎΡ‚Π΄Π΅Π»ΡŒΠ½Ρ‹Ρ… Π·Π½Π°Ρ‡Π΅Π½ΠΈΠΉ силы с массой ΠΈ ускорСниСм. Анализ ΠΊΠ°ΠΆΠ΄ΠΎΠ³ΠΎ ΠΎΡ‚Π΄Π΅Π»ΡŒΠ½ΠΎΠ³ΠΎ ΠΎΠ±ΡŠΠ΅ΠΊΡ‚Π° Π³Π΅Π½Π΅Ρ€ΠΈΡ€ΡƒΠ΅Ρ‚ ΡƒΡ€Π°Π²Π½Π΅Π½ΠΈΠ΅ с нСизвСстным. Π’ Ρ€Π΅Π·ΡƒΠ»ΡŒΡ‚Π°Ρ‚Π΅ получаСтся систСма Π΄Π²ΡƒΡ… ΡƒΡ€Π°Π²Π½Π΅Π½ΠΈΠΉ с двумя нСизвСстными. БистСма ΡƒΡ€Π°Π²Π½Π΅Π½ΠΈΠΉ Ρ€Π΅ΡˆΠ°Π΅Ρ‚ΡΡ с Ρ†Π΅Π»ΡŒΡŽ опрСдСлСния нСизвСстных Π·Π½Π°Ρ‡Π΅Π½ΠΈΠΉ.


            Π’ качСствС ΠΏΠ΅Ρ€Π²ΠΎΠ³ΠΎ ΠΏΡ€ΠΈΠΌΠ΅Ρ€Π° ΠΈΠ· Π΄Π²ΡƒΡ… ΠΏΠΎΠ΄Ρ…ΠΎΠ΄ΠΎΠ² ΠΊ Ρ€Π΅ΡˆΠ΅Π½ΠΈΡŽ Π·Π°Π΄Π°Ρ‡ Π΄Π²ΡƒΡ… Ρ‚Π΅Π» рассмотрим ΡΠ»Π΅Π΄ΡƒΡŽΡ‰ΠΈΠΉ ΠΏΡ€ΠΈΠΌΠ΅Ρ€ Π·Π°Π΄Π°Ρ‡ΠΈ.

            ΠŸΡ€ΠΈΠΌΠ΅Ρ€ Π·Π°Π΄Π°Ρ‡ΠΈ 1:

            ΠšΠΎΡ€ΠΎΠ±ΠΊΠ° 5,0 ΠΊΠ³ ΠΈ ΠΊΠΎΡ€ΠΎΠ±ΠΊΠ° 10,0 ΠΊΠ³ ΡΠΎΠΏΡ€ΠΈΠΊΠ°ΡΠ°ΡŽΡ‚ΡΡ Π΄Ρ€ΡƒΠ³ с Π΄Ρ€ΡƒΠ³ΠΎΠΌ. К 5-ΠΊΠΈΠ»ΠΎΠ³Ρ€Π°ΠΌΠΌΠΎΠ²ΠΎΠΌΡƒ ящику прилагаСтся Π³ΠΎΡ€ΠΈΠ·ΠΎΠ½Ρ‚Π°Π»ΡŒΠ½Π°Ρ сила 45,0 Н для ускорСния ΠΎΠ±ΠΎΠΈΡ… ящиков ΠΏΠΎ ΠΏΠΎΠ»Ρƒ. НС ΠΎΠ±Ρ€Π°Ρ‰Π°ΠΉΡ‚Π΅ внимания Π½Π° силы трСния ΠΈ ΠΎΠΏΡ€Π΅Π΄Π΅Π»ΠΈΡ‚Π΅ ускорСниС ΠΊΠΎΡ€ΠΎΠ±ΠΎΠΊ ΠΈ силу, Π΄Π΅ΠΉΡΡ‚Π²ΡƒΡŽΡ‰ΡƒΡŽ ΠΌΠ΅ΠΆΠ΄Ρƒ ΠΊΠΎΡ€ΠΎΠ±ΠΊΠ°ΠΌΠΈ.

            ΠŸΠ΅Ρ€Π²Ρ‹ΠΉ ΠΏΠΎΠ΄Ρ…ΠΎΠ΄ ΠΊ этой ΠΏΡ€ΠΎΠ±Π»Π΅ΠΌΠ΅ ΠΏΡ€Π΅Π΄ΠΏΠΎΠ»Π°Π³Π°Π΅Ρ‚ Π΄Π²ΠΎΠΉΠ½ΠΎΠ΅ сочСтаниС систСмного Π°Π½Π°Π»ΠΈΠ·Π° ΠΈ Π°Π½Π°Π»ΠΈΠ·Π° ΠΎΡ‚Π΄Π΅Π»ΡŒΠ½ΠΎΠ³ΠΎ ΠΎΠ±ΡŠΠ΅ΠΊΡ‚Π°.Как ΡƒΠΆΠ΅ ΡƒΠΏΠΎΠΌΠΈΠ½Π°Π»ΠΎΡΡŒ, систСмный Π°Π½Π°Π»ΠΈΠ· ΠΈΡΠΏΠΎΠ»ΡŒΠ·ΡƒΠ΅Ρ‚ΡΡ для опрСдСлСния ускорСния, Π° Π°Π½Π°Π»ΠΈΠ· ΠΎΡ‚Π΄Π΅Π»ΡŒΠ½ΠΎΠ³ΠΎ ΠΎΠ±ΡŠΠ΅ΠΊΡ‚Π° ΠΈΡΠΏΠΎΠ»ΡŒΠ·ΡƒΠ΅Ρ‚ΡΡ для опрСдСлСния сил, Π΄Π΅ΠΉΡΡ‚Π²ΡƒΡŽΡ‰ΠΈΡ… ΠΌΠ΅ΠΆΠ΄Ρƒ ΠΎΠ±ΡŠΠ΅ΠΊΡ‚Π°ΠΌΠΈ. Π’ систСмном Π°Π½Π°Π»ΠΈΠ·Π΅ Π΄Π²Π° ΠΎΠ±ΡŠΠ΅ΠΊΡ‚Π° Ρ€Π°ΡΡΠΌΠ°Ρ‚Ρ€ΠΈΠ²Π°ΡŽΡ‚ΡΡ ΠΊΠ°ΠΊ ΠΎΠ΄ΠΈΠ½ ΠΎΠ±ΡŠΠ΅ΠΊΡ‚. Π Π°Π·Π΄Π΅Π»ΠΈΡ‚Π΅Π»ΡŒΠ½Π°Ρ линия, Ρ€Π°Π·Π΄Π΅Π»ΡΡŽΡ‰Π°Ρ ΠΎΠ±ΡŠΠ΅ΠΊΡ‚Ρ‹, игнорируСтся. Масса систСмы ΠΈΠ· Π΄Π²ΡƒΡ… ΠΎΠ±ΡŠΠ΅ΠΊΡ‚ΠΎΠ² 15,0 ΠΊΠ³. Π‘Ρ…Π΅ΠΌΠ° свободного Ρ‚Π΅Π»Π° для систСмы ΠΏΠΎΠΊΠ°Π·Π°Π½Π° справа. На систСму Π΄Π΅ΠΉΡΡ‚Π²ΡƒΡŽΡ‚ Ρ‚Ρ€ΠΈ силы – сила тяТСсти (ЗСмля тянСт Π²Π½ΠΈΠ· Π½Π° 15.0 ΠΊΠ³ массы), Π½ΠΎΡ€ΠΌΠ°Π»ΡŒΠ½Π°Ρ сила (ΠΏΠΎΠ» Ρ‚ΠΎΠ»ΠΊΠ°Π΅Ρ‚ систСму Π²Π²Π΅Ρ€Ρ…, Ρ‡Ρ‚ΠΎΠ±Ρ‹ Π²Ρ‹Π΄Π΅Ρ€ΠΆΠ°Ρ‚ΡŒ Π΅Π΅ вСс) ΠΈ прилоТСнная сила (Ρ€ΡƒΠΊΠ° Π΄Π°Π²ΠΈΡ‚ Π½Π° заднюю Ρ‡Π°ΡΡ‚ΡŒ систСмы). Π‘ΠΈΠ»Π°, Π΄Π΅ΠΉΡΡ‚Π²ΡƒΡŽΡ‰Π°Ρ ΠΌΠ΅ΠΆΠ΄Ρƒ ящиком 5,0 ΠΊΠ³ ΠΈ ящиком 10,0 ΠΊΠ³, Π½Π΅ учитываСтся Π² систСмном Π°Π½Π°Π»ΠΈΠ·Π΅, Ρ‚Π°ΠΊ ΠΊΠ°ΠΊ это внутрСнняя сила . Π’Π°ΠΊ ΠΆΠ΅, ΠΊΠ°ΠΊ силы, ΡƒΠ΄Π΅Ρ€ΠΆΠΈΠ²Π°ΡŽΡ‰ΠΈΠ΅ вмСстС Π°Ρ‚ΠΎΠΌΡ‹ Π²Π½ΡƒΡ‚Ρ€ΠΈ ΠΎΠ±ΡŠΠ΅ΠΊΡ‚Π°, Π½Π΅ Π²ΠΊΠ»ΡŽΡ‡Π΅Π½Ρ‹ Π² Π΄ΠΈΠ°Π³Ρ€Π°ΠΌΠΌΡƒ свободного Ρ‚Π΅Π»Π°, Ρ‚Π°ΠΊ ΠΈ силы, ΡƒΠ΄Π΅Ρ€ΠΆΠΈΠ²Π°ΡŽΡ‰ΠΈΠ΅ вмСстС части систСмы, ΠΈΠ³Π½ΠΎΡ€ΠΈΡ€ΡƒΡŽΡ‚ΡΡ. Π­Ρ‚ΠΎ считаСтся Π²Π½ΡƒΡ‚Ρ€Π΅Π½Π½ΠΈΠΌΠΈ силами; ΠΏΡ€ΠΈ построСнии Π΄ΠΈΠ°Π³Ρ€Π°ΠΌΠΌ свободного Ρ‚Π΅Π»Π° ΡƒΡ‡ΠΈΡ‚Ρ‹Π²Π°ΡŽΡ‚ΡΡ Ρ‚ΠΎΠ»ΡŒΠΊΠΎ внСшниС силы.Π’Π΅Π»ΠΈΡ‡ΠΈΠ½Π° силы тяТСсти составляСт m β€’ g ΠΈΠ»ΠΈ 147 Н. Π’Π΅Π»ΠΈΡ‡ΠΈΠ½Π° Π½ΠΎΡ€ΠΌΠ°Π»ΡŒΠ½ΠΎΠΉ силы Ρ‚Π°ΠΊΠΆΠ΅ составляСт 147 Н, ΠΏΠΎΡΠΊΠΎΠ»ΡŒΠΊΡƒ ΠΎΠ½Π° Π΄ΠΎΠ»ΠΆΠ½Π° Π²Ρ‹Π΄Π΅Ρ€ΠΆΠΈΠ²Π°Ρ‚ΡŒ вСс (147 Н) систСмы. ΠŸΡ€ΠΈΠ»ΠΎΠΆΠ΅Π½Π½Π°Ρ сила составляСт 45,0 Н. Π’Ρ‚ΠΎΡ€ΠΎΠΉ Π·Π°ΠΊΠΎΠ½ ΠΡŒΡŽΡ‚ΠΎΠ½Π° (a = F net / ΠΌ) ΠΌΠΎΠΆΠ½ΠΎ ΠΈΡΠΏΠΎΠ»ΡŒΠ·ΠΎΠ²Π°Ρ‚ΡŒ для опрСдСлСния ускорСния. ΠŸΡ€ΠΈ 45,0 Н для F , Π½Π΅Ρ‚Ρ‚ΠΎ ΠΈ 15,0 ΠΊΠ³ для ΠΌ, ускорСниС составляСт 3,0 ΠΌ / с 2 .

            Π’Π΅ΠΏΠ΅Ρ€ΡŒ, ΠΊΠΎΠ³Π΄Π° ускорСниС ΠΎΠΏΡ€Π΅Π΄Π΅Π»Π΅Π½ΠΎ, для ΠΊΠ°ΠΆΠ΄ΠΎΠ³ΠΎ ΠΎΠ±ΡŠΠ΅ΠΊΡ‚Π° ΠΌΠΎΠΆΠ½ΠΎ Π²Ρ‹ΠΏΠΎΠ»Π½ΠΈΡ‚ΡŒ Π°Π½Π°Π»ΠΈΠ· ΠΎΡ‚Π΄Π΅Π»ΡŒΠ½ΠΎΠ³ΠΎ ΠΎΠ±ΡŠΠ΅ΠΊΡ‚Π°, Ρ‡Ρ‚ΠΎΠ±Ρ‹ ΠΎΠΏΡ€Π΅Π΄Π΅Π»ΠΈΡ‚ΡŒ силу, Π΄Π΅ΠΉΡΡ‚Π²ΡƒΡŽΡ‰ΡƒΡŽ ΠΌΠ΅ΠΆΠ΄Ρƒ Π½ΠΈΠΌΠΈ.НСваТно, ΠΊΠ°ΠΊΠΎΠΉ ΠΎΠ±ΡŠΠ΅ΠΊΡ‚ Π²Ρ‹Π±Ρ€Π°Π½; Ρ€Π΅Π·ΡƒΠ»ΡŒΡ‚Π°Ρ‚ Π±ΡƒΠ΄Π΅Ρ‚ ΠΎΠ΄ΠΈΠ½Π°ΠΊΠΎΠ²Ρ‹ΠΌ Π² любом случаС. Π—Π΄Π΅ΡΡŒ Π°Π½Π°Π»ΠΈΠ· ΠΎΡ‚Π΄Π΅Π»ΡŒΠ½ΠΎΠ³ΠΎ ΠΎΠ±ΡŠΠ΅ΠΊΡ‚Π° проводится Π½Π° ΠΎΠ±ΡŠΠ΅ΠΊΡ‚Π΅ массой 10,0 ΠΊΠ³ (Ρ‚ΠΎΠ»ΡŒΠΊΠΎ ΠΏΠΎΡ‚ΠΎΠΌΡƒ, Ρ‡Ρ‚ΠΎ Π½Π° Π½Π΅Π³ΠΎ дСйствуСт Π½Π° ΠΎΠ΄Π½Ρƒ силу мСньшС). Π‘ΠΏΡ€Π°Π²Π° ΠΏΠΎΠΊΠ°Π·Π°Π½Π° Π΄ΠΈΠ°Π³Ρ€Π°ΠΌΠΌΠ° свободного Ρ‚Π΅Π»Π° для ΠΎΠ±ΡŠΠ΅ΠΊΡ‚Π° вСсом 10,0 ΠΊΠ³. На Π½Π΅Π³ΠΎ Π΄Π΅ΠΉΡΡ‚Π²ΡƒΡŽΡ‚ Ρ‚ΠΎΠ»ΡŒΠΊΠΎ Ρ‚Ρ€ΠΈ силы: сила тяТСсти Π½Π° 10,0 ΠΊΠ³, сила ΠΎΠΏΠΎΡ€Ρ‹ (ΠΎΡ‚ ΠΏΠΎΠ»Π°, Ρ‚ΠΎΠ»ΠΊΠ°ΡŽΡ‰Π°Ρ Π²Π²Π΅Ρ€Ρ…) ΠΈ сила ΠΏΡ€Π°Π²ΠΎΠ³ΠΎ ΠΊΠΎΠ½Ρ‚Π°ΠΊΡ‚Π° (F , ΠΊΠΎΠ½Ρ‚Π°ΠΊΡ‚ ). По ΠΌΠ΅Ρ€Π΅ Ρ‚ΠΎΠ³ΠΎ ΠΊΠ°ΠΊ ΠΎΠ±ΡŠΠ΅ΠΊΡ‚ вСсом 5,0 ΠΊΠ³ ускоряСтся Π²ΠΏΡ€Π°Π²ΠΎ, ΠΎΠ½ Π±ΡƒΠ΄Π΅Ρ‚ Ρ‚ΠΎΠ»ΠΊΠ°Ρ‚ΡŒ Π²ΠΏΡ€Π°Π²ΠΎ Π½Π° 10.0-ΠΊΠΈΠ»ΠΎΠ³Ρ€Π°ΠΌΠΌΠΎΠ²Ρ‹ΠΉ ΠΎΠ±ΡŠΠ΅ΠΊΡ‚; это извСстно ΠΊΠ°ΠΊ контактная сила (ΠΈΠ»ΠΈ Π½ΠΎΡ€ΠΌΠ°Π»ΡŒΠ½Π°Ρ сила, ΠΈΠ»ΠΈ прилоТСнная сила, или…). Π’Π΅Ρ€Ρ‚ΠΈΠΊΠ°Π»ΡŒΠ½Ρ‹Π΅ силы ΡƒΡ€Π°Π²Π½ΠΎΠ²Π΅ΡˆΠΈΠ²Π°ΡŽΡ‚ Π΄Ρ€ΡƒΠ³ Π΄Ρ€ΡƒΠ³Π°, ΠΏΠΎΡΠΊΠΎΠ»ΡŒΠΊΡƒ Π½Π΅Ρ‚ Π²Π΅Ρ€Ρ‚ΠΈΠΊΠ°Π»ΡŒΠ½ΠΎΠ³ΠΎ ускорСния. ЕдинствСнная Π½Π΅ΡƒΡ€Π°Π²Π½ΠΎΠ²Π΅ΡˆΠ΅Π½Π½Π°Ρ сила, Π΄Π΅ΠΉΡΡ‚Π²ΡƒΡŽΡ‰Π°Ρ Π½Π° ΠΎΠ±ΡŠΠ΅ΠΊΡ‚ вСсом 10,0 ΠΊΠ³, – это Fcontact. Π­Ρ‚Π° сила прСдставляСт собой Ρ‡ΠΈΡΡ‚ΡƒΡŽ силу ΠΈ Ρ€Π°Π²Π½Π° m β€’ a, Π³Π΄Π΅ m Ρ€Π°Π²Π½ΠΎ 10,0 ΠΊΠ³ (ΠΏΠΎΡΠΊΠΎΠ»ΡŒΠΊΡƒ этот Π°Π½Π°Π»ΠΈΠ· ΠΏΡ€Π΅Π΄Π½Π°Π·Π½Π°Ρ‡Π΅Π½ для ΠΎΠ±ΡŠΠ΅ΠΊΡ‚Π° вСсом 10,0 ΠΊΠ³), Π° a ΡƒΠΆΠ΅ Π±Ρ‹Π»ΠΎ ΠΎΠΏΡ€Π΅Π΄Π΅Π»Π΅Π½ΠΎ Ρ€Π°Π²Π½Ρ‹ΠΌ 3,0 ΠΌ / с 2 . Чистая сила Ρ€Π°Π²Π½Π° 30,0 Н. Π­Ρ‚Π° чистая сила прСдставляСт собой силу, с ΠΊΠΎΡ‚ΠΎΡ€ΠΎΠΉ ΠΎΠ±ΡŠΠ΅ΠΊΡ‚ массой 5,0 ΠΊΠ³ Ρ‚ΠΎΠ»ΠΊΠ°Π΅Ρ‚ 10.ΠžΠ±ΡŠΠ΅ΠΊΡ‚ вСсом 0 ΠΊΠ³ справа; ΠΎΠ½Π° ΠΈΠΌΠ΅Π΅Ρ‚ Π²Π΅Π»ΠΈΡ‡ΠΈΠ½Ρƒ 30,0 Н. Π’Π°ΠΊΠΈΠΌ ΠΎΠ±Ρ€Π°Π·ΠΎΠΌ, ΠΎΡ‚Π²Π΅Ρ‚Ρ‹ Π½Π° Π΄Π²Π° нСизвСстных для этой ΠΏΡ€ΠΎΠ±Π»Π΅ΠΌΡ‹: 3,0 ΠΌ / с 2 ΠΈ 30,0 Н.

            Π’Π΅ΠΏΠ΅Ρ€ΡŒ рассмотрим Ρ€Π΅ΡˆΠ΅Π½ΠΈΠ΅ этой ΠΆΠ΅ ΠΏΡ€ΠΎΠ±Π»Π΅ΠΌΡ‹, ΠΈΡΠΏΠΎΠ»ΡŒΠ·ΡƒΡ Π²Ρ‚ΠΎΡ€ΠΎΠΉ ΠΏΠΎΠ΄Ρ…ΠΎΠ΄ – использованиС Π°Π½Π°Π»ΠΈΠ·Π° Π΄Π²ΡƒΡ… ΠΎΡ‚Π΄Π΅Π»ΡŒΠ½Ρ‹Ρ… ΠΎΠ±ΡŠΠ΅ΠΊΡ‚ΠΎΠ². Π’ процСссС этого Π²Ρ‚ΠΎΡ€ΠΎΠ³ΠΎ ΠΏΠΎΠ΄Ρ…ΠΎΠ΄Π° ΠΌΡ‹ ΠΏΡ€ΠΎΠΈΠ³Π½ΠΎΡ€ΠΈΡ€ΡƒΠ΅ΠΌ Ρ‚ΠΎΡ‚ Ρ„Π°ΠΊΡ‚, Ρ‡Ρ‚ΠΎ Π·Π½Π°Π΅ΠΌ ΠΎΡ‚Π²Π΅Ρ‚Ρ‹, ΠΈ Π±ΡƒΠ΄Π΅ΠΌ ΠΏΡ€Π΅Π΄ΠΏΠΎΠ»Π°Π³Π°Ρ‚ΡŒ, Ρ‡Ρ‚ΠΎ Ρ€Π΅ΡˆΠ°Π΅ΠΌ ΠΏΡ€ΠΎΠ±Π»Π΅ΠΌΡƒ Π²ΠΏΠ΅Ρ€Π²Ρ‹Π΅. ΠŸΡ€ΠΈ Ρ‚Π°ΠΊΠΎΠΌ ΠΏΠΎΠ΄Ρ…ΠΎΠ΄Π΅ Π²Ρ‹ΠΏΠΎΠ»Π½ΡΡŽΡ‚ΡΡ Π΄Π²Π° ΠΎΡ‚Π΄Π΅Π»ΡŒΠ½Ρ‹Ρ… Π°Π½Π°Π»ΠΈΠ·Π° Π΄ΠΈΠ°Π³Ρ€Π°ΠΌΠΌ свободного Ρ‚Π΅Π»Π°.На Π΄ΠΈΠ°Π³Ρ€Π°ΠΌΠΌΠ°Ρ… Π½ΠΈΠΆΠ΅ ΠΏΠΎΠΊΠ°Π·Π°Π½Ρ‹ Π΄ΠΈΠ°Π³Ρ€Π°ΠΌΠΌΡ‹ свободного Ρ‚Π΅Π»Π° для Π΄Π²ΡƒΡ… ΠΎΠ±ΡŠΠ΅ΠΊΡ‚ΠΎΠ².

            ΠžΠ±Ρ€Π°Ρ‚ΠΈΡ‚Π΅ Π²Π½ΠΈΠΌΠ°Π½ΠΈΠ΅, Ρ‡Ρ‚ΠΎ Π½Π° ΠΎΠ±ΡŠΠ΅ΠΊΡ‚ массой 5,0 ΠΊΠ³ сзади Π΄Π΅ΠΉΡΡ‚Π²ΡƒΡŽΡ‚ Ρ‡Π΅Ρ‚Ρ‹Ρ€Π΅ силы. Π”Π²Π΅ Π²Π΅Ρ€Ρ‚ΠΈΠΊΠ°Π»ΡŒΠ½Ρ‹Π΅ силы – F grav ΠΈ F norm – ΠΎΡ‡Π΅Π²ΠΈΠ΄Π½Ρ‹Π΅ силы. ΠŸΡ€ΠΈΠ»ΠΎΠΆΠ΅Π½Π½Π°Ρ сила 45,0 Н (F ΠΏΡ€ΠΈΠ»ΠΎΠΆΠ΅Π½ΠΈΠ΅ ) являСтся Ρ€Π΅Π·ΡƒΠ»ΡŒΡ‚Π°Ρ‚ΠΎΠΌ давлСния Ρ€ΡƒΠΊΠΈ Π½Π° Π·Π°Π΄Π½ΠΈΠΉ ΠΎΠ±ΡŠΠ΅ΠΊΡ‚, ΠΊΠ°ΠΊ описано Π² Ρ„ΠΎΡ€ΠΌΡƒΠ»ΠΈΡ€ΠΎΠ²ΠΊΠ΅ ΠΏΡ€ΠΎΠ±Π»Π΅ΠΌΡ‹ ΠΈ ΠΏΠΎΠΊΠ°Π·Π°Π½ΠΎ Π½Π° Π΄ΠΈΠ°Π³Ρ€Π°ΠΌΠΌΠ΅. Π‘ΠΈΠ»Π° Π»Π΅Π²ΠΎΠ³ΠΎ ΠΊΠΎΠ½Ρ‚Π°ΠΊΡ‚Π° Π½Π° 5.ΠžΠ±ΡŠΠ΅ΠΊΡ‚ вСсом 0 ΠΊΠ³ – это сила, с ΠΊΠΎΡ‚ΠΎΡ€ΠΎΠΉ ΠΎΠ±ΡŠΠ΅ΠΊΡ‚ вСсом 10,0 ΠΊΠ³ Ρ‚ΠΎΠ»ΠΊΠ°Π΅Ρ‚ Π²Π»Π΅Π²ΠΎ ΠΎΠ±ΡŠΠ΅ΠΊΡ‚ вСсом 5,0 ΠΊΠ³. Когда дСлаСтся ΠΏΠΎΠΏΡ‹Ρ‚ΠΊΠ° ΠΏΠΎΠ΄Ρ‚ΠΎΠ»ΠΊΠ½ΡƒΡ‚ΡŒ Π·Π°Π΄Π½ΠΈΠΉ ΠΎΠ±ΡŠΠ΅ΠΊΡ‚ (ΠΎΠ±ΡŠΠ΅ΠΊΡ‚ вСсом 5,0 ΠΊΠ³) Π²ΠΏΠ΅Ρ€Π΅Π΄, ΠΏΠ΅Ρ€Π΅Π΄Π½ΠΈΠΉ ΠΎΠ±ΡŠΠ΅ΠΊΡ‚ (ΠΎΠ±ΡŠΠ΅ΠΊΡ‚ вСсом 10,0 ΠΊΠ³) Ρ‚ΠΎΠ»ΠΊΠ°Π΅Ρ‚ Π΅Π³ΠΎ Π½Π°Π·Π°Π΄. Π­Ρ‚Π° сила Ρ€Π°Π²Π½Π° ΠΈ ΠΏΡ€ΠΎΡ‚ΠΈΠ²ΠΎΠΏΠΎΠ»ΠΎΠΆΠ½Π° Π·Π°Π΄Π½Π΅ΠΌΡƒ ΠΎΠ±ΡŠΠ΅ΠΊΡ‚Ρƒ, Ρ‚ΠΎΠ»ΠΊΠ°ΡŽΡ‰Π΅ΠΌΡƒ Π²ΠΏΠ΅Ρ€Π΅Π΄ ΠΏΠ΅Ρ€Π΅Π΄Π½ΠΈΠΉ ΠΎΠ±ΡŠΠ΅ΠΊΡ‚. Π­Ρ‚Π° сила просто ΠΎΠ±ΠΎΠ·Π½Π°Ρ‡Π΅Π½Π° ΠΊΠ°ΠΊ F , ΠΊΠΎΠ½Ρ‚Π°ΠΊΡ‚ для ΠΎΠ±Π΅ΠΈΡ… Π΄ΠΈΠ°Π³Ρ€Π°ΠΌΠΌ свободного Ρ‚Π΅Π»Π°. На Π΄ΠΈΠ°Π³Ρ€Π°ΠΌΠΌΠ΅ свободного Ρ‚Π΅Π»Π° для ΠΎΠ±ΡŠΠ΅ΠΊΡ‚Π° массой 10,0 ΠΊΠ³ ΠΏΡ€ΠΈΡΡƒΡ‚ΡΡ‚Π²ΡƒΡŽΡ‚ Ρ‚ΠΎΠ»ΡŒΠΊΠΎ Ρ‚Ρ€ΠΈ силы. Π•Ρ‰Π΅ Ρ€Π°Π·, Π΄Π²Π΅ Π²Π΅Ρ€Ρ‚ΠΈΠΊΠ°Π»ΡŒΠ½Ρ‹Π΅ силы – F grav ΠΈ F norm – ΠΎΡ‡Π΅Π²ΠΈΠ΄Π½Ρ‹Π΅ силы.Π“ΠΎΡ€ΠΈΠ·ΠΎΠ½Ρ‚Π°Π»ΡŒΠ½Π°Ρ сила – это просто ΠΎΠ±ΡŠΠ΅ΠΊΡ‚ вСсом 5,0 ΠΊΠ³, Ρ‚ΠΎΠ»ΠΊΠ°ΡŽΡ‰ΠΈΠΉ Π²ΠΏΠ΅Ρ€Π΅Π΄ ΠΎΠ±ΡŠΠ΅ΠΊΡ‚ вСсом 10,0 ΠΊΠ³. ΠŸΡ€ΠΈΠ»ΠΎΠΆΠ΅Π½Π½Π°Ρ сила 45,0 Н Π½Π΅ дСйствуСт Π½Π° этот ΠΎΠ±ΡŠΠ΅ΠΊΡ‚ вСсом 10,0 ΠΊΠ³; ΠΎΠ½ΠΎ примСняСтся ΠΊ ΠΎΠ±ΡŠΠ΅ΠΊΡ‚Ρƒ массой 5,0 ΠΊΠ³ ΠΈ ΡƒΠΆΠ΅ Π±Ρ‹Π»ΠΎ рассмотрСно Π½Π° ΠΏΡ€Π΅Π΄Ρ‹Π΄ΡƒΡ‰Π΅ΠΉ Π΄ΠΈΠ°Π³Ρ€Π°ΠΌΠΌΠ΅ свободного Ρ‚Π΅Π»Π°.

            Π’Π΅ΠΏΠ΅Ρ€ΡŒ Ρ†Π΅Π»ΡŒ этого ΠΏΠΎΠ΄Ρ…ΠΎΠ΄Π° состоит Π² Ρ‚ΠΎΠΌ, Ρ‡Ρ‚ΠΎΠ±Ρ‹ ΡΠΎΠ·Π΄Π°Ρ‚ΡŒ систСму ΠΈΠ· Π΄Π²ΡƒΡ… ΡƒΡ€Π°Π²Π½Π΅Π½ΠΈΠΉ, ΠΊΠΎΡ‚ΠΎΡ€ΡƒΡŽ ΠΌΠΎΠΆΠ½ΠΎ Ρ€Π΅ΡˆΠΈΡ‚ΡŒ для Π΄Π²ΡƒΡ… нСизвСстных Π·Π½Π°Ρ‡Π΅Π½ΠΈΠΉ. ИспользованиС F net = m β€’ a с Π΄ΠΈΠ°Π³Ρ€Π°ΠΌΠΌΠΎΠΉ свободного Ρ‚Π΅Π»Π° для ΠΎΠ±ΡŠΠ΅ΠΊΡ‚Π° массой 5,0 ΠΊΠ³ даст ΡƒΡ€Π°Π²Π½Π΅Π½ΠΈΠ΅ 1 Π½ΠΈΠΆΠ΅:

            45.0 – ΠšΠΎΠ½Ρ‚Π°ΠΊΡ‚ F = 5,0 β€’ a

            ИспользованиС F net = m β€’ a с Π΄ΠΈΠ°Π³Ρ€Π°ΠΌΠΌΠΎΠΉ свободного Ρ‚Π΅Π»Π° для ΠΎΠ±ΡŠΠ΅ΠΊΡ‚Π° массой 10,0 ΠΊΠ³ даст ΡƒΡ€Π°Π²Π½Π΅Π½ΠΈΠ΅ 2 Π½ΠΈΠΆΠ΅:

            F ΠΊΠΎΠ½Ρ‚Π°ΠΊΡ‚ = 10,0 β€’ a

            (ΠžΠ±Ρ€Π°Ρ‚ΠΈΡ‚Π΅ Π²Π½ΠΈΠΌΠ°Π½ΠΈΠ΅, Ρ‡Ρ‚ΠΎ Π΅Π΄ΠΈΠ½ΠΈΡ†Ρ‹ Π±Ρ‹Π»ΠΈ ΠΈΡΠΊΠ»ΡŽΡ‡Π΅Π½Ρ‹ ΠΈΠ· Π£Ρ€Π°Π²Π½Π΅Π½ΠΈΠΉ 1 ΠΈ 2, Ρ‡Ρ‚ΠΎΠ±Ρ‹ ΠΎΡ‡ΠΈΡΡ‚ΠΈΡ‚ΡŒ уравнСния Π΄ΠΎ .) Если Π²Ρ‹Ρ€Π°ΠΆΠ΅Π½ΠΈΠ΅ 10.0 β€’ a подставляСтся Π² Π£Ρ€Π°Π²Π½Π΅Π½ΠΈΠ΅ 1 для F ΠΊΠΎΠ½Ρ‚Π°ΠΊΡ‚Π° , Ρ‚ΠΎΠ³Π΄Π° Π£Ρ€Π°Π²Π½Π΅Π½ΠΈΠ΅ 1 сводится ΠΊ СдинствСнному ΡƒΡ€Π°Π²Π½Π΅Π½ΠΈΠ΅ с ΠΎΠ΄Π½ΠΎΠΉ нСизвСстной.Π£Ρ€Π°Π²Π½Π΅Π½ΠΈΠ΅ ΠΏΡ€ΠΈΠ½ΠΈΠΌΠ°Π΅Ρ‚ Π²ΠΈΠ΄

            45,0 – 10,0 β€’ a = 5,0 β€’ a

            ΠŸΠ°Ρ€Π° шагов Π°Π»Π³Π΅Π±Ρ€Ρ‹ ΠΏΡ€ΠΈΠ²ΠΎΠ΄ΠΈΡ‚ ΠΊ Π·Π½Π°Ρ‡Π΅Π½ΠΈΡŽ ускорСния 3,0 ΠΌ / с 2 . Π­Ρ‚ΠΎ Π·Π½Π°Ρ‡Π΅Π½ΠΈΠ΅ a ΠΌΠΎΠΆΠ½ΠΎ ΠΏΠΎΠ΄ΡΡ‚Π°Π²ΠΈΡ‚ΡŒ ΠΎΠ±Ρ€Π°Ρ‚Π½ΠΎ Π² ΡƒΡ€Π°Π²Π½Π΅Π½ΠΈΠ΅ 2, Ρ‡Ρ‚ΠΎΠ±Ρ‹ ΠΎΠΏΡ€Π΅Π΄Π΅Π»ΠΈΡ‚ΡŒ ΠΊΠΎΠ½Ρ‚Π°ΠΊΡ‚Π½ΠΎΠ΅ усилиС:

            ΠšΠΎΠ½Ρ‚Π°ΠΊΡ‚ F = 10,0 β€’ a = 10,0 β€’ 3,0
            ΠšΠΎΠ½Ρ‚Π°ΠΊΡ‚ F = 30,0 Н

            Как ΠΌΠΎΠΆΠ½ΠΎ Π²ΠΈΠ΄Π΅Ρ‚ΡŒ, использованиС Π²Ρ‚ΠΎΡ€ΠΎΠ³ΠΎ ΠΏΠΎΠ΄Ρ…ΠΎΠ΄Π° ΠΊ Ρ€Π΅ΡˆΠ΅Π½ΠΈΡŽ Π΄Π²ΡƒΡ… тСлСсных Π·Π°Π΄Π°Ρ‡ Π΄Π°Π΅Ρ‚ Ρ‚Π΅ ΠΆΠ΅ Π΄Π²Π° ΠΎΡ‚Π²Π΅Ρ‚Π° для Π΄Π²ΡƒΡ… нСизвСстных.Π’Π΅ΠΏΠ΅Ρ€ΡŒ ΠΌΡ‹ ΠΏΠΎΠΏΡ€ΠΎΠ±ΡƒΠ΅ΠΌ Ρ‚Π΅ ΠΆΠ΅ Π΄Π²Π° ΠΏΠΎΠ΄Ρ…ΠΎΠ΄Π° ΠΊ ΠΎΡ‡Π΅Π½ΡŒ ΠΏΠΎΡ…ΠΎΠΆΠ΅ΠΉ Π·Π°Π΄Π°Ρ‡Π΅, которая Π²ΠΊΠ»ΡŽΡ‡Π°Π΅Ρ‚ Π² сСбя силу трСния.


            ΠŸΡ€ΠΈΠΌΠ΅Ρ€ ΠΏΡ€ΠΎΠ±Π»Π΅ΠΌΡ‹ 2:

            ΠšΠΎΡ€ΠΎΠ±ΠΊΠ° 5,0 ΠΊΠ³ ΠΈ ΠΊΠΎΡ€ΠΎΠ±ΠΊΠ° 10,0 ΠΊΠ³ ΡΠΎΠΏΡ€ΠΈΠΊΠ°ΡΠ°ΡŽΡ‚ΡΡ Π΄Ρ€ΡƒΠ³ с Π΄Ρ€ΡƒΠ³ΠΎΠΌ. К 5-ΠΊΠΈΠ»ΠΎΠ³Ρ€Π°ΠΌΠΌΠΎΠ²ΠΎΠΌΡƒ ящику прилагаСтся Π³ΠΎΡ€ΠΈΠ·ΠΎΠ½Ρ‚Π°Π»ΡŒΠ½Π°Ρ сила 45,0 Н для ускорСния ΠΎΠ±ΠΎΠΈΡ… ящиков ΠΏΠΎ ΠΏΠΎΠ»Ρƒ. ΠšΠΎΡΡ„Ρ„ΠΈΡ†ΠΈΠ΅Π½Ρ‚ кинСтичСского трСния 0.200. ΠžΠΏΡ€Π΅Π΄Π΅Π»ΠΈΡ‚Π΅ ускорСниС ΠΈ ΠΊΠΎΠ½Ρ‚Π°ΠΊΡ‚Π½ΠΎΠ΅ усилиС.

            НашС ΠΏΠ΅Ρ€Π²ΠΎΠ΅ Ρ€Π΅ΡˆΠ΅Π½ΠΈΠ΅ этой ΠΏΡ€ΠΎΠ±Π»Π΅ΠΌΡ‹ Π±ΡƒΠ΄Π΅Ρ‚ Π²ΠΊΠ»ΡŽΡ‡Π°Ρ‚ΡŒ Π΄Π²ΠΎΠΉΠ½ΡƒΡŽ ΠΊΠΎΠΌΠ±ΠΈΠ½Π°Ρ†ΠΈΡŽ систСмного Π°Π½Π°Π»ΠΈΠ·Π° ΠΈ Π°Π½Π°Π»ΠΈΠ·Π° ΠΎΡ‚Π΄Π΅Π»ΡŒΠ½ΠΎΠ³ΠΎ ΠΎΠ±ΡŠΠ΅ΠΊΡ‚Π°. Как Π²Ρ‹, вСроятно, Π·Π°ΠΌΠ΅Ρ‚ΠΈΠ»ΠΈ, ΠΏΡ€ΠΈΠΌΠ΅Ρ€ ΠΏΡ€ΠΎΠ±Π»Π΅ΠΌΡ‹ 2 Π°Π½Π°Π»ΠΎΠ³ΠΈΡ‡Π΅Π½ ΠΏΡ€ΠΈΠΌΠ΅Ρ€Ρƒ ΠΏΡ€ΠΎΠ±Π»Π΅ΠΌΡ‹ 1, Π·Π° ΠΈΡΠΊΠ»ΡŽΡ‡Π΅Π½ΠΈΠ΅ΠΌ Ρ‚ΠΎΠ³ΠΎ, Ρ‡Ρ‚ΠΎ ΠΏΠΎΠ²Π΅Ρ€Ρ…Π½ΠΎΡΡ‚ΡŒ Π½Π΅ лишСна трСния Π² ΠΏΡ€ΠΈΠΌΠ΅Ρ€Π΅ Π·Π°Π΄Π°Ρ‡ΠΈ 2. ΠŸΠΎΡΡ‚ΠΎΠΌΡƒ ΠΏΡ€ΠΈ ΠΏΡ€ΠΎΠ²Π΅Π΄Π΅Π½ΠΈΠΈ Π°Π½Π°Π»ΠΈΠ·Π° систСмы Π² этом Π²Ρ‚ΠΎΡ€ΠΎΠΌ ΠΏΡ€ΠΈΠΌΠ΅Ρ€Π΅ Π½Π΅ΠΎΠ±Ρ…ΠΎΠ΄ΠΈΠΌΠΎ ΡƒΡ‡ΠΈΡ‚Ρ‹Π²Π°Ρ‚ΡŒ Ρ‚Ρ€Π΅Π½ΠΈΠ΅ Π² систСмС вСсом 15 ΠΊΠ³. . Π˜Ρ‚Π°ΠΊ, Π΄ΠΈΠ°Π³Ρ€Π°ΠΌΠΌΠ° свободного Ρ‚Π΅Π»Π° для систСмы Ρ‚Π΅ΠΏΠ΅Ρ€ΡŒ Π²ΠΊΠ»ΡŽΡ‡Π°Π΅Ρ‚ Ρ‡Π΅Ρ‚Ρ‹Ρ€Π΅ силы – Ρ‚Π΅ ΠΆΠ΅ Ρ‚Ρ€ΠΈ, Ρ‡Ρ‚ΠΎ ΠΈ Π² ΠΏΡ€ΠΈΠΌΠ΅Ρ€Π΅ Π·Π°Π΄Π°Ρ‡ΠΈ 1, плюс Π»Π΅Π²ΡƒΡŽ силу трСния.Π‘ΠΈΠ»Π° трСния Π² систСмС ΠΌΠΎΠΆΠ΅Ρ‚ Π±Ρ‹Ρ‚ΡŒ рассчитана ΠΊΠ°ΠΊ ΞΌ β€’ F norm , Π³Π΄Π΅ F norm – Π½ΠΎΡ€ΠΌΠ°Π»ΡŒΠ½Π°Ρ сила, испытываСмая систСмой. Норма F систСмы Ρ€Π°Π²Π½Π° силС тяТСсти, Π΄Π΅ΠΉΡΡ‚Π²ΡƒΡŽΡ‰Π΅ΠΉ Π½Π° систСму массой 15,0 ΠΊΠ³; это Π·Π½Π°Ρ‡Π΅Π½ΠΈΠ΅ составляСт 147 Н. Π˜Ρ‚Π°ΠΊ,

            F frict = ΞΌ β€’ F norm = (0.200) β€’ (147 N) = 29.4 N

            Π’Π΅Ρ€Ρ‚ΠΈΠΊΠ°Π»ΡŒΠ½Ρ‹Π΅ силы ΡƒΡ€Π°Π²Π½ΠΎΠ²Π΅ΡˆΠΈΠ²Π°ΡŽΡ‚ Π΄Ρ€ΡƒΠ³ Π΄Ρ€ΡƒΠ³Π° – Π² соотвСтствии с Ρ‚Π΅ΠΌ Ρ„Π°ΠΊΡ‚ΠΎΠΌ, Ρ‡Ρ‚ΠΎ Π½Π΅Ρ‚ Π²Π΅Ρ€Ρ‚ΠΈΠΊΠ°Π»ΡŒΠ½ΠΎΠ³ΠΎ ускорСния.Π“ΠΎΡ€ΠΈΠ·ΠΎΠ½Ρ‚Π°Π»ΡŒΠ½Ρ‹Π΅ силы Π½Π΅ ΡƒΡ€Π°Π²Π½ΠΎΠ²Π΅ΡˆΠΈΠ²Π°ΡŽΡ‚ Π΄Ρ€ΡƒΠ³ Π΄Ρ€ΡƒΠ³Π°. Чистая сила ΠΌΠΎΠΆΠ΅Ρ‚ Π±Ρ‹Ρ‚ΡŒ ΠΎΠΏΡ€Π΅Π΄Π΅Π»Π΅Π½Π° ΠΊΠ°ΠΊ вСкторная сумма F app ΠΈ F frict . Π’ΠΎ Π΅ΡΡ‚ΡŒ F net = 45,0 Н, правая + 29,4 Н, лСвая; ΠΎΠ½ΠΈ Π΄ΠΎΠ±Π°Π²Π»ΡΡŽΡ‚ ΠΊ 15,6 Н, Π²Π΅Ρ€Π½ΠΎ. Π’Π΅ΠΏΠ΅Ρ€ΡŒ ускорСниС ΠΌΠΎΠΆΠ½ΠΎ Ρ€Π°ΡΡΡ‡ΠΈΡ‚Π°Ρ‚ΡŒ, ΠΈΡΠΏΠΎΠ»ΡŒΠ·ΡƒΡ Π²Ρ‚ΠΎΡ€ΠΎΠΉ Π·Π°ΠΊΠΎΠ½ ΠΡŒΡŽΡ‚ΠΎΠ½Π°.

            a = F , Π½Π΅Ρ‚Ρ‚ΠΎ / ΠΌ = (15,6 Н / 15,0 ΠΊΠ³) = 1,04 ΠΌ / с 2

            Π’Π΅ΠΏΠ΅Ρ€ΡŒ, ΠΊΠΎΠ³Π΄Π° систСмный Π°Π½Π°Π»ΠΈΠ· Π±Ρ‹Π» использован для опрСдСлСния ускорСния, ΠΌΠΎΠΆΠ½ΠΎ Π²Ρ‹ΠΏΠΎΠ»Π½ΠΈΡ‚ΡŒ Π°Π½Π°Π»ΠΈΠ· ΠΎΡ‚Π΄Π΅Π»ΡŒΠ½ΠΎΠ³ΠΎ ΠΎΠ±ΡŠΠ΅ΠΊΡ‚Π° для любого ΠΎΠ±ΡŠΠ΅ΠΊΡ‚Π°, Ρ‡Ρ‚ΠΎΠ±Ρ‹ ΠΎΠΏΡ€Π΅Π΄Π΅Π»ΠΈΡ‚ΡŒ силу, Π΄Π΅ΠΉΡΡ‚Π²ΡƒΡŽΡ‰ΡƒΡŽ ΠΌΠ΅ΠΆΠ΄Ρƒ Π½ΠΈΠΌΠΈ.ΠžΠΏΡΡ‚ΡŒ ΠΆΠ΅, Π½Π΅ ΠΈΠΌΠ΅Π΅Ρ‚ значСния, ΠΊΠ°ΠΊΠΎΠΉ ΠΎΠ±ΡŠΠ΅ΠΊΡ‚ Π²Ρ‹Π±Ρ€Π°Π½; Ρ€Π΅Π·ΡƒΠ»ΡŒΡ‚Π°Ρ‚ Π±ΡƒΠ΄Π΅Ρ‚ ΠΎΠ΄ΠΈΠ½Π°ΠΊΠΎΠ²Ρ‹ΠΌ Π² любом случаС. ΠžΠ±ΡŠΠ΅ΠΊΡ‚ массой 10 ΠΊΠ³ Π²Ρ‹Π±Ρ€Π°Π½ для Π°Π½Π°Π»ΠΈΠ·Π° ΠΎΡ‚Π΄Π΅Π»ΡŒΠ½ΠΎΠ³ΠΎ ΠΎΠ±ΡŠΠ΅ΠΊΡ‚Π°, ΠΏΠΎΡΠΊΠΎΠ»ΡŒΠΊΡƒ Π½Π° Π½Π΅Π³ΠΎ дСйствуСт Π½Π° ΠΎΠ΄Π½Ρƒ силу мСньшС; это ΡƒΠΏΡ€ΠΎΡ‰Π°Π΅Ρ‚ Ρ€Π΅ΡˆΠ΅Π½ΠΈΠ΅. На ΠΎΠ±ΡŠΠ΅ΠΊΡ‚ вСсом 10,0 ΠΊΠ³ Π΄Π΅ΠΉΡΡ‚Π²ΡƒΡŽΡ‚ Ρ‡Π΅Ρ‚Ρ‹Ρ€Π΅ силы. Π”Π²Π΅ Π²Π΅Ρ€Ρ‚ΠΈΠΊΠ°Π»ΡŒΠ½Ρ‹Π΅ силы ΠΎΡ‡Π΅Π²ΠΈΠ΄Π½Ρ‹ – сила тяТСсти (98,0 Н) ΠΈ Π½ΠΎΡ€ΠΌΠ°Π»ΡŒΠ½Π°Ρ сила (равная силС тяТСсти). Π“ΠΎΡ€ΠΈΠ·ΠΎΠ½Ρ‚Π°Π»ΡŒΠ½Ρ‹Π΅ силы – это сила трСния слСва ΠΈ сила 5,0-ΠΊΠΈΠ»ΠΎΠ³Ρ€Π°ΠΌΠΌΠΎΠ²ΠΎΠ³ΠΎ ΠΎΠ±ΡŠΠ΅ΠΊΡ‚Π°, Ρ‚ΠΎΠ»ΠΊΠ°ΡŽΡ‰Π΅Π³ΠΎ 10.0-ΠΊΠΈΠ»ΠΎΠ³Ρ€Π°ΠΌΠΌΠΎΠ²Ρ‹ΠΉ ΠΎΠ±ΡŠΠ΅ΠΊΡ‚ Π²ΠΏΠ΅Ρ€Π΅Π΄; Π½Π° схСмС свободного Ρ‚Π΅Π»Π° ΠΎΠ½ ΠΎΠ±ΠΎΠ·Π½Π°Ρ‡Π΅Π½ ΠΊΠ°ΠΊ F , ΠΊΠΎΠ½Ρ‚Π°ΠΊΡ‚ . Π§ΠΈΡΡ‚ΡƒΡŽ силу – Π²Π΅ΠΊΡ‚ΠΎΡ€Π½ΡƒΡŽ сумму всСх сил – всСгда ΠΌΠΎΠΆΠ½ΠΎ Π½Π°ΠΉΡ‚ΠΈ, слоТив силы Π² Π½Π°ΠΏΡ€Π°Π²Π»Π΅Π½ΠΈΠΈ ускорСния ΠΈ вычтя Ρ‚Π΅, ΠΊΠΎΡ‚ΠΎΡ€Ρ‹Π΅ Π΄Π΅ΠΉΡΡ‚Π²ΡƒΡŽΡ‚ Π² ΠΏΡ€ΠΎΡ‚ΠΈΠ²ΠΎΠΏΠΎΠ»ΠΎΠΆΠ½ΠΎΠΌ Π½Π°ΠΏΡ€Π°Π²Π»Π΅Π½ΠΈΠΈ. Π­Ρ‚ΠΎΡ‚ F net Ρ€Π°Π²Π΅Π½ F contact – F frict . ΠŸΡ€ΠΈΠΌΠ΅Π½Π΅Π½ΠΈΠ΅ Π²Ρ‚ΠΎΡ€ΠΎΠ³ΠΎ Π·Π°ΠΊΠΎΠ½Π° ΠΡŒΡŽΡ‚ΠΎΠ½Π° ΠΊ этому ΠΎΠ±ΡŠΠ΅ΠΊΡ‚Ρƒ Π΄Π°Π΅Ρ‚ ΡƒΡ€Π°Π²Π½Π΅Π½ΠΈΠ΅:

            F ΠΊΠΎΠ½Ρ‚Π°ΠΊΡ‚ – F frict = (10,0 ΠΊΠ³) β€’ (1.04 ΠΌ / с 2 )

            Π‘ΠΈΠ»Π° трСния Π½Π° этом ΠΎΠ±ΡŠΠ΅ΠΊΡ‚Π΅ вСсом 10,0 ΠΊΠ³ отличаСтся ΠΎΡ‚ силы трСния Π² систСмС (ΠΏΠΎΡΠΊΠΎΠ»ΡŒΠΊΡƒ систСма вСсила ΠΊΠ³ ΠΈ ). Π—Π½Π°Ρ‡Π΅Π½ΠΈΠ΅ F frict ΠΌΠΎΠΆΠ½ΠΎ Π²Ρ‹Ρ‡ΠΈΡΠ»ΠΈΡ‚ΡŒ ΠΊΠ°ΠΊ ΞΌ β€’ F norm , Π³Π΄Π΅ F norm – Π½ΠΎΡ€ΠΌΠ°Π»ΡŒΠ½Π°Ρ сила, испытываСмая ΠΎΠ±ΡŠΠ΅ΠΊΡ‚ΠΎΠΌ вСсом 10,0 ΠΊΠ³. Норма F для 10,0-ΠΊΠ³ ΠΎΠ±ΡŠΠ΅ΠΊΡ‚Π° Ρ€Π°Π²Π½Π° силС тяТСсти, Π΄Π΅ΠΉΡΡ‚Π²ΡƒΡŽΡ‰Π΅ΠΉ Π½Π° 10-ΠΊΠΈΠ»ΠΎΠ³Ρ€Π°ΠΌΠΌΠΎΠ²Ρ‹ΠΉ ΠΎΠ±ΡŠΠ΅ΠΊΡ‚; это Π·Π½Π°Ρ‡Π΅Π½ΠΈΠ΅ 98.0 Н. Π˜Ρ‚Π°ΠΊ,

            F frict = ΞΌ β€’ F norm = (0.200) β€’ (98,0 Н) = 19,6 Н

            Π˜Ρ‚Π°ΠΊ, Ρ‚Π΅ΠΏΠ΅Ρ€ΡŒ Π·Π½Π°Ρ‡Π΅Π½ΠΈΠ΅ 19,6 Н ΠΌΠΎΠΆΠ΅Ρ‚ Π±Ρ‹Ρ‚ΡŒ подставлСно Π² ΠΏΡ€ΠΈΠ²Π΅Π΄Π΅Π½Π½ΠΎΠ΅ Π²Ρ‹ΡˆΠ΅ ΡƒΡ€Π°Π²Π½Π΅Π½ΠΈΠ΅ ΠΈ F ΠΊΠΎΠ½Ρ‚Π°ΠΊΡ‚ ΠΌΠΎΠΆΠ΅Ρ‚ Π±Ρ‹Ρ‚ΡŒ вычислСн:

            F ΠΊΠΎΠ½Ρ‚Π°ΠΊΡ‚ – 19,6 Н = (10,0 ΠΊΠ³) β€’ (1,04 ΠΌ / с 2 )
            ΠšΠΎΠ½Ρ‚Π°ΠΊΡ‚ F = (10,0 ΠΊΠ³) β€’ (1,04 ΠΌ / с 2 ) + 19,6 Н
            ΠšΠΎΠ½Ρ‚Π°ΠΊΡ‚ F = 30,0 Н

            Π’Π°ΠΊΠΈΠΌ ΠΎΠ±Ρ€Π°Π·ΠΎΠΌ, использованиС Π΄Π²ΠΎΠΉΠ½ΠΎΠΉ ΠΊΠΎΠΌΠ±ΠΈΠ½Π°Ρ†ΠΈΠΈ систСмного Π°Π½Π°Π»ΠΈΠ·Π° ΠΈ ΠΈΠ½Π΄ΠΈΠ²ΠΈΠ΄ΡƒΠ°Π»ΡŒΠ½ΠΎΠ³ΠΎ Π°Π½Π°Π»ΠΈΠ·Π° Ρ‚Π΅Π»Π° позволяСт Π½Π°ΠΌ ΠΎΠΏΡ€Π΅Π΄Π΅Π»ΠΈΡ‚ΡŒ Π΄Π²Π° нСизвСстных значСния – 1.04 ΠΌ / с 2 для ускорСния ΠΈ 30,0 Н для ΠΊΠΎΠ½Ρ‚Π°ΠΊΡ‚Π° F . Π’Π΅ΠΏΠ΅Ρ€ΡŒ ΠΌΡ‹ ΡƒΠ²ΠΈΠ΄ΠΈΠΌ, ΠΊΠ°ΠΊ Π΄Π²Π° Π°Π½Π°Π»ΠΈΠ·Π° ΠΎΡ‚Π΄Π΅Π»ΡŒΠ½Ρ‹Ρ… ΠΎΠ±ΡŠΠ΅ΠΊΡ‚ΠΎΠ² ΠΌΠΎΠ³ΡƒΡ‚ Π±Ρ‹Ρ‚ΡŒ ΠΎΠ±ΡŠΠ΅Π΄ΠΈΠ½Π΅Π½Ρ‹, Ρ‡Ρ‚ΠΎΠ±Ρ‹ ΡΠ³Π΅Π½Π΅Ρ€ΠΈΡ€ΠΎΠ²Π°Ρ‚ΡŒ систСму ΠΈΠ· Π΄Π²ΡƒΡ… ΡƒΡ€Π°Π²Π½Π΅Π½ΠΈΠΉ, способных Ρ€Π°Π·Ρ€Π΅ΡˆΠΈΡ‚ΡŒ Π΄Π²Π΅ нСизвСстныС. ΠœΡ‹ Π΅Ρ‰Π΅ Ρ€Π°Π· Π½Π°Ρ‡Π½Π΅ΠΌ Π°Π½Π°Π»ΠΈΠ·, прСдполагая, Ρ‡Ρ‚ΠΎ ΠΌΡ‹ Ρ€Π΅ΡˆΠ°Π΅ΠΌ ΠΏΡ€ΠΎΠ±Π»Π΅ΠΌΡƒ Π²ΠΏΠ΅Ρ€Π²Ρ‹Π΅ ΠΈ Π½Π΅ Π·Π½Π°Π΅ΠΌ Π½ΠΈ ускорСниС, Π½ΠΈ ΠΊΠΎΠ½Ρ‚Π°ΠΊΡ‚Π½ΡƒΡŽ силу. Π”ΠΈΠ°Π³Ρ€Π°ΠΌΠΌΡ‹ свободного Ρ‚Π΅Π»Π° для ΠΎΡ‚Π΄Π΅Π»ΡŒΠ½Ρ‹Ρ… ΠΎΠ±ΡŠΠ΅ΠΊΡ‚ΠΎΠ² ΠΏΠΎΠΊΠ°Π·Π°Π½Ρ‹ Π½ΠΈΠΆΠ΅.

            Π’Π΅ΠΏΠ΅Ρ€ΡŒ ΠΏΡΡ‚ΡŒ сил Π½Π° 5.ΠžΠ±ΡŠΠ΅ΠΊΡ‚ вСсом 0 ΠΊΠ³ сзади. Π”Π²Π΅ Π²Π΅Ρ€Ρ‚ΠΈΠΊΠ°Π»ΡŒΠ½Ρ‹Π΅ силы – F grav ΠΈ F norm – ΠΎΡ‡Π΅Π²ΠΈΠ΄Π½Ρ‹Π΅ силы. ΠŸΡ€ΠΈΠ»ΠΎΠΆΠ΅Π½Π½Π°Ρ сила 45,0 Н (F , ΠΏΡ€ΠΈΠ»ΠΎΠΆΠ΅Π½ΠΈΠ΅ ) являСтся Ρ€Π΅Π·ΡƒΠ»ΡŒΡ‚Π°Ρ‚ΠΎΠΌ давлСния Ρ€ΡƒΠΊΠΈ Π½Π° Π·Π°Π΄Π½ΠΈΠΉ ΠΎΠ±ΡŠΠ΅ΠΊΡ‚. Π‘ΠΈΠ»Π° Π»Π΅Π²ΠΎΠ³ΠΎ ΠΊΠΎΠ½Ρ‚Π°ΠΊΡ‚Π° с ΠΎΠ±ΡŠΠ΅ΠΊΡ‚ΠΎΠΌ массой 5,0 ΠΊΠ³ – это сила, с ΠΊΠΎΡ‚ΠΎΡ€ΠΎΠΉ ΠΎΠ±ΡŠΠ΅ΠΊΡ‚ массой 10 ΠΊΠ³ Ρ‚ΠΎΠ»ΠΊΠ°Π΅Ρ‚ Π²Π»Π΅Π²ΠΎ ΠΎΠ±ΡŠΠ΅ΠΊΡ‚ массой 5,0 ΠΊΠ³. Π•Π³ΠΎ Π·Π½Π°Ρ‡Π΅Π½ΠΈΠ΅ Ρ‚Π°ΠΊΠΎΠ΅ ΠΆΠ΅, ΠΊΠ°ΠΊ сила ΠΊΠΎΠ½Ρ‚Π°ΠΊΡ‚Π°, прилагаСмая ΠΊ ΠΏΠ΅Ρ€Π΅Π΄Π½Π΅ΠΌΡƒ ΠΎΠ±ΡŠΠ΅ΠΊΡ‚Ρƒ вСсом 10,0 ΠΊΠ³ Π·Π°Π΄Π½ΠΈΠΌ ΠΎΠ±ΡŠΠ΅ΠΊΡ‚ΠΎΠΌ вСсом 5,0 ΠΊΠ³. Π­Ρ‚Π° сила просто ΠΎΠ±ΠΎΠ·Π½Π°Ρ‡Π΅Π½Π° ΠΊΠ°ΠΊ F , ΠΊΠΎΠ½Ρ‚Π°ΠΊΡ‚ для ΠΎΠ±Π΅ΠΈΡ… Π΄ΠΈΠ°Π³Ρ€Π°ΠΌΠΌ свободного Ρ‚Π΅Π»Π°.НаконСц, сила трСния Π²Π»Π΅Π²ΠΎ являСтся Ρ€Π΅Π·ΡƒΠ»ΡŒΡ‚Π°Ρ‚ΠΎΠΌ трСния ΠΎ ΠΏΠΎΠ», ΠΏΠΎ ΠΊΠΎΡ‚ΠΎΡ€ΠΎΠΌΡƒ двиТСтся ΠΎΠ±ΡŠΠ΅ΠΊΡ‚ вСсом 5,0 ΠΊΠ³. На Π΄ΠΈΠ°Π³Ρ€Π°ΠΌΠΌΠ΅ свободного Ρ‚Π΅Π»Π° для ΠΎΠ±ΡŠΠ΅ΠΊΡ‚Π° вСсом 10,0 ΠΊΠ³ Ρ‚Π΅ΠΏΠ΅Ρ€ΡŒ Ρ‡Π΅Ρ‚Ρ‹Ρ€Π΅ силы. Π”Π²Π΅ Π²Π΅Ρ€Ρ‚ΠΈΠΊΠ°Π»ΡŒΠ½Ρ‹Π΅ силы – F grav ΠΈ F norm – ΠΎΡ‡Π΅Π²ΠΈΠ΄Π½Ρ‹. ΠŸΡ€Π°Π²ΠΎΠ΅ ΠΊΠΎΠ½Ρ‚Π°ΠΊΡ‚Π½ΠΎΠ΅ усилиС (F contact ) – это просто ΠΎΠ±ΡŠΠ΅ΠΊΡ‚ вСсом 5,0 ΠΊΠ³, Ρ‚ΠΎΠ»ΠΊΠ°ΡŽΡ‰ΠΈΠΉ Π²ΠΏΠ΅Ρ€Π΅Π΄ ΠΎΠ±ΡŠΠ΅ΠΊΡ‚ вСсом 10,0 ΠΊΠ³. А сила трСния Π²Π»Π΅Π²ΠΎ – это Ρ€Π΅Π·ΡƒΠ»ΡŒΡ‚Π°Ρ‚ трСния ΠΎ ΠΏΠΎΠ». И снова Π½Π° этот 10 Π½Π΅ дСйствуСт прилоТСнная сила 45,0 Н.0-ΠΊΠΈΠ»ΠΎΠ³Ρ€Π°ΠΌΠΌΠΎΠ²Ρ‹ΠΉ ΠΎΠ±ΡŠΠ΅ΠΊΡ‚; ΠΎΠ½ΠΎ примСняСтся ΠΊ ΠΎΠ±ΡŠΠ΅ΠΊΡ‚Ρƒ массой 5,0 ΠΊΠ³ ΠΈ ΡƒΠΆΠ΅ Π±Ρ‹Π»ΠΎ рассмотрСно Π½Π° ΠΏΡ€Π΅Π΄Ρ‹Π΄ΡƒΡ‰Π΅ΠΉ Π΄ΠΈΠ°Π³Ρ€Π°ΠΌΠΌΠ΅ свободного Ρ‚Π΅Π»Π°. Π‘ΠΈΠ»Π° трСния для ΠΊΠ°ΠΆΠ΄ΠΎΠ³ΠΎ ΠΎΠ±ΡŠΠ΅ΠΊΡ‚Π° ΠΌΠΎΠΆΠ΅Ρ‚ Π±Ρ‹Ρ‚ΡŒ ΠΎΠΏΡ€Π΅Π΄Π΅Π»Π΅Π½Π° ΠΊΠ°ΠΊ ΞΌ β€’ Fnorm, Π³Π΄Π΅ F norm – Π½ΠΎΡ€ΠΌΠ°Π»ΡŒΠ½Π°Ρ сила, испытываСмая ΠΎΡ‚Π΄Π΅Π»ΡŒΠ½Ρ‹ΠΌΠΈ ΠΎΠ±ΡŠΠ΅ΠΊΡ‚Π°ΠΌΠΈ. ΠšΠ°ΠΆΠ΄Ρ‹ΠΉ ΠΎΠ±ΡŠΠ΅ΠΊΡ‚ испытываСт Π½ΠΎΡ€ΠΌΠ°Π»ΡŒΠ½ΡƒΡŽ силу, Ρ€Π°Π²Π½ΡƒΡŽ Π΅Π³ΠΎ вСсу (ΠΏΠΎΡΠΊΠΎΠ»ΡŒΠΊΡƒ Π²Π΅Ρ€Ρ‚ΠΈΠΊΠ°Π»ΡŒΠ½Ρ‹Π΅ силы Π΄ΠΎΠ»ΠΆΠ½Ρ‹ ΡƒΡ€Π°Π²Π½ΠΎΠ²Π΅ΡˆΠΈΠ²Π°Ρ‚ΡŒΡΡ). Π’Π°ΠΊΠΈΠΌ ΠΎΠ±Ρ€Π°Π·ΠΎΠΌ, силы трСния для ΠΎΠ±ΡŠΠ΅ΠΊΡ‚Π° вСсом 5,0 ΠΊΠ³ (вСс 49,0 Н) ΠΈ ΠΎΠ±ΡŠΠ΅ΠΊΡ‚Π° вСсом 10,0 ΠΊΠ³ (вСс 98,0 Н) ΡΠΎΡΡ‚Π°Π²Π»ΡΡŽΡ‚ 0,200 β€’ 49,0 Н ΠΈ 0,200 β€’ 98,0 Н соотвСтствСнно.

            Π˜ΡΠΏΠΎΠ»ΡŒΠ·ΡƒΡ эти значСния F frict ΠΈ Π²Ρ‚ΠΎΡ€ΠΎΠΉ Π·Π°ΠΊΠΎΠ½ ΠΡŒΡŽΡ‚ΠΎΠ½Π°, ΠΌΠΎΠΆΠ½ΠΎ Π·Π°ΠΏΠΈΡΠ°Ρ‚ΡŒ систСму ΠΈΠ· Π΄Π²ΡƒΡ… ΡƒΡ€Π°Π²Π½Π΅Π½ΠΈΠΉ, ΠΏΠΎΠ·Π²ΠΎΠ»ΡΡŽΡ‰ΡƒΡŽ Π½Π°ΠΉΡ‚ΠΈ Π΄Π²Π° нСизвСстных значСния. ИспользованиС F net = m β€’ a с Π΄ΠΈΠ°Π³Ρ€Π°ΠΌΠΌΠΎΠΉ свободного Ρ‚Π΅Π»Π° для ΠΎΠ±ΡŠΠ΅ΠΊΡ‚Π° массой 5,0 ΠΊΠ³ даст ΡƒΡ€Π°Π²Π½Π΅Π½ΠΈΠ΅ 3 Π½ΠΈΠΆΠ΅:

            45,0 – F ΠΊΠΎΠ½Ρ‚Π°ΠΊΡ‚ – 9,8 = 5,0 β€’ a

            ИспользованиС F net = m β€’ a с Π΄ΠΈΠ°Π³Ρ€Π°ΠΌΠΌΠΎΠΉ свободного Ρ‚Π΅Π»Π° для ΠΎΠ±ΡŠΠ΅ΠΊΡ‚Π° массой 10,0 ΠΊΠ³ даст ΡƒΡ€Π°Π²Π½Π΅Π½ΠΈΠ΅ 4 Π½ΠΈΠΆΠ΅:

            F ΠΊΠΎΠ½Ρ‚Π°ΠΊΡ‚ – 19.6 = 10,0 β€’

            (ΠžΠ±Ρ€Π°Ρ‚ΠΈΡ‚Π΅ Π²Π½ΠΈΠΌΠ°Π½ΠΈΠ΅, Ρ‡Ρ‚ΠΎ Π΅Π΄ΠΈΠ½ΠΈΡ†Ρ‹ Π±Ρ‹Π»ΠΈ ΠΈΡΠΊΠ»ΡŽΡ‡Π΅Π½Ρ‹ ΠΈΠ· Π£Ρ€Π°Π²Π½Π΅Π½ΠΈΠΉ 3 ΠΈ 4, Ρ‡Ρ‚ΠΎΠ±Ρ‹ ΠΎΡ‡ΠΈΡΡ‚ΠΈΡ‚ΡŒ уравнСния Π΄ΠΎ .) Из УравнСния 4, ΠΊΠΎΠ½Ρ‚Π°ΠΊΡ‚ F = 10,0 β€’ a + 19,6. ΠŸΠΎΠ΄ΡΡ‚Π°Π²Π»ΡΡ это Π²Ρ‹Ρ€Π°ΠΆΠ΅Π½ΠΈΠ΅ для ΠΊΠΎΠ½Ρ‚Π°ΠΊΡ‚Π° F Π² ΡƒΡ€Π°Π²Π½Π΅Π½ΠΈΠ΅ 3 ΠΈ выполняя ΠΏΡ€Π°Π²ΠΈΠ»ΡŒΠ½Ρ‹Π΅ алгСбраичСскиС манипуляции, ΠΏΠΎΠ»ΡƒΡ‡Π°Π΅ΠΌ Π·Π½Π°Ρ‡Π΅Π½ΠΈΠ΅ ускорСния:

            45,0 – (10,0 β€’ a + 19,6) – 9,8 = 5,0 β€’ a
            45,0 – 19,6 – 9,8 = 15,0 β€’
            15,6 = 15,0 β€’
            а = (15,6 / 15.0) = 1,04 м / с 2

            Π­Ρ‚ΠΎ Π·Π½Π°Ρ‡Π΅Π½ΠΈΠ΅ ускорСния ΠΌΠΎΠΆΠ½ΠΎ ΠΏΠΎΠ΄ΡΡ‚Π°Π²ΠΈΡ‚ΡŒ ΠΎΠ±Ρ€Π°Ρ‚Π½ΠΎ Π² Π²Ρ‹Ρ€Π°ΠΆΠ΅Π½ΠΈΠ΅ для F contact , Ρ‡Ρ‚ΠΎΠ±Ρ‹ ΠΎΠΏΡ€Π΅Π΄Π΅Π»ΠΈΡ‚ΡŒ ΠΊΠΎΠ½Ρ‚Π°ΠΊΡ‚Π½ΠΎΠ΅ усилиС:

            ΠšΠΎΠ½Ρ‚Π°ΠΊΡ‚ F = 10,0 β€’ a + 19,6 = 10,0 β€’ (1,04) + 19,6
            ΠšΠΎΠ½Ρ‚Π°ΠΊΡ‚ F = 30,0 Н

            ΠžΠΏΡΡ‚ΡŒ ΠΆΠ΅, ΠΌΡ‹ ΠΎΠ±Π½Π°Ρ€ΡƒΠΆΠΈΠ²Π°Π΅ΠΌ, Ρ‡Ρ‚ΠΎ Π²Ρ‚ΠΎΡ€ΠΎΠΉ ΠΏΠΎΠ΄Ρ…ΠΎΠ΄ с использованиСм Π°Π½Π°Π»ΠΈΠ·Π° Π΄Π²ΡƒΡ… ΠΎΡ‚Π΄Π΅Π»ΡŒΠ½Ρ‹Ρ… ΠΎΠ±ΡŠΠ΅ΠΊΡ‚ΠΎΠ² Π΄Π°Π΅Ρ‚ Ρ‚ΠΎΡ‚ ΠΆΠ΅ Π½Π°Π±ΠΎΡ€ ΠΎΡ‚Π²Π΅Ρ‚ΠΎΠ² для Π΄Π²ΡƒΡ… нСизвСстных. ПослСдний ΠΏΡ€ΠΈΠΌΠ΅Ρ€ Π·Π°Π΄Π°Ρ‡ΠΈ Π±ΡƒΠ΄Π΅Ρ‚ Π²ΠΊΠ»ΡŽΡ‡Π°Ρ‚ΡŒ Π²Π΅Ρ€Ρ‚ΠΈΠΊΠ°Π»ΡŒΠ½ΠΎΠ΅ Π΄Π²ΠΈΠΆΠ΅Π½ΠΈΠ΅.ΠŸΠΎΠ΄Ρ…ΠΎΠ΄Ρ‹ останутся ΠΏΡ€Π΅ΠΆΠ½ΠΈΠΌΠΈ.

            ΠŸΡ€ΠΈΠΌΠ΅Ρ€ Π·Π°Π΄Π°Ρ‡ΠΈ 3:

            ΠœΡƒΠΆΡ‡ΠΈΠ½Π° Π²Ρ…ΠΎΠ΄ΠΈΡ‚ Π² Π»ΠΈΡ„Ρ‚ с двумя ΠΊΠΎΡ€ΠΎΠ±ΠΊΠ°ΠΌΠΈ – ΠΎΠ΄Π½Π° Π½Π° Π΄Ρ€ΡƒΠ³ΠΎΠΉ. Π’Π΅Ρ€Ρ…Π½ΠΈΠΉ ящик ΠΈΠΌΠ΅Π΅Ρ‚ массу 6,0 ΠΊΠ³, Π° Π½ΠΈΠΆΠ½ΠΈΠΉ – 8,0 ΠΊΠ³. ΠœΡƒΠΆΡ‡ΠΈΠ½Π° ставит Π΄Π²Π΅ ΠΊΠΎΡ€ΠΎΠ±ΠΊΠΈ ΠΏΠΎ мСтричСской шкалС Π½Π° ΠΏΠΎΠ». ΠŸΡ€ΠΈ ускорСнии Π²Π²Π΅Ρ€Ρ… ΠΈΠ· состояния покоя ΠΌΡƒΠΆΡ‡ΠΈΠ½Π° Π·Π°ΠΌΠ΅Ρ‡Π°Π΅Ρ‚, Ρ‡Ρ‚ΠΎ шкала ΠΏΠΎΠΊΠ°Π·Ρ‹Π²Π°Π΅Ρ‚ Π·Π½Π°Ρ‡Π΅Π½ΠΈΠ΅ 166 Н; это сила, направлСнная Π²Π²Π΅Ρ€Ρ… Π½Π° ниТнюю ΠΊΠΎΡ€ΠΎΠ±ΠΊΡƒ.ΠžΠΏΡ€Π΅Π΄Π΅Π»ΠΈΡ‚Π΅ ускорСниС Π»ΠΈΡ„Ρ‚Π° (ΠΈ ящиков) ΠΈ ΠΎΠΏΡ€Π΅Π΄Π΅Π»ΠΈΡ‚Π΅ силы, Π΄Π΅ΠΉΡΡ‚Π²ΡƒΡŽΡ‰ΠΈΠ΅ ΠΌΠ΅ΠΆΠ΄Ρƒ ящиками.

            Для Ρ€Π΅ΡˆΠ΅Π½ΠΈΡ этой ΠΏΡ€ΠΎΠ±Π»Π΅ΠΌΡ‹ Π±ΡƒΠ΄ΡƒΡ‚ ΠΈΡΠΏΠΎΠ»ΡŒΠ·ΠΎΠ²Π°Ρ‚ΡŒΡΡ ΠΎΠ±Π° ΠΏΠΎΠ΄Ρ…ΠΎΠ΄Π°. ΠŸΠ΅Ρ€Π²Ρ‹ΠΉ ΠΏΠΎΠ΄Ρ…ΠΎΠ΄ ΠΏΡ€Π΅Π΄ΠΏΠΎΠ»Π°Π³Π°Π΅Ρ‚ Π΄Π²ΠΎΠΉΠ½ΠΎΠ΅ сочСтаниС систСмного Π°Π½Π°Π»ΠΈΠ·Π° ΠΈ Π°Π½Π°Π»ΠΈΠ·Π° ΠΎΡ‚Π΄Π΅Π»ΡŒΠ½ΠΎΠ³ΠΎ ΠΎΠ±ΡŠΠ΅ΠΊΡ‚Π°. Для систСмного Π°Π½Π°Π»ΠΈΠ·Π° Π΄Π²Π΅ ΠΊΠΎΡ€ΠΎΠ±ΠΊΠΈ ΡΡ‡ΠΈΡ‚Π°ΡŽΡ‚ΡΡ Π΅Π΄ΠΈΠ½ΠΎΠΉ систСмой массой 14,0 ΠΊΠ³. На эту систСму Π΄Π΅ΠΉΡΡ‚Π²ΡƒΡŽΡ‚ Π΄Π²Π΅ силы – сила тяТСсти ΠΈ Π½ΠΎΡ€ΠΌΠ°Π»ΡŒΠ½Π°Ρ сила. Π‘Ρ…Π΅ΠΌΠ° свободного Ρ‚Π΅Π»Π° ΠΏΠΎΠΊΠ°Π·Π°Π½Π° справа.Π‘ΠΈΠ»Π° тяТСсти рассчитываСтся ΠΎΠ±Ρ‹Ρ‡Π½Ρ‹ΠΌ ΠΎΠ±Ρ€Π°Π·ΠΎΠΌ с использованиСм массы 14,0 ΠΊΠ³.

            F grav = m β€’ g = 14,0 ΠΊΠ³ β€’ 9,8 Н / ΠΊΠ³ = 137,2 Н

            ΠŸΠΎΡΠΊΠΎΠ»ΡŒΠΊΡƒ сущСствуСт Π²Π΅Ρ€Ρ‚ΠΈΠΊΠ°Π»ΡŒΠ½ΠΎΠ΅ ускорСниС, Π²Π΅Ρ€Ρ‚ΠΈΠΊΠ°Π»ΡŒΠ½Ρ‹Π΅ силы Π½Π΅ ΡƒΡ€Π°Π²Π½ΠΎΠ²Π΅ΡˆΠΈΠ²Π°ΡŽΡ‚ΡΡ; F grav Π½Π΅ соотвСтствуСт Π·Π½Π°Ρ‡Π΅Π½ΠΈΡŽ F norm . ΠΠΎΡ€ΠΌΠ°Π»ΡŒΠ½Π°Ρ сила ΡƒΠΊΠ°Π·Π°Π½Π° Π² Ρ„ΠΎΡ€ΠΌΡƒΠ»ΠΈΡ€ΠΎΠ²ΠΊΠ΅ Π·Π°Π΄Π°Ρ‡ΠΈ. Π­Ρ‚Π° Π½ΠΎΡ€ΠΌΠ°Π»ΡŒΠ½Π°Ρ сила 166 Н прСдставляСт собой Π½Π°ΠΏΡ€Π°Π²Π»Π΅Π½Π½ΡƒΡŽ Π²Π²Π΅Ρ€Ρ… силу, Π΄Π΅ΠΉΡΡ‚Π²ΡƒΡŽΡ‰ΡƒΡŽ Π½Π° ниТнюю ΠΊΠΎΡ€ΠΎΠ±ΠΊΡƒ; ΠΎΠ½ слуТит силой для систСмы, ΠΏΠΎΡΠΊΠΎΠ»ΡŒΠΊΡƒ Π½ΠΈΠΆΠ½ΠΈΠΉ ящик являСтся Ρ‡Π°ΡΡ‚ΡŒΡŽ систСмы.Чистая сила – это вСкторная сумма этих Π΄Π²ΡƒΡ… сил. Π˜Ρ‚Π°ΠΊ,

            F net = 166 N, Π²Π²Π΅Ρ€Ρ… + 137,2 N, Π²Π½ΠΈΠ· = 28,8 N, Π²Π²Π΅Ρ€Ρ…

            УскорСниС ΠΌΠΎΠΆΠ½ΠΎ Ρ€Π°ΡΡΡ‡ΠΈΡ‚Π°Ρ‚ΡŒ, ΠΈΡΠΏΠΎΠ»ΡŒΠ·ΡƒΡ Π²Ρ‚ΠΎΡ€ΠΎΠΉ Π·Π°ΠΊΠΎΠ½ ΠΡŒΡŽΡ‚ΠΎΠ½Π°:

            a = F Π½Π΅Ρ‚Ρ‚ΠΎ / ΠΌ = 28,8 Н / 14,0 ΠΊΠ³ = 2,0571 ΠΌ / с 2 = ~ 2,1 ΠΌ / с 2

            Π’Π΅ΠΏΠ΅Ρ€ΡŒ, ΠΊΠΎΠ³Π΄Π° систСмный Π°Π½Π°Π»ΠΈΠ· Π±Ρ‹Π» использован для опрСдСлСния ускорСния, ΠΌΠΎΠΆΠ½ΠΎ Π²Ρ‹ΠΏΠΎΠ»Π½ΠΈΡ‚ΡŒ Π°Π½Π°Π»ΠΈΠ· ΠΎΡ‚Π΄Π΅Π»ΡŒΠ½ΠΎΠ³ΠΎ ΠΎΠ±ΡŠΠ΅ΠΊΡ‚Π° для любого Π±Π»ΠΎΠΊΠ°, Ρ‡Ρ‚ΠΎΠ±Ρ‹ ΠΎΠΏΡ€Π΅Π΄Π΅Π»ΠΈΡ‚ΡŒ силу, Π΄Π΅ΠΉΡΡ‚Π²ΡƒΡŽΡ‰ΡƒΡŽ ΠΌΠ΅ΠΆΠ΄Ρƒ Π½ΠΈΠΌΠΈ.Как ΠΈ Π² ΠΏΡ€Π΅Π΄Ρ‹Π΄ΡƒΡ‰ΠΈΡ… Π·Π°Π΄Π°Ρ‡Π°Ρ…, Π½Π΅ ΠΈΠΌΠ΅Π΅Ρ‚ значСния, ΠΊΠ°ΠΊΠΎΠΉ ящик Π²Ρ‹Π±Ρ€Π°Π½; Ρ€Π΅Π·ΡƒΠ»ΡŒΡ‚Π°Ρ‚ Π±ΡƒΠ΄Π΅Ρ‚ ΠΎΠ΄ΠΈΠ½Π°ΠΊΠΎΠ²Ρ‹ΠΌ Π² любом случаС. Π’Π΅Ρ€Ρ…Π½ΠΈΠΉ ящик ΠΈΡΠΏΠΎΠ»ΡŒΠ·ΡƒΠ΅Ρ‚ΡΡ Π² этом Π°Π½Π°Π»ΠΈΠ·Π΅, ΠΏΠΎΡΠΊΠΎΠ»ΡŒΠΊΡƒ ΠΎΠ½ встрСчаСт Π½Π° ΠΎΠ΄Π½Ρƒ силу мСньшС. Π‘Ρ…Π΅ΠΌΠ° свободного Ρ‚Π΅Π»Π° ΠΏΠΎΠΊΠ°Π·Π°Π½Π° справа. Π‘ΠΈΠ»Π° тяТСсти Π½Π° Π²Π΅Ρ€Ρ…Π½Π΅ΠΌ ящикС Ρ€Π°Π²Π½Π° m β€’ g, Π³Π΄Π΅ m = 6,0 ΠΊΠ³. Π‘ΠΈΠ»Π° тяТСсти составляСт 58,8 Н. Π‘ΠΈΠ»Π°, направлСнная Π²Π²Π΅Ρ€Ρ…, нСизвСстна, Π½ΠΎ ΠΌΠΎΠΆΠ΅Ρ‚ Π±Ρ‹Ρ‚ΡŒ рассчитана, Ссли сСтка F = m β€’ ΡƒΡ€Π°Π²Π½Π΅Π½ΠΈΠ΅ примСняСтся ΠΊ Π΄ΠΈΠ°Π³Ρ€Π°ΠΌΠΌΠ΅ свободного Ρ‚Π΅Π»Π°. ΠŸΠΎΡΠΊΠΎΠ»ΡŒΠΊΡƒ ускорСниС Π½Π°ΠΏΡ€Π°Π²Π»Π΅Π½ΠΎ Π²Π²Π΅Ρ€Ρ…, сторона Fnet уравнСния Π±ΡƒΠ΄Π΅Ρ‚ Ρ€Π°Π²Π½Π° силС Π² Π½Π°ΠΏΡ€Π°Π²Π»Π΅Π½ΠΈΠΈ ускорСния (F ΠΊΠΎΠ½Ρ‚Π°ΠΊΡ‚ ) минус сила, которая Π΅ΠΌΡƒ противодСйствуСт (F grav ).Π˜Ρ‚Π°ΠΊ,

            F ΠΊΠΎΠ½Ρ‚Π°ΠΊΡ‚ – 58,8 Н = (6,0 ΠΊΠ³) β€’ (2,0571 ΠΌ / с2)

            (ΠžΠ±Ρ€Π°Ρ‚ΠΈΡ‚Π΅ Π²Π½ΠΈΠΌΠ°Π½ΠΈΠ΅, Ρ‡Ρ‚ΠΎ здСсь ΠΈΡΠΏΠΎΠ»ΡŒΠ·ΡƒΠ΅Ρ‚ΡΡ Π½Π΅ΠΎΠΊΡ€ΡƒΠ³Π»Π΅Π½Π½ΠΎΠ΅ Π·Π½Π°Ρ‡Π΅Π½ΠΈΠ΅ ускорСния; ΠΎΠΊΡ€ΡƒΠ³Π»Π΅Π½ΠΈΠ΅ ΠΏΡ€ΠΎΠΈΠ·ΠΎΠΉΠ΄Π΅Ρ‚, ΠΊΠΎΠ³Π΄Π° Π±ΡƒΠ΄Π΅Ρ‚ ΠΎΠΏΡ€Π΅Π΄Π΅Π»Π΅Π½ ΠΎΠΊΠΎΠ½Ρ‡Π°Ρ‚Π΅Π»ΡŒΠ½Ρ‹ΠΉ ΠΎΡ‚Π²Π΅Ρ‚.) РСшСниС для ΠΊΠΎΠ½Ρ‚Π°ΠΊΡ‚Π° F Π΄Π°Π΅Ρ‚ 71,14 Н. Π­Ρ‚ΠΎ число ΠΌΠΎΠΆΠ΅Ρ‚ Π±Ρ‹Ρ‚ΡŒ ΠΎΠΊΡ€ΡƒΠ³Π»Π΅Π½ΠΎ Π΄ΠΎ Π΄Π²ΡƒΡ… Π·Π½Π°Ρ‡Π°Ρ‰ΠΈΡ… Ρ†ΠΈΡ„Ρ€ – 71 Н. Π’Π°ΠΊΠΈΠΌ ΠΎΠ±Ρ€Π°Π·ΠΎΠΌ, Π΄Π²ΠΎΠΉΠ½ΠΎΠ΅ ΠšΠΎΠΌΠ±ΠΈΠ½Π°Ρ†ΠΈΡ систСмного Π°Π½Π°Π»ΠΈΠ·Π° ΠΈ ΠΈΠ½Π΄ΠΈΠ²ΠΈΠ΄ΡƒΠ°Π»ΡŒΠ½ΠΎΠ³ΠΎ Π°Π½Π°Π»ΠΈΠ·Π° Ρ‚Π΅Π»Π° ΠΏΡ€ΠΈΠ²ΠΎΠ΄ΠΈΡ‚ ΠΊ ΡƒΡΠΊΠΎΡ€Π΅Π½ΠΈΡŽ 2,1 ΠΌ / с 2 ΠΈ силС ΠΊΠΎΠ½Ρ‚Π°ΠΊΡ‚Π° 71 Н.

            Π’Π΅ΠΏΠ΅Ρ€ΡŒ Π²Ρ‚ΠΎΡ€ΠΎΠΉ ΠΏΠΎΠ΄Ρ…ΠΎΠ΄ ΠΊ Ρ€Π΅ΡˆΠ΅Π½ΠΈΡŽ ΠΏΡ€ΠΎΠ±Π»Π΅ΠΌΡ‹ Π±ΡƒΠ΄Π΅Ρ‚ использован для Ρ€Π΅ΡˆΠ΅Π½ΠΈΡ Ρ‚ΠΎΠΉ ΠΆΠ΅ ΠΏΡ€ΠΎΠ±Π»Π΅ΠΌΡ‹. Π’ этом Ρ€Π΅ΡˆΠ΅Π½ΠΈΠΈ Π΄Π²Π° Π°Π½Π°Π»ΠΈΠ·Π° ΠΎΡ‚Π΄Π΅Π»ΡŒΠ½Ρ‹Ρ… ΠΎΠ±ΡŠΠ΅ΠΊΡ‚ΠΎΠ² Π±ΡƒΠ΄ΡƒΡ‚ ΠΎΠ±ΡŠΠ΅Π΄ΠΈΠ½Π΅Π½Ρ‹ для создания систСмы Π΄Π²ΡƒΡ… ΡƒΡ€Π°Π²Π½Π΅Π½ΠΈΠΉ, способных Ρ€Π΅ΡˆΠΈΡ‚ΡŒ для Π΄Π²ΡƒΡ… нСизвСстных. ΠœΡ‹ Π½Π°Ρ‡Π½Π΅ΠΌ этот Π°Π½Π°Π»ΠΈΠ· с прСдполоТСния, Ρ‡Ρ‚ΠΎ ΠΌΡ‹ Ρ€Π΅ΡˆΠ°Π΅ΠΌ ΠΏΡ€ΠΎΠ±Π»Π΅ΠΌΡƒ Π²ΠΏΠ΅Ρ€Π²Ρ‹Π΅ ΠΈ Π½Π΅ Π·Π½Π°Π΅ΠΌ Π½ΠΈ ускорСниС, Π½ΠΈ ΠΊΠΎΠ½Ρ‚Π°ΠΊΡ‚Π½ΡƒΡŽ силу. Π”ΠΈΠ°Π³Ρ€Π°ΠΌΠΌΡ‹ свободного Ρ‚Π΅Π»Π° для ΠΎΡ‚Π΄Π΅Π»ΡŒΠ½Ρ‹Ρ… ΠΎΠ±ΡŠΠ΅ΠΊΡ‚ΠΎΠ² ΠΏΠΎΠΊΠ°Π·Π°Π½Ρ‹ Π½ΠΈΠΆΠ΅.

            ΠžΠ±Ρ€Π°Ρ‚ΠΈΡ‚Π΅ Π²Π½ΠΈΠΌΠ°Π½ΠΈΠ΅, Ρ‡Ρ‚ΠΎ значСния F grav для Π΄Π²ΡƒΡ… ящиков Π±Ρ‹Π»ΠΈ Π²ΠΊΠ»ΡŽΡ‡Π΅Π½Ρ‹ Π² Π΄ΠΈΠ°Π³Ρ€Π°ΠΌΠΌΡƒ.Они Π±Ρ‹Π»ΠΈ рассчитаны с использованиСм F grav = m β€’ g, Π³Π΄Π΅ m = 6,0 ΠΊΠ³ для Π²Π΅Ρ€Ρ…Π½Π΅Π³ΠΎ ящика ΠΈ m = 8,0 ΠΊΠ³ для Π½ΠΈΠΆΠ½Π΅Π³ΠΎ ящика. ΠšΠΎΠ½Ρ‚Π°ΠΊΡ‚Π½Π°Ρ сила (F , ΠΊΠΎΠ½Ρ‚Π°ΠΊΡ‚ ) Π½Π° Π²Π΅Ρ€Ρ…Π½Π΅ΠΌ Π±Π»ΠΎΠΊΠ΅ Π½Π°ΠΏΡ€Π°Π²Π»Π΅Π½Π° ​​ввСрх, ΠΏΠΎΡΠΊΠΎΠ»ΡŒΠΊΡƒ Π½ΠΈΠΆΠ½ΠΈΠΉ Π±Π»ΠΎΠΊ Ρ‚ΠΎΠ»ΠΊΠ°Π΅Ρ‚ Π΅Π³ΠΎ Π²Π²Π΅Ρ€Ρ…, ΠΊΠΎΠ³Π΄Π° систСма ΠΈΠ· Π΄Π²ΡƒΡ… ΠΎΠ±ΡŠΠ΅ΠΊΡ‚ΠΎΠ² ускоряСтся Π²Π²Π΅Ρ€Ρ…. ΠšΠΎΠ½Ρ‚Π°ΠΊΡ‚Π½ΠΎΠ΅ усилиС (F ΠΊΠΎΠ½Ρ‚Π°ΠΊΡ‚ ) Π½Π° Π½ΠΈΠΆΠ½Π΅ΠΌ Π±Π»ΠΎΠΊΠ΅ Π½Π°ΠΏΡ€Π°Π²Π»Π΅Π½ΠΎ Π²Π½ΠΈΠ·, ΠΏΠΎΡΠΊΠΎΠ»ΡŒΠΊΡƒ Π²Π΅Ρ€Ρ…Π½ΠΈΠΉ Π±Π»ΠΎΠΊ Ρ‚ΠΎΠ»ΠΊΠ°Π΅Ρ‚ Π²Π½ΠΈΠ· Π½ΠΈΠΆΠ½ΠΈΠΉ Π±Π»ΠΎΠΊ, ΠΊΠΎΠ³Π΄Π° происходит ускорСниС. Π­Ρ‚ΠΈ Π΄Π²Π΅ ΠΊΠΎΠ½Ρ‚Π°ΠΊΡ‚Π½Ρ‹Π΅ силы Ρ€Π°Π²Π½Ρ‹ Π΄Ρ€ΡƒΠ³ Π΄Ρ€ΡƒΠ³Ρƒ, ΠΏΠΎΡΠΊΠΎΠ»ΡŒΠΊΡƒ ΠΎΠ½ΠΈ Π²ΠΎΠ·Π½ΠΈΠΊΠ°ΡŽΡ‚ Π² Ρ€Π΅Π·ΡƒΠ»ΡŒΡ‚Π°Ρ‚Π΅ взаимодСйствия ΠΌΠ΅ΠΆΠ΄Ρƒ двумя ΠΊΠΎΡ€ΠΎΠ±ΠΊΠ°ΠΌΠΈ.Π’Ρ€Π΅Ρ‚ΡŒΡ сила Π½Π° Π½ΠΈΠΆΠ½Π΅ΠΌ ящикС – это сила, с ΠΊΠΎΡ‚ΠΎΡ€ΠΎΠΉ вСсы Ρ‚ΠΎΠ»ΠΊΠ°ΡŽΡ‚ Π΅Π³ΠΎ Π²Π²Π΅Ρ€Ρ… с силой 166 Н; это Π·Π½Π°Ρ‡Π΅Π½ΠΈΠ΅ Π±Ρ‹Π»ΠΎ ΡƒΠΊΠ°Π·Π°Π½ΠΎ Π² постановкС Π·Π°Π΄Π°Ρ‡ΠΈ.

            ΠŸΡ€ΠΈΠΌΠ΅Π½Π΅Π½ΠΈΠ΅ Π²Ρ‚ΠΎΡ€ΠΎΠ³ΠΎ Π·Π°ΠΊΠΎΠ½Π° ΠΡŒΡŽΡ‚ΠΎΠ½Π° ΠΊ этим Π΄Π²ΡƒΠΌ Π΄ΠΈΠ°Π³Ρ€Π°ΠΌΠΌΠ°ΠΌ свободного Ρ‚Π΅Π»Π° ΠΏΡ€ΠΈΠ²ΠΎΠ΄ΠΈΡ‚ ΠΊ ΡƒΡ€Π°Π²Π½Π΅Π½ΠΈΡŽ 5 (для ΠΊΠΎΠ½Ρ‚Π΅ΠΉΠ½Π΅Ρ€Π° вСсом 6,0 ΠΊΠ³) ΠΈ ΡƒΡ€Π°Π²Π½Π΅Π½ΠΈΡŽ 6 (для ΠΊΠΎΠ½Ρ‚Π΅ΠΉΠ½Π΅Ρ€Π° вСсом 8,0 ΠΊΠ³).

            F ΠΊΠΎΠ½Ρ‚Π°ΠΊΡ‚ – 58,8 = 6,0 β€’ a

            166 – F ΠΊΠΎΠ½Ρ‚Π°ΠΊΡ‚ – 78,4 = 8,0 β€’ a

            Π’Π΅ΠΏΠ΅Ρ€ΡŒ, ΠΊΠΎΠ³Π΄Π° Ρ€Π°Π·Ρ€Π°Π±ΠΎΡ‚Π°Π½Π° систСма Π΄Π²ΡƒΡ… ΡƒΡ€Π°Π²Π½Π΅Π½ΠΈΠΉ, Π°Π»Π³Π΅Π±Ρ€Ρƒ ΠΌΠΎΠΆΠ½ΠΎ ΠΈΡΠΏΠΎΠ»ΡŒΠ·ΠΎΠ²Π°Ρ‚ΡŒ для Ρ€Π΅ΡˆΠ΅Π½ΠΈΡ Π΄Π²ΡƒΡ… нСизвСстных.Π£Ρ€Π°Π²Π½Π΅Π½ΠΈΠ΅ 5 ΠΌΠΎΠΆΠ½ΠΎ ΠΈΡΠΏΠΎΠ»ΡŒΠ·ΠΎΠ²Π°Ρ‚ΡŒ для записи выраТСния для ΠΊΠΎΠ½Ρ‚Π°ΠΊΡ‚Π½ΠΎΠΉ силы (F , ΠΊΠΎΠ½Ρ‚Π°ΠΊΡ‚ ) Ρ‡Π΅Ρ€Π΅Π· ускорСниС (a).

            F ΠΊΠΎΠ½Ρ‚Π°ΠΊΡ‚ = 6,0 β€’ a + 58,8

            Π­Ρ‚ΠΎ Π²Ρ‹Ρ€Π°ΠΆΠ΅Π½ΠΈΠ΅ для ΠΊΠΎΠ½Ρ‚Π°ΠΊΡ‚Π° F ΠΌΠΎΠΆΠ½ΠΎ Π·Π°Ρ‚Π΅ΠΌ ΠΏΠΎΠ΄ΡΡ‚Π°Π²ΠΈΡ‚ΡŒ Π² ΡƒΡ€Π°Π²Π½Π΅Π½ΠΈΠ΅ 6. Π£Ρ€Π°Π²Π½Π΅Π½ΠΈΠ΅ 6 Ρ‚ΠΎΠ³Π΄Π° прСобразуСтся Π²

            .

            166 – (6,0 β€’ a + 58,8) – 78,4 = 8,0 β€’ a

            Π‘Π»Π΅Π΄ΡƒΡŽΡ‰ΠΈΠ΅ алгСбраичСскиС шаги Π²Ρ‹ΠΏΠΎΠ»Π½ΡΡŽΡ‚ΡΡ Π½Π°Π΄ ΠΏΡ€ΠΈΠ²Π΅Π΄Π΅Π½Π½Ρ‹ΠΌ Π²Ρ‹ΡˆΠ΅ ΡƒΡ€Π°Π²Π½Π΅Π½ΠΈΠ΅ΠΌ, Ρ‡Ρ‚ΠΎΠ±Ρ‹ Π½Π°ΠΉΡ‚ΠΈ ускорСниС.

            166 – 6,0 β€’ a – 58,8 – 78,4 = 8,0 β€’ a
            166 – 58,8 – 78,4 = 8,0 β€’ Π° + 6,0 β€’ Π°
            28,8 = 14,0 Π°
            a = 2,0571 м / с 2 = ~ 2,1 м / с 2

            Π’Π΅ΠΏΠ΅Ρ€ΡŒ Π·Π½Π°Ρ‡Π΅Π½ΠΈΠ΅ ускорСния (a) ΠΌΠΎΠΆΠ½ΠΎ ΠΏΠΎΠ΄ΡΡ‚Π°Π²ΠΈΡ‚ΡŒ ΠΎΠ±Ρ€Π°Ρ‚Π½ΠΎ Π² Π²Ρ‹Ρ€Π°ΠΆΠ΅Π½ΠΈΠ΅ для F ΠΊΠΎΠ½Ρ‚Π°ΠΊΡ‚ (F ΠΊΠΎΠ½Ρ‚Π°ΠΊΡ‚ = 6.0 β€’ a + 58.8), Ρ‡Ρ‚ΠΎΠ±Ρ‹ Π½Π°ΠΉΡ‚ΠΈ F ΠΊΠΎΠ½Ρ‚Π°ΠΊΡ‚ . ΠšΠΎΠ½Ρ‚Π°ΠΊΡ‚Π½ΠΎΠ΅ усилиС составляСт 71,14 Н (~ 71 Н).

            Π‘Π»Π΅Π΄ΡƒΠ΅Ρ‚ ΠΎΡ‚ΠΌΠ΅Ρ‚ΠΈΡ‚ΡŒ, Ρ‡Ρ‚ΠΎ Π²Ρ‚ΠΎΡ€ΠΎΠΉ ΠΏΠΎΠ΄Ρ…ΠΎΠ΄ ΠΊ этой ΠΏΡ€ΠΎΠ±Π»Π΅ΠΌΠ΅ Π΄Π°Π΅Ρ‚ Ρ‚Π΅ ΠΆΠ΅ числСнныС ΠΎΡ‚Π²Π΅Ρ‚Ρ‹, Ρ‡Ρ‚ΠΎ ΠΈ ΠΏΠ΅Ρ€Π²Ρ‹ΠΉ ΠΏΠΎΠ΄Ρ…ΠΎΠ΄.Π‘Ρ‚ΡƒΠ΄Π΅Π½Ρ‚Π°ΠΌ прСдлагаСтся ΠΈΡΠΏΠΎΠ»ΡŒΠ·ΠΎΠ²Π°Ρ‚ΡŒ Π½Π°ΠΈΠ±ΠΎΠ»Π΅Π΅ ΡƒΠ΄ΠΎΠ±Π½Ρ‹ΠΉ для Π½ΠΈΡ… ΠΏΠΎΠ΄Ρ…ΠΎΠ΄.

            Для Π΄ΠΎΠΏΠΎΠ»Π½ΠΈΡ‚Π΅Π»ΡŒΠ½ΠΎΠΉ ΠΏΡ€Π°ΠΊΡ‚ΠΈΠΊΠΈ рассмотритС ΡΠ»Π΅Π΄ΡƒΡŽΡ‰ΠΈΠ΅ Π·Π°Π΄Π°Ρ‡ΠΈ с двумя Ρ‚Π΅Π»Π°ΠΌΠΈ. Для ΠΊΠ°ΠΆΠ΄ΠΎΠΉ ΠΏΡ€ΠΎΠ±Π»Π΅ΠΌΡ‹ Π±Ρ‹Π»Π° прСдоставлСна ​​сокращСнная вСрсия Ρ€Π΅ΡˆΠ΅Π½ΠΈΡ . К Ρ‚Π΅ΠΌΠ΅ Π·Π°Π΄Π°Ρ‡ Π΄Π²ΡƒΡ… Ρ‚Π΅Π» ΠΌΡ‹ вСрнСмся Π² ΡΠ»Π΅Π΄ΡƒΡŽΡ‰Π΅ΠΉ Π³Π»Π°Π²Π΅, ΠΊΠΎΠ³Π΄Π° ΠΌΡ‹ рассмотрим ситуации, Π² ΠΊΠΎΡ‚ΠΎΡ€Ρ‹Ρ… ΡˆΠΊΠΈΠ²Ρ‹ ΠΈ ΠΎΠ±ΡŠΠ΅ΠΊΡ‚Ρ‹ двиТутся Π² Ρ€Π°Π·Π½Ρ‹Ρ… направлСниях.

            ΠŸΡ€ΠΎΠ²Π΅Ρ€ΡŒΡ‚Π΅ своС ΠΏΠΎΠ½ΠΈΠΌΠ°Π½ΠΈΠ΅

            1.Π“Ρ€ΡƒΠ·ΠΎΠ²ΠΈΠΊ Π²Π΅Π·Π΅Ρ‚ ΠΌΠ°ΡˆΠΈΠ½Ρƒ ΠΏΠΎ пСрСсСчСнной мСстности. Масса Π³Ρ€ΡƒΠ·ΠΎΠ²ΠΈΠΊΠ° 4,00Ρ…10 3 ΠΊΠ³, масса автомобиля 1,60Ρ…10 3 ΠΊΠ³. Если сила тяги, Π²ΠΎΠ·Π½ΠΈΠΊΠ°ΡŽΡ‰Π°Ρ Π² Ρ€Π΅Π·ΡƒΠ»ΡŒΡ‚Π°Ρ‚Π΅ ΠΏΠΎΠ²ΠΎΡ€ΠΎΡ‚Π° колСс Π³Ρ€ΡƒΠ·ΠΎΠ²ΠΈΠΊΠ°, составляСт 2,50×10 4 Н, Ρ‚ΠΎ ΠΎΠΏΡ€Π΅Π΄Π΅Π»ΠΈΡ‚Π΅ ускорСниС автомобиля (ΠΈΠ»ΠΈ Π³Ρ€ΡƒΠ·ΠΎΠ²ΠΈΠΊΠ°) ΠΈ силу, с ΠΊΠΎΡ‚ΠΎΡ€ΠΎΠΉ Π³Ρ€ΡƒΠ·ΠΎΠ²ΠΈΠΊ тянСт Π½Π° Π°Π²Ρ‚ΠΎΠΌΠΎΠ±ΠΈΠ»ΡŒ. ΠŸΡ€Π΅Π΄ΠΏΠΎΠ»ΠΎΠΆΠΈΠΌ, Ρ‡Ρ‚ΠΎ силы сопротивлСния Π²ΠΎΠ·Π΄ΡƒΡ…Π° Π½Π΅Π·Π½Π°Ρ‡ΠΈΡ‚Π΅Π»ΡŒΠ½Ρ‹.

            2. Π―Ρ‰ΠΈΠΊ вСсом 7,00 ΠΊΠ³ ΠΏΡ€ΠΈΠΊΡ€Π΅ΠΏΠ»Π΅Π½ ΠΊ 3.Π―Ρ‰ΠΈΠΊ массой 00 ΠΊΠ³ Π½Π° Π²Π΅Ρ€Π΅Π²ΠΊΠ΅ 1. Π―Ρ‰ΠΈΠΊ массой 7,00 ΠΊΠ³ тянут Π²Π΅Ρ€Π΅Π²ΠΊΠΎΠΉ 2 с усилиСм 25,0 Н. ΠžΠΏΡ€Π΅Π΄Π΅Π»ΠΈΡ‚Π΅ ускорСниС ящиков ΠΈ натяТСниС Π²Π΅Ρ€Π΅Π²ΠΊΠΈ 1. ΠšΠΎΡΡ„Ρ„ΠΈΡ†ΠΈΠ΅Π½Ρ‚ трСния ΠΌΠ΅ΠΆΠ΄Ρƒ Π·Π΅ΠΌΠ»Π΅ΠΉ ΠΈ ящиками составляСт 0,120 .

            3. Π’Ρ€Π°ΠΊΡ‚ΠΎΡ€ Ρ‚Π°Ρ‰ΠΈΡ‚ Π΄Π²Π° Π±ΠΎΠ»ΡŒΡˆΠΈΡ… Π±Ρ€Π΅Π²Π½Π° Ρ‡Π΅Ρ€Π΅Π· ΠΏΠΎΠ»Π΅. Π¦Π΅ΠΏΠΎΡ‡ΠΊΠ° соСдиняСт Π±Ρ€Π΅Π²Π½Π° Π΄Ρ€ΡƒΠ³ с Π΄Ρ€ΡƒΠ³ΠΎΠΌ; ΠΏΠ΅Ρ€Π΅Π΄Π½Π΅Π΅ Π±Ρ€Π΅Π²Π½ΠΎ соСдиняСтся с Ρ‚Ρ€Π°ΠΊΡ‚ΠΎΡ€ΠΎΠΌ ΠΎΡ‚Π΄Π΅Π»ΡŒΠ½ΠΎΠΉ Ρ†Π΅ΠΏΡŒΡŽ. Масса Π»ΠΎΠ±ΠΎΠ²ΠΎΠ³ΠΎ Π±Ρ€Π΅Π²Π½Π° 180 ΠΊΠ³.Масса Π·Π°Π΄Π½Π΅Π³ΠΎ Π±Ρ€Π΅Π²Π½Π° 220 ΠΊΠ³. ΠšΠΎΡΡ„Ρ„ΠΈΡ†ΠΈΠ΅Π½Ρ‚ трСния ΠΌΠ΅ΠΆΠ΄Ρƒ Π±Ρ€Π΅Π²Π½Π°ΠΌΠΈ ΠΈ ΠΏΠΎΠ»Π΅ΠΌ составляСт ΠΏΡ€ΠΈΠΌΠ΅Ρ€Π½ΠΎ 0,45. НатяТСниС Ρ†Π΅ΠΏΠΈ, ΡΠΎΠ΅Π΄ΠΈΠ½ΡΡŽΡ‰Π΅ΠΉ Ρ‚Ρ€Π°ΠΊΡ‚ΠΎΡ€ с ΠΏΠ΅Ρ€Π΅Π΄Π½ΠΈΠΌ Π±Ρ€Π΅Π²Π½ΠΎΠΌ, составляСт 1850 Н. ΠžΠΏΡ€Π΅Π΄Π΅Π»ΠΈΡ‚Π΅ ускорСниС Π±Ρ€Π΅Π²Π΅Π½ ΠΈ натяТСниС Ρ†Π΅ΠΏΠΈ, ΡΠΎΠ΅Π΄ΠΈΠ½ΡΡŽΡ‰Π΅ΠΉ Π΄Π²Π° Π±Ρ€Π΅Π²Π½Π°.

            4. Π”Π²Π° ящика скрСплСны ΠΏΡ€ΠΎΡ‡Π½ΠΎΠΉ ΠΏΡ€ΠΎΠ²ΠΎΠ»ΠΎΠΊΠΎΠΉ ΠΈ ΠΏΡ€ΠΈΠΊΡ€Π΅ΠΏΠ»Π΅Π½Ρ‹ ΠΊ ΠΏΠΎΡ‚ΠΎΠ»ΠΊΡƒ Π»ΠΈΡ„Ρ‚Π° Π²Ρ‚ΠΎΡ€Ρ‹ΠΌ ΠΏΡ€ΠΎΠ²ΠΎΠ΄ΠΎΠΌ (см. Π‘Ρ…Π΅ΠΌΡƒ).Масса приставки 14,2 ΠΊΠ³; масса Π½ΠΈΠΆΠ½Π΅Π³ΠΎ ящика 10,4 ΠΊΠ³. Π›ΠΈΡ„Ρ‚ поднимаСтся Π²Π²Π΅Ρ€Ρ… со ΡΠΊΠΎΡ€ΠΎΡΡ‚ΡŒΡŽ 2,84 ΠΌ / с 2 . (ΠŸΡ€Π΅Π΄ΠΏΠΎΠ»ΠΎΠΆΠΈΠΌ, Ρ‡Ρ‚ΠΎ ΠΏΡ€ΠΎΠ²ΠΎΠ΄ ΠΎΡ‚Π½ΠΎΡΠΈΡ‚Π΅Π»ΡŒΠ½ΠΎ бСзмассовый .)

            (a) НайдитС натяТСниС Π²Π΅Ρ€Ρ…Π½Π΅Π³ΠΎ троса (Ρ‚ΠΎΡ‡ΠΊΠΈ соСдинСния A ΠΈ B).

            (b) НайдитС натяТСниС Π½ΠΈΠΆΠ½Π΅ΠΉ ΠΏΡ€ΠΎΠ²ΠΎΠ»ΠΎΠΊΠΈ (Ρ‚ΠΎΡ‡ΠΊΠΈ соСдинСния C ΠΈ D).

            6.1 РСшСниС ΠΏΡ€ΠΎΠ±Π»Π΅ΠΌ с ΠΏΠΎΠΌΠΎΡ‰ΡŒΡŽ Π·Π°ΠΊΠΎΠ½ΠΎΠ² ΠΡŒΡŽΡ‚ΠΎΠ½Π° – УнивСрситСтская Ρ„ΠΈΠ·ΠΈΠΊΠ°, Ρ‚ΠΎΠΌ 1

            Π‘ΠΈΠ»Π° сопротивлСния Π½Π° Π±Π°Ρ€ΠΆΠ΅
            Π”Π²Π° буксира Ρ‚ΠΎΠ»ΠΊΠ°ΡŽΡ‚ Π±Π°Ρ€ΠΆΡƒ ΠΏΠΎΠ΄ Ρ€Π°Π·Π½Ρ‹ΠΌΠΈ ΡƒΠ³Π»Π°ΠΌΠΈ (рис. 6.4). ΠŸΠ΅Ρ€Π²Ρ‹ΠΉ буксир ΠΏΡ€ΠΈΠΊΠ»Π°Π΄Ρ‹Π²Π°Π΅Ρ‚ силу 2,7 Γ— 105 Н2,7 Γ— 105 Н Π² Π½Π°ΠΏΡ€Π°Π²Π»Π΅Π½ΠΈΠΈ x , Π° Π²Ρ‚ΠΎΡ€ΠΎΠΉ буксир ΠΏΡ€ΠΈΠΊΠ»Π°Π΄Ρ‹Π²Π°Π΅Ρ‚ силу 3,6 Γ— 105 Н3,6 Γ— 105 Н Π² Π½Π°ΠΏΡ€Π°Π²Π»Π΅Π½ΠΈΠΈ y . Масса Π±Π°Ρ€ΠΆΠΈ составляСт 5,0 Γ— 106 ΠΊΠ³ 5,0 Γ— 106 ΠΊΠ³, Π° Π΅Π΅ ускорСниС составляСт 7,5 Γ— 10–2 ΠΌ / с27,5 Γ— 10–2 ΠΌ / с2 Π² ΡƒΠΊΠ°Π·Π°Π½Π½ΠΎΠΌ Π½Π°ΠΏΡ€Π°Π²Π»Π΅Π½ΠΈΠΈ. Какова сила сопротивлСния Π²ΠΎΠ΄Ρ‹ Π±Π°Ρ€ΠΆΠ΅, ΡΠΎΠΏΡ€ΠΎΡ‚ΠΈΠ²Π»ΡΡŽΡ‰Π΅ΠΉΡΡ двиТСнию? ( ΠŸΡ€ΠΈΠΌΠ΅Ρ‡Π°Π½ΠΈΠ΅: Π‘ΠΈΠ»Π° сопротивлСния – это сила трСния, создаваСмая Тидкостями, Ρ‚Π°ΠΊΠΈΠΌΠΈ ΠΊΠ°ΠΊ Π²ΠΎΠ·Π΄ΡƒΡ… ΠΈΠ»ΠΈ Π²ΠΎΠ΄Π°.Π‘ΠΈΠ»Π° сопротивлСния прСпятствуСт двиТСнию ΠΎΠ±ΡŠΠ΅ΠΊΡ‚Π°. ΠŸΠΎΡΠΊΠΎΠ»ΡŒΠΊΡƒ Π±Π°Ρ€ΠΆΠ° ΠΈΠΌΠ΅Π΅Ρ‚ плоскоС Π΄Π½ΠΎ, ΠΌΠΎΠΆΠ½ΠΎ ΠΏΡ€Π΅Π΄ΠΏΠΎΠ»ΠΎΠΆΠΈΡ‚ΡŒ, Ρ‡Ρ‚ΠΎ сила сопротивлСния Π½Π°ΠΏΡ€Π°Π²Π»Π΅Π½Π° ​​в Π½Π°ΠΏΡ€Π°Π²Π»Π΅Π½ΠΈΠΈ, ΠΏΡ€ΠΎΡ‚ΠΈΠ²ΠΎΠΏΠΎΠ»ΠΎΠΆΠ½ΠΎΠΌ двиТСнию Π±Π°Ρ€ΠΆΠΈ.) Рис. 6.4 (a) Π’ΠΈΠ΄ свСрху Π΄Π²ΡƒΡ… буксиров, Ρ‚ΠΎΠ»ΠΊΠ°ΡŽΡ‰ΠΈΡ… Π±Π°Ρ€ΠΆΡƒ. (b) Π‘Ρ…Π΅ΠΌΠ° свободного Ρ‚Π΅Π»Π° для корабля содСрТит Ρ‚ΠΎΠ»ΡŒΠΊΠΎ силы, Π΄Π΅ΠΉΡΡ‚Π²ΡƒΡŽΡ‰ΠΈΠ΅ Π² плоскости Π²ΠΎΠ΄Ρ‹. Π’ Π½Π΅ΠΌ Π½Π΅ ΡƒΡ‡ΠΈΡ‚Ρ‹Π²Π°ΡŽΡ‚ΡΡ Π΄Π²Π΅ Π²Π΅Ρ€Ρ‚ΠΈΠΊΠ°Π»ΡŒΠ½Ρ‹Π΅ силы – вСс Π±Π°Ρ€ΠΆΠΈ ΠΈ подъСмная сила ΠΏΠΎΠ΄Π΄Π΅Ρ€ΠΆΠΈΠ²Π°ΡŽΡ‰Π΅ΠΉ Π΅Π΅ Π²ΠΎΠ΄Ρ‹, ΠΊΠΎΡ‚ΠΎΡ€Ρ‹Π΅ Π½Π΅ ΠΏΠΎΠΊΠ°Π·Π°Π½Ρ‹. ΠžΠ±Ρ€Π°Ρ‚ΠΈΡ‚Π΅ Π²Π½ΠΈΠΌΠ°Π½ΠΈΠ΅, Ρ‡Ρ‚ΠΎ F β†’ appF β†’ app – это общая прилоТСнная сила буксиров.
            БтратСгия
            НаправлСния ΠΈ Π²Π΅Π»ΠΈΡ‡ΠΈΠ½Ρ‹ ускорСния ΠΈ ΠΏΡ€ΠΈΠ»ΠΎΠΆΠ΅Π½Π½Ρ‹Ρ… сил ΠΏΠΎΠΊΠ°Π·Π°Π½Ρ‹ Π½Π° РисункС 6.4 (Π°). ΠœΡ‹ опрСдСляСм ΠΎΠ±Ρ‰ΡƒΡŽ силу буксиров Π½Π° Π±Π°Ρ€ΠΆΠ΅ ΠΊΠ°ΠΊ F β†’ appF β†’ app Ρ‚Π°ΠΊ, Ρ‡Ρ‚ΠΎΠ±Ρ‹ F β†’ ΠΏΡ€ΠΈΠ»ΠΎΠΆΠ΅Π½ΠΈΠ΅ = F β†’ 1 + F β†’ 2.F β†’ ΠΏΡ€ΠΈΠ»ΠΎΠΆΠ΅Π½ΠΈΠ΅ = F β†’ 1 + F β†’ 2.

            Π‘ΠΎΠΏΡ€ΠΎΡ‚ΠΈΠ²Π»Π΅Π½ΠΈΠ΅ Π²ΠΎΠ΄Ρ‹ F β†’ DF β†’ D Π½Π°ΠΏΡ€Π°Π²Π»Π΅Π½ΠΎ Π² Π½Π°ΠΏΡ€Π°Π²Π»Π΅Π½ΠΈΠΈ, ΠΏΡ€ΠΎΡ‚ΠΈΠ²ΠΎΠΏΠΎΠ»ΠΎΠΆΠ½ΠΎΠΌ Π½Π°ΠΏΡ€Π°Π²Π»Π΅Π½ΠΈΡŽ двиТСния Π»ΠΎΠ΄ΠΊΠΈ; эта сила, Ρ‚Π°ΠΊΠΈΠΌ ΠΎΠ±Ρ€Π°Π·ΠΎΠΌ, дСйствуСт ΠΏΡ€ΠΎΡ‚ΠΈΠ² F β†’ app, F β†’ app, ΠΊΠ°ΠΊ ΠΏΠΎΠΊΠ°Π·Π°Π½ΠΎ Π½Π° Π΄ΠΈΠ°Π³Ρ€Π°ΠΌΠΌΠ΅ свободного Ρ‚Π΅Π»Π° Π½Π° РисункС 6.4 (b). Π—Π΄Π΅ΡΡŒ прСдставляСт интСрСс систСма Π±Π°Ρ€ΠΆΠ°, ΠΏΠΎΡΠΊΠΎΠ»ΡŒΠΊΡƒ Π½Π° Π½Π΅Π΅ Π΄Π΅ΠΉΡΡ‚Π²ΡƒΡŽΡ‚ силы, Π° Ρ‚Π°ΠΊΠΆΠ΅ Π΅Π΅ ускорСниС.ΠŸΠΎΡΠΊΠΎΠ»ΡŒΠΊΡƒ ΠΏΡ€ΠΈΠ»ΠΎΠΆΠ΅Π½Π½Ρ‹Π΅ силы пСрпСндикулярны, оси x ΠΈ y находятся Π² Ρ‚ΠΎΠΌ ΠΆΠ΅ Π½Π°ΠΏΡ€Π°Π²Π»Π΅Π½ΠΈΠΈ, Ρ‡Ρ‚ΠΎ ΠΈ F β†’ ​​1F β†’ 1 ΠΈ F β†’ ​​2.F β†’ 2. ΠŸΡ€ΠΎΠ±Π»Π΅ΠΌΠ° быстро становится ΠΎΠ΄Π½ΠΎΠΌΠ΅Ρ€Π½ΠΎΠΉ Π² Π½Π°ΠΏΡ€Π°Π²Π»Π΅Π½ΠΈΠΈ F β†’ ​​appF β†’ app, ΠΏΠΎΡΠΊΠΎΠ»ΡŒΠΊΡƒ Ρ‚Ρ€Π΅Π½ΠΈΠ΅ происходит Π² Π½Π°ΠΏΡ€Π°Π²Π»Π΅Π½ΠΈΠΈ, ΠΏΡ€ΠΎΡ‚ΠΈΠ²ΠΎΠΏΠΎΠ»ΠΎΠΆΠ½ΠΎΠΌ F β†’ app.F β†’ app. Наша стратСгия состоит Π² Ρ‚ΠΎΠΌ, Ρ‡Ρ‚ΠΎΠ±Ρ‹ Π½Π°ΠΉΡ‚ΠΈ Π²Π΅Π»ΠΈΡ‡ΠΈΠ½Ρƒ ΠΈ Π½Π°ΠΏΡ€Π°Π²Π»Π΅Π½ΠΈΠ΅ чистой ΠΏΡ€ΠΈΠ»ΠΎΠΆΠ΅Π½Π½ΠΎΠΉ силы F β†’ appF β†’ app, Π° Π·Π°Ρ‚Π΅ΠΌ ΠΏΡ€ΠΈΠΌΠ΅Π½ΠΈΡ‚ΡŒ Π²Ρ‚ΠΎΡ€ΠΎΠΉ Π·Π°ΠΊΠΎΠ½ ΠΡŒΡŽΡ‚ΠΎΠ½Π° для опрСдСлСния силы сопротивлСния F β†’ D.F β†’ D.

            РСшСниС
            ΠŸΠΎΡΠΊΠΎΠ»ΡŒΠΊΡƒ FxFx ΠΈ FyFy пСрпСндикулярны, ΠΌΡ‹ ΠΌΠΎΠΆΠ΅ΠΌ ΠΎΠΏΡ€Π΅Π΄Π΅Π»ΠΈΡ‚ΡŒ Π²Π΅Π»ΠΈΡ‡ΠΈΠ½Ρƒ ΠΈ Π½Π°ΠΏΡ€Π°Π²Π»Π΅Π½ΠΈΠ΅ F β†’ appF β†’ app Π½Π°ΠΏΡ€ΡΠΌΡƒΡŽ.Π’ΠΎ-ΠΏΠ΅Ρ€Π²Ρ‹Ρ…, Ρ€Π΅Π·ΡƒΠ»ΡŒΡ‚ΠΈΡ€ΡƒΡŽΡ‰Π°Ρ Π²Π΅Π»ΠΈΡ‡ΠΈΠ½Π° опрСдСляСтся Ρ‚Π΅ΠΎΡ€Π΅ΠΌΠΎΠΉ ΠŸΠΈΡ„Π°Π³ΠΎΡ€Π°: Fapp = F12 + F22 = (2.7 Γ— 105N) 2+ (3.6 Γ— 105N) 2 = 4.5 Γ— 105N. Fapp = F12 + F22 = (2.7 Γ— 105N) 2+ (3.6 Γ— 105N) 2 = 4.5 Γ— 105N.

            Π£Π³ΠΎΠ» Ρ€Π°Π²Π΅Π½

            ΞΈ = tan βˆ’ 1 (F2F1) = tan βˆ’ 1 (3,6 Γ— 105N2,7 Γ— 105N) = 53,1 Β°. ΞΈ = tan βˆ’ 1 (F2F1) = tan βˆ’ 1 (3,6 Γ— 105N2,7 Γ— 105N) = 53,1 Β°.

            Из ΠΏΠ΅Ρ€Π²ΠΎΠ³ΠΎ Π·Π°ΠΊΠΎΠ½Π° ΠΡŒΡŽΡ‚ΠΎΠ½Π° ΠΌΡ‹ Π·Π½Π°Π΅ΠΌ, Ρ‡Ρ‚ΠΎ это Ρ‚ΠΎ ΠΆΠ΅ Π½Π°ΠΏΡ€Π°Π²Π»Π΅Π½ΠΈΠ΅, Ρ‡Ρ‚ΠΎ ΠΈ ускорСниС. ΠœΡ‹ Ρ‚Π°ΠΊΠΆΠ΅ Π·Π½Π°Π΅ΠΌ, Ρ‡Ρ‚ΠΎ F β†’ DF β†’ D находится Π² ΠΏΡ€ΠΎΡ‚ΠΈΠ²ΠΎΠΏΠΎΠ»ΠΎΠΆΠ½ΠΎΠΌ Π½Π°ΠΏΡ€Π°Π²Π»Π΅Π½ΠΈΠΈ ΠΎΡ‚ F β†’ app, F β†’ app, ΠΏΠΎΡΠΊΠΎΠ»ΡŒΠΊΡƒ ΠΎΠ½ замСдляСт ускорСниС. Π‘Π»Π΅Π΄ΠΎΠ²Π°Ρ‚Π΅Π»ΡŒΠ½ΠΎ, чистая внСшняя сила ΠΈΠΌΠ΅Π΅Ρ‚ Ρ‚ΠΎ ΠΆΠ΅ Π½Π°ΠΏΡ€Π°Π²Π»Π΅Π½ΠΈΠ΅, Ρ‡Ρ‚ΠΎ ΠΈ F β†’ ​​app, F β†’ app, Π½ΠΎ Π΅Π΅ Π²Π΅Π»ΠΈΡ‡ΠΈΠ½Π° Π½Π΅ΠΌΠ½ΠΎΠ³ΠΎ мСньшС, Ρ‡Π΅ΠΌ F β†’ app.F β†’ ΠΏΡ€ΠΈΠ»ΠΎΠΆΠ΅Π½ΠΈΠ΅. ΠŸΡ€ΠΎΠ±Π»Π΅ΠΌΠ° Ρ‚Π΅ΠΏΠ΅Ρ€ΡŒ одномСрная. Из Π΄ΠΈΠ°Π³Ρ€Π°ΠΌΠΌΡ‹ свободного Ρ‚Π΅Π»Π° Π²ΠΈΠ΄Π½ΠΎ, Ρ‡Ρ‚ΠΎ

            Fnet = Fapp-FD.Fnet = Fapp-FD.

            Однако Π²Ρ‚ΠΎΡ€ΠΎΠΉ Π·Π°ΠΊΠΎΠ½ ΠΡŒΡŽΡ‚ΠΎΠ½Π° гласит, Ρ‡Ρ‚ΠΎ

            Π’Π°ΠΊΠΈΠΌ ΠΎΠ±Ρ€Π°Π·ΠΎΠΌ,

            Fapp-FD = ma.Fapp-FD = ma.

            Π­Ρ‚ΠΎ ΠΌΠΎΠΆΠ΅Ρ‚ Π±Ρ‹Ρ‚ΡŒ Ρ€Π΅ΡˆΠ΅Π½ΠΎ для Π²Π΅Π»ΠΈΡ‡ΠΈΠ½Ρ‹ силы сопротивлСния Π²ΠΎΠ΄Π½ΠΎΠ³ΠΎ FDFD Π² Π²ΠΈΠ΄Π΅ извСстных Π²Π΅Π»ΠΈΡ‡ΠΈΠ½:

            FD = Fapp-ma. FD = Fapp-ma.

            ΠŸΠΎΠ΄ΡΡ‚Π°Π½ΠΎΠ²ΠΊΠ° извСстных Π·Π½Π°Ρ‡Π΅Π½ΠΈΠΉ Π΄Π°Π΅Ρ‚

            FD = (4,5 Γ— 105 Н) – (5,0 Γ— 106 ΠΊΠ³) (7,5 Γ— 10–2 ΠΌ / с2) = 7,5 Γ— 104 Н. FD = (4,5 Γ— 105 Н) – (5,0 Γ— 106 ΠΊΠ³) (7,5 Γ— 10–2 ΠΌ / с2) = 7,5 Γ— 104 Н.

            НаправлСниС F β†’ DF β†’ D ΡƒΠΆΠ΅ ΠΎΠΏΡ€Π΅Π΄Π΅Π»Π΅Π½ΠΎ ΠΊΠ°ΠΊ Π½Π°ΠΏΡ€Π°Π²Π»Π΅Π½ΠΈΠ΅, ΠΏΡ€ΠΎΡ‚ΠΈΠ²ΠΎΠΏΠΎΠ»ΠΎΠΆΠ½ΠΎΠ΅ F β†’ app, F β†’ app, ΠΈΠ»ΠΈ ΠΏΠΎΠ΄ ΡƒΠ³Π»ΠΎΠΌ 53 Β° 53 Β° ΠΊ ΡŽΠ³Ρƒ ΠΎΡ‚ Π·Π°ΠΏΠ°Π΄Π°.


Π”ΠΎΠ±Π°Π²ΠΈΡ‚ΡŒ ΠΊΠΎΠΌΠΌΠ΅Π½Ρ‚Π°Ρ€ΠΈΠΉ

Π’Π°Ρˆ адрСс email Π½Π΅ Π±ΡƒΠ΄Π΅Ρ‚ ΠΎΠΏΡƒΠ±Π»ΠΈΠΊΠΎΠ²Π°Π½. ΠžΠ±ΡΠ·Π°Ρ‚Π΅Π»ΡŒΠ½Ρ‹Π΅ поля ΠΏΠΎΠΌΠ΅Ρ‡Π΅Π½Ρ‹ *